You are on page 1of 394

Departamento de F sica, Facultad de Ciencias, Universidad de Chile. n Las Palmeras 3425, Nuoa.

. Casilla 653, Correo 1, Santiago fono: 562 678 7276 fax: 562 271 2973 e-mail: secretaria@sica.ciencias.uchile.cl

INTRODUCCION A LA MECANICA
Herbert Massmann

Transcriptores: V ctor Muoz G. n Max Ram G. rez

Indice general
1. Expansiones y Trigonometr a 1.1. Expansiones y series . . . . . . . . . 1.2. Elementos de trigonometr . . . . . a 1.3. Problemas . . . . . . . . . . . . . . . 1.4. Solucin a algunos de los problemas o 1 1 4 11 20 25 25 33 36 37 47 54 57 57 62 68 70 81

. . . .

. . . .

. . . .

. . . .

. . . .

. . . .

. . . .

. . . .

. . . .

. . . .

. . . .

. . . .

. . . .

. . . .

. . . .

. . . .

. . . .

. . . .

. . . .

. . . .

. . . .

. . . .

2. Cinemtica en una dimensin a o 2.1. Posicin, velocidad y aceleracin . . . . . . . o o 2.2. El camino inverso . . . . . . . . . . . . . . . . 2.3. Mximos y m a nimos . . . . . . . . . . . . . . 2.4. Problemas . . . . . . . . . . . . . . . . . . . . 2.5. Solucin a algunos de los problemas . . . . . o 2.6. Elementos del clculo innitesimal e integral . a 3. Cinemtica en dos y tres dimensiones a 3.1. Vectores . . . . . . . . . . . . . . . . . 3.2. Cinemtica . . . . . . . . . . . . . . . a Coordenadas polares . . . . . . . . . 3.3. 3.4. Problemas . . . . . . . . . . . . . . . . 3.5. Solucin a algunos de los problemas . o 4. Las 4.1. 4.2. 4.3. 4.4. 4.5. 4.6. leyes de Newton Espacio y tiempo . . . . . . . . . . . Las leyes de Newton . . . . . . . . . Uso de las leyes de Newton . . . . . Roce cintico y esttico . . . . . . . e a Problemas . . . . . . . . . . . . . . . Solucin a algunos de los problemas o

. . . . . .

. . . . . .

. . . . . .

. . . . . .

. . . . . .

. . . . . .

. . . . . .

. . . . . .

. . . . . .

. . . . . .

. . . . . .

. . . . . .

. . . . . .

. . . . . .

. . . . . .

. . . . . .

. . . . . .

. . . . .

. . . . .

. . . . .

. . . . .

. . . . .

. . . . .

. . . . .

. . . . .

. . . . .

. . . . .

. . . . .

. . . . .

. . . . .

. . . . .

. . . . .

. . . . .

. . . . .

. . . . .

. . . . .

. . . . .

. . . . .

. . . . . .

. . . . . .

. . . . . .

. . . . . .

. . . . . .

. . . . . .

. . . . . .

. . . . . .

. . . . . .

. . . . . .

. . . . . .

. . . . . .

. . . . . .

. . . . . .

. . . . . .

. . . . . .

. . . . . .

. . . . . .

. . . . . .

. . . . . .

. . . . . .

87 . 87 . 88 . 91 . 96 . 100 . 112 123 123 129 132 135 147

5. Trabajo y Energ a 5.1. Trabajo y energ para movimientos en una dimensin a o 5.2. Trabajo para un movimiento en tres dimensiones . . . 5.3. Ejemplos . . . . . . . . . . . . . . . . . . . . . . . . . 5.4. Problemas . . . . . . . . . . . . . . . . . . . . . . . . . 5.5. Solucin a algunos de los problemas . . . . . . . . . . o

. . . . .

. . . . .

. . . . .

. . . . .

. . . . .

. . . . .

. . . . .

. . . . .

. . . . .

. . . . .

. . . . .

. . . . .

II

INDICE GENERAL 155 155 157 160 162 169 174 183 183 186 188 190 192 196 204 213 213 215 220 224 229 229 232 237 249 259 259 262 265 268 273 273 273 274 277 280 283 289 291 295 296

6. Momento lineal y colisiones 6.1. Conservacin del momento lineal . . o 6.2. Colisiones . . . . . . . . . . . . . . . 6.3. Impulso . . . . . . . . . . . . . . . . 6.4. Problemas . . . . . . . . . . . . . . . 6.5. Solucin a algunos de los problemas o 6.6. Colisin de dos discos . . . . . . . . o

. . . . . .

. . . . . .

. . . . . .

. . . . . .

. . . . . .

. . . . . .

. . . . . .

. . . . . .

. . . . . .

. . . . . .

. . . . . .

. . . . . .

. . . . . .

. . . . . .

. . . . . .

. . . . . .

. . . . . .

. . . . . .

. . . . . .

. . . . . .

. . . . . .

. . . . . .

7. Torque, centro de masas y equilibrio 7.1. Producto vectorial . . . . . . . . . . . . . 7.2. Torque . . . . . . . . . . . . . . . . . . . . 7.3. Centro de masas . . . . . . . . . . . . . . 7.4. Evaluacin numrica del centro de masas o e 7.5. Equilibrio . . . . . . . . . . . . . . . . . . 7.6. Problemas . . . . . . . . . . . . . . . . . . 7.7. Solucin a algunos de los problemas . . . o 8. Momento angular 8.1. Momento angular de una part cula . . 8.2. Momento angular de varias part Iculas 8.3. Problemas . . . . . . . . . . . . . . . . 8.4. Solucin a algunos de los problemas . o 9. Rotacin de un cuerpo r o gido 9.1. Las ecuaciones bsicas . . . . . . . . a 9.2. Momento de inercia . . . . . . . . . 9.3. Problemas . . . . . . . . . . . . . . . 9.4. Solucin a algunos de los problemas o 10.Fuerzas cticias 10.1. Referencial uniformemente acelerado 10.2. Referencial en rotacin uniforme . . o 10.3. Problemas . . . . . . . . . . . . . . . 10.4. Solucin a algunos de los problemas o

. . . . . . .

. . . . . . .

. . . . . . .

. . . . . . .

. . . . . . .

. . . . . . .

. . . . . . .

. . . . . . .

. . . . . . .

. . . . . . .

. . . . . . .

. . . . . . .

. . . . . . .

. . . . . . .

. . . . . . .

. . . . . . .

. . . . . . .

. . . . . . .

. . . . . . .

. . . .

. . . .

. . . .

. . . .

. . . .

. . . .

. . . .

. . . .

. . . .

. . . .

. . . .

. . . .

. . . .

. . . .

. . . .

. . . .

. . . .

. . . .

. . . .

. . . .

. . . .

. . . .

. . . .

. . . .

. . . .

. . . .

. . . .

. . . .

. . . .

. . . .

. . . .

. . . .

. . . .

. . . .

. . . .

. . . .

. . . .

. . . .

. . . .

. . . .

. . . .

. . . .

. . . .

. . . .

. . . .

. . . .

. . . .

. . . .

. . . .

. . . .

. . . .

. . . .

. . . .

. . . .

. . . .

. . . .

. . . .

. . . .

. . . .

. . . .

. . . .

. . . .

. . . .

. . . .

. . . .

11.Gravitacin o 11.1. Elipses . . . . . . . . . . . . . . . . . . . . . 11.1.1. Elipse en coordenadas cartesianas . 11.1.2. Elipse en coordenadas polares . . . . 11.2. Las leyes de Kepler . . . . . . . . . . . . . . 11.3. Satlites . . . . . . . . . . . . . . . . . . . . e 11.4. Potencial efectivo . . . . . . . . . . . . . . . 11.5. Trayectorias de los satlites . . . . . . . . . e 11.6. El campo y potencial gravitacional . . . . . 11.7. El caso elctrico: la ley de Coulomb . . . . . e 11.8. Campo gravitacional de una cscara esfrica a e

. . . . . . . . . .

. . . . . . . . . .

. . . . . . . . . .

. . . . . . . . . .

. . . . . . . . . .

. . . . . . . . . .

. . . . . . . . . .

. . . . . . . . . .

. . . . . . . . . .

. . . . . . . . . .

. . . . . . . . . .

. . . . . . . . . .

. . . . . . . . . .

. . . . . . . . . .

. . . . . . . . . .

. . . . . . . . . .

. . . . . . . . . .

. . . . . . . . . .

INDICE GENERAL 11.9. Campo gravitacional de una esfrica slida . e o 11.9.1. Densidad media de la Tierra . . . . 11.10. roblemas . . . . . . . . . . . . . . . . . . . P 11.11. olucin a algunos de los problemas . . . . S o 12.Fluidos 12.1. Conceptos Preliminares . . . . . . . . 12.2. La presin atmosfrica P0 . . . . . . . o e 12.3. Principio de Arqu medes . . . . . . . . 12.4. La frmula baromtrica . . . . . . . . o e 12.5. Tensin supercial . . . . . . . . . . . o 12.6. Capilaridad . . . . . . . . . . . . . . . 12.7. Fluidos en movimiento . . . . . . . . . 12.8. Aplicaciones del principio de Bernoulli 12.9. *Viscosidad . . . . . . . . . . . . . . . 12.10. roblemas . . . . . . . . . . . . . . . . P 12.11. olucin a algunos de los problemas . S o . . . . . . . . . . . . . . . . . . . . . . . . . . . . . . . . . . . . . . . . . . . . . . . . . . . . . . . . . . . . . . . . . . . . . . . .

III

300 301 302 310 317 317 318 320 323 326 328 329 331 335 338 349 353 353 356 358 361 364 368 372 383

. . . . . . . . . . .

. . . . . . . . . . .

. . . . . . . . . . .

. . . . . . . . . . .

. . . . . . . . . . .

. . . . . . . . . . .

. . . . . . . . . . .

. . . . . . . . . . .

. . . . . . . . . . .

. . . . . . . . . . .

. . . . . . . . . . .

. . . . . . . . . . .

. . . . . . . . . . .

. . . . . . . . . . .

. . . . . . . . . . .

. . . . . . . . . . .

. . . . . . . . . . .

. . . . . . . . . . .

. . . . . . . . . . .

. . . . . . . . . . .

. . . . . . . . . . .

13.Oscilador Armnico o 2 13.1. La ecuacin diferencial x(t) + 0 x(t) = 0 o 13.2. El oscilador armnico simple . . . . . . o 13.3. El oscilador armnico atenuado . . . . . o 13.4. El oscilador armnico forzado . . . . . . o 13.5. Osciladores armnicos acoplados . . . . o 13.6. Modos normales de una cuerda . . . . 13.7. Problemas . . . . . . . . . . . . . . . . . 13.8. Solucin a algunos de los problemas . . o

. . . . . . . .

. . . . . . . .

. . . . . . . .

. . . . . . . .

. . . . . . . .

. . . . . . . .

. . . . . . . .

. . . . . . . .

. . . . . . . .

. . . . . . . .

. . . . . . . .

. . . . . . . .

. . . . . . . .

. . . . . . . .

. . . . . . . .

. . . . . . . .

. . . . . . . .

. . . . . . . .

. . . . . . . .

. . . . . . . .

Cap tulo 1

Expansiones y Trigonometr a
En este primer cap tulo se recopilarn algunos resultados de las matemticas que son bsicos a a a para los cap tulos que siguen.

1.1.

Expansiones y series

Consideremos las expansiones: (1 + x)1 = 1 + x (1 + x)2 = 1 + 2x + x2 (1 + x)3 = 1 + 3x + 3x2 + x3 (1 + x)4 = 1 + 4x + 6x2 + 4x3 + x4 (1 + x)5 = 1 + 5x + 10x2 + 10x3 + 5x4 + x5 Generalizando, para un entero n positivo arbitrario, la expansin del binomio (1+x)n puede o escribirse en la forma (1 + x)n = 1 + n n (n 1) 2 n (n 1) (n 2) 3 x+ x + x 1! 2! 3! n (n 1) (n 2) (n 3) 4 + x + + nx(n1) + xn , 4! (1.1) donde n! 1 2 3 . . . (n 1) n. Por denicin 0! 1. La expansin 1.1 es vlida para o o a cualquier valor de x y cualquier valor de n entero no negativo. Una expresin anloga tambin se puede escribir para (1 + x) , donde es ahora cualquier o a e nmero real. En efecto, en ese caso u (1 + x) = 1 + ( 1) 2 ( 1) ( 2) 3 x+ x + x 1! 2! 3! ( 1) ( 2) ( 3) 4 + x + . 4!

(1.2)

Expansiones y Trigonometr a

Sin embargo, si no es nulo o un entero positivo, hay una diferencia importante entre las dos expresiones: la expansin (1.1), con n entero no negativo siempre tiene una cantidad o nita de trminos y se puede usar para cualquier valor de x; la serie (1.2), por otra parte, e posee innitos trminos (sumandos) y slo se puede usar (en el lenguaje tcnico, converge) e o e si |x| < 1. Ejemplos: 1. Usando la ecuacin (1.2) con = 1 se obtiene la serie geomtrica o e (1 x)1 = 1 = 1 + x + x2 + x3 + x4 + 1x (1.3)

Si bien el lado izquierdo est bien denido para cualquier valor de x, el lado derecho a slo da un resultado nito si |x| < 1. o Para x = 1/2 el lado izquierdo es igual a 2, mientras que el lado derecho da la serie 1+ 1 1 1 1 + + + + ... 2 4 8 16

que, obviamente, al sumarla, tambin da 2. e Para x = 1/10 el lado izquierdo es igual a 10/9, mientras que el lado derecho da la serie 1 + 0, 1 + 0, 01 + 0, 001 + . . . = 1, 1111 . . . . que es el desarrollo decimal de 10/9. 2. Evaluemos la suma nita SN = 1 + x + x2 + x3 + + xN . Para ello restemos de esta serie la misma serie, pero multiplicada por x, es decir: SN x SN = 1 + x + x2 + x3 + + xN = x + x2 + x3 + + xN + xN +1 .

Al restar, al lado izquierdo queda (1 x) SN , mientras que al lado derecho queda 1 xN +1 , o sea, (1 x) SN = 1 xN +1 . 1 xN +1 . 1x Si hacemos N cada vez ms grande, es decir lo hacemos tender a innito, en el lado a derecho se tendr algo nito slo si |x| < 1. En efecto, en ese caso l N xN +1 = 0 a o m y entonces 1 , l SN = 1 + x + x2 + x3 + = m N 1x resultado consistente con el del ejemplo 1. SN = Despejando SN se obtiene

1.1 Expansiones y series 3. Escribamos la relacin (1.2) para = 1/2. En ese caso se obtiene o (1 + x)1/2 = 1 1 1 1 + x = 1 + x x2 + x3 2 8 16

La razn por la cual esta expresin es util es que con frecuencia se requerir evaluar o o a la ra de (1 + x) para situaciones en que x es un nmero muy pequeo. En ese caso z u n los trminos sucesivos de la serie son cada vez ms pequeos y es posible obtener un e a n resultado satisfactorio usando slo los dos o tres primeros trminos del lado derecho. o e La tabla adjunta muestra un pequeo anlisis para x = 0,1: n a lado izquierdo 1,04880884817 lado derecho 1,0 1,05 1,04875 1,0488125 # de trminos e 1 2 3 4 error 4,9 % 0,11 % 0,0059 % 0,00037 %

Ejercicio: Verique que para valores de x ms pequeos, la convergencia del resultado a n de la serie truncada hacia el resultado exacto es aun ms rpida. a a 4. Sea = 0 un nmero real arbitrario y evaluemos [(1 + x) 1]/x para valores de u x muy pequeos. Observe que para valores de x cada vez ms pequeos, tanto el n a n numerador como el denominador tienden a cero. De acuerdo a la ecuacin (1.2), para x muy pequeo vale la aproximacin o n o (1 + x) 1+x

(o sea, estamos despreciando todos los trminos de la serie excepto los dos primeros). e Usando esta aproximacin se encuentra que (para x muy pequeo) o n (1 + x) 1 x 1 + x 1 x = = . x x

Verique numricamente este resultado usando una calculadora. e

Algunas aproximaciones que se obtienen a partir de la ecuacin (1.2) para |x| pequeo, que o n se usarn con frecuencia, y conviene tener siempre presentes, son: a (1 + x) 1 1+x 1 1x 1+x 1 + x , (1.4)

1x, 1+x, 1+ x . 2

(1.5) (1.6) (1.7)

Expansiones y Trigonometr a

Figura 1.1

Para abreviar la escritura de series, se usa frecuentemente la letra griega sigma mayscula u ( ). Ilustramos el uso de este s mbolo con algunos ejemplos:
6

j = 1 + 2 + 3 + 4 + 5 + 6 = 21 ,
j=1 4

j 2 = 12 + 22 + 32 + 42 = 30 ,
j=1 2

j k = j 2 + j 1 + 1 + j + j 2 ,
k=2 n=0

1 2

=1+

1 1 1 + + + = 2 . 2 4 8

1.2.

Elementos de trigonometr a

Consideremos los tringulos rectngulos (ABC) y (AB C ) mostrados en la gura 1.1. a a De acuerdo a un teorema de la geometr elemental, la razn (entre trazos) AC : AB es a o e o a igual a la razn AC : AB , dependiendo sta slo del valor del ngulo . Se ha convenido o llamar a tal razn cos ; o sea, en un tringulo rectngulo, el cuociente entre el cateto o a a adyacente y la hipotenusa dene el coseno del ngulo que forman esos dos lados: a longitud del lado adyacente AC = . longitud de la hipotenusa AB Tambin el cuociente entre el cateto opuesto al ngulo y la hipotenusa es independiente e a del tamao del tringulo rectngulo y slo depende del valor de . A esta razn se la llama n a a o o seno del ngulo, tenindose a e cos = sin = BC longitud del lado opuesto = . longitud de la hipotenusa AB

1.2 Elementos de trigonometr a Es util denir tambin la funcin tangente: e o tan longitud del lado opuesto sin = . longitud del lado adyacente cos

Evaluemos sin2 + cos2 . Se tiene:


2 2

cos + sin = =

AC BC + AB AB 2 + (BC)2 (AC) . (AB)2

Pero, de acuerdo al teorema de Pitgoras, (AC)2 + (BC)2 = (AB)2 , luego a cos2 + sin2 = 1 . Dos relaciones trigonomtricas importantes son: e sin( + ) = sin cos + sin cos y cos( + ) = cos cos sin sin . (1.9) (1.8)

Figura 1.2

Demostremos al menos una de ellas; la primera. Para ello consideremos la gura 1.2. Partiendo del tringulo (ABC), prolongamos el lado BC y gracamos las alturas CD y AE. a Note que el ngulo < ACE resulta ser igual a + . El rea de un tringulo es la mitad del a ) a a producto de su base por la altura. De la gura 1.2, para el rea del a (ABC), obtenemos 2 Area [ (ABC)] = BC EA = AB CD .

Expansiones y Trigonometr a

En la ultima ecuacin hemos escrito el producto base por altura del tringulo (ABC) de o a dos maneras distintas: en la primera igualdad, BC es la base y EA la altura, mientras que en la segunda, AB es la base y CD la altura. Partiendo de la ultima igualdad, dividiendo ambos lados por AC y CB, se obtiene BC EA AB CD = , BC AC AC CB o sea,

EA AC

= =

(AD + DB) CD AC BC AD CD DB CD + . AC BC BC AC

Usando las deniciones de seno y coseno, se deduce nalmente que sin( + ) = sin cos + sin cos . Como casos particulares de las ecuaciones (1.8) y (1.9), se encuentra cos(2) = cos2 sin2 y sin(2) = 2 cos sin . (1.11) (1.10)

Existen muchas identidades trigonomtricas de este tipo que resultan ser utiles para llee var adelante diferentes tipos de clculos. Dejamos que el lector demuestre las siguientes a identidades: sin sin = 2 sin cos , (1.12) 2 2 cos + cos = 2 cos + 2 + 2 cos 2 2 , (1.13)

cos cos = 2 sin

sin

(1.14)

tan 2 =

2 tan . 1 tan2

(1.15)

La denicin del seno y coseno que hemos dado es vlida para ngulos entre 0 y 90 o a a grados. Para denir estas funciones para otros ngulos es conveniente considerar un c a rculo de radio R = 1 centrado en el origen (ver gura 1.3). Por convencin, los ngulos se miden o a desde el eje x en el sentido contrario a los punteros del reloj.

1.2 Elementos de trigonometr a

Figura 1.3 Consideremos el punto A sobre el c rculo, formando un ngulo con el eje x. Usando el a hecho que la hipotenusa vale 1, es fcil convencerse de que las coordenadas x e y del punto a A coinciden con los valores de cos y sin , respectivamente. Es sta la propiedad que se usa para denir el valor del seno y coseno para cualquier ngulo e a . El procedimiento es el siguiente: i) Encontrar el punto P sobre el c rculo que forma un a ngulo con el eje x (en la gura 1.3, esto se muestra para = 210 ); ii) luego, proyectar el punto P sobre los ejes para encontrar xp e yp . Entonces cos = xp y sin = yp . Para el caso mostrado en la gura 1.3, cos(210 ) = 3/2 = 0, 8660 . . . y sin(210 ) = 1/2. Es evidente que, para todos los ngulos , siempre se cumple a 1 cos 1 y 1 sin 1 . Podemos gracar las proyecciones del punto P a medida que variamos . De esta manera se obtiene el grco de las funciones coseno y seno (ver gura 1.4). a

Figura 1.4 Recordemos que los ngulos tambin pueden ser medidos en radianes (unidad adimensional a e que se abrevia por rad ). El valor del ngulo , en radianes, es igual al largo del arco a subtendido sobre el c rculo unitario desde donde lo cruza el eje x hasta el punto A (ver

Expansiones y Trigonometr a

gura 1.3). De acuerdo a la denicin, un ngulo de 360 , o sea, la circunferencia completa, o a corresponder a un ngulo igual a 2 rad. El ngulo recto es igual a /2. No es dif a a a cil vericar que 360 1 rad = = 57, 3 . 2 Para llegar al punto P (gura 1.3) originalmente se recorri un ngulo desde el eje x o a positivo. Al continuar y dar una vuelta completa para volver al punto P , habremos recorrido desde el eje x un ngulo 2 + . Sucesivas rotaciones nos llevarn nuevamente al punto P , a a habindose recorrido ngulos 4 + , 6 + , etc. Cada vez que, desde el eje x positivo, e a recorremos un ngulo ms un mltiplo de 2, estaremos en el punto P . Se trata de un a a u movimiento que se repite y se dice que es peridico en el ngulo , con per o a odo igual a 2. Se tiene (ver gura 1.4) que, para cualquier ngulo , a cos( + n 2) = cos y sin( + n 2) = sin , donde n es un entero. Note que, cuando el ngulo se expresa en radianes, se cumplen las a siguientes relaciones: sin( ) = sin sin(/2 ) = cos cos( ) = cos cos(/2 ) = sin cos( + /2) = sin sin( + /2) = cos . Cuando el argumento (en radianes) de una funcin trigonomtrica es muy pequeo, sta o e n e puede aproximarse con una expresin simple. En efecto, consideremos el tringulo rectnguo a a lo ABC mostrado en la gura 1.5. A medida que decrece, el cateto opuesto a se hace cada vez ms parecido al arco de c a rculo s con centro en A.

Figura 1.5

1.2 Elementos de trigonometr a Usando la denicin de la funcin seno se tiene o o sin = a c s . c

Pero el cuociente s/c es precisamente el ngulo en radianes, luego, para ngulos pequeos a a n (y stos expresados en radianes) e sin Sabemos que cos2 = 1 sin2 . Luego, para ngulos pequeos a n cos2 o sea, cos Ejemplo: Evale, usando una calculadora, las funciones sin y cos para = 5 . Compare los valores u obtenidos con aqullos que resultan de usar las expresiones aproximadas escritas ms arriba. e a = 5 2/360 rad en una calculadora, obtenemos: Ingresando el valor = 5 sin 5 = 0, 0871557 y cos 5 = 0, 9961947 . Si ahora hacemos uso de las expresiones aproximadas, obtenemos sin 5 y cos 5 = 1 5 2 = 0, 087266 360 1 2 5 2 360
2

(1.16)

1 2 , 1 1 2 . 2

1 2

(1.17)

= 0, 9961923

Note que los valores aproximados dieren poco de los obtenidos con la calculadora. Para el coseno el error es inferior al 0,003 %. Cabe destacar que las funciones sin y cos pueden ser expresadas como una suma innita de trminos proporcionales a diferentes potencias del ngulo (expresado en radianes): e a cos = 1 y sin = 2 4 6 + + , 2! 4! 6! 3 5 7 + + , 3! 5! 7! (1.18)

10

Expansiones y Trigonometr a

donde n! n (n 1) (n 2) . . . 3 2 1. Para || 1, estas series convergen rpidamente, a lo que permite representar las funciones seno y coseno con pocos trminos. e Ejemplo: Representemos en un mismo grco, para el intervalo t [, 2] , las siguientes cinco a funciones: i) ii) iii) iv) v) f0 (t) = cos t f1 (t) = 1 f2 (t) = 1 t2 /2! f3 (t) = 1 t2 /2! + t4 /4! f4 (t) = 1 t2 /2! + t4 /4! t6 /6!

Observe que de acuerdo a la ecuacin (1.18), las funciones f1 (t), f2 (t), etc., para t pequeo o n son aproximaciones cada vez mejores de f0 (t) = cos t. Este comportamiento se observa claramente en la gura 1.6 (pgina siguiente) donde se han gracado las diversas funciones. a

Figura 1.6 Funciones trigonomtricas inversas e En ocasiones, lo que se conoce es x = cos y lo que se desea conocer es el ngulo . Esta a operacin inversa se denota por o = arccos(x) . Es importante darse cuenta de que esta funcin inversa, llamada arcocoseno, es una o funcin multivaluada, o sea, que la respuesta no es unica. Hay varios ngulos distintos o a para los cuales el coseno del ngulo tiene el mismo valor. Las calculadoras, al evaluar las a

1.3 Problemas

11

funciones trigonomtricas inversas, slo dan la solucin que est en el intervalo [0, ] para el e o o a arcocoseno y el intervalo [/2, +/2] para la funcin arcoseno y la funcin arcotangente. o o En ocasiones la solucin entregada por la calculadora no es la f o sicamente aceptable, en cuyo caso uno debe preocuparse de encontrar la solucin correcta (en el lenguaje tcnico: o e elegir la rama adecuada). Algo similar ocurre cuando uno extrae ra ces: puede ocurrir que la ra de 9 de inters f z e sico sea 3 y no la solucin que entrega la calculadora (que es +3). o Para la funcin arcocoseno la calculadora, al evaluar = arccos(x) con |x| 1, siempre o dar la respuesta que se ubica en el intervalo [0, ] (si est usando la calculadora en a a radianes) o en el intervalo [0, 180 ] si la calculadora est calculando en grados. a Ejercicio: Sea |x| 1 cierto valor dado y suponga que deseamos encontrar todos los a ngulos (en radianes) para los cuales cos = x. Suponga adems que hemos, de alguna a manera, encontrado una solucin = 0 (por ejemplo, el ngulo que muestra la calculadora o a al evaluar arccos(x) ). Demuestre que todas las dems soluciones a nuestro problema vienen a dadas por = 0 + j 2 y = 0 + j 2, con j cualquier valor entero. Para la funcin arcoseno la calculadora, al evaluar = arcsin(x) con |x| 1, siempre o dar la respuesta que se ubica en el intervalo [/2, /2] (si est usando la calculadora a a en radianes) o en el intervalo [90 , +90 ] si la calculadora est calculando en grados. a Ejercicio: Sea |x| 1 cierto valor dado y suponga que deseamos encontrar todos los a ngulos (en radianes) para los cuales sin = x. Suponga adems que hemos, de alguna a manera, encontrado una solucin = 0 (por ejemplo, el ngulo que muestra la calculadora o a al evaluar arccos(x) ). Demuestre que todas las dems soluciones a nuestro problema vienen a dadas por = 0 + j 2 y = ( 0 ) + j 2, con j cualquier valor entero. Por ser frecuentemente fuente de errores reiteramos lo dicho unos prrafos antes: al evaluar a funciones trigonomtricas inversas la solucin entregada por la calculadora no es siempre e o la f sicamente aceptable. El alumno debe asegurarse de que la respuesta mostrada por la calculadora efectivamente resuelve completamente su problema, en caso contrario, debe analizar si alguna de las otras soluciones, que se obtuvieron en los dos ejercicios anteriores, sirve.

1.3.
1.

Problemas
Evale las siguientes sumatorias u a) S=
n = 1, 2 m = 1, 2, 3

nm

b)

S=
j=3,...,8

12

Expansiones y Trigonometr a

c)

S=
j=0

d)

S=
i, j = 1, . . . , 4 i>j

1 |i j|

Respuestas: a) 17 , b) 12 , c) N (N + 1)/2 , d) 13/3 2. Encuentre una expresin para [ (x + ) x ]/, en el l o mite en que tiende a cero. En otras palabras, tiene un valor nito pero peque nsimo (tan pequeo como n se quiera); al nal del clculo se permite poner = 0. a Usando una notacin y un lenguaje ms tcnico, el enunciado de este problema ser o a e a: Evale u f (x) = l m
0

1 [ (x + ) x ] .

Respuesta: f (x) = x1 . cos(x + ) cos x sin x.

3.

Evale u

para ||

1.

Respuesta:

4.

Represente en forma cuidadosa, en un mismo grco, para el intervalo t [1, 1] , a las siguientes cuatro funciones: a) b) c) d) f0 (t) = 1/(1 t) f1 (t) = 1 + t f2 (t) = 1 + t + t2 f3 (t) = 1 + t + t2 + t3

Observe que, de acuerdo a la ecuacin (1.3), f1 (t), f2 (t) y f3 (t) son sucesivamente o aproximaciones cada vez mejores (para t pequeo) de la funcin f0 (t). n o

5.

Demuestre las siguientes relaciones trigonomtricas: e (a) sin = tan 1 + tan2

1.3 Problemas

13

(b)

tan( + ) =

tan + tan 1 tan tan + 2 cos 2 .

(c)

sin + sin = 2 sin

6.

Sea r el radio del c rculo circunscrito de un pentgono regular (ver gura 1.7). a a) b) c) Cunto mide el ngulo interior (en radianes)? a a Determine el largo del lado s en funcin de r. o Determine el rea del pentgono. a a

Figura 1.7 Respuestas: a) = 3/5 radianes ; 7. c) rea = a


5 2

Figura 1.8 r2 sin(2/5).

Una camionada de arena seca se descarga formando un cono de 4 metros de dimetro. a 3 y el el ngulo del cono (ver gura 1.8) Si la densidad de la arena seca es =1.7 g/cm a es de = 32 , calcule la masa de la arena (en toneladas). Encuentre todos los valores de x en el intervalo [5, +5] (cuando no se especica nada se asume que las unidades son radianes) para los cuales se cumple la relacin o sin x tan x = Respuesta: x = 4/3 , 2/3 , 2/3 , 4/3 . 3 . 2

8.

9.

Represente en un mismo grco, para t en el intervalo [, 2] , las siguientes cuatro a funciones: a) b) f0 (t) = sin t f1 (t) = t

14 c) d) f2 (t) = t t3 /3! f3 (t) = t t3 /3! + t5 /5!

Expansiones y Trigonometr a

Aqu nuevamente f1 (t), f2 (t) y f3 (t) son sucesivamente aproximaciones cada vez mejores (para t pequeo) de la funcin f0 (t). n o 10. Al incidir luz sobre una interfase, por ejemplo, al pasar del aire al vidrio o viceversa, sta generalmente sufre un cambio de direccin (ver gura 1.9). Este fenmeno se e o o conoce con el nombre de refraccin de la luz. La ecuacin que describe este fenmeno o o o es la Ley de Snell: sin v = aire , sin vvidrio donde vaire y vvidrio corresponden a la velocidad de la luz en el aire y el vidrio, respectivamente. (Para el vidrio comn se tiene vaire /vvidrio 1,5 .) u

Figura 1.9 a) Supongamos que un haz de luz incide sobre un vidrio de 2 cm de espesor, con un ngulo de incidencia = 40 . Encuentre la distancia d por la cual el haz de a luz emergente se encontrar paralelamente desplazado respecto al haz incidente a (ver gura 1.10). Considere ahora un haz de luz incidiendo sobre un prisma en la forma que se muestra en la gura 1.11. Encuentre el ngulo para = 20 , 40 , 50 y a 70 . Para qu ngulo = 0 se obtiene = 90 ? Para > 0 el haz de luz es ea reejado especularmente (como si fuese un espejo) por la supercie interior del prisma, fenmeno conocido con el nombre de reexin total. o o

b)

Figura 1.10

Figura 1.11

1.3 Problemas 11.

15

La gura 1.12 adjunta indica la diferencia entre un d sideral y un d solar. Para a a facilitar la explicacin supongamos que es posible observar las estrellas durante el o d (Por supuesto que las estrellas estn all y de hecho los radioastrnomos observan a. a o algunas de ellas.) Para un observador en el Ecuador, el d solar es el per a odo que transcurre entre dos pasos consecutivos del sol por el zenit (posicin del sol justo sobre nuestras cabezas). o El d sideral consiste en el mismo fenmeno pero que ahora ocurre con una estrella a o muy lejana. La diferencia entre ambas deniciones se debe a la traslacin de la tierra o alrededor del sol. Determine el valor del ngulo que se muestra en la gura y calcule a la diferencia entre el d sideral y el d solar en segundos. a a

Figura 1.12

Figura 1.13

12.

Un tambor de 50 cm de radio y 1.5 m de largo se encuentra acostado y lleno con parana hasta una altura h =60 cm (ver gura 1.13). Cuntos litros de parana hay a en el tambor?

13.

La esfericidad de la tierra fue postulada por Pitgoras y conrmada por Aristteles a o al observar la forma circular de la sombra que proyecta la tierra en la supercie de la luna durante un eclipse lunar. El primer clculo que se conoce del radio de la tierra se debe a Eratstenes (276 A.C. a o 194 A.C.), quien a la fecha estaba a cargo del Museo de Alejandr El mtodo que a. e us se bas en observar el ngulo con que inciden los rayos solares sobre la supercie o o a de la tierra, el mismo d y a la misma hora, en dos lugares separados entre s por a una gran distancia. Los lugares elegidos fueron Siena (S) (hoy Asun) y Alejandr a a (A).

16

Expansiones y Trigonometr a

Figura 1.14 Eratstenes sab que al mediod del 22 de junio el Sol ca verticalmente en Siena, o a a a pues la luz se reejaba directamente en el fondo de una noria. El mismo d a la misma a, hora, midi la sombra que proyectaba en Alejandr un alto obelisco, que le indic que o a o los rayos solares formaban un ngulo de 7,2 con la vertical (ver gura 1.14). a Dado que el sol est a gran distancia de la tierra se puede suponer que los rayos que a llegan a ambas ciudades son paralelos. Eso quiere decir que la separacin angular entre o Siena y Alejandr medida con respecto al centro de la tierra es tambin 7,2 . Sabiendo a e que la distancia entre Siena y Alejandr (arco de c a rculo) es de aproximadamente 800 km, estime el radio de la tierra. Respuesta: Radio 6100 km. (El resultado que obtuvo Eratstenes en su poca fue o e incorrecto, debido a la imprecisin con que estim la distancia entre los dos lugares.) o o 14. Una persona ubicada en el punto P observa dos montaas que la rodean, una a la n derecha y la otra a la izquierda. Sean y los ngulos de elevacin, respectivamente a o (ver gura 1.15). Si la montaa de la izquierda tiene una altura h y la separacin entre n o las proyecciones de las cimas sobre el nivel de la supercie terrestre es D, calcule la altura del otro monte.

Figura 1.15 15. En el ao 1752 los astrnomos Landale y Lacaille determinaron en Berl (B) y en la n o n ciudad del Cabo (C), a la misma hora, el ngulo entre la normal y la recta entre su a

1.3 Problemas

17

posicin y un punto predeterminado del borde de la luna. Los ngulos que determinao a ron fueron = 32,08 en Berl y = 55,72 en El Cabo. Ambas ciudades se ubican n en el mismo meridiano y se encuentran en las latidudes B = 52,52 y C = 33,93 , respectivamente (ver gura 1.16). Usando para el radio terrestre el valor de 6370 km, determine la distancia entre la tierra y la luna.

Figura 1.16 16. 17. Encuentre el ngulo entre dos diagonales de un cubo. a a) Teorema del seno. Demuestre que en un tringulo cualquiera se cumplen las a siguientes relaciones: a b c = = , sin sin sin donde , y son los ngulos interiores del tringulo y a, b y c los lados opuestos a a a cada uno de estos ngulos. a b) Teorema del coseno. Demuestre que en un tringulo cualquiera se cumplen las a siguientes relaciones:

c2 = a2 + b2 2ab cos b2 = a2 + c2 2ac cos y a2 = b2 + c2 2cb cos

18 18. Determine el largo m nimo que debe tener una cadena para unir dos poleas de radios R y r, separadas por una distancia D (ver gura 1.17).

Expansiones y Trigonometr a

Respuesta: Rr D

Figura 1.17

L = 2 (R r) arcsin

+2

D2 (R r)2 + (r + R) .

19.

Un tetraedro regular es la gura geomtrica que se obtiene al formar una pirmide con e a cuatro tringulos equilteros idnticos. Encuentre el ngulo entre dos de sus caras. a a e a

20.

La altura de un edicio se puede determinar midiendo su ngulo de elevacin y la a o distancia a la que uno se encuentra del edicio. Suponga que el instrumento que tiene a disposicin le permite medir ngulos con un error de 1 . Determine el menor error o a porcentual con que, con tal instrumento, usted puede medir la altura de un edicio.

21.

Dos observadores A y B miden a ngulos de elevacin de un avin que o o los sobrevuela a una altura constante. En cierto instante los ngulos a medidos por A y B son = 60 y = 40 , respectivamente. Diez segundos ms tarde, A mide un ngua a (ver gulo de elevacin = 110 o ra 1.18). La separacin entre A y B o es D = 1 km. A qu altura vuela el e avin? Cul es su velocidad? o a

Figura 1.18

22.

Graque, usando un computador, la funcin f (t) = cos(t) + cos(0, 9t) para t o [0, 40] y observe el fenmeno de pulsaciones. o

23.

Para qu latitud el paralelo terrestre tiene 1/3 de la longitud del Ecuador? e

1.3 Problemas 24. Una cuneta de forma angular est caracterizada por los ngulos a a y respecto a la horizontal. Una bola de acero de radio R posa sobre la cuneta, ver gura 1.19. Determine el nivel m nimo de agua, medido desde el punto ms bajo de la cuneta, a necesario para cubrir la bola completamente.

19

Figura 1.19

25.

Son las 12 del d Determine en cunto rato ms se vuelven a juntar los punteros del a. a a reloj. a) Calcule la razn entre las reas del c o a rculo y del tringulo equiltero que lo cira a cunscribe (ver gura 1.20a). b) Haga el mismo clculo anterior pero para el caso en que el tringulo contenga a a n(n + 1)/2 discos de radio R dispuestos como se muestra en la gura 1.20b.

26.

Figura 1.20a 27.

Figura 1.20b

Usted se plantea tener un atardecer de 24 horas de duracin en el Ecuador, para lo cual o cuenta con un aeroplano. Calcule la velocidad con que deber volar y la direccin a o que debe tomar para lograr su propsito. Si un amigo viaja a la misma velocidad o relativa a la tierra, pero en sentido opuesto, calcule el tiempo que transcurrir hasta a encontrarse nuevamente con l. e Hay que decidir el tipo de empaque que se le va a dar a pelotas de tenis en una bandeja de forma cuadrada. Decida cual de las dos conguraciones mostradas en la gura 21 resulta ms conveniente. Justique su respuesta cuantitativamente. a

28.

20

Expansiones y Trigonometr a

Figura 1.21a

Figura 1.21b

1.4.

Solucin a algunos de los problemas o

Solucin al problema 15 o

Figura 1.22 Inspeccionando la gura 1.22 se deduce de inmediato que = + y = + B |C | . Usando el teorema del seno (ver problema 17) en los tringulos OBL y OLC, se obtienen a las expresiones sin sin( ) = R D

1.4 Solucin a algunos de los problemas o y sin sin( ) = . R D Como y son ngulos pequeos podemos usar las aproximaciones a n sin y sin De esta manera se obtienen y Sumando estas ecuaciones se deduce que = + o sea, R (sin + sin ) R (sin + sin ) = . + B |C | Sustituyendo en esta ecuacin los valores numricos se encuentra que o e D D 367,000 km , R (sin + sin ) , D R sin . D .

21

R sin D

valor muy cercano al actualmente aceptado para el radio de la rbita lunar, que es de o 384.000 km. Solucin al problema 16 o Consideremos un cubo de lados a. Sea A un vrtice de una diagonal y B el vrtice de otra e e diagonal del cubo. De los dos vtices de la segunda diagonal, denotaremos por B vrtice e al e que est a una distancia a de A (el otro vrtice se encontrar a una distancia a 2 de A). a e a Sea O el punto central del cubo. El tringulo AOB es issceles: con base a o AB = a y lados b AO = BO = 23 a. El a ngulo =< (AOB) es el ngulo buscado. ) a Se tiene que sin a/2 1 = = , 2 b 3

de donde se deduce que = 70,529 . El ngulo complementario < (AOC) = a ) . 109,47 Figura 1.23

22 Solucin al problema 21 o

Expansiones y Trigonometr a

Sea a = AP y d = P Q. Usando el teorema del seno en el tringulo AP B se obtiene a sin sin ( ) = , a D o sea, a=D sin . sin( )

Usando el teorema del seno en el tringulo AQP se deduce que a sin( ) sin( ) = . a d Usando las dos ecuaciones anteriores se obtiene para d la expresin o d=D sin sin( ) . sin( ) sin

Reemplazando los valores numricos se encuentra que la distancia recorrida por el avin en e o 10 segundos es d = 1, 53 km. La velocidad del avin es, por lo tanto, v = 552 km/h. La o altura a la que vuela el avin viene dada por o h = a sin = 1628 [m] .

Figura 1.24

Solucin al problema 24 o Primero giremos la cuneta de manera que quede simtrica respecto a la horizontal, es decir, e con un ngulo ( + )/2 a cada lado (ver gura 25a). a

1.4 Solucin a algunos de los problemas o

23

Figura 1.25a El ngulo < a )ABC tambin es ( + )/2, luego e AB = R cos + 2 .

Figura 1.25b

Para volver a poner la cuneta en la orientacin original debemos girarla en un ngulo o a ( )/2. Por lo tanto, (ver gura 1.25b)
2

BD = AB cos

=R

cos 2 cos + 2

Para la altura del nivel de agua se obtiene nalmente la expresin o h = R 1 + cos 2 cos + 2 .

24

Expansiones y Trigonometr a

Cap tulo 2

Cinemtica en una dimensin a o


2.1. Posicin, velocidad y aceleracin o o

Cinemtica es la descripcin del movimiento de un cuerpo sin considerar las causas que lo a o producen. Ms tarde, al estudiar las leyes de Newton, analizaremos el origen del movimiento. a Para simplicar la discusin, comenzaremos por estudiar el movimiento de objetos cuya o ubicacin queda determinada especicando la posicin de un solo punto. Este tipo de objeto o o recibe el nombre de part cula. Contrariamente a lo que pudiera pensarse, no es necesario que los objetos sean pequeos para que puedan ser considerados part n culas. Por ejemplo, cuando se estudia el movimiento de la tierra en torno al sol, la distancia relevante es la distancia Tierrasol. En este caso, el tamao de la Tierra no es importante, pudindose n e tratar como una part cula ubicada en el centro de la tierra. El movimiento ms simple de una part a cula se tiene cuando la posicin de sta viene descrita o e por una unica coordenada; por ejemplo, el movimiento de una part cula que se traslada a lo largo de una l nea recta. (En el presente cap tulo nos restringiremos a este tipo de movimientos.) La eleccin de un origen divide naturalmente a la recta en dos zonas. En o forma arbitraria llamamos a una de ellas el lado positivo y a la otra el lado negativo (ver gura 2.1).

Figura 2.1 La posicin de una part o cula queda determinada dando simplemente un nmero (la cooru denada x). La descripcin de su movimiento es completa si conocemos la funcin x(t) que o o indica la posicin que ocupa en cada instante t. o La diferencia entre la coordenada de una part cula entre dos instantes t1 y t2 (con t2 > t1 ) se denomina desplazamiento:
Desplazamiento x2 x1 x .

26

Cinemtica en una dimensin a o

El desplazamiento es una cantidad que tiene signo. Si la coordenada x de la part cula se incrementa durante cierto intervalo de tiempo, entonces el desplazamiento es positivo; si, por el contrario, decrece, el desplazamiento es negativo. Se dene velocidad media de una part cula durante el intervalo [t1 , t2 ] como la razn entre o el desplazamiento y la duracin del intervalo de tiempo, o v(t1 , t2 ) = x(t2 ) x(t1 ) . t2 t1

En un grco x(t) en funcin de t, esta denicin corresponde a la tangente del ngulo que a o o a forma la recta que une (x1 , t1 ) y (x2 , t2 ) con el eje del tiempo (ver gura 2.2).

Figura 2.2 La velocidad promedio entrega una informacin global sobre el movimiento que realiza una o part cula en un cierto intervalo de tiempo. Si se desea tener una informacin ms precisa o a acerca de la velocidad durante el movimiento, es necesario subdividir el intervalo de tiempo original en subintervalos y calcular en cada uno de ellos una velocidad media. Mientras ms pequeo es el tamao de esos subintervalos, ms precisa es la informacin acerca de a n n a o las variaciones que experimenta la velocidad de la part cula mientras se desplaza. El valor que se mide para la velocidad media en un cierto intervalo de tiempo pequeo, donde n es nito pero tan pequeo como nosotros deseamos, se denomina velocidad instantnea. n a Para determinar la velocidad instantnea de la part a cula en un instante t, se evala la u velocidad promedio durante un intervalo muy pequeo que comienza en t y termina en n t + , donde es un incremento de tiempo innitesimal (ms adelante, al nalizar el clculo, a a haremos 0). Expl citamente: v(t, t + ) = x(t + ) x(t) .

Al hacer 0, se obtiene la velocidad instantnea de la part a cula en el instante t. Esta la denotaremos por v(t) o x(t). Se tiene v(t) = l m x(t + ) x(t) = x(t) . (2.1)

2.1 Posicin, velocidad y aceleracin o o

27

Este proceso de l mite est ilustrado en la Figura 2.3. All se observa cmo cambia el valor a o de la velocidad media de la part cula en un intervalo [t, t + t] cuando es evaluada para diferentes valores de t. En el caso l mite, cuando 0, se observa que la velocidad instantnea queda representada por la tangente del ngulo (pendiente) que forma la recta a a tangente a la curva x(t) vs. t con el eje del tiempo. De aqu en adelante el trmino velocidad siempre se referir a la velocidad instantnea. e a a

Figura 2.3

Ejemplos: 1. Supongamos que la posicin de una part o cula viene dada por x(t) = x0 + v0 t, con m . El grco x(t) en funcin de t da lugar a la recta que se x0 = 1 m y v0 = 0,5 s a o muestra en la gura 2.4. Esa curva corresponde a una part cula que se mueve con velocidad uniforme. La inclinacin de la recta con respecto al eje del tiempo es una medida de la velocidad de o la part cula. Una recta horizontal corresponde a una part cula en reposo mientras que una recta perpendicular al eje del tiempo representa un objeto que tiene velocidad innita. Evaluemos expl citamente la velocidad en un instante t cualquiera. Usando la ecuacin o (2.1) y la expresin para x(t) de este ejercicio, se obtiene o x(t + ) x(t) [x0 + v0 (t + )] [x0 + v0 t] = l m 0 0 v0 = l m = l v0 = v0 . m 0 0

v(t) = l m

28

Cinemtica en una dimensin a o

Figura 2.4 Este resultado indica que la expresin para x(t) escrita ms arriba efectivamente o a corresponde al movimiento de una part cula con velocidad constante v0 (i.e., independiente del tiempo). 2. Supongamos ahora que la posicin de una part o cula viene dada por z(t) = z0 1 2 gt , 2

con z0 = 10 m y g = 9,81 m . Al gracar la posicin en funcin del tiempo se encuentra o o s2 la curva (parbola) mostrada en la gura 2.5. a Evaluemos la velocidad en un instante t cualquiera. Usando la ecuacin (2.1), se o obtiene [z0 1 g (t + )2 ] [z0 1 g t2 ] z(t + ) z(t) 2 2 = l m 0 0 1 g (2t + ) g (2t + ) 2 = l m = g t . = l m 0 0 2

v(t) = l m

La gura 2.6 muestra el grco de la velocidad instantnea en funcin del tiempo. a a o Se observa que sta decrece linealmente a medida que transcurre el tiempo. El signo e negativo de la velocidad signica que la part cula se est desplazando en el sentido a negativo del eje z. Sin embargo, el mdulo de la velocidad de la part o cula (magnitud que en algunos textos es denominada rapidez) aumenta a medida que transcurre el tiempo: |v(t)| = g t . El movimiento descrito por la funcin z(t) de este ejemplo corresponde a la ca o da libre de una part cula en el campo gravitacional terrestre y desde una altura z0 .

2.1 Posicin, velocidad y aceleracin o o

29

Figura 2.5

Figura 2.6

Si la velocidad de una part cula cambia a medida que transcurre el tiempo, entonces la part cula tiene una aceleracin. o La aceleracin media (o promedio) que tiene la part o cula durante el intervalo [t1 , t2 ] es igual al cambio de velocidad que ocurre durante el intervalo, dividido por la duracin de ste, es o e decir v(t2 ) v(t1 ) a(t1 , t2 ) = . t2 t1 Para determinar en un instante t la aceleracin instantnea de la part o a cula, evaluamos la aceleracin promedio durante un intervalo muy pequeo que comienza en t. Sea [t, t + ] ese o n intervalo, donde es un tiempo innitesimal (de hecho, al nalizar el clculo nuevamente a tomaremos 0). Entonces a(t, t + ) = v(t + ) v(t) .

Al hacer 0 se obtiene la aceleracin instantnea de la part o a cula (en el instante t). Esta la denotaremos con a(t), x(t) o v(t). Se obtiene a(t) = l m v(t + ) v(t) = x(t) = v(t) . (2.2)

De aqu en adelante el trmino aceleracin siempre se referir a la aceleracin instantnea. e o a o a Ejemplos: 1. Para el movimiento rectil neo uniforme, la posicin de una part o cula viene dada por x(t) = x0 + v0 t. Ya hemos visto que, en ese caso, su velocidad es constante e igual a v0 . Demostremos ahora, usando la ecuacin (2.2), que en este caso la part o cula efectivamente no tiene aceleracin. De hecho, o a(t) = l m v(t + ) v(t) v0 v0 = l m = l 0 = 0 . m 0 0

30 2.

Cinemtica en una dimensin a o En un ejemplo anterior vimos que la posicin y velocidad de una part o cula que cae libremente bajo la accin de la aceleracin de gravedad terrestre estn dadas por las o o a siguientes ecuaciones 1 2 gt 2

z(t) = z0 y

v(t) = g t . Evaluemos la aceleracin: o a(t) = l m [g (t + )] (g t)] v(t + ) v(t) = l m 0 0 g = l m = l (g) = g . m 0 0

El resultado indica que la aceleracin es constante y negativa. Eso signica que la o part cula acelera en el sentido negativo del eje z. Generalizando, podemos concluir que cuando el grco v(t) en funcin del tiempo t es a o una recta, el movimiento de la part cula corresponde a un movimiento uniformemente acelerado. El caso particular en que la recta es horizontal corresponder a la situacin a o donde la aceleracin es nula. o En el grco x(t) en funcin de t, las aceleraciones se maniestan en la curvatura del a o grco. Se dice que un grco tiene curvatura positiva, si sta tiene la misma oriena a e tacin que la curvatura de un pocillo, y negativa si la curvatura tiene la orientacin o o de la de un paraguas. Si en un grco x(t) vs. t la curvatura es positiva dentro de un cierto intervalo, a entonces tambin lo ser la aceleracin en ese intervalo. Por ejemplo, en la gura 2.5 e a o (que corresponde a la ca libre) la curvatura es negativa, luego tambin lo ser la da e a aceleracin. o 3. Consideremos una part cula de masa m, cuya posicin a medida que transcurre el o tiempo viene dada por z(t) = A cos(t) , donde A y son constantes. Tal movimiento de la part cula es un movimiento oscilatorio peridico. La amplitud de las oscilaciones es A y el per o odo del movimiento (es decir, el tiempo que debe transcurrir hasta que una conguracin se vuelva a repetir) o es T = 2/ . Al inverso de T se le llama frecuencia: = 1/T . A la magnitud se le llama frecuencia angular. Se tiene que = 2.

2.1 Posicin, velocidad y aceleracin o o Evaluemos la velocidad de la part cula: v(t) = l m = = z(t + ) z(t) 0 1 l m [A cos((t + )) A cos(t)] 0 A l m [cos(t) cos() sin(t) sin() cos(t)] 0 A 2 2 l m cos(t) 1 sin(t) () cos(t) 0 2 2 2 A cos(t) sin(t) () l m 0 2 2 l A cos(t) m sin(t) 0 2 A sin(t)

31

= = =

Una vez conocida la velocidad podemos, en forma anloga, calcular la aceleracin: a o a(t) = l m v(t + ) v(t) 0 1 = l m [A sin((t + )) (A) sin(t)] 0 A [sin(t) cos() + cos(t) sin() sin(t)] = l m 0 2 2 A sin(t) 1 + cos(t) sin(t) l m 0 2 2 = l A sin(t) m + cos(t) 0 2 = A 2 cos(t) La gura 2.7 muestra la posicin, velocidad y aceleracin de la part o o cula en funcin o del tiempo.

32 Figura 2.7

Cinemtica en una dimensin a o

Notemos que para todo t, a(t) = 2 z(t). El lector ya familiarizado con la ecuaciones de Newton (que analizaremos recin en el cap e tulo 4) puede establecer una interesante relacin con la Ley de Hooke. En efecto, al hacer uso de la ecuacin de Newton o o F = m a, se encuentra que la fuerza neta que acta sobre la part u cula de masa m debe satisfacer la relacin o F = (m 2 ) z . Denotando a la constante (m 2 ) por k, se tiene F = kz. Esto nos muestra que la fuerza neta sobre la part cula es proporcional al desplazamiento. El signo negativo indica que la direccin en que acta la fuerza es opuesta al desplazamiento. Un ejemplo o u concreto en que aparece una fuerza del tipo F = kz es una masa m colgando de un resorte. En ese caso k es la constante del resorte y a F = kz se le llama Ley de Hooke.

4.

Una persona levanta un peso P , sujetando una cuerda que pasa por una polea y caminando horizontalmente con velocidad v0 . Cul es la a velocidad del peso P ? Supongamos que el largo de la cuerda es 2h (o sea, cuando la persona est en x = 0, el cuerpo P est en a a el suelo encontrndose la cuerda esa tirada). Se tiene Figura 2.8

(h y) + o sea, y(t) =

h2 + x2 = 2h ,

h2 + x2 (t) h =

2 h2 + v0 t2 h .

Para la velocidad obtenemos

2.2 El camino inverso

33

y(t) = v(t) = l m

y(t + ) y(t) 0 1 2 2 = l m h2 + v0 (t + )2 h h2 + v0 t2 h 0 1 2 2 2 2 = l m (h2 + v0 t2 ) + (2v0 t + v0 2 ) h2 + v0 t2 0 = l m 1 0


2 h2 + v0 t2

1+ 1+

2 2 2v0 t + v0 2 2 t2 1 h2 + v0

= l m

1 0

2 h2 + v0 t2

2 2 1 2v0 t + v0 2 1 2 2 h2 + v0 t2

= l m =

2 2 1 1 2v0 t + v0 2 2 0 2 h2 + v0 t2 2 v0 t 2 h2 + v0 t2

Ejercicio: Demuestre que la aceleracin de P viene dada por: o

2 a(t) = y (t) = v0

h2
2 h2 + v0 t2 3/2

2.2.

El camino inverso

En la seccin anterior se present el procedimiento que permite evaluar, partiendo del o o conocimiento de la posicin en funcin del tiempo, la velocidad y luego la aceleracin. En o o o esta seccin analizaremos el camino inverso, es decir, conociendo la aceleracin en funcin o o o del tiempo, calcular la velocidad y posicin. o Suponga que la velocidad de una part cula en funcin del tiempo viene dada por el grco o a mostrado en la gura 2.9.

34

Cinemtica en una dimensin a o

Figura 2.9 Cul ser la distancia recorrida por la part a a cula entre los instantes ti y tf ? Entre esos dos instantes la velocidad de la part cula es constante (igual a v0 ), por lo tanto la distancia recorrida ser x(tf ) x(ti ) = v0 (tf ti ). Podemos escribir a x(tf ) = x(ti ) + v0 (tf ti ) , o sea, si una part cula entre dos instantes (inicial y nal) se mueve a una velocidad constante, entonces la posicin nal es igual a la posicin inicial ms el rea de la funcin v(t) entre o o a a o los instantes ti y tf . Cuando la funcin v(t) no es constante la situacin es ms compleja. Intentemos evaluar o o a la distancia que recorre la part cula entre los instantes t1 y t4 . Como la velocidad no es constante, tomaremos algunas mediciones intermedias, separadas por un intervalo de tiempo t. Entre t1 y t2 la distancia recorrida ser aproximadamente v(t1 ) (t2 t1 ) = v(t1 ) t, a entre t2 y t3 ser v(t2 )(t3 t2 ) = v(t2 )t, y nalmente entre t3 y t4 ser aproximadamente a a v(t3 ) (t4 t3 ) = v(t3 ) t. La distancia total recorrida ser aproximadamente a
3

x(t4 ) x(t1 )
j=1

v(tj ) t ,

(2.3)

donde t = (t4 t1 )/3. Observe que el lado derecho de la ecuacin (2.3) es igual al o a rea de los rectngulos mostrados en la gura 2.10. Evidentemente el resultado anterior es a slo aproximado: hemos tomado 3 mediciones intermedias y hemos supuesto que entre las o mediciones la velocidad es constante (igual al valor de la ultima medicin). Tambin es claro o e que si aumentamos el nmero de mediciones intermedias obtendremos un resultado ms u a preciso. Para un nmero muy grande (innito) de mediciones intermedias, el procedimiento u ser exacto; en ese caso el rea de los rectngulos ser igual al rea entre la funcin v(t) a a a a a o y el eje . De esta manera hemos encontrado un resultado completamente general: t x(tf ) = x(ti ) + (Area entre v(t) y el eje t entre t = ti y tf ) . (2.4)

Otra manera de proceder es la siguiente: dividir el intervalo [ti , tf ] en much simos (innitos) intervalos de ancho dt. Entonces v(t)(dt) es igual a la distancia recorrida entre los instantes

2.2 El camino inverso

35

t y t + dt. Para obtener la distancia recorrida entre ti y tf , habr que sumar todas las a contribuciones. Se tiene entonces que
tf

x(tf ) = x(ti ) +
ti t

v(t) dt .

(2.5)

a a mbolo desde t = ti El s mbolo tif signica sume las contribuciones que estn detrs del s hasta t = tf . Por supuesto que
tf ti

v(t) dt = (Area delimitada por v(t) y el eje t entre t = ti y tf ) .

Ejemplos: 1. Movimiento uniforme: Consideremos una part cula cuya velocidad es constante v(t) = v0 en todo instante. Si la part cula en el instante t = 0 se encuentra en xi , dnde se encontrar en el o a instante t? Usando la ecuacin (2.4) se obtiene o
x(t) = x(0) + Area entre v0 y el eje t, entre t = 0 y t .

= x(0) + v0 t 2. Movimiento uniformemente acelerado: Consideremos una part cula cuya velocidad viene dada por v(t) = v0 + a0 t , (ver gura 2.10). Observe que v0 es la velocidad de la part cula en el instante t = 0. Al calcular la aceleracin se encuentra que o a(t) = l m v(t + ) v(t) = a0 ,

o sea, la expresin para la velocidad corresponde a una part o cula que en todo instante sufre una aceleracin constante a0 . o Encontremos el desplazamiento entre los instantes t = 0 y el instante t = tf . Usando la ecuacin (2.4) se obtiene o
x(tf ) = x(0) + Area entre v(t) y el eje t, entre t = 0 y t = tf 1 = x(0) + v0 tf + (v(tf ) v0 ) tf 2 1 = x(0) + v0 tf + a0 t2 . f 2

36

Cinemtica en una dimensin a o

Figura 2.10 Conociendo la posicin x(t) de una part o cula, siempre es posible determinar su velocidad. El rec proco no es cierto: si se conoce la velocidad v(t) no es posible determinar la posicin; o lo unico que se puede determinar es el desplazamiento entre dos instantes. En otras pala bras, si conocemos v(t), debemos conocer adems la posicin en algn instante para poder a o u determinar x(t). Las relaciones que permiten obtener la velocidad si se conoce la aceleracin a(t), son anloo a gas a las que relacionan la posicin con la velocidad: o
v(tf ) = v(ti ) + Area entre a(t) y el eje t entre t = ti y tf .

(2.6)

o v(tf ) = v(ti ) +

tf

a(t) dt .
ti

(2.7)

Ejemplo: Movimiento uniformemente acelerado. Suponga que la aceleracin de una part o cula es constante (a(t) = a0 , t). Usando (2.6) se deduce que v(t) = v(0) + a0 t . Haciendo uso del resultado obtenido en el ejemplo anterior se obtiene nalmente que 1 x(t) = x(0) + v(0) t + a0 t2 . 2 Observe que x(0) y v(0) son la posicin y la velocidad de la part o cula en el instante t = 0.

2.3.

Mximos y m a nimos

Considere una funcin f (t) suave (o sea, sin saltos ni puntas). Ya sabemos (ver ultimo o (t) est relacionado con la pendiente de las tangentes problema de la seccin anterior) que f o a

2.4 Problemas

37

de la funcin f (t). Observemos que para valores de t en los cuales f(t) = 0, la funcin f (t) o o tiene un mximo o m a nimo (local). Tambin podemos invertir la argumentacin: encontrar e o los mximos y m a nimos de una funcin f (z) es equivalente a encontrar los ceros de la funcin o o derivada f (z + ) f (z) g(z) = l m . 0

Ejemplo: Suponga que un agricultor tiene L metros de malla para construir un corral rectangular. El agricultor desea aprovechar una muralla de piedra (recta) para obtener un corral mayor. Qu dimensioe nes deber tener el corral para que su rea a a sea mxima? a Figura 2.11 Solucin: Sean a y b los largos del gallinero (ver gura 2.11). El largo de la malla es o L = 2a + b, mientras que el rea del gallinero es A = a b. Despejando b de la primera a ecuacin y sustituyndolo en la segunda se obtiene: o e A = a (L 2a) . El rea es una funcin de a. Tanto para a = 0 como para a = L/2 se tiene que A = 0. a o Para algn valor intermedio el rea del gallinero ser mxima. Para resolver el problema u a a a debemos encontrar el mximo de la funcin f (a) = a (L 2a). Para ello encontremos los a o ceros de la funcin derivada o g(a) = l m
0

1 f (a + ) f (a) = l m [(a + ) (L 2(a + )) a (L 2a)] = L 4a . 0

La funcin g(a) tiene un (nico) cero para a = L/4. Luego para ese valor de a el rea del o u a gallinero ser mxima. a a

2.4.
1.

Problemas
Suponga que la altura de cierto proyectil en funcin del tiempo viene dada por la o relacin z(t) = a0 (t t0 )2 + z0 , con z0 = 125 m, t0 = 5 s y a0 = 5 m/s2 . o a) b) c) Graque la altura del proyectil en funcin del tiempo desde t = 0 hasta t = 12 s. o En qu instante choca el proyectil contra el suelo? e Encuentre grcamente la velocidad instantnea (es decir, mida las pendientes a a de las tangentes) en los instantes t=0 s, t=2 s, t=4 s, t=6 s, t=8 s y t=10 s. Graque su resultado.

2.

Un conductor maneja su coche 10 km a una velocidad de 90 km/h y luego otros 10 km a 70 km/h. Cul es la rapidez promedio durante el trayecto de 20 km? a (La respuesta no es 80 km/h.)

38 3.

Cinemtica en una dimensin a o La gura 2.12 muestra la posicin de una part o cula en funcin del tiempo. Encuentre o la velocidad promedio durante los siguientes intervalos de tiempo: a) b) c) d) 0s < t <4s 7 s < t < 10 s 0 s < t < 13 s 10 s < t < 13 s (Respuesta: v = 0,154 m/s )

Figura 2.12

4.

La gura 2.13 muestra la posicin de una part o cula en funcin del tiempo. En qu inso e tantes o en qu intervalos de tiempo e a) b) c) d) e) f) la velocidad (instantnea) es cero? a la velocidad es positiva? la velocidad es negativa? el mdulo de la velocidad es mximo? o a la velocidad es constante? la aceleracin es negativa? o

2.4 Problemas

39

Figura 2.13

5.

Suponga que la posicin de una part o cula en funcin del tiempo (medido en segundos) o viene dada por t [m] z(t) = 1 + t2 a) b) Graque z(t) en el intervalo de tiempo 4 s < t < +4 s. Encuentre la velocidad instantnea en funcin del tiempo evaluando a o v(t) = l m Graque v(t). z(t + t) z(t) . t0 t

6.

La gura 2.14 muestra la posicin de una part o cula en funcin del tiempo. o a) b) c) d) e) Encuentre la velocidad promedio en el intervalo de tiempo 2 s < t < 10 s. Encuentre la velocidad instantnea para t = 10 s. a En qu instante o instantes la velocidad (instantnea) de la part e a cula es nula? En qu instante la rapidez es mxima? e a En qu instante la aceleracin es nula? e o

40

Cinemtica en una dimensin a o

Figura 2.14

7.

Suponga que la posicin de una part o cula en funcin del tiempo (medido en segundos) o viene dada por z(t) = t 4 cos t [m] a) b) Graque z(t) en el intervalo de tiempo 0 < t < +6 s. A partir del grco responda las siguientes preguntas: a 1) 2) 3) 4) c) En qu instante la velocidad es nula? e En qu instantes la part e cula se encuentra en el origen? En qu intervalos de tiempo la velocidad es negativa? e En qu intervalos de tiempo la aceleracin es positiva? e o

Encuentre la velocidad instantnea en funcin del tiempo evaluando a o v(t) = l m z(t + t) z(t) . t

t0

d)

Graque v(t) encontrada en la parte anterior. A partir del grco responda las a siguientes preguntas: 1) 2) 3) En qu instante la velocidad es nula? e En qu intervalos de tiempo la velocidad es negativa? e En qu intervalos de tiempo la aceleracin es positiva? e o

(Compare las respuestas con las de la parte b)). 8. La gura 2.15 muestra la velocidad de una part cula en funcin del tiempo. o

2.4 Problemas

41

Figura 2.15 En qu instantes o en qu intervalos de tiempo: e e a) b) c) d) e) f) g) h) i) j) La velocidad es cero? La velocidad es constante? La velocidad es positiva? La aceleracin es nula? o La aceleracin es positiva? o El mdulo de la velocidad es mximo? o a El mdulo de la aceleracin es mximo? o o a Cul es la distancia que recorre la part a cula entre t = 2 s y t = 4 s? Si en el instante t = 0 la part cula se encuentra en el origen (es decir, si s(0) = 0), haga un grco aproximado del desplazamiento s(t). a Haga un grco aproximado de s(t) si s(0) = 4 m. a

Respuestas: a) En t = 2 s y t = 8,5 s; b) A partir de t = 10 s, se podr decir tambin a e que en el instante t = 6 s la velocidad es constante; c) Entre t = 2 s y t = 8,5 s; d) Misma respuesta de la parte b); e) Entre t = 0 s y t = 6 s; f) En t = 6 s; g) Entre t = 7 s y t = 9 s; h) Entre t = 2 s y t = 4 s la velocidad media es de 1 m/s, luego la distancia recorrida es de 2 m (note que esto coincide con el rea bajo la curva). a 9. La gura 2.16 muestra la aceleracin de una part o cula en funcin del tiempo. o a) b) c) Si en el instante t = 0 s la part cula est en reposo, encuentre la velocidad de la a part cula en cada instante. Graque! Calcule el tamao de las reas I, II y III. Qu unidades tienen? Qu relacin n a e e o hay entre estas reas y la parte a) de este problema? a Repita lo hecho en la parte a), pero suponiendo que en el instante t = 0 la part cula tiene una velocidad v0 = 8 m/s. Graque!

42

Cinemtica en una dimensin a o

Figura 2.16 10. En cada una de las siguientes expresiones para la posicin s(t) de una part o cula, encuentre una expresin anal o tica para la velocidad instantnea: a a) s(t) = at2 + bt + c b) s(t) = at c) s(t) = a cos (t + ) En las ecuaciones anteriores a, b, c, , y son constantes. 11. Para cada una de las siguientes expresiones para la aceleracin a(t) de una part o cula 2 (a en m/s y t en s), encuentre la expresin ms general para la velocidad v(t) y la o a posicin x(t). o a) b) a(t) = a0 a(t) = a0 cos (t)

En las expresiones anteriores, a0 y son constantes. 12. Un observador suelta una piedra desde el techo de un edicio. El sonido de la piedra chocando contra el suelo se escucha despus de t0 = 6 s. e a) Si la velocidad del sonido es c = 340 m/s, encuentre la altura del edicio. (Ignore los efectos del roce del aire, que en la prctica, para este problema, no son a despreciables.) Demuestre que si gt0 /c mente dada por 1, entonces la altura del edicio viene aproximada1 h = gt2 2 0 1 gt0 c .

b)

2.4 Problemas 13.

43

Dos trenes A y B, inicialmente separados por una distancia de 13 km, viajan hacia su encuentro a una velocidad de 30 km/h. Desde A parte una paloma mensajera que llega al tren B 10 minutos despus. Calcule la velocidad con que vuela la paloma e respecto al tren A. Resuelva el problema en forma grca y luego en forma anal a tica.

Figura 2.17 14. La gura 2.17 muestra la velocidad de una part cula en funcin del tiempo. o a) b) 15. Si en el instante t = 0 s la part cula se encuentra en el origen (es decir, x(0) = 0), encuentre la posicin de la part o cula en cada instante. Graque. Repita lo hecho en la parte a), pero suponiendo que en el instante t = 0 se tiene x(0) = 3 m.

Desde un puente de 60 m de altura se deja caer una piedra. Una segunda piedra se arroja verticalmente hacia abajo 1 s ms tarde. Ambas piedras llegan al suelo a simultneamente. Cul fue la velocidad inicial de la segunda piedra? (Desprecie el a a roce del aire.) Un cohete se dispara verticalmente, subiendo con aceleracin constante de 20 m/s2 o respecto a la plataforma de lanzamiento durante 1 minuto. En ese momento se agota su combustible y contina movindose slo bajo la accin de la aceleracin de gravedad. u e o o o a) b) c) Cul es la mxima altura que alcanza? a a Cul es el tiempo transcurrido desde que despega hasta volver a caer sobre la a plataforma? Graque la posicin y velocidad en funcin del tiempo. o o

16.

17.

Panchito deja caer una pelota desde una altura h. La pelota, cada vez que choca contra el suelo, rebota con una rapidez igual a aqulla con la cual lleg al suelo multiplicada e o por , donde es una constante 0 < < 1. Encuentre:

44 a) b) c) d) e)

Cinemtica en una dimensin a o La altura que alcanza la pelota despus del primer rebote. e La altura que alcanza despus del segundo rebote. e La altura que alcanza despus del k-simo rebote. e e La distancia total recorrida desde que se solt la pelota hasta el k-simo rebote. o e La distancia total recorrida por la pelota hasta que se detiene (tome k en la expresin anterior). o
2(k1) 1 2 1

Respuestas: c) 2k h ; d) h + 2h2 18.

Un automovilista pasa a exceso de velocidad frente a un retn policial. 5 minutos e ms tarde sale en su persecusin un polic motorizado a una velocidad de 120 km/h. a o a Despus de 40 minutos, el polic da alcance al infractor. Cul era la velocidad del e a a infractor? Consideremos el movimiento de una esfera en un medio viscoso (en ausencia de fuerzas gravitacionales). La aceleracin que sufre la esfera es proporcional a su velocidad, pero o en direccin contraria, es decir a(t) = v(t), donde es una constante. Supongamos o que = 0,01 s1 y la velocidad inicial de la esfera es |v0 | = 50 m/s. Encuentre numricamente la distancia s(t) recorrida por la esfera y graf e quela. Para resolver el problema note que, si es un pequeo intervalo de tiempo, entonces n v(t + ) s(t + ) v(t) + a(t) s(t) + v(t) .

19.

20.

Considere dos varillas muy largas: una ja horizontalmente y la otra formando un a ngulo constante con la primera, y movindose verticalmente con rapidez v0 conse tante (ver gura 2.18). Determine la velocidad con que se mueve el punto de interseccin de las dos varillas (tal punto de interseccin no corresponde al movimiento de o o algn objeto f u sico real).

Figura 2.18 21. Un pasajero corre con velocidad de 4 m/s para alcanzar un tren. Cuando est a a una distancia d de la portezuela ms prxima, el tren comienza a moverse con una a o aceleracin constante a=0.4 m/s2 , alejndose del pasajero. o a

2.4 Problemas a) b) Si d=12 m y el pasajero sigue corriendo, alcanzar a subirse al tren? a

45

Haga un grco de la funcin xt (t) del tren. En el mismo grco dibuje la funa o a cin xp (t) correspondiente al pasajero para diversos valores de la distancia de o separacin d. Encuentre el valor cr o tico dc para el cual el pasajero alcanza apenas el tren. Para la separacin cr o tica dc , cul es la velocidad del tren cuando el pasajero lo a alcanza?

c)

22.

Desde un edicio se lanza una piedra A con una velocidad inicial vertical hacia abajo v0 = 30 m/s. Desde el suelo, al pie del edicio y en el mismo instante, se lanza una piedra B hacia arriba. Las dos piedras chocan a una altura h = 30 m, siendo en ese instante la rapidez de ambas piedras la misma. Encuentre el tiempo que transcurre entre el lanzamiento y la colisin. (Use para g el valor 10 m/s2 .) o Respuesta: t = 3 1 s.

23.

Considere un avin de pasajeros cuya velocidad de aterrizaje es de unos 400 km/h. o Suponga que la desaceleracin del avin es uniforme. Encuentre el valor que debe o o tener sta para que el avin llegue al reposo en una pista de 1200 m. e o Respuesta: a =5,15 m/s2 Cul ser la forma del cilindro de mximo volumen que puede ser inscrito en una a a a esfera de radio R?

24.

25.

En Paine un agricultor tiene la posibilidad de realizar una (y slo una) exportacin de o o sand de su plantacin. Al comienzo de la temporada el precio es bueno, pero la proas o duccin no es grande. En efecto, al comienzo tiene 6 toneladas para vender y el precio o es de $40,000/ton. . Por cada d que demore la exportacin puede exportar 0.5 toa o neladas adicionales; sin embargo, el precio disminuye en aproximadamente $800/ton. Cunto tiempo deber esperar para realizar la exportacin si desea maximizar las a a o entradas? Respuesta: 19 d as.

26.

A partir de un tronco de 27 cm de dimetro se desea aserrar una viga de seccin a o rectangular que tenga la mayor resistencia posible. La resistencia de una viga horizontal apoyada en sus extremos, en primera aproximacin, es proporcional al ancho o y proporcional al cuadrado de su altura. Cules sern las dimensiones de la viga? a a

46 27. Un salvavidas ubicado en el punto A en una playa debe socorrer a un nadador ubicado en el punto B (ver gura 2.19). La velocidad con que puede correr el salvavidas en la arena es v1 y la velocidad con que avanza en el agua es v2 . Sea P el lugar ptimo en el cual el salvavidas debe o ingresar al agua para que tarde el menor tiempo posible en el trayecto de A a B. Demuestre que en ese caso se satisface sin v1 . = sin v2

Cinemtica en una dimensin a o

Figura 2.19

Notemos que esta expresin es anloga a la ley de Snell para la refraccin de un rayo o a o de luz. 28. Qu dimensiones (interiores) tiene un recipiente cil e ndrico, cuya capacidad es de un litro, si la forma se ha elegido de tal manera que en su confeccin se use la menor o cantidad de material posible? Considere cierto objeto A que se mueve a lo largo del eje x tal como se describe a continuacin: o i) En el instante t = 0 se encuentra en x0 = 4 [m] y su velocidad es v0 = 2 [m/s]. ii) Durante los primeros cuatro segundos la velocidad permanece constante. iii) A partir del instante t = 4 [s], el objeto frena uniformemente hasta quedar con la mitad de la velocidad. Durante este proceso de frenado la part cula avanza 3 [m]. iv) Luego mantiene esa velocidad durante 2 [s]. v) Luego la part cula acelera (en sentido negativo) con una aceleracin constante o 2 ] hasta que la velocidad sea v = 3 [m/s]. a0 = 2 [m/s 1 vi) A continuacin se desplaza con la velocidad v1 hasta llegar a dos metros del o punto de partida. vii) Finalmente la part cula A frena uniformemente hasta quedar en reposo en el punto de partida (x0 = 4 [m]). a) b) c) Haga un grco detallado de x(t) y v(t). a Encuentre la velocidad media de la part cula A entre los instantes t = 6 [s] y t = 13 [s]. En qu instante el alejamiento desde el punto de partida es mximo y cunto e a a es ese alejamiento?

29.

2.5 Solucin a algunos de los problemas o d)

47

Un segundo mvil B parte en t = 0 desde el origen y se deplaza con velocidad o constante vB = 1 [m/s] a lo largo de la misma recta que A. Suponga que cuando los dos mviles se encuentran por primera vez, B se detiene. En qu instante o e volvern a encontrarse? a

30.

Un malabarista desea hacer piruetas manteniendo en forma rotativa, con una mano, tres manzanas en el aire. Si el malabarista desea hacer lanzamientos cada 0,5 s, determine la altura a la cual usted le aconsejar lanzar cada manzana. a Desde la altura H con respecto al piso se deja caer un macetero. En ese instante, y desde el primer piso, un ascensor acelera hacia arriba con aceleracin g, ( < 1). o Si el ascensor tiene una altura h, (h < H) y parte del reposo, calcule el tiempo que demora el macetero en pasar desde el techo al piso del mismo. Para no hacer trgica a la situacin, suponga que la trayectoria (recta) del macetero pasa al lado del ascensor. o Dos mviles A y B (puntuales) estn restringidos a moverse sobre el eje x de cierto o a sistema de coordenadas. Inicialmente A se desplaza a 10 m/s, mientras que B se encuentra en reposo en el origen del sistema de coordenadas. En t = 0 cuando A se encuentra en xA = 100 m, el mvil B comienza a ser uniformemente acelerado en la o direccin positiva del eje x con aceleracin a1 = 1 m/s2 . Este movimiento contina o o u hasta que B se encuentra a 22 m de A. Entonces B deja de acelerar y simultneamente a env un mensaje al mvil A, que demora 0,5 s en llagar a destino. Tan pronto A recibe a o el mensaje, se detiene. Cul es la velocidad c con que se propaga el mensaje entre A y B? Suponga a que la velocidad con que viaja el mensaje es constante. b) Cul es la velocidad de B en el instante en que env el mensaje? a a c) Cul es el desplazamiento de B entre t = 0 y el instante en que choca con A? a d ) Cul es la velocidad media de B entre t = 0 y el instante en que choca con A? a a)

31.

32.

2.5.

Solucin a algunos de los problemas o

Solucin al problema 19 o Sea x la direccin a lo largo de la cual ocurre el movimiento y denotemos, respectivamente, o con s(t), v(t) y a(t) a la posicin , velocidad y aceleracin que tiene la part o o cula en el instante t. Las condiciones iniciales son s(0) = 0 y v(0) = 50 m/s. Conociendo s(0), v(0) podemos encontrar a(0). En efecto a(0) = v(0). Usando las expresiones v(t + ) s(t + ) v(t) + a(t) s(t) + v(t) () .

48

Cinemtica en una dimensin a o

y eligiendo cierto valor pequeo para , podemos encontrar s() y v(). n Conociendo s() y v() podemos encontrar a(). En efecto a() = v(). Usando nuevamente las relaciones (*) (pero ahora con t = ), podemos encontrar s(2) y v(2), y a partir del ultimo tambin a(2). Etc... e Todo el proceso anterior se puede automatizar. En la prxima pgina se presenta un proo a grama en QUICKBASIC (para un PC compatible) que resuelve numricamente el problema e y graca los resultados en la pantalla del computador. Al resolver numricamente el problema, repita el clculo con distintos valores de y observe e a como el resultado no depende de este parmetro cuando es lo sucientemente chico. a Tambin repita el clculo para distintos valores de y analice como este parmetro afecta e a a al resultado.

2.5 Solucin a algunos de los problemas o


CLS SCREEN 12 VIEW (160, 20)-(580,310) TMIN = 0 TMAX = 500 YMIN = 0 YMAX = 6000 WINDOW (TMIN, YMIN)-(TMAX, YMAX) LINE (TMIN, YMIN)-(TMAX, YMAX), , B FOR I = 0 TO 6 YP = I * 1000 PSET (TMIN, YP) DRAW R8" PSET (TMAX - 10, YP) DRAW R8" NEXT I LOCATE 2, 17 PRINT "60" LOCATE 2, 74 PRINT YMAX LOCATE 2, 18 PRINT "0" LOCATE 20, 74 PRINT YMIN LOCATE 11, 17 PRINT "30" LOCATE 11, 76 PRINT "X" LOCATE 2, 13 PRINT "V" FOR I = 0 TO 10 XP = TMIN + I * (TMAX - TMIN) / 10 PSET (XP, YMIN) DRAW 5" U NEXT I LOCATE 21, 20 PRINT TMIN LOCATE 21, 71 PRINT TMAX LOCATE 23, 44 PRINT "TIEMPO" DT = 1 T = 0 X = 0 V = 40 ETA = 0.01 TF = 500 LOCATE 1, 36 PRINT "DT="; DT; "ETA="; ETA; 10 T = T + DT IF T >TF THEN STOP A = -ETA * V X = X + V * DT V = V + A * DT PSET (T, X), 12 PSET (T, V * 100), 14 GOTO 10 LIMPIA PANTALLA ELIGE SUPERVGA COLOR DEFINE AREA DE TRABAJO MINIMO DE ABSISA MAXIMI DE ABSISA MINIMO DE ORDENADA MAXIMO DE ORDENADA fIJA VALORES ANTERIORES GRAFICA EJES (CAJA) EVALUA POSICION DE TIC POSICIONA EL LAPIZ EN ORDENADA (IZQ) GRAFICA TIC POSICIONA EL LAPIZ EN ORDENADA (DER) GRAFICA TIC POSICIONA LAPIZ IMPRIME 60 EN ORDENADA IZQUIERDA POSICIONA LAPIZ IMPRIME EN ORDENADA DERECHA POSICIONA LAPIZ IMPRIME POSICIONA LAPIZ IMPRIME POSICIONA LAPIZ IMPRIME POSICIONA LAPIZ IMPRIME LEYENDA DE ORDENADA DERECHA POSICIONA LAPIZ IMPRIME LEYENDA DE ORDENADA IZQUIERDA EVALUA POSICION DE TICS DE ABSISA POSICIONA LAPIZ GRAFICA TIC POSICIONA LAPIZ IMPRIME POSICIONA LAPIZ IMPRIME POSICIONA LAPIZ IMPRIME LEYENDA DE ABSISA SE ELIGE DT TIEMPO INICIAL POSICION INICIAL VELOCIDAD INICIAL SE FIJA PARAMETRO DE FRICCION TIEMPO FINAL POSICIONA LAPIZ IMPRIME TITULO EL CALCULO EMPIEZA AQUI !! SE INCREMENTA EL TIEMPO SI T>TF EL CALCULO TERMINA EVALUACION DE LA ACELERACION NUEVA POSICION NUEVA VELOCIDAD GRAFICA PUNTO (T,X) GRAFICA PUNTO (T,V)

49

50 Solucin al problema 27 o Los tiempos t1 , que el salvavidas tarda para correr de A a P y t2 , que tarda para nadar de P a B vienen dados por t1 = y t2 =
2 (L x)2 + zb 2 x2 + za . v1

Cinemtica en una dimensin a o

v1

Por lo tanto, el tiempo total que tarda en ir de A a B es T =


2 x2 + za + v1 2 (L x)2 + zb

Figura 2.20

v1

En la expresin anterior L, za y zb son jos; el valor de x se debe determinar de manera o que T sea m nimo. Encontrar el m nimo de T en funcin de x es equivalente a encontrar los ceros de la funcin o o derivada dT /dx: dT (x) T (x + ) T (x) = l m = 0 dx v1 La derivada tiene ceros si x v1 Pero x2 +
2 za

x x2 +
2 za

v2

(L x)
2 (L x)2 + zb

= v2 x

(L x)
2 (L x)2 + zb

x2 y

2 + za

= sin

(L x)
2 (L x)2 + zb

= sin ,

luego, T (x) tiene un extremo en funcin de x cuando o sin sin = . v1 v2 No es dif convencerse que tal extremo corresponde a un m cil nimo (y no a un mximo). a

2.5 Solucin a algunos de los problemas o Solucin al problema 29 o

51

a) Impl citamente supondremos que las distancias estarn expresadas en metros, el tiempo a en segundos, las velocidades en m/s y las aceleraciones en m/s2 . De acuerdo al enunciado se tiene: Punto de partida: x(0) = 4, v(0) = 2 Entre t = 0 y 4, v(t) = 2, lo que corresponde a una l nea horizontal en el grco v en a funcin de t (ver gura 2.21). o Entre t = 0 y 4 se tiene una recta con pendiente 2, en el grco x(t) en funcin de t (ver a o gura 2.22). La posicin en t = 4 es x(4) = x(0) + v0 4 = 4 + 2 4 = 4. o A partir de t = 4, en el grco v en funcin de t, la velocidad estar representada por una a o a recta hasta llegar a v0 /2 = 1. Durante el proceso de frenado que tarda hasta cierto instante t, la part cula avanza 3 metros, o sea, el rea bajo la curva v(t) entre t = 4 y t debe ser 3. a debe ser 6. No es dif darse cuenta de que t cil La aceleracin entre t = 4 y t = 6 es a1 = 0,5 (es la pendiente en el grco 2.21). o a De acuerdo al enunciado, la part cula avanza 3 metros durante el frenado, o sea, x(6) = x(4) + 3 = 7. El grco de x(t), entre t = 4 y t = t = 6 ser parablico con curvatura a a o negativa. Otra forma de encontrar la posicin en t = 6 es usando la expresin x(6) = o o 2 , o sea, x(6) = 4 + 2 2 0,5 0,5 22 = 7. x(4) + v(4) (6 4) + 0,5 a1 (6 4) De t = 6 hasta t = 8 (durante 2 segundos) la velocidad se mantiene constante. El grco a de v(t) es una recta horizontal con velocidad 1. El rea bajo el grco v(t) entre t = 6 y 8 nos da la distancia que A avanza en ese intervalo. a a Tal rea es 2, luego x(8) = 7 + 2 = 9. Durante este intervalo x(t) es representado por una a recta (velocidad constante). Se tiene que v(8) = 1. La part cula desacelera con aceleracin a0 = 2 hasta que la o velocidad sea 3. Se observa inmediatamente que para ello debe desacelerar durante 2 segundos. Entonces v(10) = v(8) + a0 (10 8) = 1 2 (10 8) = 1 4 = 3. Entre t = 8 y 10 el grco de v(t) es una recta (aceleracin constante). a o Podemos encontrar la posicin de la part o cula en t = 10: x(10) = x(8) + v(8) (10 8) + 0,5 a1 (10 8)2 , o sea, x(10) = 9 + 1 2 + 0,5 (2) 22 = 7. En t = 10 la part cula se encuentra en x(10) = 7 y su velocidad es v(10) = 3. La part cula sigue a velocidad constante hasta llegar a dos metros del punto de partida (o sea, hasta llegar a 2 metros). La part cula, por lo tanto, deber recorrer 9 metros. Con v1 = 3 [m/s] a tardar para ello 3 segundos. O sea, entre t = 10 y t = 13 la velocidad ser constante (linea a a horizontal) en el grco v en funcin de t. a o A partir de t = 13 la part cula frena uniformemente hasta quedar en reposo en el punto de partida. El grco de v(t) es por lo tanto una recta hasta cero. El rea bajo la curva a a entre t = 13 y el instante en que queda en reposo debe ser 3 (la part cula A debe recorrer an dos metros hacia la izquierda para llegar al punto de partida). Es claro que para ello u tardar 4/3 segundos. a

52

Cinemtica en una dimensin a o

Entre t = 13 y t = 14, 3, la part cula recorre 2 metros. El grco de x(t) es una parbola a a curvada hacia arriba que llega a t = 14, 3 con pendiente nula.

Figura 2.21

Figura 2.22 b) En t = 6 y t = 13 la part cula A se encuentra en x(6) = 7 y x(13) = 2, respectivamente. La velocidad media entre esos dos instantes es v= (2) 7 = 9/7 m/s . 13 6

2.5 Solucin a algunos de los problemas o

53

c) En t = 8 la velocidad es 1 m/s. A partir de ese instante la part cula acelera con aceleracin o a0 = 2, o sea, tarda 0.5 s para quedar temporalmente en reposo. En ese instante (8,5 s) ocurre el alejamiento mximo. Se tiene a 1 x(8, 5) = x(8) + v(8) (8, 5 8) + a0 (8, 5 8)2 2 1 = 9 + 1 0, 5 2 0, 52 = 9, 25 [m] . 2 d) Gracando xB (t) en la gura 2.21 se encuentra que los dos mviles se vuelven a encontrar o en el instante t = 11 s. Solucin al problema 30 o Cada manzana debe tardar t0 = 30, 5 = 1, 5 segundos en subir y bajar. Al lanzar un objeto con velocidad v0 hacia arriba tarda un tiempo v0 /g hasta llegar arriba y un tiempo igual hasta volver al punto de partida. Tenemos t0 = 2v0 = 1, 5 [s] . g

Esta ecuacin nos permite evaluar la velocidad con que se debe lanzar la manzana, v0 = o t0 g/2. 2 La altura a la que llega es un objeto lanzado con velocidad v0 es h = v0 /(2g). Combinando las dos ultimas ecuaciones se encuentra para h la expresin o h= Con g 10 [m/s2 ] se encuentra h 1 2 gt . 8 0

3 metros.

Solucin al problema 32 o

a) Cuando B env el mensaje se encuentra a 22 m de A. El mensaje tarda 1/2 s en a llegar a su destino. Durante ese intervalo el mvil A seguir movindose desplazndose o a e a 10 0, 5 = 5 metros. El mensaje deber recorrer en 0,5 s una distancia de (22+5)=27 a metros. La velocidad del mensaje ser c = 27/0, 5 = 54 [m/s]. a b) Las ecuaciones de movimiento de los mviles, para 0 < t y el instante en que B env o a el mensaje (llammoslo t1 ), son e xA (t) = xA (0) + vA (0)t = 100 + 10 t 1 1 xB (t) = a1 t2 = t2 2 2 vA (t) = vA (0) = 100

54

Cinemtica en una dimensin a o vB (t) = a1 t = t . (En las expresiones anteriores estamos suponiendo que los tiempos estn dados en a segundos, las distancias en metros, las velocidades en [m/s] y las aceleraciones en [m/s2 ].) Sabemos que en t = t1 la separacin entre A y B es de 22 metros, o sea, o xA (t1 ) xB (t1 ) = 100 + 10 t1 1 2 t = 22 . 2 1

Resolviendo esta ecuacin cuadrtica para t1 se encuentra que t1 = 10 16. En el o a contexto del problema slo la solucin positiva tiene sentido, o sea, t1 = 26 [s]. o o La velocidad de B en el instante t1 es vB (t1 ) = 26 [m/s]. c) Desde que B env el mensaje hasta chocar con A, el mvil B debe recorrer una a o distancia de 22+5=27 metros. En el instante t1 se encuentra a xB (t1 ) = (26)2 /2 = 338 m del origen. La distancia total que B debe recorrer desde que parte del origen hasta que choca con A es (338+27)=365 m. d) Desde que B env el mensaje hasta chocar con A, el mvil B debe recorrer una a o distancia de 22+5=27 metros. Como su velocidad (a partir de t1 ) es de 26 m/s, tardar 27/26 segundos. El tiempo total, desde que B parte del origen hasta que a choca con A es (26+27/26) s. Para la velocidad media de B se encuentra v= 365 26 + 27 26 13, 5 [m/s] .

2.6.

Elementos del clculo innitesimal e integral a

A continuacin se presenta un resumen de algunos resultados del clculo que se usarn o a a extensivamente en lo que sigue. Se dejar para los cursos de matemticas la demostracin a a o rigurosa de los resultados. Supondremos impl citamente que las funciones que se usan ms a abajo tienen todas las propiedades necesarias para que los teoremas planteados sean vlidos a (por ejemplo, sean funciones continuas, derivables, acotadas, etc.). Sean f (t) y g(t) dos funciones y un nmero (real o complejo). La funcin derivada df (t)/dt, u o relacionada con la pendiente de la funcin f (t), por denicin es o o df (t) 1 = f(t) = l m [f (t + ) f (t)] . 0 dt Propiedades: a) b) c) d(f (t)) = f(t) . dt d(f (t) + g(t)) = f(t) + g(t) . dt d(f (t) g(t)) = f(t) g(t) + f (t) g(t) . dt

2.6 Elementos del clculo innitesimal e integral a d) df (g(t)) = f(g(t)) g(t) . dt

55

Demostracin de c): o De la denicin de la derivada se deduce que, para muy pequeo o n f (t + ) = f (t) + f(t) . Con esta relacin, y una anloga para la funcin g(t), se deduce que o a o d(f (t) g(t)) dt 1 [f (t + ) g(t + ) f (t) g(t)] 0 1 = l m (f (t) + f(t)) (g(t) + g(t)) f (t) g(t) 0 1 f(t) g(t) + f (t) g(t) + 2 f(t) g(t) = l m 0 = f(t) g(t) + f (t) g(t) . = l m ()

Demostracin de d): o d 1 f (g(t)) = l m [f (g(t + )) f (g(t))] 0 dt 1 = l m [f (g(t) + g(t)) f (g(t))] 0 Pero, usando nuevamente la ecuacin (), se tiene o f (g + g) = f (g) + ( g) f(g) , luego d 1 f (g(t)) + g(t) f(g(t)) f (g(t)) f (g(t)) = l m 0 dt = f(g(t)) g(t) .

En un grco de la funcin f (t) en funcin a o o de t, la expresin (integral) o


b

A=
a

f (t) dt

representa al rea delimitado por la funa cin f (t) y el eje t entre t = a y t = b (ver o gura).

Figura 2.23 Propiedades:

56 a)
a b b

Cinemtica en una dimensin a o

f (t) dt =
a b b

f (t) dt .
b

b)
a

[ f (t) + g(t) ] dt =
a b c

f (t) dt +
a b

g(t) dt .

c)
a

f (t) dt =
a

f (t) dt +
c

f (t) dt .

En muchos casos es posible evaluar la integral A anal ticamente. Para ello, se debe encontrar una funcin F (t) tal que su derivada sea la funcin que aparece tras el s o o mbolo integral, o sea, tal que dF (t)/dt = f (t). Entonces
b b

A=
a

f (t) dt = F (t)
a

= F (a) F (b) .

Cap tulo 3

Cinemtica en dos y tres a dimensiones


En este cap tulo extenderemos la descripcin del movimiento de una part o cula a dos y tres dimensiones. Esto nos lleva a introducir el concepto de vector, cuya denicin y propiedades o ilustraremos con los vectores desplazamiento, velocidad y aceleracin. o

3.1.

Vectores

Consideremos el movimiento de una part cula en un plano. La posicin de la o part cula podr ser claramente especicaa da si se introduce un sistema de ejes perpendiculares que se intersectan en un punto, que llamaremos el origen (ver gura 3.1). Por ejemplo, el punto P en la gura 3.1 se encuentra a 3 m a la derecha del origen, medidos a lo largo de la direccin del eje o x, a 2 m sobre el origen, medidos a lo largo del eje y. En general, la posicin de un o punto cualquiera queda determinada dando un par ordenado (x, y) de nmeros, en u el sentido que siempre el primer nmero u corresponder a la proyeccin sobre el eje a o x y el segundo nmero a aqulla sobre el u e eje y .

Figura 3.1

El trazo que une el origen O con el punto P , en el sentido que indica la punta de echa en la gura 3.1, se denomina el vector de posicin rp del punto P . La magnitud de este vector o es igual a la longitud del trazo OP y se denota por |rp | o simplemente como rp (sin echa). Rigurosamente, un vector es un objeto que, ms all de poseer las caracter a a sticas descritas, est denido por la existencia de una operacin de suma entre vectores y la multiplicacin a o o

58

Cinemtica en dos y tres dimensiones a

de un vector por un nmero (escalar), operaciones que satisfacen reglas muy precisas. u Introduzcamos estas ideas a travs de ejemplos. e Supongamos que la part cula en un instante t se encuentra en P y en un instante posterior t > t se encuentra en el punto Q (ver gura 3.1). El vector que une el origen O con Q es el nuevo vector de posicin de la part o cula. Al vector conformado por el trazo P Q y cuyo sentido va desde P hacia Q, se llama vector desplazamiento, r (ver gura 3.1). Suma de Vectores Sean A y B dos vectores. Traslademos paralelamente a s mismo al vector B hasta que su extremo romo se superponga con el extremo aguzado (punta de echa) del vector A. El vector suma A + B C se dene como el trazo que comienza en el extremo romo de A y termina en el extremo aguzado de B. Esta denicin se conoce o con el nombre de regla del paralelgramo. o Figura 3.2 Ejemplo: Un excursionista parte desde una cierta posicin y camina 4 km hacia el Este y o luego 3 km hacia el Sur. Cul es el vector a desplazamiento resultante C? El vector C es la suma vectorial de los desplazamientos parciales realizados por el excursionista, hacia el este A y luego hacia el sur B. Grcamente la situacin a o est ilustrada en la gura 3.3. La magnia tud del desplazamiento resultante se calcula utilizando el teorema de Pitgoras a C= A2 + B 2 = 9 + 16 = 5 km .

Figura 3.3

La direccin de C queda denida por el ngulo que forma el vector C con la direccin o a o OesteEste. Consideraremos un ngulo positivo cuando se mide en sentido contrario a los a punteros del reloj, luego 3 tan = = 0,75 , es decir, = 36,9 . 4 Que el ngulo sea negativo signica que est medido en el mismo sentido de los punteros a a del reloj. Propiedades de la suma de vectores.

3.1 Vectores i) Conmutatividad: ii) Asociatividad: A+B =B+A . A + (B + C) = (A + B) + C . A+0=A .

59

iii) Existe un vector nulo tal que

iv) Para cada vector A existe un vector opuesto, que denotaremos por A, tal que A + (A) = 0 .

Multiplicacin de un vector por un escalar real. o La multiplicacin de un vector A por un nmero real (escalar real) se dene como un o u nuevo vector B de magnitud |A|, cuyo sentido coincide con el de A si > 0 y es opuesto al de ste si < 0. e Propiedades de la multiplicacin por un escalar real. o Sean y dos nmeros reales y A y B dos vectores, entonces: u i) ii) iii) iv) (A + B) = A + B. ( + )A = A + A. ()A = ( A). Para todo vector A se cumple que 1 A = A.

Ejercicio: Compruebe grcamente, con algunos ejemplos concretos, que se cumplen todas a las propiedades de los vectores recin sealadas. e n Note que dos vectores son iguales si tienen la misma magnitud y que apuntan en la misma direccin. En la gura 3.4 se o muestra un conjunto de vectores iguales, dibujados en diferentes posiciones del plano xy.

Figura 3.4 Componentes cartesianas y polares de un vector. Consideremos nuevamente al vector desplazamiento r de la gura 3.1. Proyectando los extremos del vector desplazamiento sobre el eje x, se obtienen los puntos xP y xQ . La diferencia xQ xP se llama componente cartesiana x del vector r. De la misma forma, las l neas perpendiculares al eje y, trazadas desde los extremos del vector r, denen su componente cartesiana y, o sea, r = (xQ xP , yQ yP ) .

60 Sea A = (Ax , Ay ) un vector cualquiera del plano xy, con componentes cartesianas Ax y Ay . Expresemos las componentes del vector en funcin de su magnitud o y del ngulo que forma con el semieje x a positivo. La gura 3.5 muestra que Ax = A cos donde A = |A| = (A2 + A2 ) x y y tan = Ay . Ax Ay = A sin ,

Cinemtica en dos y tres dimensiones a

Figura 3.5

De esta manera, un vector en un plano queda determinado si se conocen sus componentes cartesianas, o si se conoce su magnitud A y el ngulo que forma con el semieje x positivo a (referidos a un sistema de coordenadas dado). Los nmeros (A, ) reciben el nombre de u coordenadas polares del vector A. Vectores Unitarios. Al dividir un vector A por su magnitud se obtiene un nuevo vector a, de mdulo uno, cuya o direccin y sentido coinciden con aquellos del vector A. En efecto, o a= A = A Ax Ay , A A A2 + A2 x y =1 . A2

|| = a

(Ax /A)2 + (Ay /A)2 =

A cada vector se le puede asociar un vector unitario. Existen, sin embargo, tres vectores unitarios que merecen mencin especial. Estos son los vectores unitarios x, y y z que apuntan o en sentido positivo sobre cada uno de los ejes coordenados de un sistema cartesiano en tres dimensiones. La gura 3.6 muestra la descomposicin o de un vector arbitrario A en la suma de tres vectores: un vector Ax x , paralelo al eje x, otro Ay y paralelo al eje y y un ter cero Az z paralelo al eje z. Es decir, A = Ax x + Ay y + Az z .

Figura 3.6 Producto escalar o producto punto de dos vectores

3.1 Vectores Sean A = (Ax , Ay , Az ) = Ax x + Ay y + Az z y B = (Bx , By , Bz ) = Bx x + By y + Bz z dos vectores arbitrarios. Se dene el producto punto entre los vectores A y B mediante la expresin o Figura 3.7 A B |A| |B| cos , donde es el ngulo entre los dos vectores (ver gura 3.7). a

61

De la denicin se desprende que el producto punto de dos vectores es un nmero real. o u Adems, y esto es muy importante, es independiente de la orientacin del sistema de coora o denadas. Usando la denicin de producto punto es inmediato que o xx=yy =zz =1 y xy =xz =yz =0. Otras caracter sticas importantes del producto punto son su conmutatividad AB =BA y distributividad A (B + C) = A B + A C .

Evaluemos el producto punto entre los dos vectores A y B en trminos de sus coordenadas. e Se tiene A B = (Ax x + Ay y + Az z ) (Bx x + By y + Bz z ) = Ax Bx x x + Ax By x y + Ax Bz x z + Ay Bx y x + Ay By y y + +Ay Bz y z + Az Bx z x + Az By z y + Az Bz z z = Ax Bx + Ay By + Az Bz . Resumen: El mdulo de un vector y la suma y producto punto de dos vectores vienen dados o por |A| = A2 + A2 + A2 x y z

A + B = (Ax + Bx , Ay + By , Az + Bz ) = (Ax + Bx ) x + (Ay + By ) y + (Az + Bz ) z

62 y

Cinemtica en dos y tres dimensiones a

A B = |A||B| cos = Ax Bx + Ay By + Az Bz . Note que la ultima expresin permite evaluar el ngulo entre dos vectores si se conocen sus o a componentes cartesianas.

Ejemplo Evaluemos nuevamente el ngulo entre dos a diagonales de un cubo. Sea A el vector a lo largo de la diagonal que une el punto (0,0,0) con el punto (1,1,1) y B el vector a lo largo de la diagonal que une el punto (1,0,0) con el punto (0,1,1). Los vectores A y B, por lo tanto, pueden escribirse en coordenadas cartesianas de la forma A=x+y+z y B = + y + z . x Figura 3.8

Evaluemos el producto punto de estos dos vectores. Se tiene A B = |A| |B| cos = 3 3 cos , donde es el ngulo entre los dos vectores (o sea, el ngulo entre las dos diagonales). Por a a otra parte, usando coordenadas cartesianas A B = 1 (1) + 1 1 + 1 1 = 1 . De las dos ecuaciones anteriores se deduce que cos = 1/3, o sea, = 70,53 .

3.2.

Cinemtica a

La generalizacin de los conceptos de la cinemtica de una a dos y tres dimensiones es o a directa. Supongamos que r (t) representa la posicin de cierta part o cula. Entonces su velocidad y aceleracin (instantnea) vendrn dadas por o a a v(t) = r (t) = l m y v(t + ) v(t) a(t) = v(t) = r (t) = l m . 0 De la expresin anterior se deduce que si o r (t) = x(t) + y(t) + z(t) , x y z r(t + ) r(t)

3.2 Cinemtica a

63

Figura 3.9 donde x(t), y(t) y z(t) son las componentes del vector de posicin, entonces o v(t) = x(t) + y(t) + z(t) , x y z o sea, para encontrar la velocidad se puede derivar cada componente del vector posicin o por separado. Introduzcamos tambin el concepto de velocidad relativa. Supongamos que una part e cula A se mueve con velocidad vA y otra part cula B con velocidad vB , entonces la velocidad con que A observa que se mueve B, viene dada por v = vB vA . Se dice que v es la velocidad relativa de B respecto a A. Ejemplo: Suponga que la corriente de un canal tiene una velocidad de 10 km/h en direccin Este. o Un transbordador navega en la direccin de 30 Noroeste, a una velocidad de 20 km/hora o con respecto a la corriente del canal (ver gura 3.9). Cul es la velocidad y direccin del a o transbordador segn un observador situado en la ribera? u Para resolver el problema introduciremos un sistema de coordenadas x, y cuyo origen O se mueve junto al agua del canal. Para el observador O , un punto jo en la orilla se mueve con velocidad vA = [10, 0] km/h mientras que el transbordador se aleja con una velocidad vt = [20 sin(30 ), 20 cos(30 )] km/h = [10, 10 3] km/h .

Luego, la velocidad con que el observador parado en la orilla en el punto A ve alejarse al transbordador (o sea, la velocidad relativa entre el transbordador y la orilla), ser a v = vt vA = [0, 10 3] km/h = 10 3 y km/h .

64

Cinemtica en dos y tres dimensiones a

Figura 3.10 Analicemos ahora el problema de otra forma. Supongamos que nos damos un intervalo de tiempo arbitrario, por ejemplo, 1 hora (porque es el ms fcil de usar en este caso) e a a imaginemos que durante ese intervalo la corriente del canal est detenida. Calculamos el a desplazamiento del transbordador en este caso. En una hora el ferry se desplaza 20 km desde O hasta el punto P . En seguida y siempre en nuestra imaginacin dejemos uir o la corriente del canal durante una hora, pero ahora con el ferry detenido (dejando que simplemente ote en la corriente). El desplazamiento debido al arrastre del canal llevar al a ferry desde el punto P hasta P (10 km hacia la derecha), como mostramos en la gura 3.10. El desplazamiento total del ferry es el vector de O hasta P . Este desplazamiento, como es fcil de demostrar, coincide con el que el ferry hubiese tenido en una hora si los dos a movimientos hubiesen estado presentes simultneamente. Es decir, para resolver el problema a podemos descomponer el movimiento en dos movimientos separados, congelando uno y otro sucesivamente. El movimiento total es la superposicin de ambos movimientos. Esta o operacin, slo posible en la imaginacin, arroja los mismos resultados que se observan en o o o la vida real. Demos otro ejemplo del uso del principio de superposicin. Consideremos un anillo que o rueda (sin resbalar) por una supercie horizontal con velocidad constante. Tomemos un punto cualquiera sobre el anillo y analicemos su movimiento. Para un observador O en reposo respecto a la supercie, el movimiento del punto tendr un aspecto complicado. Sin a embargo, al trasladarnos uniformemente con la misma velocidad que el centro del anillo, el movimiento del punto se tornar muy simple: es un movimiento circular uniforme. As el a , movimiento complicado que observa O se puede descomponer en dos movimientos simples, un movimiento de traslacin uniforme superpuesto a un movimiento circular uniforme (ver o problema 13 de la seccin 3.3). o

Ca libre da Galileo fue el primero en considerar la ca de una part da cula como una superposicin de o dos movimientos. La gura 3.11, a la izquierda, muestra la posicin de una pelota en ca libre durante o da varios instantes equiespaciados. A la derecha se muestra la situacin que se observa si el o cuerpo adems inicialmente tiene una velocidad horizontal. La trayectoria en este caso es a

3.2 Cinemtica a

65

Figura 3.11 una parbola. Antes de Galileo, los lsofos se esforzaron mucho para intentar explicar este a o movimiento. Galileo centr su inters buscando la descripcin ms sencilla y directa. De o e o a hecho, lo analiz como una superposicin de dos movimientos: i) la tendencia natural de los o o cuerpos a mantener su velocidad (ley de inercia) y ii) la ca libre de un cuerpo debida a da la atraccin gravitatoria. Ambos movimientos se superponen simultneamente y dan origen o a al movimiento parablico. o Una vez aceptado que el movimiento de una part cula en un campo gravitatorio uniforme se puede describir como una superposicin de dos desplazamientos que ocurren simultneao a mente, continuamos con la descripcin de este movimiento. o Para comenzar, especiquemos el sistema de referencia. El eje x lo elejimos de manera que su direccin coincida con la proyeccin de la velocidad sobre el plano horizontal, mientras o o que el eje z lo elegimos hacia arriba (o sea, una part cula al caer acelera en la direccin o ). De acuerdo a nuestra hiptesis, la aceleracin en todo instante es a(t) = g. Tambin z o o z e (0) (0) supondremos que la velocidad en el instante t = 0 viene dada por v(0) = vx x + vz z y que la part cula se encuentra en el lugar r(0) = r0 = x0 x + z0 z . Analicemos cada una de las componentes por separado. Componente x : La aceleracin no tiene componente en la direccin x, o sea, o o ax = 0 . La velocidad vx es, por lo tanto, constante, igual al valor inicial:
(0) vx (t) = vx t .

Para el desplazamiento en la direccin x se encuentra que o


(0) x(t) = x(0) + vx t .

66

Cinemtica en dos y tres dimensiones a

Figura 3.12 Componente z : La aceleracin es o az = g . La velocidad vz y el desplazamiento en la direccin z vendrn dados por o a
(0) vz (t) = vz gt

y 1 (0) z(t) = z(0) + vz t gt2 . 2 Estos resultados los podemos condensar escribindolos en forma vectorial: e a(t) = g z v(t) = v (0) gt z 1 r(t) = r0 + v (0) t gt2 z . 2 Ejemplo Un bombardero vuela con una velocidad horizontal v0 , constante, y a una altura h en una trayectoria que pasa directamente por sobre su objetivo. A qu ngulo de visin debe ea o soltar la bomba, de forma que sta llegue a su objetivo? (Ignore el efecto debido al roce del e aire.) La bomba en el instante en que se deja libre tiene la misma velocidad que el bombardero. Denimos el sistema de coordenadas de acuerdo a lo que se observa en la gura 3.12. Entonces la posicin y la velocidad inicial de la bomba vienen dadas por r0 = h y v0 = v0 x, o z respectivamente. Cunto demora la bomba en caer? La bomba llegar al suelo cuando a a z(t) = h gt2 /2 = 0. Esto ocurre en el instante = (2h/g). Durante el intervalo de tiempo la bomba alcanza a recorrer una distancia horizontal L = v0 . Luego para el a ngulo de visin obtenemos o tan = L v0 = h h 2h = g
2 2v0 . gh

3.2 Cinemtica a Movimiento circular uniforme

67

Consideremos una part cula que gira con rapidez constante sobre una trayectoria circular de radio R (que dene el plano xy). Eligiendo el origen al centro del c rculo, el ngulo del a vector posicin con el eje x aumentar uniformemente: o a (t) = 0 + 0 t , donde 0 es el ngulo en el instante t = 0 y 0 es una constante que determina cun rpido a a a var el ngulo (por esta razn se le suele llamar velocidad angular). Las componentes x e a a o y del vector posicin vienen dadas por o x(t) = R cos (t) = R cos(0 + 0 t) e y(t) = R sin (t) = R sin(0 + 0 t). El vector posicin es, por lo tanto, o r(t) = R cos(0 +0 t) +R sin(0 +0 t) . x y Derivando r(t) se encuentra la velocidad v(t) = R0 sin(0 + 0 t) x + R0 cos(0 + 0 t) . y Evaluemos el mdulo de la velocidad (rao pidez): Figura 3.13 v = |v(t)| = = vx (t)2 + vy (t)2

2 2 R2 0 sin2 (0 + 0 t) + R2 0 cos2 (0 + 0 t) = R0 .

A pesar de que la rapidez es constante (no depende del tiempo), la velocidad no lo es, ya que continuamente cambia de sentido. Esta ultima ecuacin ensea que la velocidad angular es o n la rapidez de la part cula dividida por el radio de giro. Evaluando el producto punto entre r y v: r(t) v(t) = x(t)vx (t) + y(t)vy (t) = 0 se encuentra que ste es nulo. Como el producto punto de dos vectores no nulos vale cero e slo si los dos vectores son perpendiculares, se halla que la velocidad de una part o cula en un movimiento circular uniforme es siempre perpendicular al radio. Derivando la velocidad se encuentra la aceleracin: o
2 2 a(t) = R0 cos(0 + 0 t) R0 sin(0 + 0 t) . x y

68 Note que en todo instante

Cinemtica en dos y tres dimensiones a

2 a(t) = 0 r(t) ,

o sea, la aceleracin siempre apunta hacia el origen (razn por la cual se llama aceleracin o o o centr peta). La magnitud de la aceleracin siempre es constante y vale o
2 a = |a(t)| = R0 .

3.3.

Coordenadas polares

Los vectores unitarios r y . Hemos visto que el movimiento de un punto P en el plano x, y se puede especicar usando dos funciones que describan sus coordenadas cartesianas del punto, o sea, r(t) = x(t) x + y(t) y . Tambin podemos especicar el movimiento P usando coordenadas polares, es decir, dando e las funciones r(t) y (t). Al usar coordenadas polares para describir el movimiento de un punto P , resulta sumamente conveniente introducir los vectores unitarios r y denidos por r = cos x + sin y y = sin x + cos y . Observe que estos vectores unitarios generalmente (cuando = (t) depende del tiempo) son tiempo dependientes. El vector r apunta en la direccin radial, mientras que el vector o es tangencial al c rculo que pasa por P y tiene su centro en el origen. Ejercicio: Demuestre que los vectores r y efectivamente son unitarios. Tambien demuestre que son ortonormales, es decir, r . Figura 3.14 Encontremos la derivada temporal de estos vectores unitarios, es decir, analicemos como var a medida que transcurre el tiempo. Se tiene an

3.3 Coordenadas polares

69

d r =r = dt

d [ cos x + sin y ] dt d cos (t) d sin (t) = x+ y dt dt = sin((t)) (t) x + cos((t)) (t) y = (t) [ sin((t)) x + cos((t)) y ] =

y d = = dt d [ sin x + cos y ] dt d sin (t) d cos (t) = x+ y dt dt = cos((t)) (t) x sin((t)) (t) y = (t) [ cos((t)) x + sin((t)) y ] = r

Resumen: r= = r . (3.1) (3.2)

Movimiento circular (en coordenadas polares). Consideremos un punto P que se mueve entorno al origen sobre un c rculo de radio R y sea (t) el ngulo polar (medido respecto al eje x y en el sentido contrario al avance del reloj). a El vector posicin del punto P es: o r(t) = R r(t) Derivando esta relacin (slo r depende de t) encontramos la velocidad: o o r(t) = R r = R . Tal como se esperaba, la direccin de la velocidad es tangencial al c o rculo con centro en O que pasa por P . La rapidez es |v| = R . (t) es un ngulo, por esa razn a se le llama velocidad angular. Si el movimiento circular a o es uniforme (siempre con la misma rapidez) entonces (t) = 0 t. Para el movimiento circular uniforme la velocidad angular es simplemente (t) = 0 . Determinemos ahora la aceleracin para el movimiento circular. Derivando el vector veloo cidad se encuentra d v(t) = R dt = R + = R R 2 r

70

Cinemtica en dos y tres dimensiones a

El primer trmino nos da la aceleracin tangencial mientras que el segundo es la aceleracin e o o radial. Para el movimiento circular uniforme (es decir, si (t) = 0 t) se obtiene
2 a(t) = r(t) = R0 r ,

o sea, el mismo resultado encontrado en la seccin anterior. o

3.4.
1.

Problemas
Sean A, B y C los vectores A = 2 + y , B = 3 + y 2 y C = x + 3 z . x x z y a) b) Encuentre el mdulo de A, B y C. o Encuentre el mdulo del vector suma, o sea, evale o u D = |D| = |A + B + C| . c) d) Cul vector es ms largo: A + B o A + C ? En vista de lo calculado en la parte a a a), le sorprende este resultado? Encuentre el ngulo entre los vectores B y C. a 49,860 .

Respuesta: d) 2.

Demuestre que los vectores: A = cos () + sin () x y B = cos () + sin () x y son vectores unitarios que forman un ngulo y con el eje x, respectivamente. a Evale A B y encuentre una frmula para cos ( ). u o

3.

Considere los tres puntos cuyas coordenadas cartesianas vienen dadas por: P1 = (1, 1, 1), P2 = (1, 2, 0) y P3 = (2, 3, 1). Demuestre que ellos denen los vrtices de un e tringulo rectngulo. a a Encuentre un vector unitario A que sea simultneamente perpendicular a los vectores a u = 2 + y z y v = x y + z . Cuntos vectores unitarios A existen con esta x a propiedad? Denamos los vectores:

4.

5.

1 s = ( + y ) x 2 1 t = ( + y ) x 2

3.4 Problemas a) b) c) Graque s y t . Evale s = |s | y t = |t | . u

71

Encuentre el ngulo entre s y t . a Comentario: Note que s y t pueden considerarse como un nuevo conjunto de ejes de referencia (, t ). Para indicar que s y t son vectores unitarios se ha usado la s convencin de reemplazar las echas por tongos. o Considere los vectores A = x + 2 y B = 2 3. Exprese estos vectores en y x y trminos de los nuevos vectores unitarios, es decir, escriba A y B de la forma e A = as s + at t B = bs s + bt t y evale las constantes as , at , bs y bt . u

d)

e)

Evale A B de dos maneras distintas: primero usando las componentes respecto u al sistema de referencia (, y ) y luego usando las componentes respecto al sistema x ). de referencia (, t s

6.

Sea A = x + 3 2. Encuentre un vector B en el plano x, y que sea perpendicular z y a A. Respuesta: B = (2 + y ), donde es un nmero real no nulo. x u

7.

Considere la siguiente situacin en nuestro espacio f o sico de tres dimensiones: Desde cierto origen emergen cuatro vectores de igual tamao, de manera que los ngulos n a entre cualquier par de vectores sean iguales. Encuentre el valor de ese ngulo. (Para a resolver este problema relacinelo con el de las diagonales de un cubo considerado en o la seccin 3.1.) o Comentario: Las puntas de los cuatro vectores forman los vrtices de un tetraedro e regular. La molcula de metano CH4 es un ejemplo de lo arriba planteado. En tal e molcula el tomo de carbono se encuentra al centro de los cuatro tomos de hidrgeno e a a o que estn distribuidos de la manera ms regular posible. a a

8.

Encuentre el ngulo entre dos vectores de 8 y 10 unidades de largo, si el vector suma a forma un ngulo de 50 con el mayor de ellos. Encuentre tambin la magnitud del a e vector suma. La suma de dos vectores mide 30 unidades y forma ngulos de 25 y 50 con ellos. a Cul es la magnitud de cada uno de los vectores? a Suponga que la posicin r de una part o cula en funcin del tiempo t viene dada por: o r = r (t) = r0 cos t t0 x + sin t t0 y ,

9.

10.

72

Cinemtica en dos y tres dimensiones a con t0 = 1 min y r0 = 3 cm. Qu trayectoria recorre la part e cula? Cunto tiempo a tarda la part cula en volver al punto de partida?

11.

Supongamos que la posicin r de una part o cula en funcin del tiempo t viene dada o por r = at x + (b ct2 ) y , con a = 2 m/s, b = 10 m y c = 9,8 m/s2 . Graque la trayectoria. Qu tipo de e trayectoria es? En qu instante la part e cula cruza el eje x?

12.

Un barco a vapor se dirige hacia el sur con una velocidad vb = 25 km/h en un a rea donde sopla un viento desde el suroeste con velocidad v0 = 18 km/h. Encuentre el ngulo 0 que forma el humo a emitido por el vapor con la direccin o nortesur (ver gura 3.15). Respuesta: 0 18, 64o

Figura 3.15 13. Considere un disco de radio R = 50 cm que rueda sobre una recta (el eje x ) con una velocidad angular = 2 s1 . Considere un punto P ubicado en el per metro del disco, y designe por r al vector que va desde el origen hacia el punto P . Encuentre una expresin pao ra r = r (t); suponga que en el instante t = 0 el punto P est en el origen. a

Figura 3.16

Haga un grco de r (t) para el intervalo t [0 s , 10 s ]. Cunto tarda la rueda en a a dar una vuelta completa?

14.

Una part cula recorre una trayectoria circular en el plano xy, cuyo radio es R = 5 m con una velocidad constante v0 = 15 m/s y en el sentido del reloj. Encuentre el vector posicin r(t), el vector velocidad v(t) y el vector aceleracin a(t) (en coordenadas o o cartesianas) si en el instante t = 0 la part cula se encuentra en r0 = 5. y

3.4 Problemas 15. Considere un disco de radio R en el plano xy. Sea el ngulo de un punto a ubicado en el borde del disco respecto al eje x. Suponga que el disco gira con una aceleracin angular constante o = 0 ). Encuentre la 0 (es decir, (t) velocidad y aceleracin de P en funcin o o del tiempo. Suponga que en el instante t = 0 el punto P se encontraba en reposo sobre el eje x.

73

Figura 3.17

16.

Estime (en m/s y km/h) la velocidad mxima con la que usted puede lanzar una a piedra.

17.

Una pelota sale rodando del descanso de una escalera con velocidad horizontal v0 = 1,52 m/s. Los escalones son de 20 cm de alto y 20 cm de ancho. Cul ser el primer a a escaln al que llegue la pelota? Dibuje una gura para ilustrar el problema. o

18.

Un can se encuentra a una distancia no D de un edicio. Encuentre el ngua lo de elevacin 0 y la velocidad v0 de o la bala de manera que el proyectil entre horizontalmente por la ventana que se encuentra a una altura h (ver gura 3.18). Figura 3.18

19.

Considere un r de ancho L en el cual o el agua uye con velocidad v0 . Un nadador recorre el trayecto A B A, mientras que un segundo nada el trayecto C D C (ver gura 3.19). Los puntos C y D estn ana clados jamente al fondo del r y la o separacin entre C y D es la misma o que entre A y B. Si ambos nadan con la misma velocidad v respecto al agua, quin ganar la carrera? e a

Figura 3.19

74 20. Un pato vuela horizontalmente en l nea recta con velocidad vp a una altura h. Un nio con una honda, n que puede disparar piedras con una velocidad v0 , hace uso de su arma en el instante que el pato lo sobrevuela. a) Cul es el ngulo respecto a a a la normal con el cual debe disparar la piedra? Qu distancia d alcanza a ree correr el pato antes de ser alcanzado por el proyectil?

Cinemtica en dos y tres dimensiones a

b)

Figura 3.20

(c) Cul es la velocidad m a nima que debe tener el proyectil para que ste llegue al e pato? 21. Se lanza un proyectil con cierto a ngulo de elevacin 0 . El alcance o del proyectil es R (ver gura 3.21). Si se desprecia el roce con el aire, demuestre que la trayectoria viene dada por la ecuacin o y(x) = tan 0 R x2 + x tan 0 . Figura 3.21 Note que esta ecuacin corresponde a una parbola. Demuestre tambin que el ngulo o a e a de la tangente en el punto x viene impl citamente dado por tan = 1 22. 2x tan 0 . R

Graque en papel polar la trayectoria de una part cula si su posicin en coordenadas o polares, en funcin del tiempo, viene dada por: o a) r(t) = r0 (t) = t/t0

con r0 = 1 [m] y t0 = 2 [s]. r(t) = At b) (t) = t/t0 con A = 1/(4) [m/s] y t0 = 2 [s]. r(t) = r0 + B cos (t/2t0 ) c) (t) = t/t0 con r0 = 1 [m], t0 = 2 [s] y B = 0,5 [m].

3.4 Problemas 23.

75

Una part cula se encuentra en el instante t = 0 en el lugar r(0) = 10 cm y tiene una y velocidad v(0) = 2 cm/s. La aceleracin en todo instante es x o a = G r , r3

con G=200 cm/s2 . Encuentre numricamente la trayectoria de la part e cula para t [0, 3,5 s]. Graque! Indicacin: programe las siguientes relaciones o r(t + ) v(t + ) r(t) + v(t) v(t) + a(t)

a(t + ) = Gr(t + )/r3 (t + ) .

24.

Calcule la mxima distancia que un a objeto puede alejarse del borde de un peldao para evitar ser alcanzado n por los objetos lanzados con velocidad v0 desde el punto A. La distancia desde A al borde del peldao es L y la altura n de ste es H. e Figura 3.22

25.

Un proyectil se lanza con velocidad inicial v0 y ngulo de lanzamiento , ama bos conocidos. El proyectil sobrepasa una barrera rectangular de ancho a conocido, pero altura h desconocida, rozando sus dos vrtices A y B (ver gue ra 3.23). Encuentre la distancia d que separa el punto de lanzamiento con la pared ms cercana al obstculo. Tama a bin encuentre la altura h de la barree ra.

Figura 3.23

26.

Una part cula tiene un vector posicin dado por r = 30 t x + (40 t 5 t2 ), donde o y r est en metros y t en segundos. Encuentre los vectores velocidad y aceleracin a o instantneas. a

76 27. Desde una distancia d del borde recto de un tobogn se dispara una a bengala. Si el tobogn tiene una ala tura h y un largo b, determinar ambas componentes de la velocidad inicial del proyectil para que ste atee rrice sobre el vrtice superior del toe bogn de manera que su velocidad a sea paralela al plano inclinado.

Cinemtica en dos y tres dimensiones a

Figura 3.24 Respuesta: v=d gb x + (2 b + d) 2 h (b + d) hg z . 2 b (b + d)

28.

Supongamos que r(t) y (t) son las coordenadas polares de un punto que se mueve en un plano. Demuestre que la velocidad de tal punto, en coordenadas cartesianas, viene dada por dr d dr d cos r sin x + sin + r cos y dt dt dt dt r cos r sin x + r sin + r cos y .

v(t) = =

Encuentre la velocidad en coordenadas cartesianas para los tres casos del problema 22.

29.

Una part cula tiene aceleracin constante o a = (6 x + 4 y )[m/s2 ] . En t = 0 la velocidad es cero y el vector posicin es x0 = 10 x [m]. o a) Encuentre los vectores velocidad y posicin en un instante t cualquiera. o b) Encuentre la ecuacin de la trayectoria en el plano y dibjela. o u

30.

De un can se disparan dos proyectiles: el primero con un ngulo de elevacin 1 = no a o 60 y el segundo con un ngulo de elevacin 2 = 45 . La velocidad de los proyectiles, a o al emerger del can es v0 = 250 m/s. Despreciando la resistencia del aire, encuentre no el intervalo de tiempo entre los dos disparos que asegure que los proyectiles choquen.

3.4 Problemas 31. La gura indica la conexin en una caja o de cambios de un automvil. Encuentre o la razn entre los radios de ambos eno granajes, que es la misma para ambos pares, si uno desea que en la primera marcha, con el motor a 2000 RPM, el auto tenga una velocidad de 30 Km/h. Por cada cinco vueltas en la salida de la caja de cambios, las ruedas, cuyo radio es de 50 cm, dan una vuelta. Consideremos una turbina hidrulica. a Supongamos que el agua ingresa a la turbina con una velocidad v, con v = |v| = 15 m/s, formando un ngulo con a la tangente al rotor en el punto de entrada = 30 (ver gura 3.26). Suponga adems que el radio externo del a rotor es R = 2 m y que, en su estado estacionario, el rotor gira a 30 RPM (o sea, con frecuencia = 0, 5 s1 ). La forma de las paletas de un rotor de una turbina hidrulica es tal que la vea locidad relativa entre el agua que ingresa a la turbina y la paleta en el punto de entrada, sea tangente a la paleta (de esta manera el agua ingresa a la turbina sin choques).

77

Figura 3.25

32.

Figura 3.26

Determine el ngulo entre la paleta del rotor y la tangente al rotor en el punto de a entrada de agua. Encuentre tambin la velocidad relativa vr del agua (respecto a la e paleta) en ese punto. Respuesta: tan = v sin ; v cos 2R vr = 10, 06 [m/s] .

33.

Una part cula se mueve en el plano xy con una velocidad (que depende de la posicin) o v = a+bx, donde a y b son constantes. En el instante inicial la part x y cula se encuentra en el origen (x(0) = y(0) = 0). Encuentre la ecuacin de la trayectoria y(x). o Respuesta: y(x) = b 2 x . 2a

78 34. Un mono est colgado a una altura a h de un rbol. Un cazador apunta a con una cerbatana directamente al mono desde una distancia d (ver gura 3.27). En el mismo instante en que el cazador sopla el dardo envenenado el mono se suelta del rbol. a Sobrevivir el mono? (Desprecie el a efecto de friccin del dardo con el o aire)

Cinemtica en dos y tres dimensiones a

Figura 3.27

35.

Una rueda gira en torno a un eje horizontal a 30 rpm (1 rpm = una revolucin por o minuto = 1 vuelta por minuto), de manera que su parte inferior queda a nivel del suelo, pero sin rozarlo. (O sea, la rueda gira sin rodar). Sobre el borde de la rueda se han adosado dos piedrecitas, en posiciones diametralmente opuestas. a) Suponga que cuando el dimetro que une a las piedras pasa por la posicin a o horizontal, stas se desprenden del borde, en forma simultnea (gura 3.28a), y e a una de ellas llega al suelo antes que la otra. Se observa que durante el intervalo de tiempo entre la llegada al suelo de una y otra piedra, la rueda da una vuelta completa. Determine el radio de la rueda. Qu ngulo debe formar la l e a nea que une a ambas piedras con la vertical para que, si las piedras se desprenden en esa posicin, lleguen al suelo al mismo o tiempo?

b)

Figura 3.28a

Figura 3.28b

3.4 Problemas 36. Un globo sonda es soltado desde la tierra y se aleja con velocidad constante en trayectoria recta la cual forma un a ngulo de 30 con la vertical. La velocidad del viento con respecto al suelo es de 10 [km/h], estable, hacia el norte. a) b) Calcule la velocidad del globo respecto al aire. Calcule el tiempo que tarda el globo en alcanzar una altura de 1 km con respecto al suelo. Figura 3.29

79

37.

Una rueda de radio 0,25 [m] ha estado girando en forma uniforme a razn de una o revolucin por segundo. En cierto instante la rueda es frenada y se detiene, uniforo memente, despus de haber girado media vuelta. Calcule la aceleracin tangencial y e o centr peta de un punto jo en el borde de la rueda cuando sta comienza a ser frenada. e Dos proyectiles son lanzados simultneamente desde el mismo punto a en un plano horizontal. Los proyectiles son lanzados con igual rapidez y con a ngulos con respecto a la horizontal y , respectivamente ( < ). Ambos proyectiles llegan al mismo punto en la horizontal pero a instantes diferentes. Demuestre que lo descrito es posible y encuentre la razn entre los tiempos o de llegada. (Expresar el resultado en trminos de ). e Un proyectil es lanzado desde un plano inclinado cuyo ngulo de inclinacin a o con la horizontal es . Si el proyectil es lanzado con rapidez v0 y con un ngua lo de eyeccin con respecto al plano o (ver gura 3.31), calcule el alcance D del proyectil a lo largo del plano. Figura 3.31

38.

Figura 3.30

39.

40.

El avix, una apetitosa ave del tiempo de las cavernas, desarroll por un proceso de o evolucin, una coraza en la parte inferior de su cuerpo de manera que los trogloditas o no pod cazarlas con arcos y echas. an

80

Cinemtica en dos y tres dimensiones a Og, un ingenioso troglodita, desarroll un mtodo para cazarla aprovechando que el u o e ave no tiene coraza sobre el dorso. El disparaba echas que impactaran al avix por arriba. Dados la velocidad del ave vave , la altura h a la que vuela, la velocidad v0 con que la echa es impulsada por el arco y el ngulo (respecto a la horizontal) con que el a troglodita dispara la echa, calcular: a) b) El tiempo que le toma a la echa pasar por la altura h la segunda vez. El valor de la distancia d entre el ave y la vertical por el punto de lanzamiento, en el instante del lanzamiento, para que la echa impacte al ave.

Figura 3.32

41.

Se lanzan dos proyectiles A y B de modo que tienen igual alcance horizontal L. A se lanza horizontalmente desde una altura h, que es igual a la altura mxima que a alcanza B durante su vuelo (ver gura 3.33) a) b) Calcule la razn entre los tiempos o de vuelo de A y B. Calcule la razn entre las compoo nentes horizontales de la velocidad de los proyectiles. Cul es la rapidez (magnitud de a la velocidad) de cada uno de ellos al llegar al suelo? Figura 3.33

c)

3.5 Solucin a algunos de los problemas o

81

3.5.

Solucin a algunos de los problemas o

Solucin al problema 18. o Coloquemos el origen en el lugar en que est ubicado el can y sean x y z los ejes horizontal a no y vertical, respectivamente. La posicin de la bala (siendo t = 0 el instante del disparo) o vendr dada por las coordenadas a y x(t) = v0 cos 0 t

1 z(t) = v0 sin 0 t gt2 . 2 La componente vertical de la velocidad de la bala ser a vz (t) = v0 sin 0 gt . Sea t el instante en que la bala penetra por la ventana. En ese instante deben cumplirse las relaciones v0 cos 0 t = D y 1 v0 sin 0 t gt2 = h . 2 La condicin de que la bala penetre en forma horizontal por la ventana exige que en t la o velocidad vertical de la bala sea nula. O sea, adems de las dos relaciones anteriores, debe a cumplirse que v0 sin 0 gt = 0 . Despejando t de la ultima relacin y reemplazndola en las dos anteriores se obtiene o a y
2 v0 sin 0 cos 0 = Dg

(1) (2)

2 v0 sin2 0 = 2hg .

Dividindo la ultima por la antepenltima se encuentra e u tan 0 = 2h . D

Esta relacin permite encontrar el ngulo de elevacin del disparo 0 . Para determinar el o a o valor de v0 elevamos al cuadrado la ecuacin (1): o
4 v0 sin2 0 (1 sin2 0 ) = D2 g 2 .

Despejando sin2 0 de (2), sustituyndolo en la ultima ecuacin se encuentra para v0 la e o expresin o (D2 + 4h2 )g 2 v0 = . 2h

82 Solucin al problema 30. o

Cinemtica en dos y tres dimensiones a

Sea xy el plano en que se mueven los proyectiles, z el eje que apunta hacia arriba y colo quemos el origen en el lugar en que se encuentra el can. no Sea t el tiempo transcurrido desde el disparo de la bala # 1. La posicin de esa bala viene o dada por 1 z1 (t) = v0 sin 1 t 2 gt2 x1 (t) = v0 cos 1 t . Sea t el tiempo transcurrido desde el disparo de la bala # 2. La posicin de la segunda o bala viene, anlogamente, dada por a z2 (t ) = v0 sin 2 t 1 gt 2 2 x2 (t ) = v0 cos 2 t . Para que las balas choquen deben coincidir las dos coordenadas de ambas balas, o sea, debe cumplirse cos1 t = cos2 t (3.3) y 1 1 (3.4) v0 sin 1 t gt2 = v0 sin 2 t gt 2 . 2 2 Despejando t de la primera de estas ecuaciones y reemplazndola en la segunda se obtiene a 1 cos 1 1 cos2 1 2 v0 sin 1 t gt2 = v0 sin 2 t g t . 2 cos 2 2 cos2 2 Luego dividimos por t, multiplicamos por cos 2 y reordenamos los trminos: e v0 (cos 2 sin 1 sin 2 cos 1 ) = gt (cos2 2 cos2 1 ) . 2 cos 2 (3.5)

Sea t el tiempo entre ambos disparos. Se tiene entonces que t = t t. Sustituyendo esto en (5.3) se encuentra que t= cos 2 cos 2 cos 1 t . (3.6)

Sustituyendo esta relacin a su vez en (5.6), se obtiene: o v0 (cos 2 sin 1 sin 2 cos 1 ) = o sea, t = 2v0 sin(1 2 ) g cos 1 + cos 2 11 s . g (cos2 2 cos2 1 ) t , 2 cos 2 cos 1

3.5 Solucin a algunos de los problemas o Solucin al problema 33. o

83

Sea r (t) = x(t) x + y(t) y la posicin de la part o cula. Derivando respecto al tiempo se encuentra su velocidad: v(t) = x(t) x + y(t) y . Por otra parte, de acuerdo al enunciado, sabemos que v(t) = a x + bx(t) y . Igualando ambas expresiones, componente a componente, obtenemos x(t) = a y y(t) = bx(t) . La primera de estas expresiones indica que, para la componente a lo largo del eje x, el movimiento es uniforme, o sea, x(t) = x(0) + at . Pero, de acuerdo al enunciado, x(0) = 0, luego x(t) = at. Sustituyendo esto en la ecuacin o para y(t) se encuentra y(t) = bat . De aqu se deduce que el movimiento a lo largo del eje y es uniformemente acelerado, luego 1 1 y(t) = y(0) + bat2 = bat2 . 2 2 De esta manera hemos encontrado que las coordenadas x e y de la part cula, en funcin del o tiempo, vienen dadas por x = at ab y = t2 . 2 Despejando t de la primera de estas ecuaciones y reemplazndolo en la segunda, se obtiene a nalmente la ecuacin de la trayectoria o y = y(x) = Solucin al problema 36. o a) Sea v0 la velocidad del globo respecto a un observador jo en la Tierra. La velocidad vertical y horizontal sern a v0 3 vz = v0 cos 30 = 2 y v0 vx = v0 sin 30 = , 2 b 2 x . 2a

84

Cinemtica en dos y tres dimensiones a

respectivamente. La componente horizontal de la velocidad del globo debe coincidir con la del viento, o sea, vx = v0 /2 = vv . De aqu se deduce que v0 = 2vv = 20 km/h. La componente vertical de la velocidad del globo es precisamente la velocidad con que ste se mueve respecto al aire (su movimiento horizontal se debe al viento). Esta velocidad e vertical viene dada por vz = v0 3/2 = 17, 3... km/h. b) Conociendo vz es fcil evaluar el tiempo t que demora el globo en alcanzar una altura a de h = 1 km. Este viene dado por t = h vz 1 [h] 17, 3 3, 46 [minutos] .

Solucin al problema 37. o Sea 0 la velocidad angular de la rueda antes de ser frenada: 0 = 2 s1 . Sea la aceleracin angular que sufre la rueda al ser frenada. Si t = 0 es el instante en que se aplica o el freno, se tiene que la velocidad angular vendr dada por a (t) = 0 + t , mientras que el ngulo que rotar la rueda ser a a a 1 1 (t) = (0) + 0 t + t2 = 0 t + t2 . 2 2 Sea t el tiempo que tarda la rueda en quedar en reposo. De acuerdo al enunciado del problema, debe cumplirse que (t ) = 0 y (t ) = , o sea, 1 = 0 t + t 2 2 y 0 + t = 0 .

De estas ecuaciones podemos despejar t y . En particular para la aceleracin angular se o obtiene 2 = 0 = 2 [s2 ] . 2 La magnitud de la aceleracin tangencial y centr o peta (ver seccin 3.3) vienen dadas por o at = R y ac = R 2 . Usando estas expresiones con R = 0, 25 [m] y = 0 = 2 s1 se encuentra que la aceleraciones tangencial y centr peta de un punto jo en el borde de la rueda, cuando sta comienza a ser frenada, son at = 1, 57 [m/s2 ] y ac = 9, 87 [m/s2 ]. e Solucin al problema 41. o a) Lo que A tarda en llegar hasta el suelo es igual a lo que demora B desde su punto mximo (ambos ah tienen una velocidad vertical nula). B demora lo mismo en subir que a en bajar, luego la razn entre los tiempos de vuelo de A y B es o tA 1 = . tB 2

3.5 Solucin a algunos de los problemas o

85

b) La velocidad horizontal de ambos proyectiles es constante. Ambos recorren la misma distancia horizontal y como B para ello demora el doble que A, se deduce que la velocidad horizontal de B debe ser la mitad de la de A. c) La velocidad vertical con que A y B llegan al suelo es la misma (la de una ca libre de da una altura h). Esta es vv = 2gh. El tiempo de ca de A es t = (2h/g). En ese tiempo da A avanza en direccin horizontal una distancia horizontal L. Como la velocidad horizontal o es uniforme se deduce que sta (para la part e cula A) debe ser vh = L/t = L g/(2h). La rapidez de A cuando llega al suelo es, por lo tanto, |vA (t )| =
2 2 vv + vh =

2gh +

L2 g . 2h

Para la part cula B la componente vertical de la velocidad es la misma, mientras que la componente horizontal es la mitad de la de A, o sea, |vB (t )| =
2 vv + (vh /2)2 =

2gh +

L2 g . 8h

86

Cinemtica en dos y tres dimensiones a

Cap tulo 4

Las leyes de Newton


En el presente cap tulo enunciaremos y analizaremos las as llamadas Leyes de Newton. Recurrir a estas leyes para formular la mecnica clsica presenta algunos inconvenientes, a a pues permite que se le hagan objeciones desde un punto de vista lgico. A pesar de estas o dicultades persistiremos en este camino, es decir, tomaremos las leyes de Newton como el punto de partida para el desarrollo de la mecnica. Las razones para ello son dos: por una a parte esta forma de proceder corresponde ms de cerca al desarrollo histrico y, por otra, a o tiene la ventaja de ser una formulacin menos abstracta que las otras alternativas. o Antes de enunciar las famosas leyes de Newton, debemos discutir algunos conceptos preliminares.

4.1.

Espacio y tiempo

En la Mecnica de Newtoniana se supone que las part a culas, como tambin los observadores, e viven en un espacio euclideano tridimensional. Eso signica, entre otras cosas, que la suma de los ngulos interiores de cualquier tringulo que imaginemos en este espacio, es siempre a a o . Otra caracter 180 stica de un espacio euclideano es, por ejemplo, que la suma de dos vectores de desplazamiento es conmutativa. Para darse cuenta como estos conceptos fracasan cuando el espacio es noeuclideano es util considerar el espacio bidimensional formado por la supercie de una esfera. Tal espacio es noeuclideano y en l se presentan varias situaciones curiosas. Por ejemplo, al viajar en e l nea recta en ese espacio, en algn instante uno vuelve al punto de partida. La suma de u los ngulos interiores de un tringulo dibujado sobre tal esfera es mayor a 180o y tambin a a e la suma de dos vectores es no conmutativa. El espacio que Newton usa para desarrollar la mecnica no slo es euclideano sino que tama o bin homogneo e istropo. Esto signica que todos los lugares del espacio son equivalentes e e o y que el espacio tiene las mismas propiedades en todas las direcciones. Para desarrollar la mecnica tambin es indispensable decir algo sobre el concepto de tiema e po. Newton us la suposicin de que: El tiempo matemtico, absoluto y verdadero uye, o o a

88

Las leyes de Newton

debido a su propia naturaleza, parejamente y en forma independiente a cualquier agente externo. Si bien la mayor de las personas sienten simpat por esta concepcin del tiempo, a a o hay que darse cuenta de que desde el punto de vista estrictamente lgico esta concepcin o o es insatisfactoria ya que sin el concepto tiempo la palabra parejamente no tiene signicado. No es fcil decir algo sobre la nocin tiempo que sea mejor o que clarique lo expresado por a o Newton, consecuentemente, no intentaremos hacerlo aqu Ms bien apelaremos a nuestra . a intuicin, experiencia y conocimiento sobre lo que es el tiempo: es algo que permea a todo o el espacio y avanza en forma homognea y continua, independiente de la posicin, del e o observador, de la velocidad independiente de cualquier cosa. El tiempo se mide usando relojes. Generalmente un reloj posee alguna caracter stica que hace que ste se comporte en forma peridica. Con la suposicin de que el tiempo transcue o o rrido entre dos repeticiones es siempre el mismo, podemos usar ese movimiento peridico o como reloj. Por ejemplo, el movimiento rotatorio de la tierra en torno al sol se usa para denir la unidad de tiempo llamada ao; el movimiento de la tierra en torno a su propio eje n puede usarse para denir d solar. Un pndulo, o una masa colgada de un resorte, tambin a e e puede usarse como reloj. Supongamos que un observador O tiene numerosos relojes idnticos a su disposicin, que e o los ha sincronizado y que tales relojes no modican su ritmo si se los aleja, cada uno de los dems. De esta manera el observador O puede tener en todos los lugares del espacio a relojes sincronizados con el que l posee. Para el observador O, dos eventos que ocurren e en lugares distintos, sern simultneos si los relojes ubicados en los dos lugares marcan a a la misma hora al ocurrir los eventos. Una consecuencia de la concepcin newtoniana del o tiempo es que si dos eventos son simultneos para un observador, tambin lo sern para a e a todos los dems observadores. En la mecnica newtoniana el concepto simultaneidad tiene a a una validez absoluta. Al comenzar con el estudio de la f sica es dif argumentar a favor o en contra de esta cil concepcin newtoniana del tiempo. Las experiencias vividas por la gran mayor de las o a personas, sugieren aceptar esta concepcin como vlida (o al menos plausible). o a Sealamos, sin embargo, que ms adelante (en futuros cursos) nos veremos forzados a n a abandonar este concepto intuitivo del tiempo. Y no solamente del tiempo; en algn momento u nos veremos obligados a revisar muchos otros conceptos que ya cre amos tener claramente establecidos. Pasamos a enunciar las leyes de Newton. Sin embargo, deseamos hacer notar desde la partida que las leyes de Newton slo sern aplicables a fenmenos que usualmente observamos en o a o nuestro mundo macroscpico; no son aplicables ni en el mundo microscpico, ni a fenmenos o o o que ocurren a escalas cosmolgicas. Las leyes de Newton tambin fracasan estrepitosamente o e al describir con ellas sistemas en que algunas (o todas) de las part culas se desplazan a velocidades comparables a la velocidad de la luz.

4.2.

Las leyes de Newton

Presentamos a continuacin los postulados fundamentales de la mecnica que Isaac Newton o a public en su libro Principia en 1687. o

4.2 Las leyes de Newton Primera ley: Cada cuerpo material persiste en su estado de reposo o de movimiento uniforme en l nea recta, a menos que una fuerza, que acta sobre el cuerpo, lo conmine u a cambiar de estado.

89

Qu realmente quiere decir esta ley, que se conoce tambin con el nombre de ley de inere e cia? En su redaccin aparece la palabra fuerza, luego para interpretar la ley de inercia o debemos apelar a nuestro conocimiento intuitivo sobre qu es una fuerza: una fuerza es lo e que hacemos, por ejemplo, al usar nuestros msculos para empujar un objeto. La primera u ley entonces establece que cualquier cuerpo material, al que nadie ni nada empuja o tira, se trasladar con una velocidad constante (es decir, se mover en l a a nea recta con una rapidez uniforme). Si la velocidad es cero, o sea, el cuerpo est en reposo, continuar en reposo. a a Consideremos ahora un observador O que observa una part cula sobre la cual no actan u fuerzas. Si el observador O, mientras observa, realiza saltos mortales, la part cula no le parecer estar movindose con velocidad constante. Slo si el sistema de referencia que usa a e o O para observar a la part cula satisface ciertas condiciones, el cuerpo se mover (para O) a con velocidad constante. O sea, la primera ley de Newton es vlida slo si el movimiento a o del cuerpo se observa desde ciertos sistemas de referencia bien particulares. Tales sistemas de referencia se llaman inerciales. En otras palabras, la primera ley de Newton en realidad no es otra cosa que la denicin de un sistema inercial. o Para enunciar la segunda ley debemos denir previamente el concepto de cantidad de movimiento o momentum de una part cula. El momentum de una part cula es el producto de la masa de la part cula por su velocidad. Como el producto de un escalar (la masa) por un vector (la velocidad), es un vector, el momentum de una part cula es un vector: p = mv . La masa m de un cuerpo ser una magnitud que es proporcional a su peso, es decir, proa porcional al esfuerzo que es necesario realizar para levantarlo o suspenderlo. Si un cuerpo pesa ms que otro, esto se debe a que el primero tiene una masa mayor que el segundo. a La unidad de masa en el sistema internacional de unidades SI es el kilgramo, y corresponde o a la masa del kilgramo patrn guardado en una ocina en Par Sin embargo, para la o o s. mayor de los efectos prcticos podemos denir a un kilgramo como la cantidad de masa a a o que posee un litro de agua dulce. Una hiptesis que se hace en la mecnica newtoniana es que la cantidad de materia no camo a bia. Efectivamente, nuestra experiencia nos muestra que, por ejemplo, si hacemos colisionar dos relojes de manera que ellos se desintegren, la masa de todos los fragmentos y partes seguir siendo igual a la de los dos relojes originales. Otro ejemplo, al agregarle un litro de a agua a un balde de arena seca encontraremos que la arena mojada pesar ahora un kilgraa o mo ms que cuando la arena estaba seca. Esta hiptesis, de que la masa de un sistema a o cerrado no cambia, pareciera estar bien fundamentada por numerosas observaciones. Pasamos a enunciar la segunda ley de Newton. Segunda ley:

90

Las leyes de Newton El cambio de momentum p de una part cula es proporcional a la fuerza neta que acta sobre el cuerpo, como tambin al intervalo t durante el cual ella se u e aplica, y apunta en la direccin y sentido de esta fuerza, o sea, o p = F t .

Como primer comentario es necesario decir que esta ley slo es vlida si la fuerza F es o a constante durante el intervalo t y si las magnitudes son observadas desde un sistema de referencia inercial. La segunda ley debemos considerarla como denicin del concepto fuerza. Si sobre una o part cula acta una fuerza durante un cierto intervalo de tiempo t, necesariamente camu biar su velocidad (y por consiguiente tambin su momentum). La fuerza media que acta a e u sobre la part cula durante el intervalo t es el cuociente entre el cambio de momentum y el intervalo de tiempo: p F = . t La fuerza instantnea se obtiene en el l a mite t0, o sea, viene dada por F dp . dt

Note que la fuerza tambin es una magnitud vectorial. e Si la masa de una part cula no var a medida que transcurre el tiempo, entonces a F = dp d( mv ) dv = =m = ma . dt dt dt

En palabras, la fuerza neta que acta sobre una part u cula es igual al producto de su masa y su aceleracin. o Si la masa se mide en kg y la aceleracin en (m/s2 ), entonces la fuerza viene dada en o Newtons (N). O sea, por denicin, en el sistema de unidades SI o 1 N 1 kg 1 m . s2

Tercera ley: Si un cuerpo A ejerce una fuerza sobre otro B, entonces este ultimo ejercer so a bre A una fuerza de igual magnitud y en la misma direccin, pero en sentido o opuesto. De acuerdo a la tercera ley, una fuerza nunca aparece en forma solitaria, sino que siempre vendr acompaada de otras fuerzas, de manera que la suma vectorial de todas ellas sea a n nula. Es importante sealar que estas fuerzas, denominadas de accin y reaccin, actan n o o u siempre sobre objetos diferentes. O sea, la suma vectorial de todas las fuerzas que actan u

4.3 Uso de las leyes de Newton

91

sobre un cuerpo no necesariamente tiene que ser nula. Para que sobre un cuerpo pueda actuar una fuerza neta no nula es necesario que exista al menos un segundo cuerpo. A pesar de que no se menciona expl citamente, al aplicar la tercera ley se supone que la accin y reaccin aparecen en forma simultnea. Como dos cuerpos pueden interactuar o o a a distancia (por ejemplo, a travs de la interaccin gravitacional), el ultimo comentario e o implica que en la mecnica newtoniana debe existir una manera de transmitir la informacin a o de un cuerpo a otro con una velocidad innita. En la naturaleza tales velocidades innitas no existen; hoy en d sabemos que la velocidad de la luz en el vac es un l a o mite superior para las velocidades con que se puede trasladar algo material o informacin de un lugar a o otro. Por esta razn, la tercera ley es generalmente una muy buena aproximacin, pero no o o tiene una validez universal; por ejemplo, en colisiones atmicas no es siempre aplicable. o

4.3.

Uso de las leyes de Newton

Para aprender a manejar las leyes de Newton y comprender su signicado, lo mejor es ilustrar su uso en algunas situaciones concretas. Ejemplos: 1. Analicemos las fuerzas que actan sobre un cuerpo que cae. u Debido a la atraccin gravitatoria, todo objeto sufrir una fuerza que apunta hacia o a el centro de la tierra. Es esta fuerza la que acelera al cuerpo durante su ca da. Cul es el tamao de esta fuerza? Sabemos que al realizar experimentos con cuerpos a n sobre la supercie terrestre, al soltarlos todos ellos caen con la misma aceleracin o hacia la supercie. Esta aceleracin constante, llamada aceleracin de la gravedad, se o o denota por g, y su valor es aproximadamente g = 9,81 m/s2 . (En realidad, al realizar estos experimentos hay que asegurarse de que los efectos de la densidad y viscosidad de la atmsfera sean despreciables. Ms an, el experimento debe realizarse sin alejarse o a u demasiadoa lo ms unas pocas decenas de kilmetrosde la supercie terrestre.) a o Conociendo la aceleracin del cuerpo y su masa m podemos (usando la segunda o ley de Newton) establecer cul es la fuerza gravitacional que acta sobre el cuerpo. a u Deniendo al vector unitario z como un vector que apunta hacia arriba, el vector aceleracin del cuerpo vendr dado por a = g. La fuerza sobre el cuerpo es entonces o a z F = m (g z ) = mg . z A la magnitud de esta fuerza gravitacional es lo que se llama peso del objeto. Usando la letra W para denotar al peso se tiene |F | W = m g = peso del objeto .

92 2.

Las leyes de Newton Analicemos las fuerzas que actan sobre un libro de masa M , en reposo sobre una u mesa (supercie horizontal). Ya sabemos que sobre el libro acta una fuerza, debido a la gravedad terrestre, que u es W = M g . z Por otra parte, debido a que el libro se encuentra (y se mantiene) en reposo, la fuerza neta sobre el libro debe ser nula. Quin o qu ejerce otra fuerza, igual a W , sobre e e el libro? La respuesta es: la mesa. Efectivamente, el libro se apoya sobre la mesa y la supercie de ella ejerce sobre el libro una fuerza hacia arriba, llamada reaccin, cuya o magnitud es igual al peso del libro. Introduzcamos los as llamados diagramas de cuerpo libre: Al analizar las fuerzas que se ejercen sobre un cuerpo es conveniente aislarlo del resto de los objetos que interactan con l. Para ello cada objeto que u e interacta con este cuerpo es sustituido por una fuerza que cumple con la u tercera ley de Newton. El resultado de esta operacin es el as llamado o diagrama de cuerpo libre del objeto. Para el caso del libro, la interaccin de o ste con la tierra se reemplaza por el e vector W que apunta hacia abajo y cuya magnitud coincide con el peso del libro; el efecto de la mesa sobre el libro se reemplaza por el vector R, (ver gura 4.1). Si el libro se mantiene en reposo, la segunda ley de Newton requiere que W + R = 0.

Figura 4.1

3.

Consideremos un objeto de masa m que cuelga del techo sujetado por una cuerda ideal (ver gura 4.2). Cul es la fuerza que la cuerda ejerce sobre el gancho en el a techo y cul es la tensin de la cuerda? a o Una cuerda ideal es una cuerda que, a menos que se especique lo contrario, no tiene masa, es perfectamente exible y no es extensible. Que una cuerda sea perfectamente exible quiere decir que slo es capaz de transmitir una fuerza a o lo largo de ella; no puede ejercer fuerzas transversales.

Figura 4.2

Sobre el objeto actan dos fuerzas; una es el peso W = mg y la otra es la fuerza u z F1 ejercida por la cuerda. Como el objeto no acelera, la fuerza neta (es decir, la suma de todas las fuerzas que actan sobre l) debe ser nula. Por consiguiente, F1 = W . u e

4.3 Uso de las leyes de Newton

93

Denotemos por F1 a la fuerza ejercida por el objeto sobre la cuerda. Debido al principio de accin y reaccin, F1 = F1 . La cuerda por otra parte ejerce una fuerza F2 o o sobre el gancho (la direccin de esta fuerza es hacia abajo). A su vez, el gancho o ejercer una fuerza F2 sobre la cuerda. Nuevamente debido al principio de accin y a o reaccin, F2 = F2 . o Ahora, debido a que la cuerda no tiene masa, las unicas fuerzas que actan sobre ella u sern F1 y F2 . Al estar en equilibrio (la cuerda no acelera), la suma de ambas fuerzas a debe ser cero, luego F2 = F1 . Resumiendo, tenemos que mg = W = F1 = F1 = F2 = F2 , z o sea, la fuerza F2 que la cuerda ejerce sobre el gancho es igual al peso mg. z Cada uno de los extremos de la cuerda ejerce una fuerza sobre los objetos a los cuales est unida. Cuando la masa de la cuerda es nula, la magnitud de esa fuerza es la a misma. A esta magnitud se le llama tensin de la cuerda. A lo largo de una cuerda o ideal, que no tiene masa, la tensin no var Para la cuerda del presente problema, o a. la tensin es = mg. La tensin es un escalar. o o 4. Mquina de Atwood. a Consideremos dos masas m1 y m2 unidas por una cuerda ideal sin masa que pasa sobre una polea ideal (ver gura 4.3). Deseamos encontrar la aceleracin de las masas o y las tensiones de las cuerdas. Con la expresin polea ideal nos estamos o reriendo a una polea que no tiene masa y gira sin roce. El objetivo de la polea es simplemente cambiar la direccin de la o cuerda y, por lo tanto, de la fuerza (que acta siempre a lo largo de la cuerda). La u tensin a la que est sometida una cuero a da no se modica al pasar por una polea ideal. Sea la tensin de la cuerda que une amo bas masas y a1 = a0 z la aceleracin que o sufrir la masa 1. La fuerza neta que acta a u sobre la masa 1 es (m1 g + ), luego, de z acuerdo a la segunda ley de Newton (m1 g + ) = m1 a1 = m1 a0 z . z

Figura 4.3

De esta relacin se deduce que o m1 g = m1 a0 . (4.1)

94

Las leyes de Newton Debido a que la cuerda es inextensible, la aceleracin que sufrir la masa 2 es la o a opuesta a la de la masa 1, o sea, a2 = a0 z . Aplicando la segunda ley de Newton a la segunda masa se obtiene la expresin o m2 g = m2 a0 . De las ecuaciones (4.1) y (4.2) podemos despejar las dos incgnitas a0 y : o =2 y a0 = m1 m2 g. m1 + m2 m1 m2 g m1 + m2 (4.2)

Como la polea no tiene masa y sta no sufre aceleraciones, la tensin de la cuerda que e o la sujeta deber ser igual a 2 . a Casos particulares: Si m1 = m2 , entonces a0 = 0 y = m1 g = m2 g. Tal como era de esperarse, si las masas son iguales, ninguna de ellas acelera. Si m1 > m2 entonces a0 resulta ser una magnitud negativa. Esto quiere decir que a1 = a0 z es una aceleracin que apunta hacia abajo; tal como debe ser, la masa 1 o baja, mientras que la masa 2 sube. Si m1 es muy parecida a m2 , entonces |a0 | g. O sea, cada una de las masas realizar un movimiento uniformemente acelerado, pero con una aceleracin mucho a o menor que g. Si m1 = 0, entonces a0 = g y = 0. En este caso la cuerda deja de tener tensin, y o por consiguiente la part cula 2 caer con aceleracin g. a o 5. Considere una cuerda exible de masa M que cuelga entre dos paredes, siendo el a ngulo que forma la cuerda con la pared (ver gura 4.4). Se desea encontrar la tensin que la cuerda tiene en el punto m o nimo. Para resolver el problema consideremos como nuestro sistema slo la mitad deo recha de la cuerda. Hay tres fuerzas que actan sobre ese sistema: u i) El peso W = 1 M g. z 2 ii) La fuerza F1 ejercida por la parte izquierda de la cuerda. La magnitud de esta fuerza es igual a la tensin de la cuerda en el m o nimo, que llamaremos 0 . Se tiene que F1 = 0 x .

Figura 4.4

4.3 Uso de las leyes de Newton

95

iii) La fuerza que ejerce el gancho sobre la cuerda. Como la cuerda es exible la fuerza necesariamente es a lo largo de la tangente de la cuerda. Si a la magnitud de esta fuerza la llamamos f0 , se tiene que F2 = f0 cos z + f0 sin x. Como nuestro sistema est en equilibrio (no acelera), la suma de las tres fuerzas debe a ser nula: 1 W + F1 + F2 = M g 0 x + f0 cos z + f0 sin x = 0 . z 2 Pero para que un vector sea cero es necesario que cada una de sus componentes sea nula. Este hecho nos da las siguientes ecuaciones: componente z: y componente x: 0 + f0 sin = 0 . De estas dos ecuaciones podemos despejar 0 y f0 , obtenindose e 1 0 = M g tan 2 y f0 =
2 0 +

1 M g + f0 cos = 0 2

Mg 2

Notemos cmo para 90 , o sea, a medida que la cuerda se cuelga en forma ms o a tirante, la tensin de la cuerda tiende a innito. o 6. Consideremos una masa m que gira en el plano x, y, en un c rculo de radio R y con una velocidad angular constante, 0 . Encontremos la fuerza neta a la que est soa metida la masa. En la seccin 3.3 ya analizamos el moo vimiento circular y demsotramos que la aceleracin de la masa m viene dada por o 2 a(t) = R0 r. De acuerdo a la tercera ley de Newton, el hecho que la masa m est acelerada implica que sobre ella e est actuando una fuerza neta a
2 F = ma = Rm0 r .

Figura 4.5

Esta fuerza (de magnitud constante) apunta hacia el origen y por esta razn se le o denomina fuerza centr peta.

96

Las leyes de Newton Debido a la importancia de este resultado lo reiteramos: Una masa m que realiza un movimiento circular uniforme, est sometida a una fuerza que apunta hacia el centro a de giro. La magnitud de esta fuerza centr peta es
2 Fcent = mR0 =

mv 2 , R

donde R es el radio del c rculo, 0 la velocidad angular y v = 0 R el mdulo de la o velocidad de la part cula.

4.4.

Roce cintico y esttico e a

Si un cuerpo se desliza sobre otro, tarde o temprano se detendr a menos que exista una a fuerza externa que perpete el movimiento. La fuerza que se opone al deslizamiento relativo u entre los dos cuerpos se denomina fuerza de roce cintico. Se origina en la interaccin de e o ambas supercies en contacto. La fuerza de roce no slo aparece cuando dos cuerpos estn en movimiento relativo, sino o a que tambin puede estar presente cuando los dos cuerpos se encuentran en reposo relativo. e En efecto, si, por ejemplo, intentamos deslizar una mesa por el piso, notamos que aparece una fuerza que impide que este deslizamiento comience. A esta fuerza se le denomina fuerza de roce esttico. a Tambin existen otras fuerzas de roce que aparecen en diversas circunstancias (por ejemplo, e el roce rodante, el roce viscoso, etc), sin embargo, en el presente cap tulo centraremos nuestro inters en las fuerzas de roce cintico y esttico. e e a Se sabe relativamente poco acerca de ambos y es dif cuanticarlos porque dependen de cil la naturaleza de los materiales y de propiedades de la supercie como el pulido, la existencia de xidos en la interfase, etc. Lo que diculta an ms la cuanticacin de la fuerza de roce o u a o es su dependencia de la historia de las supercies: el paso del roce esttico al roce dinmico a a depende de si las supercies se han deslizado previamente o no. Las fuerzas de roce tienen un origen microscpico. Dos supercies, por suaves que parezcan, o a nivel microscpico tienen irregularidades. Estas protuberancias forman, en algunos casos, o microsoldaduras, y son el origen de la fuerza adicional que uno debe aplicar para poder iniciar un movimiento relativo entre los cuerpos. Una vez que stos estn en movimiento, e a estas aristas microscpicas se enganchan unas con otras y dan origen al roce cintico o e (tambin a veces llamado roce cinemtico o roce dinmico). e a a A continuacin presentamos algunos resultados fenomenolgicos y cualitativos sobre del o o roce. Estos resultados no son leyes fundamentales de la naturaleza, sino slo conclusiones o generales que fueron obtenidas despus de numerosos estudios experimentales. e Consideremos un bloque de masa M que descansa sobre una supercie, el cual intentamos deslizar aplicando sobre l una fuerza horizontal F , que incrementamos paulatinamente. e Designemos por f a la fuerza de roce que aparece debido a la friccin entre las dos supercies o y describamos la forma en que t picamente var esta fuerza. a a) Mientras la fuerza horizontal externa F = |F | var desde 0 hasta un cierto valor a (max) fe , el bloque M no se deplazar. Como no hay aceleracin, la fuerza neta horia o

4.4 Roce cintico y esttico e a

97

zontal sobre el cuerpo debe ser nula, o sea, debe haber otra fuerza horizontal sobre el bloque que exactamente cancele a la fuerza F . Esta es la fuerza de roce esttica f . a Se tiene, por lo tanto, que f = F . b) Cuando la fuerza horizontal externa F sobrepasa cierto valor fe , la fuerza de roce no sigue aumentando. Como ahora la componente horizontal de la fuerza neta no es nula, el bloque comenzar a acelerar. Tan pronto como los cuerpos se deslizan con a cierta velocidad relativa, la fuerza de roce se vuelve constante, siendo su magnitud max algn valor fc (menor que fe ) y su sentido opuesto al movimiento relativo. u De ah en adelante, si se desea mantener el bloque deslizndose con una velocidad a constante, debe aplicarse una fuerza horizontal de exactamente la magnitud fc , en la direccin de movimiento. o Este comportamiento fenomenolgio co recin descrito, que muestra la e fuerza de roce, se muestra en la gura 4.6. Emp ricamente se ha observado que, para dos supercies (secas) en contacto, tanto la fuerza de friccin dinmica fc como el mxio a a (max) mo de la friccin esttica fe o a , son proporcionales a la fuerza normal entre ambas supercies, o sea, fc = c FN y
(max) fe = e FN . (max)

Figura 4.6

FN es la fuerza normal entre las supercies (es decir, perpendicular a la interfase formada por las dos supercies) y c y e son los coecientes de friccin. Los coecientes de friccin o o de alguna manera engloban nuestra ignorancia de los distintos parmetros que intervienen a en el problema. Siempre se tiene que el coeciente de roce cinemtico es menor al coeciente a de roce dinmico: c < e . Ambas fuerzas de roce actan en la direccin paralela a las a u o supercies. El sentido de la fuerza de roce esttico es opuesto a la fuerza horizontal neta a que acta sobre el cuerpo, mientras que el sentido de la fuerza de roce dinmico es siempre u a opuesto al movimiento relativo (y no a la fuerza) entre las dos supercies. Ilustremos los conceptos anteriores con un ejemplo. Problema: Considere el montaje experimental mostrado en la gura 4.7. Supongamos que los coecientes de friccin esttico y cinemtico entre la masa M =4 Kg y el plano inclinado son o a a e = 0,4 y c = 0,3, respectivamente.

98 Qu rango de valores puede tener m para e que el sistema se encuentre en equilibrio esttico? Si la masa m justo sobrepasa ese a mximo, con qu aceleracin se mover el a e o a bloque sobre el plano?

Las leyes de Newton

Solucin: o Resolvamos primero el problema esttico. a La gura 4.8 muestra el diagrama de cuerpo libre del bloque que se encuentra sobre el plano inclinado. A priori no sabemos en que sentido apunta la fuerza de roce fr . La hemos dibujado apuntando a lo largo del plano hacia abajo; si despus de realizar e el clculo fr resulta tener un valor negatia vo entonces la fuerza de roce en realidad apunta en el sentido opuesto al mostrado en la gura. Sea M g el peso, la fuerza ejercida por la tensin de la cuerda y FN o la fuerza normal que ejerce el plano inclinado sobre el bloque. Debido al principio de accin y reaccin, FN tambin coincide o o e con la magnitud de la fuerza que el bloque ejerce sobre el plano.

Figura 4.7

Figura 4.8

Introduzcamos un sistema de coordenadas cartesianas en que el eje x es paralelo y el eje y normal al plano inclinado (ver gura 4.8). Como el bloque est en reposo, la fuerza neta a sobre el bloque debe ser nula, esto es, tanto la fuerza total a lo largo del eje x como a lo largo del eje y . Esto nos da las siguientes ecuaciones: eje x: eje y : M g sin fr = 0 FN M g cos = 0 ,

donde es el ngulo de elevacin del plano inclinado. Como la masa m no acelera, la tensin a o o de la cuerda debe ser = mg. Luego, de la primera ecuacin se deduce que o fr = mg M g sin . Recordemos que fr puede ser positivo o negativo: fr es positivo si m > M sin y negativo si m < M sin . Tambin se tiene que e |fr | e FN = e M g cos . De las ecuaciones anteriores se deduce que mg M g sin = +fr e FN = e M g cos mg + M g sin = fr e FN = e M g cos si m > M sin si m < M sin .

4.4 Roce cintico y esttico e a o sea el bloque de masa M no se desliza sobre el plano inclinado si i) para M sin < m , se cumple que m M (e cos + sin ), ii) para M sin > m , se cumple que m M (sin e cos ).

99

Para los valores numricos del enunciado, el bloque no se deslizar por el plano si 0.61 kg e a < m < 3.4 kg. Analicemos ahora lo que sucede si m sobrepasa (en una magnitud innitesimal) al valor M (e cos + sin ). En ese caso, el bloque comenzar a deslizarse hacia arriba. La fuerza a de roce, por lo tanto, ser a fr = c M g cos x . La fuerza neta sobre el bloque y su aceleracin, en la direccin x, vendrn dados por o o a Fx = fr M g sin = c M g cos M g sin . y Fx = g(c cos + sin ) . M M Por otra parte, la fuerza neta sobre la masa m y su aceleracin en la direccin vertical, o o sern a F = mg . ax = y F = g = ax . m m La ultima igualdad en la ecuacin anterior se debe a que la cuerda es inextensible; por o consiguiente, cuando el bloque acelera hacia arriba, la masa m acelerar con la misma a magnitud, pero hacia abajo. De las ecuaciones anteriores se deduce que a = ax = g
m M

c cos sin m M +1

Este resultado tambin lo podemos escribir de otra manera. Recordemos que m sobrepasa e en una magnitud innitesimal al valor M (e cos + sin ), luego m = M (e cos + sin ) , m = e cos + sin . M Sutituyendo esto en la expresin para ax se obtiene o ax = g (e c ) cos . 1 + e cos + sin o sea,

Con los valores numricos del enunciado se obtiene ax 0,047 g. e Note que la tensin de la cuerda es distinta en el caso estacionario que en el caso dinmico. o a En el primer caso es = mg, mientras que en el segundo viene dada por = m(g ax ).

100

Las leyes de Newton

4.5.
1.

Problemas
Un automvil de 2000 kg movindose a 80 km/h puede llevarse al reposo en 75 m o e mediante una fuerza de frenado constante: a) b) Cunto tiempo tardar en detenerse? a a Cul es la fuerza necesaria para detener el coche en esa distancia? Quin o a e qu ejerce esa fuerza horizontal que detiene al coche? e

2.

Una carga de 2 toneladas se levanta mediante una gra. u a) b) Inicialmente, durante cierto intervalo de tiempo, la carga sube con una aceleracin a = 1,3 m/s2 . Cul es la tensin del cable que la soporta? o a o Despus de un breve per e odo de aceleracin, la carga sigue elevndose con una o a velocidad constante. Cul es la tensin del cable en ese caso? a o

3.

Dos bloques unidos por una cuerda que pasa por una polea sin rozamiento, descansan sobre planos lisos como se muestra en la gura 4.9. a) En qu sentido se mover el e a sistema? b) c) Cul es la aceleracin de los a o bloques? Cul es la tensin de la cuera o da? Figura 4.9

4.

Una pelota de 2 kg cae libremente llegando, en cierto instante, a tener una rapidez de 6 m/s. Qu fuerza vertical constante se debe aplicar para detenerla en los prximos e o 5 m? Qu fuerza vertical constante se debe aplicar para detenerla en los prximos 5 e o s?

5.

Qu fuerza F debe aplicarse al carro e de masa M (ver gura adjunta) para que el carro de masa m2 no suba ni baje? Respuesta: F = g (M + m1 + m2 ) m2 m1

Figura 4.10

4.5 Problemas 6. Considere un pndulo que consiste en e una masa m colgada de un hilo de largo . En presencia de un campo gravitacional constante, al sacar el pndulo e de su posicin de equilibrio y soltarlo, o ste oscilar. Encuentre la aceleracin e a o de la masa m en el instante en que el pndulo forma un ngulo con la nore a mal. Si 1, demuestre que d2 (t) 2 + 0 (t) = 0 , dt2 con 0 = g/ . Figura 4.11

101

7.

Considere una masa m adosada a un resorte de constante de restitucin k. Sea x = 0 o la posicin de equilibrio del sistema. De acuerdo a la Ley de Hook, al desplazar la o masa m una distancia x desde su posicin de equilibrio, la fuerza ejercida por el o resorte sobre la masa es F = kx. Demuestre que d2 x(t) 2 + 0 x(t) = 0 , dt2 con 0 = k/m. Compare este resultado con el del problema anterior.

8.

Un cuerpo de 500 g desliza por un plano inclinado liso. El cuerpo parte del reposo y durante el tercer segundo recorre una distancia de 120 cm. Encuentre el ngulo de a inclinacin del plano. o

9.

Una esfera de masa m es mantenida en la posicin A por dos cuerdas o (ver gura 4.12). Sea TA la tensin o de la cuerda indicada. Se corta la cuerda horizontal y el pndulo oscie la hasta la posicin B. Cul es la o a razn de las tensiones TB /TA ? o Respuesta: TB /TA = cos2 . Figura 4.12

102 10. Considere el montaje mostrado en la gura 4.13, con M =1,650 kg, m=0,150 kg y d0 =4 m. El sistema est en reposo cuando d = d0 = 4 m. a Cunto tiempo transcurrir antes a a de que la masa m llegue a la base de M?

Las leyes de Newton

Figura 4.13 11. Un objeto se encuentra sobre un plano liso sin roce y es sometido a una fuerza F que var en funcin del a o tiempo de acuerdo al grco que se a acompaa. Si la masa del objeto es n m, obtenga y graque las siguientes magnitudes: Figura 4.14 a) b) c) 12. Aceleracin del objeto en funcin del tiempo. o o Velocidad del objeto, si ste parte del reposo. e Posicin del objeto en funcin del tiempo. o o

Una pesa calibrada en Newtons se coloca sobre una plataforma mvil y se hace deslizar o con una rapidez constante de 14 [m/s] sobre un terreno ondulado (ver gura 4.15). Sobre la pesa se coloca una caja que pesa 500 [N]. a) b) Cuando la plataforma pasa sobre la cresta de una colina con radio de curvatura de 100 [m], cul es la lectura de la pesa? a Cuando la plataforma pasa por la parte inferior de una hondonada con radio de curvatura de 80 [m], cul es la lectura de la pesa? a

Figura 4.15 Respuesta: (parte b) 625 [N].

4.5 Problemas 13. Un bloque de masa M es tirado hacia una muralla vertical mediante el uso de una cuerda y poleas como se muestra en la gura. El bloque se desliza sin roce sobre la supercie. La fuerza con que se tira la cuerda es F , el largo de la cuerda es 2L y la separacin inicial entre el bloo que y la muralla es L. Determine el tiempo que transcurre hasta que se encuentren la punta de la cuerda y el bloque. Un plato cnico de ngulo caraco a ter stico gira uniformemente entorno a su eje, el cual se mantiene en posicin vertical. Una piedrecilla o de masa m rota solidariamente con el plato. Suponiendo que no hay roce entre la piedrecilla y la supercie del plato, calcule el radio de la rbio ta circular que describe la piedrecilla.

103

Figura 4.16

14.

Figura 4.17

15.

Una persona se para sobre una balanza dentro del ascensor y observa que sta registra e un peso igual a un 70 % de su peso normal. Si el ascensor y el pasajero tienen masas M y m respectivamente, calcule la tensin a la que est sometido el cable que sujeta o a el ascensor. Compare esta tensin con la que se producir si el ascensor acelera con o a la misma magnitud pero en sentido opuesto. Considere el montaje mostrado en la gura 4.18. Suponga que las masas de la polea y del hilo, as como el roza miento son despreciables. Se conocen las masas m, M y el ngulo de la cua. a n Encuentre la aceleracin de la cua. o n mg sin . a= Respuesta: M + 2m(1 cos ) Figura 4.18 Dos masas m y M se encuentran unidas por una cuerda de masa despreciable y largo . En estas condiciones ambas realizan un movimiento circular uniforme (en un plano horizontal) en torno al as llamado centro de masas del sistema. Suponga que el per odo del movimiento rotatorio es T . Encuentre la distancia entre la masa m y el centro de giro (para resolver esta parte del problema no es necesario conocer la denicin de centro de masas). Calcule la tensin de la cuerda que une ambas masas. o o

16.

17.

104 Respuesta: = mM m+M 2 T


2

Las leyes de Newton

18.

Una cua lisa de masa M se desliza n bajo la accin de una fuerza horizontal o F . Sobre ella se coloca un bloque de masa m. a) Dibuje todas las fuerzas que actan sobre cada una de las u masas. Determine el valor de F para que el bloque ms pequeo no a n resbale sobre la cua. n

Figura 4.19

b)

19.

Dos bloques idnticos y de masa m e posan sobre una supercie horizontal pulida. Uno de ellos es tirado mediante una cuerda en cuyo extremo libre se aplica una fuerza horizontal igual a M g. El otro bloque es tambin tirado e horizontalmente mediante una cuerda pero en cuyo extremo libre cuelga una bola de masa M . Determine cual de los bloques se mueve ms rpido si a a ambos parten del reposo simultneaa mente. Un pintor que pesa 900 Newtons trabaja en una silla colgante en un edicio de altura. Al terminar su turno debe volver al ultimo piso para bajar a la calle. Para subir con la silla tira de la cuerda de tal forma que la fuerza que l e ejerce sobre el asiento de la silla es de 500 Newtons. La silla misma pesa 300 Newtons. a) b) Cul es la aceleracin del pintor a o y de la silla? Cul es la fuerza total sobre el a soporte de la polea? a = 2g/3 ; b) Ftot = 2000N.

Figura 4.20

20.

Figura 4.21

Respuestas: a)

4.5 Problemas 21. Considere el montaje mostrado en la gura 4.22. La masa del cuerpo # 1 es n = 4 veces mayor que la del cuerpo # 2. Suponga que las masas de las poleas y de los hilos, as como el rozamien to son despreciables por su pequeez. n Cuando el cuerpo # 2 se suelta, la masa # 1 se encuentra a una altura h. Cul es la aceleracin de la masa # 2 a o mientras m1 baja? Cul es la altura a mxima del suelo H a la que subir la a a masa # 2? (La altura mxima no es a 2h!) Respuesta: H = 6hn/(n + 4) .

105

Figura 4.22

22.

Una masa m se encuentra apoyada sobre una cua de masa M y ngulo de n a elevacin . La cua se puede desplazar o n horizontalmente sin roce sobre un plano. Dos gu restringen el movimiento as de la masa m de manera que sea slo en o direccin vertical. No hay roce entre la o masa m y la cua como tampoco entre n las gu y la masa m. as

Figura 4.23

a) Encuentre la relacin que existe entre la aceleracin vertical am de la masa m y o o la aceleracin horizontal aM de la cua. o n b) Haga los diagramas de cuerpo libre de la masa m y de la cua M . n

c) Encuentre la aceleracin aM de la cua. o n d) Si entre la cua y el suelo hay roce cunto es el valor m n a nimo que debe valer el coeciente de roce esttico e para que la cua no acelere? a n

106 23. Considere dos masas M y m unidas por un hilo que pasa por una polea ideal tal como se muestra en la gura adjunta. Inicialmente la masa M se sujeta con un hilo auxiliar (que no se muestra en la gura) y el sistema se encuentra en reposo. En cierto instante el hilo auxiliar se corta. Demuestre que la aceleracin de la masa M es (con el eje z o apuntando hacia arriba): 4M + 2m a= g . z 4M + m Demuestre que esta expresin da el vao lor correcto en los l mites M m y m M. 24. Dos objetos 1 y 2, de igual masa, estn a atados a los extremos de una cuerda ideal de largo L. El conjunto descansa sobre un disco que gira en un plano horizontal con velocidad angular constante, en torno a su centro (ver gura). Suponga que no existe friccin entre el o disco y el objeto 1, pero existe friccin o entre el objeto 2 y la supercie del disco. Los coecientes de friccin esttico o e y cintico entre la masa 2 y el disco son e e y c , respectivamente.

Las leyes de Newton

Figura 4.24

Figura 4.25

Se observa que cuando el disco gira con velocidad angular 0 , la cuerda se mantiene tensa y alineada en la direccin radial. En esta condicin el objeto 2 est en reposo a o o a una distancia R del eje de rotacin. Cuando la velocidad angular es mayor que 0 el o objeto 2 (y tambin el 1) resbala sobre el disco. Calcule el valor de 0 . e

4.5 Problemas 25.

107

Tal como el campo gravitacional ejerce sobre una masa m una fuerza, un campo elctrico E ejerce una fuerza sobre una carga q. Esta ultima viene dada por e F =qE. (En el sistema internacional de unidades SI, la unidad para la carga es el Coulomb [C] y la del campo eletrico Volt/metro= Newton/Coulomb, siendo las abreviaciones c [V/m]=[N/C]. Un campo de 1 [V/m] ejerce sobre una carga de 1 [C] una fuerza de 1 [N].) Considere un electrn, inicialmente en reposo, que es acelerado entre dos placas (un o condensador) separadas por una distancia de 1 cm. En el espacio entre las dos placas hay un campo elctrico de 900 Volt/cm. e a) b) Cul es su velocidad terminal (la velocidad con que emerge del primer condena sador)? Suponga ahora que el electrn de la parte a), despus de ser acelerado y emerger o e (por un pequeo agujero) del espacio entre las dos placas, ingresa a una regin de n o largo L = 3cm en que existe un campo elctrico transversal de magnitud |E | = e 30 Volt/cm. Cul ser el ngulo de deexin con que emerger el electrn a a a o a o del segundo condensador? (Ver gura 4.26). (En este problema Usted puede despreciar la interaccin gravitatoria, es decir, puede suponer que g = 0. La carga o de un electrn (universalmente denotada con la letra e) es e = 1, 60 1019 [C] o y su masa me = 9, 11 1031 [Kg].)

Figura 4.26

26.

Un pulso de iones de Cs+ (simplemente ionizados) que han sido acelerados desde el reposo por un campo elctrico de 1 (statvolt/cm) a lo largo de 0,33 cm, tarda un e tiempo t = 87 109 s para recorrer 1 mm despus del proceso de aceleracin (ver e o gura 4.27).

108

Las leyes de Newton

a) b)

Encuentre la masa del Cs+ . Si en lugar de Cs+ se realiza el experimento con deuterones, cunto ser el tiempo a a de traves t ? a Suponiendo que los protones y los neutrones tienen la misma masa, encuentre la masa de un neutrn. o Con este dispositivo experimental, ser posible disa tinguir entre deuterones y part culas ? Figura 4.27

c)

d)

(Un deutern es un ncleo atmico formado por un protn y un neutrn; una part o u o o o cula es equivalente a un ncleo de un tomo de He y consiste en dos protones y dos neuu a trones. El ncleo de cesio consta de 58 protones y 84 neutrones, el in Cs+ corresponde u o a un tomo de cesio que ha perdido un electrn). a o 27. Considere una carga q que en el instante t = 0 se encuentra en el origen y en reposo. A partir de t = 0 se le aplica un campo elctrico alterno de la forma e E = E0 sin (t) x . Encuentre la ecuacin diferencial que describe el movimiento de la carga y encuentre o la expresin ms general para la posicin x(t). o a o 28. Un bloque de masa M sube por un plano inclinado cuyo ngulo de elea vacin es . Los coecientes de roce o esttico y cintico entre la masa M a e y el plano son e y c , respectivamente. a) b) c)

Figura 4.28

Cul es la altura mxima que alcanza el bloque, si parte con velocidad v0 desde a a la base del plano? Qu condicin debe satisfacerse para que el bloque vuelva a descender? e o En caso de cumplirse la condicin anterior, con qu velocidad llegar a la base o e a del plano inclinado?

4.5 Problemas 29. Una masa de 100 kg se empuja a lo largo de una supercie en la cual el roce es despreciable mediante una fuerza F , de modo que su aceleracin es de 6 m/s2 (ver gura). Una o masa de 20 kg desliza a lo largo de la parte superior de la masa de 100 kg y tiene una aceleracin de 4 m/s2 o (por lo tanto desliza hacia atrs resa pecto a la masa de 100 kg). a) b) c)

109

Figura 4.29

Qu fuerza de rozamiento ejerce la masa de 100 kg sobre la masa de 20 kg? e Cul es la fuerza neta sobre la masa de 100 kg? Cul es la fuerza F ? a a Una vez que la masa de 20 kg se cae de la masa de 100 kg, cul es la aceleracin a o de la masa de 100 kg?

30.

Sea el coeciente de roce esttico a entre la masa m y el carro. Cul es a la fuerza m nima que debe aplicarse al carro para que la masa m no caiga? Respuesta: F min = (M + m)g/ .

Figura 4.30

31.

Las masas A y B son de 10 y 5 Kg respectivamente. El coeciente de roce de A con la mesa es = 0,2. Encuentre el m nimo valor de la masa C que impide el movimiento de A. Encuentre la aceleracin de A si se o saca C.

Figura 4.31

32.

Una carretera est peraltada de moa do que un automvil, desplazndose o a a 80 Km/h, puede tomar la curva de 30 m de radio, incluso si existe una capa de hielo equivalente a un coeciente de friccin aproximadamente o Figura 4.32 cero. Determinar el intervalo de velocidades a que un automvil puede tomar esta curva o sin patinar, si los coecientes de friccin esttica y cinemtica, entre la carretera y o a a las ruedas, son e = 0,3 y c = 0,26, respectivamente.

110 33. Cul es el mximo valor que puede a a tener m3 para que m1 no se caiga si el coeciente de friccin esttico o a entre m1 y m2 es e , y el de friccin o cinemtica entre m2 y la mesa es c ? a Respuesta: (m1 + m2 ) c +e 1e

Las leyes de Newton

mmax = 3

si e < 1 si e > 1

Figura 4.33

34.

Un bloque de masa M , inicialmente en reposo, resbala por un plano inclinado cuyo a ngulo de elevacin es . Despus de recorrer una distancia D el cuerpo lleva una o e velocidad igual al 50 % de la velocidad que habr adquirido en ausencia de roce. a Encuentre una expresin para el coeciente de roce cinemtico entre el plano y el o a bloque.

35.

Sea c el coeciente de roce cintie co entre un escobilln, cuya masa es o m, y el piso. Un hombre ejerce una fuerza F a lo largo del palo del escobilln. Encuentre |F | en funcin de o o . Existe una solucin para todo o entre 0 y 90 ? (El barrendero avanza con velocidad uniforme.)

Figura 4.34

36.

Una part cula de masa M descansa sobre un plano inclinado que forma un ngulo con la horizontal. Si el a coeciente de roce esttico es e , ena cuentre la m nima fuerza horizontal Fmin transversal a la pendiente del plano, que se requiere para que la part cula comience a moverse.

Figura 4.35

Respuesta: Fmin =

Mg 0

2 cos2 sin2 si e > tan e si e < tan

4.5 Problemas 37.

111

Considere un paquete, de masa m, que se mueve sin roce y con rapidez v0 sobre una supercie de hielo. En cierto instante el paquete entra en contacto con el tablero horizontal de un trineo de masa M , que a su Figura 4.36 vez puede deslizarse sin roce sobre el hielo. Suponga que el coeciente de roce entre el paquete y el trineo es y que el paquete se desliza sobre el trineo hasta nalmente quedar en reposo con respecto a ste. e a) b) c) Una vez que el paquete queda en reposo con respecto al trineo, cul es la a velocidad del trineo? Cunto tiempo demora el paquete en quedar en reposo con respecto al trineo? a Evale el momento lineal del paquete antes de que entre en contacto con el trineo u y comprelo con el momento lineal del conjunto (trineo ms paquete) una vez a a que el paquete est en reposo respecto al trineo. a

(El momento lineal de un objeto es el producto de su masa y velocidad). 38. Con dos bloques A y B se arman las conguraciones I y II que se indican en la gura adjunta. Suponga que las cuerdas y poleas tienen masas despreciables y el coeciente de roce es constante y es el mismo entre todas las supercies en contacto. El valor de las fuerzas aplicadas FI y FII es tal que el bloque A se mueve con velocidad constante en ambas situaciones. Calcule el cuociente entre el mdulo de FI y FII . o

Figura 4.37 39. Considere un cuerpo que cae en la atmsfera. El aire se opone al movimiento con una o fuerza que es proporcional al cuadrado de la velocidad, es decir Froce = kvv , Encuentre la velocidad terminal. 40. Cuando un cuerpo cae en un l quido y el ujo es laminar (es decir, no es turbulento), el u se opone al movimiento con una fuerza que es proporcional a la velocidad, do es decir Froce = v , v = |v| . v = |v| .

112

Las leyes de Newton Encuentre la velocidad terminal. (El coeciente depende del u y de la forma do del objeto).

41.

Sea el coeciente de roce cinemtia co que acta entre las supercies de la u masa m y las cuas (ver gura adjunn ta). Entre las cuas y el suelo el roce n es nulo. Suponga que el valor del roce es tal que el sistema no se encuentra en equilibrio (es decir, las cuas se sepan ran y el bloque baja). Sea el ngulo, a M la masa de las cuas y m la man sa del bloque. Determine la aceleracin o del bloque m. Sobre un plano inclinado liso, que forma un ngulo con la horizontal, se a desliza un bloque partiendo del reposo. Despus de recorrer una distancia D, e el bloque entra en un tramo rugoso. El bloque se detiene luego de recorrer una distancia D en dicho tramo. Calcule el coeciente de roce cintico entre el bloe que y la supercie rugosa.

Figura 4.38

42.

Figura 4.39

4.6.

Solucin a algunos de los problemas o

Solucin al problema 12a o Al pasar la plataforma por la cresta de la colina hay dos fuerzas actuando sobre la caja: i) El peso, W = M g. (Hemos elegido al eje z apuntando hacia arriba, M es la masa z de la caja.) ii) La reaccin de la pesa sobre la caja: Fr = Fr z . o La fuerza neta es, por lo tanto, Fneta = (Fr M g) z . Por otra parte, sabemos que la caja est realizando un movimiento circular de radio R con a rapidez constante, o sea, hay una fuerza neta sobre la caja que actua hacia el centro del c rculo (la fuerza centr peta), que es Fcent = M v2 z. R

4.6 Solucin a algunos de los problemas o La fuerza centr peta y la fuerza neta deben ser iguales, es decir, se tiene que Fr M g = Despejando Fr se obtiene Fr = M g = 500N 1 v2 gR 142 9, 81 100 400 N . M v2 . R

113

Solucin al problema 16 o Observe primero que, al moverse la cua hacia la derecha, el bloque m se mover en diagonal n a (hacia la derecha y hacia abajo). Sea rm el vector de traslacin de m cuando la cua se o n traslada en una magnitud s. Se tiene (ver gura 4.40) que rm = s(1 cos ) x s sin y .

Figura 4.40 Por supuesto que la aceleracin de la cua M y del bloque m estn relacionados. Si la o n a aceleracin de la cua es o n rM = a x , entonces rm = a (1 cos) x a sin z . (4.3)

Sea la tensin de la cuerda y R la fuerza que la cua ejerce sobre el bloque m. Debido a o n que no hay roce entre las superecies, esta fuerza de reaccin R es normal al plano inclinado. o

114 La gura 4.41 muestra el diagrama de cuerpo libre para la masa m. Las componentes horizontal y vertical de la fuerza neta que acta sobre el bloque m son u Fx y Fz
(m) (m)

Las leyes de Newton

= cos R sin

(4.4)

= mg + sin + R cos , (4.5)

Figura 4.41

respectivamente. Usando la segunda ley de Newton y las ecuaciones (4.3), (4.4) y (4.5), se encuentran las relaciones cos R sin = m a (1 cos) (4.6) y mg + sin + R cos = m a sin Sobre la cua actan 4 fuerzas: n u i) El peso M g. z ii) Una fuerza (de reaccin) R que el suelo ejerce sobre la cua. Esta fuerza, cuya mago n nitud no nos interesar, acta en la direccin +. a u o z iii) Una fuerza que el bloque m ejerce sobre la cua. Por el principio de accin esta n o fuerza es R, o sea, las componentes horizontal y vertical son R sin y R cos , respectivamente. iv) La fuerza ejercida por la roldana sobre la cua (que es igual a la fuerza ejercida por n la cuerda sobre la roldana). De la gura 4.42 se deduce que la fuerza total que ejerce la cuerda sobre la roldana es Fc = (1 cos ) sin . x z Figura 4.42 La cua slo se mueve a lo largo de la horizontal; por eso slo nos interesa esa componente n o o de la fuerza neta. Usando la segunda ley de Newton se obtiene R sin + (1 cos ) = M a . (4.8) (4.7)

Las tres ecuaciones de movimiento (4.6), (4.7) y (4.8) con las tres incgnitas a, y R, o permiten resolver el problema. Sumando (4.6) y (4.8) se obtiene = ma(1 cos ) + M a (4.9)

4.6 Solucin a algunos de los problemas o

115

Multiplicando (4.6) por cos y (4.7) por sin y sumando ambas ecuaciones se obtiene = mg sin + ma(cos 1) De (4.9) y (4.10) se deduce nalmente que a= mg sin . M + 2m(1 cos ) (4.10)

Solucin al problema 22 o La relacin entre las aceleraciones es am = aM tan . Los diagramas de cuerpo libre de la o masa m y de la cua se muestran en la gura 4.43. FN es la fuerza entre la masa m y la n cua. n

Figura 4.43 Debido a que no hay roce esta fuerza es normal al plano incliado de la cua. Fr es la fuerza n que la gu ejerce sobre el bloque m (tal fuerza es perpendicular a la gu Fp es la fuerza a a.) que el piso ejerce sobre la cua; en ausencia de roce esta fuerza es perpendicular al piso. n Las ecuaciones de movimiento para la masa m y la cua son: n mg FN cos = mam y FN sin = M aM . Usando la relacin entre las aceleraciones am y aM , podemos despejar aM , obtenindose o e aM = g m tan . M + m tan2

Si, debido al roce entre el suelo y la cua el sistema est en equilibrio, entonces la suma de n a las fuerzas sobre m debe ser nula. Esto permite evaluar FN de inmediato: FN cos = mg . Al diagrama de cuerpo libre de la cua hay que agregar una fuerza de roce fr horizontal n (apuntando hacia la izquierda). Que la suma de las fuerzas horizontales sobre la cua sean n nulas nos da la relacin o FN sin = fr ,

116 o sea, mg tan = fr . Por otra parte, la fuerza de roce debe satisfacer la relacin o

Las leyes de Newton

fr e Fp = e (M g + FN cos ) = e (M + m)g . De las relaciones anteriores se desprende que mg tan = min (M + m) g , e o sea, min = e Solucin al problema 24 o Del hecho que la velocidad angular es constante y las masas 1 y 2 siguen trayectorias circulares, se deduce que la fuerza neta que acta sobre ellas es u
2 F1 = m0 (R + L) r

m tan . m+M

y
2 F2 = m0 R r ,

respectivamente. Aqu m es la masa de cada una de part culas y r es un vector unitario que apunta en la direccin radial. o La unica fuerza radial real que acta sobre la masa 1 es la que ejerce la cuerda, luego u
2 = m 0 (R + L) ,

donde es la tensin de la cuerda. o Sobre la part cula 2 actan dos fuerzas radiales: la tensin de la cuerda r y la fuerza de u o roce fr r. Se tiene 2 fr = m0 R o sea,
2 2 fr = + m0 R = m0 (2R + L) .

Para que las masas no se deslicen la fuerza de roce debe satisfacer la desigualdad fr e mg. De las dos ultimas ecuaciones se deduce que
2 m0 (2R + L) e mg .

La velocidad angular l mite a partir de la cual las masas comienzan a deslizarse es, por lo tanto, e g 0 = . 2R + L

4.6 Solucin a algunos de los problemas o Solucin al problema 26 o

117

Sea M la masa, a la aceleracin y t el tiempo que tardan las part o culas de Cs+ en atravesar el condensador. La carga de cada in de cesio es q = e, donde e = 1, 60 1019 [C] es o la carga de un electrn (ver problema 25). Durante el proceso de aceleracin, la fuerza que o o acta sobre cada in es F = qE0 . Usando la segunda ley de Newton se obtiene que q E0 = u o M a. La aceleracin del tomo de cesio (mientras se mueve al interior del condensador) es, o a por lo tanto, a = q/(M E0 ). El movimiento es uniformemente acelerado. Durante el intervalo de tiempo [0, t ] el in alcanza a recorrer una distancia o 1 s1 = at2 = 0, 33 cm, 2 siendo la velocidad con que emerge del condensador v1 = at . A continuacin los iones de o cesio atraviesan con esa velocidad constante una regin de ancho s2 = 0, 1 cm, tardando o para ello un tiempo t = 87 109 s. Se tiene que v1 t = at t = s2 , o sea t = Por otra parte 2s1 . a Eliminando t de las dos ultimas ecuaciones se encuentra t2 = a= s2 2 . 2s1 (t)2 s2 . at

Igualando las dos expresiones que tenemos para la aceleracin podemos despejar M (la o masa de cada in de cesio): o M= 2|e|E0 s1 (t)2 = 2, 4 1025 Kg . s2 2

Cada in de Cs+ est formado por 58 protones, 84 neutrones y 57 electrones. La masa o a de los electrones es despreciable frente al de los protones y neutrones y por consiguiente, lo ignoraremos. La masa de un neutrn es muy parecida a la de un protn y, en primera o o aproximacin, podemos suponer que son iguales. En lo que a masa respecta, el in de cesio o o lo podemos pensar como un aglomerado de 58+84=142 nucleones. (Nucleones es el nombre genrico que se le da a los protones y neutrones). Dividiendo la masa del in de cesio por e o 27 Kg, valor que 142 se encuentra que la masa de un nuclen es aproximadamente 1,6910 o diere en 1 % del valor medido usando otros mtodos. e Al acelerar deuterones (un protn + un neutrn) en lugar de iones de cesio, slo cambia o o o la masa ya que, igual que en el caso del cesio, la carga neta del deutern es +|e| (o sea, la o fuerza que acta sobre la part u cula acelerada en ambos casos es la misma). El tiempo de

118 traves t es proporcional a a tiempo de traves ser a a

Las leyes de Newton M luego, al usar deuterones en lugar de iones de cesio, el

142 108 s . 2 El dispositivo experimental no es capaz de distinguir entre deuterones y part culas . La part cula (2 protones + 2 neutrones) tiene el doble de la masa del deutern y tambin o e el doble de la carga neta. Estas dos modicaciones se cancelen en cuanto a la aceleracin o respecta, siendo por consiguiente ambas iguales. td = tCs Solucin al problema 27 o La fuerza que acta sobre la carga (ver problema 25) es u F (t) = q E(t) = E0 sin(t) x . Usando la segunda ley de Newton obtenemos las ecuaciones de movimiento: m x(t) = qE0 sin(t) m y (t) = 0 y m z (t) = 0 . De las dos ultimas, usando las condiciones iniciales se deduce que y(t) = z(t) = 0 t , o sea, el movimiento slo ocurre a lo largo del eje x. o Integremos la primera ecuacin de moo vimiento. Se tiene
t

x(t) = x(0) +
0

qE0 sin(t) dt m

t qE0 = 0 cos(t) m 0 qE0 = (1 cos(t)) . m

La posicin de la carga en funcin del o o tiempo se obtiene integrando la ultima ecuacin: o x(t) = x0 + = 0+ = qE0 m
t

(1 cos(t)) dt
0

qE0 m

1 sin(t)

Figura 4.44
0

qE0 (t sin(t)) . m 2

4.6 Solucin a algunos de los problemas o

119

La gura 4.44 muestra un grco de la fuerza, la velocidad y la posicin de la carga en a o funcin del tiempo. o Solucin al problema 32 o De acuerdo con el enunciado, si la rapidez del coche es v0 = 80 km/h, no actuar ninguna fuerza de roce. Como a la trayectoria del automvil es circular, o se tiene que el movimiento es acelerado y, por lo tanto, sobre el coche acta u una fuerza neta hacia el centro de giro 0 (la fuerza centr peta): Fc =
2 mv0 r. R

Figura 4.45

Las fuerzas reales que actan sobre el auto (y cuya suma dan origen a la fuerza centr u peta) son la fuerza de gravedad Fg = mg z y la fuerza normal que la carretera ejerce sobre el coche: FN = FN cos z FN sin r . Por supesto que F c = Fg + FN , o sea,
2 mv0 r = mg z + FN cos z FN sin r . R Igualando las componentes se deduce que

FN cos = mg y
2 mv0 . R Tomando el cuociente entre estas ecuaciones se encuentra una expresin que nos permite o encontrar el ngulo del peralte de la cerretera : a

FN sin =

tan =

2 v0 . Rg

Sea v1 la mxima velocidad que el automvil puede tener sin que se deslize lateralmente a o por la carretera. Si el automvil avanza von rapidez v1 , entonces adems de la fuerza de o a gravedad y la fuerza normal, actuar tambin una fuerza de roce esttica Fr : a e a Fr = Fr cos r Fr sin z .

120

Las leyes de Newton

Cuando el automvil avanza con velocidad mxima v1 , el valor de la fuerza de roce tomar el o a a valor mximo posible Fr = e FN . (observe que el coeciente de roce que debe usarse es el a esttico y no el cinemtico). Se tiene a a Fc = o sea y 0 = mg + FN cos Fr sin . Con Fr = e FN se encuentra
2 mv1 = FN (sin + e cos ) R 2 mv1 r = mg + FN cos z FN sin r Fr cos r Fr sin z , z R 2 mv1 = FN sin Fr cos R

y mg = FN (cos e sin ) . Eliminando FN y despajando v1 de las dos ultimas ecuaciones se obtiende nalmente
2 v1 = Rg 2 v0 + Rge 2 Rg e v0

2 2 Este resultado es vlido mientras Rg > e v0 . Cuando Rg < e v0 , el coche nunca se desa lizar lateralmente hacia afuera y la velocidad mxima a la que se puede transitar por la a a carretera, en ese caso, es innita.

Sea v2 la m nima velocidad con que se puede transitar sobre la carretera sin deslizarse lateralmente hacia el interior. El anlisis en este caso es anlogo al anterior, excepto que la a a fuerza de roce esttica ahora acta en la direccin opuesta. Para v2 se encuentra a u o
2 v2 = Rg 2 v0 Rge 2 Rg + e v0

2 2 Este resultado es vlido mientras Rge < v0 . Cuando Rge > v0 , el coche nunca se deslia zar lateralmente hacia el interior, pudiendo permanecer incluso en reposo (siendo, en ese a caso, v2 = 0).

Para los valores numricos del enunciado las velocidades mxima y m e a nima con que se puede transitar por la carretera son v1 = 123 km/h y v2 = 59 km/h.

Solucin al problema 39 o Sea v la velocidad terminal que un cuerpo adquiere al caer en la atmsfera. Al caer con la o velocidad terminal el cuerpo se mover con velocidad constante. O sea, la aceleracin y la a o

4.6 Solucin a algunos de los problemas o

121

fuerza neta sobre el cuerpo deben ser nulas. Las unicas fuerzas que actan sobre el cuerpo u 2 son la fuerza de gravedad Fg = mg y la fuerza de roce Froce = kv v = kv z . Se tiene z
2 Fg + Froce = mg + kv z = 0 , z

o sea, v = mg . k

Solucin al problema 41 o Denotemos por a = a z la acelera cin del bloque m. Las fuerzas vero ticales sobre el bloque m nos dan la ecuacin de movimiento o mg+2FN cos +2Fr sin = ma . Como los bloques estn en movia miento relativo, la fuerza de roce es de origen cinemtico y se tiene que a Fr = FN . La fuerza que el bloque m ejerce sobre la cua viene dada por FN Fr . La componente n horizontal de esta fuerza nos entrega la ecuacin de movimiento o Fr cos + FN sin = M b donde b es la magnitud de la aceleracin de las cuas. Las dos aceleraciones no son indeo n pendientes sino que estn relacionadas por a a = tan . b Tenemos cuatro ecuaciones con cuatro incgnitas. Despejando la aceleracin a obtenemos o o a=g 1+2 M cos cos + sin m sin sin cos
1

Figura 4.46

Para = tan , la aceleracin es nula. o Solucin al problema 42 o Durante el primer tramo de largo D la part cula acelera uniformemente y despus desacelera e uniformemente quedando en reposo despus de recorrer otro tramo de largo D. Es evidente e que la aceleracin durante el segundo tramo debe tener la misma magnitud que en el o

122

Las leyes de Newton

primero, siendo el signo el opuesto. En otras palabras, la fuerza neta F1 que acta sobre la u masa en el tramo 1 debe ser la opuesta de la fuerza neta en el tramo 2, F2 . En el primer tramo la unica fuerza a lo largo del plano inclinado es la componente en esa direccin del peso, esto es, F1 = mg sin . o En el segundo tramo aparece adicionalmente la fuerza de roce fr . Esta, para dar una fuerza neta F2 = F1 debe ser fr = 2 F1 = 2mg sin . Por otra parte, la fuerza de roce (cinemtica) es a fr = c mg cos . Igualando las dos expresiones para fr y despejando c se obtiene, nalmente, = 2 tan .

Cap tulo 5

Trabajo y Energ a
5.1. Trabajo y energ para movimientos en una dimensin a o

Consideremos una part cula de masa m, restringida a moverse a lo largo del eje z , siendo z(t) y v(t) la posicin y velocidad de la part o cula a medida que transcurre el tiempo. En particular, sean zi y vi la posicin y velocidad de la part o cula en el instante ti , y zf y vf la las mismas magnitudes en el instante tf . Supongamos adems que, a medida que la a part cula se traslada, ejercemos sobre ella una fuerza F (z), fuerza que podr depender de a la posicin z. o Analicemos varios casos: a) Si la part cula, excepto por la fuerza que le estamos aplicando, es libre, entonces acelerar. Si la fuerza F (z) = F0 es constante (es decir, no depende de la posicin), a o entonces la aceleracin tambin lo ser, tenindose F0 = ma0 . De acuerdo a la cio e a e nemtica de un objeto uniformemente acelerado, en el intervalo de tiempo [t, t + t], a la posicin y velocidad de la part o cula cambiarn de acuerdo a las relaciones a 1 z = z(t + t) z(t) = v(t) t + a0 (t)2 2 y v = v(t + t) v(t) = a0 t . Multipliquemos ahora la primera ecuacin por F0 , y usemos, consecutivamente, la o segunda ley de Newton y la segunda ecuacin. De esta manera se obtiene o 1 F0 z = F0 v(t) t + F0 a0 (t)2 2 1 = m a0 v(t) t + m (a0 t)2 2 1 = m v(t) [ v(t + t) v(t) ] + m [ v(t + t) v(t) ]2 2 1 1 2 2 = m v (t + t) m v (t) . 2 2

124

Trabajo y Energ a Las ecuaciones anteriores son vlidas para cualquier t, en particular para t = ti y a t + t = tf , en cuyo caso 1 1 2 2 F0 (zf zi ) = mvf mvi . 2 2 La combinacin 1 mv 2 aparece en la mecnica con mucha frecuencia, siendo util bauo 2 a tizarla y designarla con un s mbolo: se le llama energ cintica y se suele denotar con a e la letra K o T . La ultima ecuacin nos indica que el cambio de energ cintica de una o a e part cula libre (excepto por la fuerza que se le est aplicando) es igual al producto a de esa fuerza y el desplazamiento que realiza. A este producto (entre la fuerza y el desplazamiento) se le llama trabajo; para denotarlo es usual usar la letra W . O sea, tenemos Wzi zf = Kf Ki .

b) Si la fuerza no es constante, entonces subdividamos la trayectoria de la part cula en N intervalos de tamao z. Denotemos las distintas posiciones por z1 , z2 , z3 , . . ., n zN , zN +1 , siendo zi = z1 y zf = zN +1 . Si en cada intervalo j la fuerza se mantiene relativamente constante, podemos usar el resultado de la parte a), o sea, Wzj zj+1 = F (zj ) (zj+1 zj ) = F (zj ) z = Kj+1 Kj . Sumando la contribucin de todos los intervalos se obtiene o
N

F (zj ) z = (K2 K1 ) + (K3 K2 ) + (K4 K3 ) + + (KN +1 KN )


j=1

= KN +1 K1 , o sea,
N

Wzi zf =
j=1

F (zj ) z = Kf Ki .

La expresin anterior es exacta en el l o mite N , de modo que el tamao de n los intervalos z se torna innitesimalmente pequeo (en cuyo caso se denota por n dz). En ese l mite la sumatoria se remplaza por una S estilizada (llamada integral), tenindose e
f

Wzi zf =
i

F (z) dz = Kf Ki .

c) Supongamos ahora que la part cula no es libre, sino que est inmersa en un campo a gravitacional constante g = g. Levantemos la part z cula desde zi hasta zf , partiendo desde el reposo y volviendo a dejarla en reposo. Elevamos la part cula aplicando una fuerza de manera que sta suba con una velocidad constante. Mientras la part e cula va subiendo, su aceleracin es nula, luego tambin la fuerza neta que acta sobre ella. o e u De lo anterior se desprende que la fuerza que debemos ejercer para elevar la part cula es F (z) = +mg. El trabajo que nosotros realizamos es, por lo tanto, Wzi zf = +mg (zf zi ) .

5.1 Trabajo y energ para movimientos en una dimensin a o

125

En este caso, el trabajo realizado por nosotros sobre la part cula no se maniesta en un cambio de su energ cintica. Lo que cambia es la potencialidad de la part a e cula para realizar trabajo o de adquirir energ cintica. a e En efecto, al dejar caer la part cula sin restricciones desde zf hasta zi , adquirir una a velocidad que, de acuerdo a las ecuaciones de la cinemtica de la ca libre, es a da vf = 2g(zf zi ). Para esta velocidad, la energ cintica es a e 1 1 2 K = mvf = m 2g(zf zi ) = mgzf mgzi , 2 2 resultado que coincide con el obtenido mas arriba. Otra forma en que puede manifestarse esta potencialidad de la part cula consiste en hacer que ella realice trabajo, por ejemplo, permitiendo que ella baje al punto inicial zi de manera que, por medio de un sistema de poleas, eleve otra masa. De acuerdo a lo desarrollado ms arriba, tambin en este ejemplo podemos expresar a e el trabajo Wzi zf como una diferencia de cierta magnitud evaluada en el punto nal menos la misma magnitud evaluada en el punto de partida: Wzi zf = U (zf ) U (zi ) , donde la energ potencial para la part a cula m en el campo gravitacional constante g = g, viene dada por z U (z) = U0 + mgz . U0 es una constante y corresponde a la energ potencial de la part a cula cuando sta se e encuentra en z = 0. En la mecnica clsica el valor de U0 no tiene mayor importancia, a a ya que lo unico relevante resultan ser diferencias de energ potencial entre dos puntos. a Esto permite elegir U0 = 0, si eso resulta conveniente y simplica las ecuaciones. Resumiendo: En el presente ejemplo, al elevar la masa en presencia de un campo gravitacional (y sin modicar su energ cintica), el trabajo realizado se transforma a e en un cambio de energ potencial. La energ potencial se designa usualmente con la a a letra U o V . d) Consideremos un resorte de constante de restitucin k, acostado sobre una suo percie horizontal sin roce y con un extremo empotrado en una pared (ver gura 5.1). Supongamos adems que el a sistema inicialmente se encuentra en reposo, con el resorte teniendo su largo Figura 5.1 natural. Evaluemos el trabajo que debemos realizar para alargar (lentamente) el resorte en una magnitud x0 . La fuerza que debemos aplicar para lograr nuestro objetivo ahora no es constante, sino que aumenta a medida que el resorte se estira: F (x) = kx x .

126

Trabajo y Energ a (Esta fuerza es la opuesta a la que el resorte ejerce sobre la masa, que, de acuerdo a la Ley de Hooke, es kx). El trabajo que debemos realizar para alargar el resorte, x desde x = 0 hasta x = x0 , viene dado por
x=x0 x0 x0

W0x0 =
x=0

F (x) dx =
0

kx dx = k
0

x dx .

Ya sabemos que la integral f (x) dx no es otra cosa que el rea bajo la curva a del grco de la funcin f (x) . Para el a o presente caso, la funcin corresponde a o una recta que pasa por el origen (ver gura 5.2), siendo el rea bajo la curva a 1 2 x0 . 2

Figura 5.2

Luego, el trabajo que debe realizarse para expandir el resorte hasta x0 es W0x0 = 1 2 kx . 2 0

(Se obtiene el mismo resultado si, en lugar de alargarlo, el resorte se comprime en una magnitud x0 ). Tambin en este ejemplo, el trabajo realizado por nosotros sobre la part e cula no se maniesta en un cambio de su energ cintica, sino lo que cambia es el estado del a e sistema. En el nuevo estado, el sistema tiene la potencialidad (al permitir que el resorte vuelva a su largo natural) de realizar trabajo, o de entregarle a la part cula adosada al resorte una energ cintica. a e Si la energ potencial de un resorte la a denimos por 1 U (x) = U0 + kx2 , 2 donde x = 0 corresponde a la posicin o de equilibrio del resorte y x es la magnitud en que ste se comprime o se alarga, e entonces nuevamente W0z0 = U (z0 ) U (0) . (La constante aditiva U0 en la expresin para la energ potencial nuevao a mente no aparece en el resultado nal; la podr amos haber elegido igual a cero.) La gura 5.3 muestra el grco a U (x) correspondiente a la energ poa tencial de un resorte.

Figura 5.3

5.1 Trabajo y energ para movimientos en una dimensin a o

127

e) Consideremos ahora una part cula que se mueve a lo largo de una recta (el eje x) sobre una mesa y supongamos que, adems de la fuerza que nosotros ejercemos sobre ella, a la unica otra fuerza se debe al roce (cintico). La fuerza de roce es fr = mgc (pero e note que sta aparece slo cuando la part e o cula se est moviendo y observe adems a a que la direccin en que apunta siempre es contraria a la direccin de movimiento). o o El trabajo que debemos realizar para empujar la part cula, partiendo desde el reposo desde xi , hasta xf (donde nuevamente la dejamos en reposo), es W = fr (xf xi ) = (mgc ) (xf xi ) . En este caso, sin embargo, el trabajo que realizamos no se maniesta en un cambio de algo que podr amos denominar energ potencial. a Hay dos problemas que aparecen cuando hay roce y que hace que la situacin sea o distinta que en los dos ultimos casos: i) En primer lugar, el trabajo que debe hacerse para llevar la part cula desde xi hasta xf , cuando hay roce, depende del camino que uno elija para ello y, por lo tanto, el trabajo no se puede escribir como una magnitud que slo dependa o del punto inicial y nal. En efecto, supongamos que xf est a la derecha de xi . a Ya vimos que, al llevar la part cula directamente desde xi a xf , el trabajo que debemos realizar es W = mgc (xf xi ). Pero si antes la empujamos hacia la izquierda en una distancia L, y recin desde ah la empujamos al punto nal xf , e el trabajo ser W = mgc (2L + xf xi ). O sea, el trabajo no slo depende a o del punto inicial y nal sin que tambin del camino. o e ii) Otra caracter stica del trabajo que se hace contra el roce es que ste no es e recuperable como energ mecnica sin una mquina trmica. Mas an, en caso a a a e u de tener una, la recuperacin del trabajo realizado slo es parcial. El trabajo o o realizado por nosotros contra la fuerza de roce se transforma y se disipa como calor. Los ejemplos unidimensionales anteriores sugieren lo siguiente: Denicin: El trabajo realizado por una fuerza F (z) que acta sobre alguna part o u cula es W =
j

F (zj ) (zj+1 zj ) =

F (z) dz ,

donde la suma (o integral) se realiza a lo largo de la trayectoria que recorre la part cula. El trabajo W que se entrega a un sistema, cuando no hay roce, se maniesta en un cambio de la energ del sistema. Hasta el momento hemos identicado las siguientes formas de a energ a: a) Energ cintica de una part a e cula de masa m. Esta viene dada por 1 K = mv 2 , 2

128

Trabajo y Energ a y se debe al movimiento de la part cula. Cuando la part cula est en reposo, su energ a a cintica es cero. e

b) Energ potencial. Esta es una energ que se debe a la posicin de la part a a o cula. La energ potencial slo aparece cuando la part a o cula no es libre, sino que est sometida a a un campo de fuerzas. La expresin para la energ potencial depende del campo de o a fuerzas. Hasta el momento hemos analizado dos casos: i) Campo gravitacional uniforme, F (z) = mg = mg, en cuyo caso la energ z a potencial es U (z) = mg (z z0 ) , donde z0 es un lugar que arbitrariamente hemos jado como el cero de la energ a potencial. ii) Campo de fuerzas de un resorte, F (x) = kx, en cuyo caso la energ potencial x a es 1 U (x) = kx2 . 2 Cuando hay roce, parte (o toda) la energ entregada al sistema (por medio del a trabajo), puede disiparse. Esta energ se maniesta en un aumento de la temperatura a de las supercies que rozan entre s En este caso, el trabajo W se transforma en calor . Q. Conservacin de la energ o a: Al entregarle a una part cula un trabajo W , entonces W = (Kf Ki ) + (Uf Ui ) + Q , (5.1)

o sea, el cambio de la energ cintica, ms el cambio de la energ potencial, ms la energ a e a a a a disipada como calor es igual al trabajo (energ aplicado al sistema. a) La ecuacin de conservacin de la energ hay que manejarla con precaucin, pues no se o o a o puede estar seguro de que uno haya identicado todas las posibles formas de energ De a. hecho, a medida que avancemos en el estudio de la f sica, en varias oportunidades nos veremos forzados a reinterpretar esa ecuacin o agregarle trminos adicionales. o e La ecuacin (5.1) tambin se puede reescribir de la siguiente manera: o e Kf + Uf = Ki + Ui + W Q . A la suma K + U suele llamarse energ mecnica y denotarse con la letra E. a a

5.2 Trabajo para un movimiento en tres dimensiones

129

5.2.

Trabajo para un movimiento en tres dimensiones

Consideremos ahora una part cula libre de masa m que se mueve en el espacio tridimensional y cuya posicin y velocidad en el instante t son r y v, respectivamente. Apliquemos sobre o esa part cula, durante un intervalo de tiempo innitesimal dt, una fuerza F . De acuerdo a la segunda ley de Newton, F = ma . Durante el intervalo de tiempo innitesimal dt, la part cula se desplaza una distancia dr = v dt . Haciendo el producto punto de los vectores que aparecen en las dos ultimas ecuaciones, se obtiene F dr = mv a dt . Evaluemos la velocidad al cuadrado en el instante t + dt. Se tiene v 2 (t + dt) = v(t + dt) v(t + dt) = (v(t) + a(t) dt) (v(t) + a(t) dt) = v(t) v(t) + v(t) a(t) dt + a(t) v(t) + a(t) a(t) (dt)2 = v 2 (t) + 2v(t) a(t) dt + (trminos del orden de (dt)2 ) , e o sea, 1 v(t) a(t) dt = v 2 (t + dt) 2 Con este resultado y la expresin deducida ms arriba, o a 1 2 v (t) . 2 obtenemos que

1 1 F dr = mv 2 (t + dt) mv 2 (t) . 2 2 La ultima ecuacin nos indica que el cambio de energ cintica de una part o a e cula sobre la cual acta una fuerza durante el intervalo de tiempo [t, t + dt] (pero por lo dems es libre), u a es igual al producto punto de esa fuerza y el desplazamiento realizado por la part cula en ese mismo intervalo. Lo anterior sugiere denir el trabajo, en el caso tridimensional, como el producto punto del vector fuerza y el vector desplazamiento. Si el movimiento no ocurre durante un intervalo de tiempo innitesimal, sino entre dos instantes ti y tf , podemos usar la ecuacin anterior siempre que el intervalo se divida o en muchos intervalos pequeos y luego se sumen los trabajos y los cambios en la energ n a cintica de cada uno de los intervalos. De esta manera se obtiene e
f

W =
i

1 1 F (r) dr = mv 2 (tf ) mv 2 (ti ) . 2 2

Este resultado es el anlogo tridimensional de la situacin considerada en la seccin anterior, a o o en las partes a) y b), para el movimiento unidimensional.

130

Trabajo y Energ a

Denicin: El trabajo realizado por una fuerza F (r) que acta sobre alguna part o u cula viene dado por W = F (r ) dr ,

donde la integral se evala a lo largo del camino recorrido por la part u cula. Volvamos a analizar el concepto energ a potencial para una part cula inmersa en un campo gravitatorio uniforme. Consideremos un objeto de masa m, en un campo gravitatorio constante g = g z (el eje z apuntando hacia arriba), y eva luemos el trabajo que debemos realizar para trasladarlo (lentamente) desde el origen hasta el punto P : (x0 , z0 ) (ver gura 5.4).

Figura 5.4

Para llevar a cabo nuestro cometido podemos usar distintos caminos. Supongamos que usamos el camino 1 , o sea, primero elevamos el objeto desde z = 0 hasta z = z0 y luego lo trasladamos hacia el lado, hasta llegar al punto P . Durante el primer tramo la fuerza que debemos realizar para elevar el objeto (con velocidad uniforme) es F = mg, siendo el z desplazamiento tambin a lo largo del eje z , es decir, dr = dz z . Luego, para este primer e tramo, el trabajo que debemos realizar es
z=z0 z=z0

W0Q =
z=0

mg dz = mg
z=0

dz = mg z0 .

Para el segundo tramo la fuerza sigue siendo F = mg; el desplazamiento, sin embargo, z ahora es a lo largo del eje x, es decir, dr = dx x. El producto punto entre la fuerza y el desplazamiento es cero (por ser uno ortogonal al otro). Luego, para trasladar el objeto desde Q a P no se requiere realizar ningn trabajo. Concluimos que el trabajo total, a lo u largo del camino 1 , es W1 =
1

F (r ) dr = mgz0 .

Evaluemos ahora el trabajo que debemos realizar para llevar el mismo objeto desde el origen al punto P a lo largo del camino 2 . La fuerza que debemos aplicar sigue siendo F = mg; z el desplazamiento, sin embargo, es a lo largo de la direccin del vector unitario s, o sea, o dr = ds s. Luego se tiene que F dr = (mg z ) (ds s) = mg sin ds . Concluimos que el trabajo total, a lo largo del camino 2 , viene dado por W2 =
1

F (r ) dr = mg(sin )
2

ds = mgL sin ,

donde L es el largo del camino. Pero L sin = z0 , luego los trabajos a lo largo del los caminos 1 y 2 son iguales.

5.2 Trabajo para un movimiento en tres dimensiones

131

No es dif demostrar que tambin el trabajo que se debe realizar para llevar el objeto cil e desde el origen al punto P a lo largo del camino 3 es igual a mgz0 . En efecto, para trasladar el objeto a lo largo de tramos horizontales no se requiere hacer trabajo, mientras que para los tramos verticales el trabajo siempre es mgh, donde h es la diferencia de altura del tramo vertical. Eligiendo el cero de la energ potencial de la part a cula cuando sta se encuentra en el e origen, su energ potencial cuando se encuentre a una altura z ser a a U (x, z) = mgz , siendo sta independiente del valor x. e Cuando un campo de fuerza tiene la propiedad de que el trabajo realizado para llevar (lentamente) una part cula entre dos puntos cualesquiera, es independiente del camino usado para unir tales puntos, entonces el campo de fuerzas se dice que es conservativo. El campo gravitacional es un ejemplo de un campo conservativo. La fuerza de roce es un ejemplo de una fuerza no conservativa. Al empujar un cajn de masa o M por el suelo de una habitacin de un lugar a otro, el trabajo realizado ser proporcional o a al largo L del camino que para ello se elige, siendo W = c M gL. Al no ser el roce una fuerza conservativa, no se puede introducir una energ potencial para esta fuerza (ya que a no existe una funcin que slo dependa de los puntos nal e inicial, y cuya diferencia sea o o igual al trabajo). El trabajo que se realiza contra la fuerza de roce se transforma en calor. La reconversin de energ calrica a energ mecnica puede hacerse slo recurriendo a o a o a a o alguna mquina y, aun as no en forma completa. a ,

Unidades En el sistema internacional de unidades SI, la unidad del trabajo (o, lo que es lo mismo, de la energ es el Joule, que se abrevia como [J]: a) 1 Joule = 1 Newton metro , o sea, kg m2 . s2 Al trabajo por unidad de tiempo se denomina potencia. En el sistema SI, la unidad de la potencia se denomina watt [W] y corresponde a 1 Joule por segundo, es decir, 1J=1Nm=1 1 W=1 J . s

Ejemplo: Considere un motor elctrico de 0.4 KW (esto corresponde a aproximadamente al e motor de una juguera). Cunto tiempo tardar tal motor, mediante un sistema de poleas, a a en levantar un automvil de 600 kg en un metro? o

132 Solucin: El trabajo requerido para levantar el automvil es o o W = m g h = 600 9,81 1 kg m2 6000 J . s2

Trabajo y Energ a

El motor es capaz de entregar 400 J por segundo (estamos suponiendo una eciencia de un 100 %), luego, para realizar un trabajo de 6000 J tardar 6000/400 = 15 s. a

5.3.

Ejemplos

A continuacin ilustremos los conceptos anteriores aplicndolos en algunos problemas cono a cretos. Ejemplo 1 Considere un bloque de masa M que incide con velocidad v0 sobre un resorte (ver gura 5.5) y lo comprime. Cul ser la a a mxima compresin que en algn instante a o u llega a tener el resorte? Figura 5.5 El bloque, al comprimir el resorte, realiza trabajo. Este trabajo, que se transforma en energ potencial del resorte, lo hace a costa de su energ cintica. La mxima compresin a a e a o se logra cuando el bloque llega a estar en reposo. En ese caso, toda la energ cintica se a e habr transformado en energ potencial, o sea, a a 1 1 2 M v0 = kx2 . 2 2 0 En la ecuacin anterior, x0 es la mxima compresin que llega a tener el resorte. Despejando o a o x0 se encuentra que M v0 . x0 = k Ejemplo 2 Un bloque de masa m resbala por un plano inclinado, partiendo del reposo desde una altura h. Sea el ngulo a de elevacin y el coeciente de roce o cinemtico entre el bloque y el plano. a Con qu velocidad llegar el bloque al e a pie del plano inclinado?

Figura 5.6

Inicialmente el bloque tiene slo una energ potencial U = mgh (el cero de la energ o a a potencial lo hemos elegido en la base del plano inclinado). Al llegar el bloque abajo, ste e 2 tiene slo energ cintica K = mvf /2. Usando el principio de conservacin de la energ o a e o a

5.3 Ejemplos

133

se tiene que la energ cintica nal debe ser igual a la energ potencial inicial menos la a e a energ disipada por el roce. Esta ultima es Q = m g (cos ) L = m g h/ tan . Se tiene a 1 mv 2 = mgh mgh/ tan , 2 f o sea,
2 vf = 2gh 1

tan

La ecuacin anterior es vlida si e tan . Si la condicin anterior no se cumple la o a o part cula no resbala. Observe cmo, en el l o mite = 90 , la velocidad vf tiende al resultado de la ca libre. da Ejemplo 3 Suponga que la energ potencial de una part a cula de masa m viene dada por la expresin o U (z) = a 1 3 z b2 z 3 ,

donde a y b son ciertas constantes positivas. Encuentre el campo de fuerza F (z) que da origen a tal energ potencial. a Sea F (z) la fuerza que el campo ejerce sobre la part cula. Para llevar lentamente la part cula desde el origen al punto z deberemos ejercer sobre la part cula una fuerza de igual magnitud pero sentido opuesto. Por consiguiente, el trabajo que debemos realizar es
z

W (z) =
0

F (z) dz .

Este trabajo es igual a la diferencia de la energ potencial entre el origen y el lugar z, o a sea, U (z) U (0). Como U (0) = 0, podemos igualar W (z) con U (z), obtenindose e
z

U (z) =
0

F (z) dz = a

1 3 z b2 z 3

Como el proceso de integracin es el inverso del proceso de derivacin (ver gura 5.7), se o o tiene que la fuerza debe ser menos la derivada de la energ potencial, o sea, a F (z) = dU (z) . dz (5.2)

Usando esta relacin se encuentra que para el presente problema, el campo de fuerzas es o F (z) = a b2 z 2 . Es importante sealar que siempre la derivada de la energ potencial es (menos) la fuerza n a que el campo conservativo ejerce sobre la part cula.

134

Trabajo y Energ a

Figura 5.7

Ejemplo 4 Una part cula en un campo de fuerzas se dice que se encuentra en equilibrio si la fuerza sobre ella es nula. Para una part cula cuya energ potencial es (la misma del a ejemplo anterior) U (z) = a 1 3 z b2 z 3 ,

la fuerza F (z) es nula cuando z = b. Note que esos puntos siempre correponden a mximos o m a nimos de la energ potena cial (ver gura 5.8). Se dice que un sistema en equilibrio es estable si al alejarlo levemente del punto de equilibrio la fuerza que aparece lo acelera nuevamente hacia dicho punto. De lo contrario, el equilibrio es inestable.

Figura 5.8

Para el caso mostrado en la gura 5.8, la part cula se encontrar en equilibrio estable si a est en reposo en el lugar z = b, e inestable si se encuentra en z = b. a Supongamos que la part cula de masa m se encuentra en el punto de equilibrio estable y que en cierto instante la sacamos levemente de su punto de equilibrio, para dejar que luego se mueva en el campo de fuerza. Demostremos que la fuerza que el campo ejerce sobre la part cula es en magnitud proporcional al desplazamiento pero en sentido opuesto. Efectivamente, si alejamos la part cula una distancia + desde el punto de equilibrio estable z = b, (con b), entonces la fuerza que aparece es F (b + ) = a b2 (b + )2 = a(2b + 2 ) 2ab .

Notemos que la fuerza asociada al potencial en la vecindad del m nimo acta en forma u anloga a la ley de Hooke que gobierna el comportamiento de un resorte: al alejar la part a cula

5.4 Problemas

135

de la posicin de equilibrio aparece una fuerza que es proporcional al desplazamiento, pero o en el sentido contrario. Al soltar la part cula (desde el reposo) en el lugar z = b + , la part cula oscilar en torno al punto de equilibrio igual como si estuviera adosada a a un resorte. El per odo de oscilacin de una masa adosada a un resorte con constante de o restitucin k es T = (2) m/k. En el presente ejemplo, 2ab juega el papel del coeciente o de restitucin k, luego el per o odo de las oscilaciones ser a T = 2 m . 2ab

5.4.
1.

Problemas
Una masa de 2 kg se lleva desde un punto A al punto B. Los vectores de posicin de o los puntos A y B son: xA = ( + 3) m x z xB = 5 m x Todo el sistema est inmerso en un cama po gravitatorio constante g. Encuentre z el trabajo realizado por la gravedad a lo largo de cada uno de los tres caminos indicados en la gura adjunta.

Figura 5.9

2.

Una bomba de agua debe elevar 200 litros de agua por minuto desde un pozo, cuyo nivel de agua est a 6 m de profundidad, para luego lanzarla con una velocidad de a 9 m/s. Suponiendo que no hay prdidas de energ de ningn tipo, qu trabajo por e a u e minuto debe realizar el motor que acciona la bomba? Cul es la potencia del motor? a (Una mquina que realiza un trabajo de 1 Joule = 1 kg m2 /s2 por segundo, tiene una a potencia de 1 Watt = 1 [W].)

3.

Sobre una part cula de masa m = 0,25 kg, que se mueve a lo largo del eje x, acta u una fuerza F = F (x) x, donde la magnitud F (x) depende de la posicin x del modo o indicado en la gura 5.10. a) b) Determine el trabajo realizado por esta fuerza sobre la part cula si ella se traslada desde x = 0 a x = 3 m. Si la part cula en el instante t = 0 se encuentra en reposo en x = 2 m, qu vee locidad tendr al llegar a x = 6 m? a

136

Trabajo y Energ a

Figura 5.10 Respuestas: a) 4 J; b) v=4 m/s. 4. El sistema mostrado en la gura adjunta se abandona, partiendo del reposo, cuando el bloque de masa m1 est a una distancia d por encima del suelo. Desprecie el roce. a a) b) Encuentre la aceleracin de la masa o mayor. (m1 > m2 .) Usando el resultado de la parte (a), encuentre la velocidad con que la masa mayor llega al suelo. Suponiendo que todo el trabajo realizado sobre el sistema se transforma en energ cintica, calcule la velocia e dad de la masa mayor justo antes de que choque contra el suelo.

c)

Figura 5.11

5.

Considere un cuerpo compuesto de N masas mj , situados en los lugares rj , con j = 1, 2, 3, . . . , N . Demuestre que la energ potencial de tal cuerpo, en un cama po gravitacional constante, se puede evaluar suponiendo que toda su masa M = m1 + m2 + + mN est concentrada en su centro de masas, dado por a rcm 1 (m1 r1 + m2 r2 + . . . + mN rN ) . M

6.

Un bloque cuya masa es m = 6 kg se desliza hacia abajo por un plano inclinado rugoso, partiendo del reposo. El ngulo de elevacin del plano es = 60 y los coecientes de a o roce esttico y cinemtico son 0.2 y 0.18, respectivamente. a a a) Describa todas las fuerzas que actan sobre el bloque y determine el trabajo u realizado por cada una de ellas, si el bloque se desliza 2 m (a lo largo del plano).

5.4 Problemas b) c) Cul es el trabajo neto realizado sobre el bloque? a Cul es la velocidad del bloque despus de recorrer una distancia de 2 m? a e

137

Resuelva el problema dos veces: la primera suponga que el sistema consiste slo del o bloque y, por lo tanto, las fuerzas de roce son parte de las fuerzas externas; la segunda vez suponga que el sistema consiste del bloque y el plano inclinado, en cuyo caso la disipacin de energ por las fuerzas de roce deben considerarse como calor. o a 7. Se desea levantar lentamente una masa M hasta una altura h, usando el sistema de poleas mostrado en la gura adjunta. a) b) c) Cul es la fuerza que debe aplicara se? Qu trabajo se realiza? e Cul es el cambio en energ potena a cial de la masa?

Figura 5.12

8.

Un bloque de m = 5 kg se sujeta contra un resorte de constante k = 1000N/m, comprimindolo en d0 = 8 cm. Al dejar el bloque en libertad, el resorte al expandirse e empuja el bloque a lo largo de una supercie horizontal rugosa con coeciente de roce cinemtico = 0,2. a a) b) c) d) e) Cul es el trabajo realizado por el resorte sobre el bloque mientras el resorte se a extiende desde la posicin comprimida hasta la posicin de equilibrio? o o Cul es el trabajo realizado por el roce sobre el bloque cuando ste recorre los a e 8 cm hasta la posicin de equilibrio? o Cul es la velocidad del bloque cuando el resorte pasa por su posicin de equia o librio? Si al pasar por la posicin de equilibrio el bloque se despega del resorte, qu diso e tancia alcanzar a recorrer antes de detenerse? a Si el bloque se mantiene sujeto al resorte, cul es la extensin mxima que a o a llegar a tener el resorte? a

9.

Un pndulo de masa m colgado de un hilo de largo , se eleva hasta formar un ngulo e a con la normal y luego se deja en libertad. 0 = 90 a) b) Encuentre la energ cintica de la masa pendular cuando el pndulo pasa por a e e su posicin de equilibrio. o Demuestre que la tensin de la cuerda, para = 0 , es 3 veces el peso de la masa o pendular.

138 10. Considere el campo de fuerza dado por F (r ) = F0 x + F0 x y . a

Trabajo y Energ a

Evale el trabajo que debe realizarse para u llevar una part cula de masa m desde el origen hasta el punto A a lo largo de los dos caminos indicados en la gura adjunta. El campo de fuerzas es conservativo?

Figura 5.13

11.

Una caja, de masa 10 Kg, descansa sobre la cubierta horizontal de una mesa. El coeciente de friccin entre la caja y la supercie de la mesa es 0,4. En cierto instante o se aplica sobre ella una fuerza F = F0 x, adquiriendo la caja una aceleracin constante o a = 2 x [m/s2 ]. a) b) Determine F0 . Determine el trabajo realizado por la fuerza F cuando la caja se ha trasladado una distancia de 5 m. Evale la diferencia entre el trabajo realizado sobre la part u cula y el calor Q disipado por el roce. Demuestre que esta diferencia coincide con la energ cintica a e nal de la caja.

c)

12.

Una masa m resbala, sin roce y debido a la gravedad, por la supercie de una semiesfera de radio R. La masa parte desde la cspide sin velocidad inicial. Sea P el u punto en el cual la masa se separa de la semiesfera. Encuentre el ngulo de elevaa cin 0 del punto P . o Figura 5.14

Respuesta: sin 0 = 2/3 .

13.

Sobre una cinta transportadora caen 5 kg de material por segundo ( = dm/dt = 5 kg/s). Suponiendo que no hay prdidas de energ de ningn tipo en todo el sistema e a u que impulsa la cinta transportadora, encuentre la fuerza F que debe aplicarse para mantener la cinta trasladndose con una velocidad constante v0 = 3 m/s. Cul es la a a m nima potencia que debe tener el motor para hacer avanzar la cinta transportadora?

5.4 Problemas

139

Figura 5.15 dW 2 = v0 = 45 W . dt

Respuesta:

P =

14.

Considere dos masas m unidas por una varilla de largo L que no tiene peso. Inicialmente el sistema est apoyado en una paa red, formando un ngulo 0 con la normal a (vea gura 5.16). El sistema comienza a resbalar sin roce debido a la gravedad. A qu altura la masa # 1 se separa de la e pared vertical? Respuesta: h = 2h0 /3 , donde h0 = L sin 0 es la altura inicial. Figura 5.16

15.

Una moneda se desliza sobre un tramo horizontal pulido. Luego entra en un tramo cil ndrico convexo de radio R=1 m. La moneda pierde contacto con la supercie cil ndrica a un ngulo de 30 con respeca to a la vertical medido desde el vrtice del e cilindro. Calcule la rapidez con que se desplazaba la moneda en el tramo horizontal.

Figura 5.17

16.

La fuerza electrosttica que ejerce una carga elctrica Q sobre otra carga q viene dada a e por la as llamada Ley de Coulomb: F = qQ r , r2

donde r = r r es el vector de posicin de q respecto a Q. Notemos que si las dos o cargas tienen el mismo signo la fuerza entre las cargas es repulsiva. Considere una carga Q ja en el origen y una carga q, que inicialmente se encuentra en el lugar r0 . a) Encuentre el trabajo que debe realizarse para trasladarla desde r0 hasta r1 .

140 b) c)

Trabajo y Energ a Repita la parte a) para varios caminos simples y demuestre que siempre obtiene el mismo resultado (en otras palabras, el campo de fuerzas es conservativo). Demuestre que la energ potencial (electrosttica) de la carga q viene dada por a a U (r) = + qQ . r

En qu lugar se ha elegido el cero para la energ potencial? e a 17. Considere una carga Q ja en el origen y otra carga q, del mismo signo, que se acerca a Q a lo largo de la recta que las une. Si q tiene una energ cintica K cuando la a e separacin entre las cargas es muy grande (innita), encuentre la m o nima distancia a la que q se acercar a Q. a Para resolver este problema use el resultado para la energ potencial obtenido en el a problema anterior. 18. Considere la conguracin de caro gas mostrada en la gura 5.18. Las cargas +Q estn jas en los lugares a x = a, y = z = 0 mientras que la carga +q puede deslizarse slo a lo o largo del eje x. a) b) c)

Figura 5.18

Encuentre una expresin para la fuerza F (x) que acta sobre la carga +q. o u Encuentre la energ potencial U (x) y graf a quela. (Especique claramente dnde o eligi el cero para la energ potencial.) o a Se comportar este sistema como un oscilador armnico para pequeos desplaa o n zamientos de q en torno al origen? (Un sistema se comporta como un oscilador armnico si, al desplazar el sistema de su posicin de equilibrio, aparece una o o fuerza proporcional al desplazamiento pero de sentido contrario ejemplo, ley de Hooke.)

19.

Considere una part cula de masa m y carga q restringida a moverse a lo largo del eje x. Adems, dos car a gas +Q se ubican jamente sobre el eje y a una distancia a del origen, tal como lo muestra la gura 5.19. a) Encuentre la energ potencial a U (x) del sistema de cargas en funcin de x. o

Figura 5.19

(b) Encuentre la fuerza electrosttica F (x) que acta sobre la carga q. a u

5.4 Problemas

141

(c) Evale la derivada dU (x)/dx y demuestre que sta coincide con F (x). u e (d) Con qu velocidad pasar la part e a cula por el origen si parte desde el innito con velocidad cero? (e) Se comportar este sistema como un oscilador armnico para pequeos desplaa o n zamientos de q en torno al origen? Si su respuesta es armativa, encuentre el per odo del movimiento peridico. o Respuestas: 2Qq a) U (x) = ; b) a2 + x2 20. F (x) = 2Qqx x ; d) v = + x2 )3/2 4Qq . ma

(a2

Un bloque de 2 Kg, situado a una altura de 1 m, se desliza por una rampa curva y lisa, partiendo del reposo. Al terminarse la rampa, el bloque resbala 6 m sobre una supercie horizontal rugosa antes de llegar al reposo. a) b) c) Cul es la velocidad del bloque en la parte inferior de la rampa? a Cul es el trabajo realizado por la fuerza de roce sobre el bloque? a Cunto vale el coeciente de roce cinemtico entre el bloque y la supercie a a horizontal?

Figura 5.20 21. En un parque de entretenciones un carro de masa m = 100 kg se desliza (sin roce) por una rampa desde una altura h, ingresando a un loop de radio R = 3 m. La altura h es la m nima que se requiere para que el carro no se salga de la v Emergiendo del loop a. el carro ingresa a la regin de frenado, donde en un trayecto de largo L, el coeciente o de roce cinemtico es c = 0, 2. Sin embargo, el carro no alcanza a detenerse durante a la primera pasada sin que pasa de largo y despus de colisionar con un resorte de o e constante k = 500N/m, vuelve a ingresar a la regin de frenado quedando en reposo o al centro de ella (o sea, en el punto C, ver gura 5.21). a) b) c) d) Encuentre la velocidad del carro en el punto B. Encuentre h. Encuentre L. Encuentre la mxima coma presin que alcanza a tener o el resorte. Figura 5.21

142 22.

Trabajo y Energ a Una part cula de masa m se mueve sobre una mesa rugosa a lo largo de un c rculo de radio R. La part cula est amarrada a un extremo de un hilo de largo R, cuyo otro a extremo est jo al centro del c a rculo. La velocidad de la part cula inicialmente es v0 . Debido al roce con la mesa, la part cula se ir frenando. Despus de completar una a e vuelta, su velocidad es v0 /2. a) b) c) Encuentre el trabajo realizado por la friccin durante una vuelta. Exprese el o resultado en funcin de m, v0 y R. o Encuentre el valor del coeciente de roce cinemtico. a Cuntas vueltas dar la part a a cula antes de detenerse?

23.

Una masa m se cuelga de dos resortes en serie, de constantes de restitucin k1 y k2 , tal como se o muestra en la gura 5.22a. Encuentre la frecuencia de oscilacin para pequeas vibraciones o n (verticales) de la masa m. Repita el clculo para el caso en a que los dos resortes estn en paa ralelo (ver gura 5.22b). Supongamos que la funcin energ poo a tencial U (x) en funcin de x, para una o part cula que slo se puede mover a lo laro go del eje x, viene dada por el grco mos a trado en la gura 5.23 a) b) c) Identique los puntos de equilibrio e indique si son estables o inestables. Para qu valor de x la fuerza tiene e su valor (mdulo) mximo? o a Describa en palabras el movimiento, a medida que transcurre el tiempo, de una part cula de energ total a E1 . (Especique claramente las condiciones iniciales que est suponiena do.)

Figura 5.22

24.

Figura 5.23

25.

Suponga que la energ potencial entre dos tomos de una molcula diatmica viene a a e o aproximadamente dada por la expresin o U (r) = U0 a r
12

a r

donde r es la separacin entre los tomos y a y U0 son constantes. o a

5.4 Problemas a) b) c) d) Graque la energ potencial. a

143

Para qu separacin r los tomos estarn en equilibrio? El equilibrio es estable e o a a o inestable? Suponga que los tomos tienen la misma masa m0 . Con qu frecuencia vibrar la a e a molcula al alejar el sistema levemente de su posicin de equilibrio? e o Si la molcula est en su estado de equilibrio, cul es la m e a a nima energ que a habr que entregarle a la molcula para disociarla, es decir, separarla en sus dos a e a tomos constituyentes?

26.

La fuerza gravitatoria entre dos masas m1 y m2 viene dada por F = Gm1 m2 r , r2

donde G es la as llamada constante gravitatoria y r = r es el vector que une los r centros de masa de ambas masas. El valor experimental de G es G = 6,6720 1011 m3 / s2 kg . a) Demuestre que el trabajo que debe hacerse para alejar las dos masas desde una separacin r1 a una separacin r2 (r2 > r1 ), viene dado por o o W = Gm1 m2 b) 1 1 r 1 r2 .

A partir del resultado anterior, demuestre que si el cero de la energ potencial a se elige en r = , entonces la energ potencial en funcin de la distancia entre a o las dos masas viene dada por U (r) = Gm2 m1 . r

c)

Suponga ahora que m1 = MT es la masa de la tierra y m2 = m es la masa de un objeto pequeo. Si tal objeto se encuentra sobre la supercie terrestre y se n eleva una pequea distancia h, demuestre que la energ potencial cambia en n a U = mgh. Note que de esta manera usted ha encontrado una expresin para o la aceleracin de gravedad g en trminos del radio de la tierra RT , la masa MT o e y la constante de gravitacin G. o Encuentre la masa de la tierra suponiendo que el radio de la tierra es aproximadamente 6380 km. (Ignore la rotacin de la tierra.) o Encuentre la velocidad de escape, es decir, la velocidad m nima que debe impartirse a una masa m (inicialmente en reposo sobre la supercie terrestre) para que ella pueda alejarse del campo gravitatorio de la tierra. (Ignore la rotacin o de la tierra.) Hasta qu distancia mxima se podr alejar el pequeo objeto si su velocidad e a a n inicial es la mitad de la velocidad de escape?

d) e)

f)

144 27.

Trabajo y Energ a Un bloque de masa M se apoya sobre un platillo de masa m sujeto a un resorte vertical de constante k y largo natural 0 . Al colocar el platillo con la masa M sobre el resorte este se comprime teniendo, en equilibrio, un largo . Comprimamos ahora el resorte otro poco de manera que inicialmente se encuentra contra a un largo < . En cierto instante se suelta el resorte, permitiendo que do ste se expanda. e a) b) Evale . u Demuestre que el si el resorte en algn instante supera el largo natuu ral 0 , entonces el bloque se separa del platillo. Cul es el m a nimo valor de la contraccin ( 0 ) que debe tener el o resorte antes de soltarlo para que el bloque alcance a separarse del platillo?

c)

Figura 5.24

d) Encuentre la altura mxima alcanzada por el bloque (en todos los casos, cuando a se separa y cuando no se separa del platillo). Respuesta: c) ( 28.
0

) = 2g(M + m)/k.

En una feria de entretenciones hay un juego que consiste en que los participantes abordan un carro en el punto P (ver gura 5.25), caen en ca libre una altura h da hasta el punto A, luego recorren un cuarto de circunferencia (AB) de 2 m de radio y una recta (BC) de 5 m, todo esto sin roce. En el punto C se ingresa a una zona de 8 m de largo con coeciente de roce c = 0,5. Como zona de seguridad, hay una distancia (DE) de 5 m sin roce, concluyendo la pista en un gran resorte cuya constante elstica a es k = 6 104 N/m. La masa del carro, con los pasajeros, es de 500 Kg. a) Calcule hasta cuntos metros por sobre el punto A se puede dejar caer el carro a para que ste se detenga en la zona de desaceleracin CD. e o Suponga ahora que el operador del juego sube el carro hasta 8 m sobre A y lo deja caer desde all . b) Encuentre el lugar en que el carro quedar sin velocidad (por primera vez). a c) Encuentre el lugar en que el carro quedar nalmente en reposo. a d) Calcule el trabajo realizado por la fuerza elstica del resorte para detener el carro a (por primera vez). e) Calcule la aceleracin del carro en el instante en que el resorte lo detiene. o

5.4 Problemas

145

Figura 5.25

29.

Considere el montaje mostrado en la gura adjunta. Suponga que las dos masas tienen el mismo valor y que 0 coincide con el largo natural del resorte cuya constante de restitucin es k = 5mg/ 0 . Suponga o adems que la masa desliza sin roce a sobre la supercie y que en el instante mostrado en la gura el sistema se encuentra momentneamente en a reposo.

Figura 5.26

a)

Demuestre que cuando la masa que se deliza por la supercie se haya desplazado en una cantidad x = 3 0 /4 hacia la derecha, esta se levantar de la supercie. a Demuestre que en el momento en que la masa se separa del plano la velocidad es v = 19g 0 /32.

b)

30.

Considere dos pequeas masas iguales m n unidos mediante cuerdas ideales de longitud = 1, 5 m, como se indica en la gura adjunta. El sistema rota con velocidad angular uniforme . El ngulo que la cuerda a atada al brazo (de longitud L = 4 m) forma con la vertical es de 60 . Encuentre el ngulo que la otra cuerda hace con a la vertical y encuentre la razn entre las o tensiones de cada cuerda.

Figura 5.27

146 31. Dos cuerpos A y B, de masas m y 2m, repectivamente, se unen mediante una cuerda ideal. El cuerpo A posa sobre una mesa de supercie spera (coeciente de roce a c ) mientras que B se deja caer como se muestra en la gura 5.28. No hay roce entre la cuerda y el punto de contacto con el borde de la mesa. Calcule el ngulo a formado por la cuerda que sostiene la masa B y la horizontal cuando el bloque A comienza a resbalar. El largo de la cuerda entre el borde de la mesa y el cuerpo B es L. Dos monos, llamados Pat y Logo, de n igual masa m estn agarrados de una cuera da que pasa por encima de una polea (sin roce), frente al Museo del Louvre. Habiendo escuchado el rumor de que en el museo hay una famosa pintura de una supuesta congnere con una enigmtica sonrisa, e a el mono Pat decide subir por la cuerda n hasta una posicin que le permita mirarla o por la ventana. Para ello debe remontar una altura h. a) b)

Trabajo y Energ a

Figura 5.28

32.

Figura 5.29

Analice como el movimiento del mono Pat afecta la posicin del mono Logo. n o Calcule el trabajo que debe realizar el mono Pat para llevar a cabo su propsito. n o

33.

Considere dos masas m unidas por un hilo de largo 2L, que caen con el hilo estirado en forma horizontal. Despus de caer e una distancia L el centro del hilo choca con un clavo, correspondiendo de ah en adelante la trayectoria de las dos masas a un movimiento circular. Si el hilo se corta cuando la tensin llega tener el valor o max = 7mg/2, encuentre el ngulo que a en ese instante forma el hilo con la horizontal (ver gura 5.30). Figura 5.30

5.5 Solucin a algunos de los problemas o

147

5.5.

Solucin a algunos de los problemas o

Solucin al problema 12 o Cuando la masa m haya resbalado hasta formar un ngulo con la horizontal, la energ a a potencial (gravitatoria) habr cambiado en a Upot = mg h = mg(R R sin ) . Como no hay roce, este cambio de energ potencial debe coincidir con la energ cintica a a e que adquiere la masa m, o sea, debe cumplirse la relacin o 1 mv 2 = mgR (1 sin ) . 2 Esto nos permite encontrar la velocidad v de la masa en funcin de : o v= 2gR(1 sin ) .

La masa m recorre un arco de c rculo de radio R, luego la fuerza centr peta (que apunta en la direccin ) es o r mv 2 Fcent = r. R (Tambin hay una fuerza tangencial que, sin embargo, aqu no es necesario evaluar.) Las e unicas fuerzas reales que actan sobre m son la normal N r y el peso mg. (Nuevamente u z hemos elegido al eje z apuntando hacia arriba.) La componente radial de la fuerza neta es (N mg sin ) r. Esta debe coincidir con la fuerza centr peta, o sea, Despejando N se obtiene N mv 2 1 = mg sin 2mgR(1 sin ) R R = mg (3 sin 2) = mg sin mv 2 = N mg sin . R

La masa m inicia su movimiento en el pice, en cuyo caso = 90 , siendo la fuerza normal a que ejerce la semiesfera sobre la masa N = mg. A medida que transcurre el tiempo, disminuye y luego tambin N . Cuando sin = 2/3, la fuerza normal se hace cero, siendo e ese el lugar en que la masa m se separa de la semiesfera.

148 Solucin al problema 14 o Supongamos por un momento que la part cula 1 nunca se despega de la pared. Cuando la part cula 1 haya bajado desde h0 hasta una altura y, entonces, por conservacin de energ o a 1 1 mg(h0 y) = my 2 + mx2 . 2 2 Sabemos que x2 +y 2 = L2 . Derivando esta relacin se deduce que 2xx + 2y y = 0, o o sea, x y=x . y Sustituyendo esto en la ecuacin de cono servacin de la energ se encuentra la reo a lacin o 1 1 x2 1 L2 mg(h0 y) = mx2 + mx2 2 = x2 2 . 2 2 y 2 y

Trabajo y Energ a

Figura 5.31

De esta ultima ecuacin podemos despejar la velocidad de la part o cula 2 en funcin de la o altura a la que se encuentra la part cula 1: x2 = 2g (h0 y) y 2 . L2

La velocidad de la part cula 2 depende de y. Observemos que la rapidez con que se mueve la part cula 2 es nula cuando y = h0 y tambin cuando y = 0, luego en algn lugar entremedio e u debe tener un mximo. a Encontremos el valor y = h para el cual x tiene su mximo. Para ello debemos encontrar el a 2 . Igualando la derivada de f (y) a cero se encuentra mximo de la funcin f (y) = (h0 y)y a o 2h0 y 3y 2 = 0 . Despejando y se encuentra y = h = 2h0 /3. Es claro que cuando la part cula 1 llegue a esa altura, se desprender de la pared (si es que no hay un mecanismo que evite que eso a ocurra). La razn es la siguiente: el unico elemento que ejerce una fuerza horizontal sobre o el sistema (las dos masas con la varilla) es la la pared vertical. Mientras y > h la part cula 2 acelera (la rapidez x aumenta) en la direccin +, luego la pared debe ejercer sobre el o x sitema una fuerza en esa direccin. Cuando y < h entonces la part o cula 2 desacelera (x vuelve a disminuir); eso implica que la pared ejerce una fuerza en la direccin sobre el o x sistema, lo que a su vez slo es posible si existe algn mecanismo que sujete a la part o u cula 1 a la pared vertical. Si tal mecanismo no existe, entonces la part cula 1 se separa de la pared.

5.5 Solucin a algunos de los problemas o Solucin al problema 18 o

149

De acuerdo a la ley de Coulomb, las cargas Q de la izquierda y de la derecha ejercen sobre q una fuerza qQ F1 = x (a + x)2 y qQ F2 = x, (a x)2 respectivamente. La fuerza total F (x) que acta sobra la carga q es la suma vectorial de u las dos fuerzas F1 y F2 , por lo tanto, F (x) = F1 + F2 = qQ 1 1 2 (a + x) (a x)2 x.

Para encontrar la energ potencial tambin podemos evaluar primero la energ potencial a e a de q con cada una de las cargas Q separadamente, para luego hacer la suma (escalar) de ellas. La energ potencial de una carga q a una distancia r de otra carga Q viene dada a por (ver problema 14) U (r) = qQ/r. Usando esta expresin se encuentra que la energ o a potencial de la carga q, cuando sta se encuentra en el lugar x, es: e U (x) = U1 (x) + U2 (x) = qQ qQ + . |a + x| |a x|

La energ potencial es cero cuando x . La gura 5.32 muestra un grco de la a a funcin U (x). o

Figura 5.32 De la gura se deduce que x = 0 es un punto de equilibrio estable del sistema. Para pequeos n desplazamientos, o sea para |x| a, se tiene U (x) = qQ 1 1 + = a+x ax qQ x x2 1 + 2 + a a a 2qQ 2qQ 2 + 3 x = U0 + a a qQ a 1+ 1+ x a
1

+ 1

x a

x x2 + a a2

2qQ 2 x . a3

150

Trabajo y Energ a

De la ecuacin anterior se deduce que, para pequeos desplazamientos de q desde el origen, o n la energ potencial es cuadrtica (es decir, similar a la expresin que se ten para una a a o a masa adosada a un resorte). La fuerza que acta sobre q al desplazarla levemente de su posicin de equilibrio es u o

F (x) =

dU (x) 4qQ = 3 x. dx a

Esta fuerza es anloga a la ley de Hooke: es proporcional y apunta en sentido contrario a al desplazamiento. El papel de la constante de restitucin k lo juega 4qQ/a3 . Luego, al o desplazar la carga q levemente de su punto de equilibrio, sta oscilar armnicamente con e a o un per odo ma3 , 4qQ

T = 2

donde m es la masa de la carga q.

Solucin al problema 25 o La gura adjunta muestra el grco de a la energ potencial. Para r > a la pena diende es positiva, para r = a es nula, mientras que para r < a es negativa. La fuerza entre los dos tomos de la a molcula es dU (r)/dr. Cuando la derie vada es nula (para r = a), la fuerza tambin es nula, luego la separacin r = a coe o rresponde a un punto de equilibrio. Para r > a, dU (r)/dr > 0 y, por consiguiente, F (r) < 0. En palabras: si la separacin o de los dos tomos de la molcula es maa e yor que a, entonces la fuerza entre ellas ser atractiva. a

Figura 5.33

Lo contrario ocurre para r < a: en ese caso dU (r)/dr < 0 y por consigiente F (r) > 0, o sea, la fuerza que aparece tratar de alejar a los dos tomos (aumentar r). Resumiendo, cada a a vez que el sistema se desplaza de su posicin de equilibrio, aparece una fuerza que trata de o llevar al sistema nuevamente a su posicin de equilibrio. (Es precisamente esto ultimo lo o que caracteriza a un punto de equilibrio estable.)

5.5 Solucin a algunos de los problemas o Sea F12 la fuerza que acta sobre el tou a mo 1 debido al tomo 2 y F21 la fuerza que a acta sobre el tomo 2 debido al tomo 1. u a a Por supuesto que, de acuerdo al principio de accin y reaccin (tercera ley de Newo o ton) F12 = F21 . Sea O un origen y r1 y r2 los vectores de posicin de cada uno de o los tomos (ver gura 5.34). a Las ecuaciones de movimiento, de acuerdo a la segunda ley de Newton, son: m0 r1 = F12 y m0 r2 = F21 . Restando una de la otra se obtiene m0 (r2 r1 ) = F21 F12 = 2F21 . La fuerza que acta sobre la part u cula 2 debida a la part cula 1 es F21 = dU (r) r, dr Figura 5.34

151

(5.3)

donde r = | r | = | r2 r1 |. Como estamos suponiendo que la molcula no rota, se tiene que e el vector unitario r, que apunta a lo largo de la l nea que une a ambos tomos, no variar a a a medida que transcurre el tiempo. Se tiene entonces que r r2 r1 = r r y r = r2 r1 = r r . Sustituyendo la ultima ecuacin en (5.3) se obtiene o m0 r r = 2F21 . Evaluemos F21 . Se tiene: F21 = dU (r) r dr U0 a = 12 a r (5.4)

a r

r.

Escribamos r de la forma r = a + s. De esta manera, s = 0 corresponder a la molcula a e en su posicin de equilibrio. Si los tomos se desplazan slo levemente de su posicin de o a o o

152 equilibrio, entonces |s| << a. En este caso F21 = 12 = 12 U0 a a a+s


6

Trabajo y Energ a

a a+s

r
7

s 6 s U0 1 1+ 1+ a a a U0 s s 12 16 1 17 r a a a U0 72 2 s r + o(s2 ) . a

(5.5)

Sutituyendo este resultado en (5.4), se obtiene m0 r r = 72 U0 sr. a

Cancelando a ambos lados r y usando el hecho que r = s, se tiene


2 s + 0 s = 0 ,

(5.6)

con
2 0 = 72

U0 2m a 0

La ecuacin diferencial (5.6) corresponde a la de un oscilador armnico. Ya sabemos que o o en ese caso, la magnitud s (el alejamiento de un tomo de su posicin de equilibrio) realia o zar oscilaciones armnicas, siendo el per a o odo de tales oscilaciones T = 2 = 0 a2 m0 . 18 U0

De la gura 5.33 tambin se deduce que para disociar a la molcula, es decir, para separar e e los tomos a una distancia r , se debe entregar al sistema una energ al menos igual a a a U0 . Solucin al problema 28 o a) Si la masa m parte de una altura h, entonces su energ (antes de entrar a la regin de a o desaceleracin) es o 1 E = mv 2 = mgh . 2 Al atravezar toda la zona de desaceleracin, la energ disipada en calor es Q = c mg CD. o a Para que la masa m quede en reposo en D, toda su energ debe ser disipada como calor, a o sea, mgh = c mg CD . Despejamos h: h = c CD = 0, 5 8 [m] = 4 [m] .

5.5 Solucin a algunos de los problemas o

153

c) Ahora h = 8 [m]. La mitad de la energ se disipar durante la primera pasada por la a a regin de desaceleracin y el resto se disipar en la segunda pasada. El carro m quedar o o a a nalmente en reposo en el punto C. b) Despus de emerger de la regin de desaceleracin por primera vez, la energ del carro e o o a ser E1 = mgh/2. Esta tendr que ser la energ potencial del resorte cuando est compria a a e mido con el carro detenido: mgh 1 = kx2 , 2 2 0 donde x0 es la compresin mxima del resorte. El carro se detendr por primera vez a una o a a distancia x0 a la derecha del punto E. Despejando x0 se encuentra (con g = 10 [m/s2 ]), x0 = mgh = k 500 10 8 [m] = 0, 816 [m] . 6 104

d) El trabajo realizado por la fuerza elstica del resorte para detener el carro es igual a la a energ con que incidi sobre el resorte, mgh/2 = 500 10 8/2 [J]= 20000 [J]. a o Tambin podemos encontrarla evaluando la integral e
x0

W =
0

1 kx dx = kx2 2

x0 0

1 1 = kx2 = 6 104 (0, 816)2 [J] . 2 0 2

e) La fuerza que ejerce el resorte cuando est comprimido es kx0 x, donde x apunta hacia a la derecha. La aceleracin del carro, por lo tanto, ser o a a= kx0 x = 97, 92 [m/s2 ] , m

aproximadamente 10 veces la aceleracin de gravedad. o Solucin al problema 33 o Despus de chocar el hilo con el clavo y al formar un ngulo con la horizontal, la energ e a a potencial de cada masa habr disminu en mgL (1 + sin ). Esta ser la energ cintica a do a a e que tendr cada masa, es decir, a 1 mv 2 = mgL (1 + sin ) . 2 Esta relacin nos permite encontrar la velocidad v = v(): o v 2 = 2gL (1 + sin ) . Como cada masa est recorriendo un c a rculo sabemos que la fuerza radial neta (la fuerza centr peta) que se est ejerciendo sobre ella es a Fcent = mv 2 r = 2mg (1 + sin ) r . L

154

Trabajo y Energ a

Las unicas fuerzas reales que estn siendo ejercidas sobre cada masa son la fuerza debido a a la tensin del hilo y la fuerza de gravedad: o Freal = r mg z . La componente radial de esta fuerza es + mg sin . Esta debe coincidir con la fuerza centr peta, o sea, + mg sin = 2mg (1 + sin ) . El hilo se corta si el ngulo es tal que = 7mg/2. Llamando 0 a ese ngulo se tiene a a 7 mg + mg sin 0 = 2mg (1 + sin 0 ) . 2 A partir de esta relacin se encuentra que sin 0 = 0, 5, o sea, 0 = 30 . o

Cap tulo 6

Momento lineal y colisiones


6.1. Conservacin del momento lineal o

Consideremos N part culas, cuyas masas y vectores de posicin vienen dados por {mj }j y o {rj }j , con j = 1, 2, . . . , N . Supongamos que sobre cada part cula j algn agente externo u ejerce una fuerza Fj . Supongamos adems que las part a culas interactan entre s siendo fji u , la fuerza que ejerce la part cula i sobre la part cula j. Por supuesto, debido al principio de accin y reaccin, fij = fji , o, en palabras, la fuerza que ejerce la part o o cula j sobre la i es la opuesta a la que la part cula i ejerce sobre la j. Usando la segunda ley de Newton, podemos escribir la ecuacin de movimiento de la part o cula j. Se tiene
N

Fj +
i=1

fji = mj rj .

Al lado izquierdo se tiene la fuerza total que acta sobre la part u cula j, mientras que al lado derecho se tiene el producto de la masa por la aceleracin de esa part o cula. Sumando las ecuaciones de movimiento de todas las part culas (es decir, sumando sobre j), se obtiene Fj +
j i,j

fji =
j

mj rj .

(6.1)

Note que, debido al principio de accin y reaccin, o o las fuerzas internas es nula. Denamos la magnitud rcm

ij

fji = 0, es decir, la suma de todas


N

m1 r1 + m2 r2 + + mN rN 1 = m1 + m2 + + mN M

mj rj ,
j=1

(6.2)

donde M m1 + m2 + + mN es la masa total. Derivando la ultima ecuacin dos veces o respecto al tiempo, queda
N

M rcm =
j=1

mj rj .

156 Sustituyendo esta expresin en (7.1), y usando la relacin o o


N

Momento lineal y colisiones fji = 0, se obtiene

ij

Fj = M rcm .
j=1

La suma j Fj corresponde a la fuerza externa total que acta sobre las part u culas, magnitud que denotaremos por Ftot . De esta manera, nalmente Ftot = M rcm . (6.3)

Esta ultima ecuacin no depende de las fuerzas internas que actan entre las distintas o u part culas. Al vector rcm se le denomina posicin del centro de masas. La ecuacin (6.3) o o ensea que la suma de las fuerzas externas que actan sobre un sistema de part n u culas acelera al centro de masas como si toda la masa del sistema estuviese centrada all . Si la fuerza (externa) total sobre el sistema de part culas es nula, o sea, si Ftot = j Fj = 0, entonces el centro de masas no acelera. En ese caso, la velocidad del centro de masas es (0) constante (designemos por vcm a tal velocidad). Por otra parte, la velocidad del centro de masas se puede obtener derivando la ecuacin (6.2) respecto al tiempo: o 1 rcm = M
N (0) mj rj = vcm . j=1

Introduciendo la denicin de momento lineal pj = mj rj de cada part o cula, la ultima ecuacin se puede reescribir de la forma o
(0) Ptot p1 + p2 + + pN = M vcm .

(6.4)

Esta ecuacin se conoce con el nombre de Principio de Conservacin del Momento Lineal. o o En palabras: cuando la fuerza externa total que acta sobre un sistema de part u culas es cero (observe que la fuerza no tiene por qu ser igual a cero individualmente sobre cada e una de las part culas), entonces la suma de los momentos lineales de las distintas part culas (o sea, el momento lineal total) no var en el tiempo, es decir, se mantiene constante (aun a cuando las part culas interacten entre s Cuando la fuerza externa total sobre un sistema u ). de part culas es cero, entonces la suma de todos los momentos lineales de las part culas en todo instante ser la misma. a Ilustremos los conceptos anteriores con un ejemplo. Problema: Considere dos masas, m = m0 y M = 3m0 , sobre las cuales no actan fuerzas u externas. Supongamos que en el instante t = 0, la part cula m se encuentra en el origen y en reposo, y que la part cula M se encuentra en rM (0) = 2 [m] movindose con una x e velocidad vM (0) = 4 [m/s]. Supongamos adems que existe cierta interaccin entre las y a o part culas y, como consecuencia de ella, ambas aceleran. Si en el instante t0 = 5 [s] la part cula m se encuentra en rm (t0 ) = (2 8) [m], en qu lugar se encontrar la otra x y e a

6.2 Colisiones masa?

157

Solucin: Como no hay fuerzas externas, el centro de masas se mueve con velocidad conso tante. Encontremos primero la velocidad del centro de masas y la posicin de ste en los o e instantes t = 0 y t = 5 [s]. En efecto rcm (0) = y vcm (0) = m vm (0) + M vM (0) 3m0 4 y = 3 [m/s] . y = m+M m0 + 3m0 m rm (0) + M rM (0) 3 3m0 2 x = x [m] = m+M m0 + 3m0 2

Como la velocidad del centro de masas es constante, en el instante t0 = 5 [s] el centro de masas se encontrar en a rcm (t0 ) = rcm (0) + vcm (0) t0 = 3 x + 3 5 y 2 [m] .

Por otra parte, en el instante t0 la posicin del centro de masas viene dada por o rcm (t0 ) = = mrm (t0 ) + M rM (t0 ) m+M 1 m0 (2 8) + 3 m0 rM (t0 ) x y = (2 8 + 3rM (t0 )) x y m0 + 3 m0 4

[m] .

Igualando las dos ecuaciones para rcm (t0 ) podemos despejar rM (t0 ): 1 3 (2 8 + 3rM (t0 )) [m] = x y x + 3 5 y 4 2 de donde se deduce que rM (t0 ) = 1 (8 + 68) [m] . x y 3 [m] ,

!Graque en el plano (, y ) los vectores rm , rM y rcm en los instantes t = 0 y t = 5 s! x

6.2.

Colisiones

Analicemos primeramente colisiones en una dimensin. Considere dos part o culas de masas m y M , restringidas a moverse (sin roce) a lo largo del eje x y estudiemos algunos casos particulares. a) Supongamos que la part cula M incide desde la izquierda con velocidad +v0 y se mueve hacia la part cula m, que inicialmente se encuentra en reposo. Suponga que las dos part culas colisionan, quedando una adosada a la otra, formando una unica part cula de masa (M + m). Con qu velocidad v se mover esta nueva part e a cula despus de la colisin? e o

158

Momento lineal y colisiones Para resolver este problema usamos el principio de conservacin del momento lineal. o Sobre el sistema no hay fuerzas externas actuando, luego el momento lineal se conserva. El momento lineal total antes de la colisin es o Ptot = M v0 + m 0 = M v0 , mientras que despus de la colisin es e o Ptot = (M + m) v . Estas dos expresiones deben ser iguales, luego v= M v0 . M +m
(f ) (i)

En el l mite M m (el caso de una locomotora chocando con una mosca) la velocidad despus de la colisin es v v0 (la velocidad de la locomotora). En el l e o mite M m (el caso de una mosca chocando contra una locomotora en reposo) la velocidad despus e de la colisin es v 0. o La energ cintica antes y despus de la colisin no es la misma, siendo la diferencia a e e o Q Kf Ki = 1 1 2 (M + m)2 M v0 v 2 2 1 m 2 = M v0 . 2 M +m

A esta diferencia se le suele llamar el valor Q de la reaccin. Cuando, como en el o presente ejemplo, Q = 0, la colisin se dice que es inelstica. Si Q < 0, la colisin es o a o endotrmicaparte de la energ que el sistema ten antes de la colisin se difunde e a a o a grados de libertad internos del sistema (por ejemplo, se pierde como calor). Si Q > 0 la colisin (o reaccin) es exotrmica. o o e b) Consideremos ahora el caso de una colisin en que se conserva la energ cintica o a e (o sea, una colisin con Q = 0) y adems se conserva la identidad de las part o a culas (es decir, el nmero de ellas y sus masas). Tales colisiones se denominan elsticas. u a Nuevamente supongamos que la part cula M incide desde la izquierda, a lo largo del eje x, con velocidad +v0 y que choca con la part cula m, que inicialmente se encuentra en reposo. Encontremos la velocidad nal vm y vM de cada una de las part culas. Para resolver este problema invocamos la conservacin del momento lineal y de la o energ (cintica). Se tiene a e M v0 = mm + M vM v y 1 1 1 2 M v0 = mm + M vM . v2 2 2 2 2 De estas dos ecuaciones podemos despejar las dos incgnitas vm y vM . Se obtiene o vm = 2M v0 M +m

6.2 Colisiones y vM = Analicemos varios casos l mites: M m v0 . M +m

159

i) M m (o sea, una locomotora con velocidad v0 colisionando con una pelota de tenis en reposo). En este l mite, vM = v0 y vm = 2v0 (es decir, la locomotora se guir como si nada hubiese pasado mientras que la pelota de tenis sale disparada a con una velocidad igual al doble de la velocidad de la locomotora). ii) M m (o sea, una pelota con velocidad v0 colisionando con una locomotora en reposo). En este l mite, vm 0 y vM = v0 (es decir, la locomotora prctica a mente no se mover y la pelota rebota con una rapidez igual a la incidente). Que a la velocidad vM en este caso sea negativa signica que la part cula se mueve en la direccin . o x iii) M = m (o sea, la colisin central de dos bolas de billar de igual masa). En ese o caso vm = v0 y vM = 0 (es decir, la part cula incidente queda en reposo mientras que la otra part cula sale disparada con velocidad v0 ).

Si la colisin no ocurre a lo largo de una l o nea recta, sino que ocurre en un plano, los conceptos involucrados son los mismos y lo unico que cambia es que la ley de conservacin del o momento lineal, siendo una ecuacin vectorial, nos dar una relacin para cada una de las o a o componentes. Demos un ejemplo. Problema: Considere una masa m1 que choca elsticamente contra una masa m2 origia nalmente en reposo. Suponga que despus del choque la masa incidente m1 emerge en una e direccin perpendicular a su direccin original (ver gura 6.1). Encuentre: o o a) El ngulo con que emerge m2 , en funcin de m1 y m2 . a o b) La velocidad v1 con que emerge m1 . c) La velocidad v2 con que emerge m2 .

Figura 6.1

160

Momento lineal y colisiones

Solucin: La energ y el momento lineal total, antes y despus de la colisin son: o a e o 1 2 Ei = m1 v0 , 2 pi = m1 v0 x , 1 1 2 2 Ef = m1 v1 + m2 v2 , 2 2 pf = m1 v1 z + m2 v2 cos m2 v2 sin . x z Debido a que el choque es elstico, se conserva la energ total y el momento lineal total; a a esto nos da las ecuaciones: 2 2 2 m1 v0 = m1 v1 + m2 v2 , m1 v0 = m2 v2 cos y m1 v1 m2 v2 sin = 0 . Estas son tres ecuaciones con tres inggnitas (las incgnitas son: v1 , v2 y ). Resolviendo o o este sistema de ecuaciones se encuentra: v1 = v0 m2 m1 , m2 + m1 2m2 1 m2 (m2 + m1 ) m2 m1 . m2 + m1

v2 = v0 y tan =

v1 = v0

Note que el problema slo tiene solucin si m2 > m1 . o o En la seccin 6.6 ilustramos los conceptos anteriores con otro ejemplo. o

6.3.

Impulso

De acuerdo a la segunda ley de Newton F = dp/dt. Multiplicando por dt e integrando se obtiene


f f

F dt =
i i

dp = pf pi ,

o sea, el cambio de momento que sufre una part cula en cierto intervalo de tiempo es igual a la integral de F dt durante ese intervalo. A la integral F dt se le llama impulso. Cuando ocurre una colisin entre dos objetos, el tiempos de colisin puede ser muy pequeo, o o n el momento transferido sin embargo puede ser grande. Por ejemplo, cuando una bolita de acero, al caer desde una altura h, rebota (elsticamente) por una supercie dura. Al llegar a

6.3 Impulso

161

a la supercie la bolita tendr la velocidad 2gh y justo despus del rebote 2gh z . El a z e cambio de momento es, por lo tanto, p = pf pi = 2m 2gh z .

Este cambio de momento debe ser igual al impulso transmitido por el suelo a la bolita, es decir,
f

F (t) dt = 2
i

2gh z .

Durante la colisin, t o picamente la fuerza que ejerce un cuerpo sobre tiene el comportamiento como el mostrado en la gura 6.2. Antes de la colisin la fuerza es o nula. En el instante t0 1 los cuerpos entran en contacto, la fuerza aumenta rpia damente llegando a un mximo en cierto a instante t0 para luego decrecer nuevamente a cero (instante t0 + 2 ). Los cuerpos estn en contacto durante un intervalo de a tiempo 1 + 2 .

Figura 6.2

El rea bajo la curca F (t) es el impulso (momento) transmitido de un cuerpo al otro. a T picamente el comportamiento F (t) puede ser representado por una parbola a F (t) = F0 1 0
1
2

(t t0 )2

para t0 < t < t0 + para t < t0 y t > t0 +

En este caso los cuerpos se mantienen en contacto durante un tiempo 2 , siendo en el instante t = t0 la fuerza mxima (F0 ) entre los cuerpos. El impulso transmitido es a
t0 +

p =
t0

F0

1
2

(t t0 )2

4 dt = F0 . 3

Con este modelo para F (t), en el caso de la bolita obtenemos 4 2m 2gh = F0 . 3 De esta relacin se deduce que la fuerza mxima entre la bolita y la supere de la cual o a rebota depende del tiempo durante el cual los dos cuerpos se mantienen en contacto. Tanto menor es este intervalo de tiempo, tanto mayor resulta ser F0 . Veamos un ejemplo numrico. e Supongamos que una bolita de 50 g se hace caer desde una altura de 1 m. Supongamos adems que el intervalo de tiempo durante el cual los dos cuerpos se mantienen en contacto a es aproximadamente 5 ms, o sea, 2 = 5 105 s. Con g = 10 m/s2 se tiene: F0 = 3 0,05 2 10 1 [N] 5 2 5 10 67 [N] ,

fuerza que es ms de cien veces mayor que el peso de la bolita. Si la bolita en lugar de acero a es de goma, el tiempo de colisin aumenta considerablemente (en un orden de magnitud y o

162

Momento lineal y colisiones

ms), siendo la fuerza F0 , por lo tanto, menor en ese mismo factor. Esto explica por que a es posible que si se lanza una bolita de acero y una bolita de goma de iguales masas y con la misma velocidad contra un vidrio, la bolita de acero rompe el vidrio mientras que la de goma rebota.

6.4.
1.

Problemas
Un muchacho de m1 = 40 kg se encuentra sobre una plataforma mvil de m2 = 10 kg o junto a dos ladrillos de m = 5 kg. La plataforma puede desplazarse, sin roce, sobre un terreno horizontal. En cierto instante el muchacho lanza horizontalmente uno y despus el otro ladrillo hacia atrs de la plataforma, con una velocidad v0 = 2 m/s e a respecto a l mismo. Qu velocidad v adquirir la plataforma? e e a Respuesta: con M m1 + m2 . v = v0 m(2M + 3m) = 0, 348 (M + m)(M + 2m) m/s ,

2.

Se lanza un proyectil con una velocidad inicial de 20 m/s formando un ngulo de a 30 respecto a la horizontal. En el transcurso de su vuelo, el proyectil explota rompindose en dos partes, una de las cuales tiene el doble de masa que la otra. Los e dos fragmentos aterrizan simultneamente. El fragmento ms ligero aterriza a 20 m a a del punto de lanzamiento y en la direccin y sentido en que se dispar el proyectil. o o Dnde aterrizar el otro fragmento? o a Una part cula de masa m y velocidad v0 choca contra otra idntica en reposo. La e part cula en reposo tiene un resorte de constante de restitucin k, al cual queda o adosada la part cula incidente (ver gura adjunta). Describa el movimiento del sistema despus de la colisin lo ms detae o a lladamente posible. Un bloque de madera de 1 kg se sujeta a un resorte de constante de restitucin o k = 200N/m. El conjunto descansa sobre una supercie lisa, como se ve en la gura. Se dispara contra el bloque un proyectil de 20 g y el resorte se comprime 13.3 cm. a) b) Encuentre la velocidad del proyectil antes del choque. Qu fraccin de la energ mecnica e o a a inicial se pierde en el choque? Figura 6.4

3.

Figura 6.3

4.

6.4 Problemas 5.

163

Una granada, que se encuentra en reposo, explota partindose en dos fragmentos. e Sean m1 y m2 las masas de los dos fragmentos. Demuestre que las energ cinticas as e de los fragmentos vienen dadas por T1 = y T2 = m1 Q , m1 + m2 m2 Q m1 + m2

donde Q es la diferencia entre la energ cintica total del sistema despus y antes de a e e la explosin. (Q es la energ liberada por la explosin de la granada.) o a o 6. Un ncleo de 236 U (masa=236 amu), inicialmente en reposo, se divide en dos fragu mentos de masas 146 y 90 amu. El valor Q de la reaccin es de 190 MeV. Encuentre la o velocidad y energ (en MeV) de cada uno de los fragmentos. (1 amu=1.7 1027 Kg, a 1 MeV=1.61013 J.) Un ascensor sube en un pozo a razn de 2 m/s. En el instante en que el ascensor o est 18 m del extremo superior del pozo, se deja caer una pelota desde la parte a superior del pozo. La pelota rebota elsticamente sobre el techo del ascensor. Hasta a qu altura subir la pelota en relacin a la parte superior del pozo? Resuelva el mismo e a o problema suponiendo que el elevador se mueve hacia abajo a 2 m/s. Una part cula A que inicialmente tiene una velocidad v0 choca contra otra que se encuentra en reposo, desvindose (la part a cula incidente) en un ngulo . La velocidad a de A despus del choque es v. Por otra parte, la segunda part e cula se desv en un a a ngulo respecto a la direccin inicial de la primera part o cula. Demuestre que tan = v sin . v0 v cos

7.

8.

9.

Considere una plataforma de ferrocarril de peso W que puede rodar sin friccin sobre o una v recta y horizontal (ver gura ada junta). Inicialmente la plataforma se mueve a la derecha con velocidad v0 . Sobre la plataforma se encuentra de pie un hombre de peso W .

Figura 6.5

Cul es el cambio de velocidad de la plataforma si el hombre corre a la izquierda, a de modo que su velocidad con relacin a la plataforma es u cuando est a punto de o a saltar del extremo izquierdo?

164 10. Un objeto de masa m = 5 kg, resbala por una rampa de masa M =10 kg, partiendo desde una altura h = 2 m. La rampa puede deslizarse, sin roce, sobre el suelo. Una vez que la masa llega abajo, con qu vee locidad se mueve la rampa?

Momento lineal y colisiones

Figura 6.6

11.

Un satlite, que se mueve en l e nea recta con una rapidez v respecto a algn sistema u de referencia, recoge part culas (polvo) csmicas (que, supondremos, se encuentran o en reposo en el sistema de referencia en cuestin) a una tasa dM/dt = k v, donde k o es una constante. Si en el instante t = 0 la velocidad del satlite es v0 y su masa es e M0 , encuentre v(t). Sobre una supercie horizontal sin friccin se encuentran en reposo N bloques de o igual masa m colocados uno al lado del otro en l nea recta separados por una pequea n distancia. (Los bloques estn enumerados de izquierda a derecha como N , N 1, a N 2, . . . , 2, 1.) Desde la izquierda incide sobre ellos un bloque de masa M > m, con velocidad v0 . Suponga que todos los choques son perfectamente elsticos. a

12.

Figura 6.7 a) b) Calcule la velocidad nal de cada bloque despus que han ocurrido todos los e choques. Muestre expl citamente (realizando los clculos antes y despus de todos los a e choques) que se conserva el momento lineal y la energ cintica del sistema. a e

13.

Aldo y su hermano Pablo han ido a patinar a una laguna helada. Al mediod Aldo a se detiene para abrir el paquete del almuerzo, cuando observa que Pablo se aproxima directamente hacia l con una rapidez v. Antes del impacto Aldo le lanza a su hermano e el paquete en direccin horizontal, el cual es atrapado en el aire por Pablo. Si ambos o hermanos tienen la misma masa M , y la masa del paquete es m, calcule el valor m nimo de la rapidez con la cual Aldo debe lanzar el paquete para que su hermano no choque con l. e

6.4 Problemas 14. Considere un pndulo consistente de e una masa m colgada de un hilo de largo L. Suponga que el pndulo inie cialmente parte con el hilo en posicin horizontal. Al llegar la masa o al punto inferior (punto O de la gura 6.8), choca elsticamente con a una masa M = 2m que se mueve con velocidad v0 x. El pndulo re e bota (hacia atrs) llegando tener coa mo amplitud mxima nuevamente la a horizontal. a) b) 15. Cul es la velocidad de M despus del choque? a e

165

Figura 6.8

Encuentre la rapidez inicial v0 en funcin de m, M , L y g. o

Considere la conguracin mostrada o en la gura adjunta. Suponga que en el instante t = 0 todas las masas estn en reposo. La masa #1, a despus de avanzar una distancia e h/2, colisiona inelsticamente con la a masa m quedando adosada a ella. Cul es la velocidad de la masa #1 a justo antes de la colisin? Cul es la o a velocidad de la masa #2 justo despus de la colisin? Con qu veloe o e cidad llega la masa #2 al suelo? Un proyectil de masa m = 20 g, que se desplaza con velocidad v, choca contra un bloque de masa M = 0, 48 kg que se encuentra en reposo sobre una supercie lisa. El proyectil queda incrustado en el bloque.

Figura 6.9

16.

Figura 6.10

A continuacin, el sistema proyectil bloque resbala sobre una zona donde el coeciente o de roce cintico con el bloque es c = 0, 3. El sistema se detiene despus de haber e e resbalado 10 m sobre esa zona rugosa. Encuentre la velocidad inicial del proyectil. 17. Sobre una plataforma horizontal lisa (sin roce) se colocan en l nea recta 99 bloques de igual volumen y masas 2m, 3m, 4m, . . . , 99m y 100m, separadas entre s por una distancia a (ver gura 6.11). Desde la izquierda incide una part cula de masa m con velocidad v0 . Todos los choques son perfectamente elsticos. a a) Calcule la velocidad de la part cula de masa 2m inmediatamente despus de la e primera colisin. o

166 b) c)

Momento lineal y colisiones Calcule la velocidad de la part cula de masa 2m inmediatamente despus que e experimenta el segundo choque. Despus de un tiempo sucientemente largo se observa que ningn bloque pere u manece sobre la plataforma. Cuntos bloque cayeron al lado izquierdo y cuntos a a al lado derecho?

Figura 6.11 18. Sobre un plano liso se encuentran tres discos iguales (de radio R y masa M ). Al disco A, que incide con velocidad v0 choca simultnea y elsticamente con a a los discos B y C, tal como se muestra en la gura 6.12. Los discos B y C inicialmente se encuentran en reposo con los centros separados en una distancia 2R+2a. Suponga que no hay roce entre los bordes de los discos cuando estn a en contacto. Encuentre la velocidad del disco A despus de la colisin. e o Respuesta: vf = v0 2 2 , 6 2 con =

Figura 6.12

R+a . R

19.

Un objeto de masa m resbala sobre la supercie lisa de una cua M . La n cu`a reposa sobre una supercie tamn bin lisa (ver gura). Originalmente el e objeto se encuentra en reposo a una altura h medida desde el tramo horizontal. Figura 6.13 a) b) Calcule las velocidades de la cua y de la masa m una vez que m ha llegado al n tramo horizontal de la cua y se desplaza hacia la derecha. n Posteriormente, la masa m choca elsticamente con la parte posterior de la cua. a n Calcule la rapidez de m y M despus del choque. e

6.4 Problemas 20. Una bola de goma se deja caer sobre una cpula semiesfrica de radio R. La u e bola se suelta a una altura H desde el suelo y a una distancia b con respecto a la vertical que pasa por el centro de la cpula. La bola choca elsticamente u a con la cpula. Calcule la altura h de u rebote de la bola con respecto al suelo. Determine el valor mximo de b para a que la respuesta tenga sentido.

167

Figura 6.14

21.

En la gura se muestran las direcciones incidente y de rebote de un cuerpo que choca contra una pared sin roce. El cuerpo incide con rapidez v0 y con una direccin que forma un ngulo con la o a normal de la pared. El cuerpo emerge con rapidez v0 (con < 1). Determine la direccin ( en la gura) con que o emerge el cuerpo y el vector impulso que la pared imprime al cuerpo.

Figura 6.15

22.

Una bala de masa 5 gramos atravieza un saco de virutas de 1 kg de masa. El saco cuelga de un cordel de 2 m de largo. A consecuencia del impacto el saco entra en movimiento y se detiene cuando el cordel forma un ngulo de 12 con a la vertical. Calcule la rapidez de la bala una vez que ha emergido del saco si su velocidad antes de la colisin es o v0 = 1000 m/s.

Figura 6.16

23. A y B son dos esferitas de igual masa m engarzadas en el eje horizontal. B est unida a a un resorte ideal de largo natural l0 y rigidez (constante de restitucin) k. Inicialmente o B est en reposo, el resorte en direccin vertical y sin deformacin. A se desliza con a o o velocidad v desconocida, choca con B y ambas permanecen unidas tras la colisin. o Calcular v, si en el instante en que el conjunto se detiene el ngulo tiene un valor a de 60 . Suponga que el roce es despreciable.

168

Momento lineal y colisiones

Figura 6.17

24.

Dos carros B y C, de masas m y 2m respectivamente, se encuentran inicialmente en reposo sobre una v horizontal recta, separados por una distancia L. Un tercer carro a A, de masa 2m, que se desplaza con velocidad v0 hacia la derecha embiste al carro B desde la izquierda (ver gura). Suponga que todos los choques que ocurren son elsticos. En estas condiciones, a a) b) Demuestre que el carro B choca dos veces con el carro A. Calcule el tiempo que media entre estos dos choques.

Figura 6.18

25.

Una cuerda (exible), de masa M y largo L se deja caer sobre una pesa. Inicialmente el extremo inferior de la cuerda justo toca la pesa y la cuerda se encuentra en reposo. Demuestre que la lectura de la pesa justo antes de que caiga sobre ella el ultimo segmento de la cuerda, ser W = 3M g. a Figura 6.19

6.5 Solucin a algunos de los problemas o

169

6.5.

Solucin a algunos de los problemas o

Solucin al problema 6 o Las energ cinticas totales antes y despus del proceso de sin son as e e o Ki = 0 y 1 1 2 2 = m1 v1 + m2 v2 . 2 2 La reaccin es exotrmica, es decir, la energ nal es superior a la energ inicial en Q = o e a a 190 MeV. Se tiene entonces que Kf = Kf + Kf
(1) (2)

Kf Ki = Kf = Kf + Kf La conservacin del momento lineal nos da la relacin o o m1 v1 = m2 v2 . Usando esta relacin se deduce que o Kf
(1) (2)

(1)

(2)

=Q.

Kf De las relaciones

1 2 2 m1 v1 1 2 2 m2 v2

m2 . m1

Kf + Kf y Kf se deduce que Kf y Kf De la relacin o Kf se deduce que


2 v1 = (1) (2) (1) (1) (2) Kf

(1)

(2)

=Q

m2 m1

= =

m2 Q = 117, 5 m1 + m2 m1 Q = 72, 5 m1 + m2

MeV MeV .

1 m2 2 = m1 v1 = Q 2 m1 + m2

m2 2Q 146 2 190 1, 6 1013 J = m1 m1 + m2 90 236 1, 7 1027 Kg v1 = 1, 57 107 m . s

2, 46 1014

m s

o sea,

170 Para la velocidad del fragmento mas pesado se obtiene v2 =

Momento lineal y colisiones

m1 m v1 = 0, 97 107 . m2 s

Solucin al problema 11 o Sea v(t) la velocidad y M (t) la masa del cohete (incluyendo la del polvo recolectado). La conservacin del momento lineal nos da la relacin o o M (t)v(t) = M (t + dt)v(t + dt) . Para un intervalo de tiempo dt innitesimal se tiene que M (t + dt) = M (t) + y v(t + dt) = v(t) + Usando estas relaciones se obtiene M (t)v(t) = M (t + dt)v(t + dt) dM dv = M (t) + dt v(t) + dt dt dt dv dM dM dv = M (t)v(t) + M (t) dt + v(t) dt + (dt)2 dt dt dt dt Despreciando el trmino de orden (dt)2 , se obtiene e M (t) dv = Pero dM/dt = kv, luego M (t) dv = kv 2 dt . Multiplicando esta relacin por v = v(t) se obtiene o M (t)v(t) dv = kv 3 dt . Pero como el momento lineal se conserva, se tiene que M (t)v(t) = M0 v0 , donde M0 y v0 son la masa y velocidad del satlite en el instante t = 0. Usando esto, la ultima ecuacin e o queda dv k = dt . 3 v M0 v0 dM v(t) dt . dt dv dt . dt dM dt dt

6.5 Solucin a algunos de los problemas o

171

Integrando la ultima ecuacin desde el instante t = 0 (en que la velocidad es v0 ) hasta el o instante t (en que la velocidad es v(t)) se obtiene
v v0

1 dv = v3
v

k M0 v0 k M0 v0 t

dt
0 t 0

1 2 2v 1 2 1 1 2 v 2 v0

=
v0

k t M0 v0

Despejando v(t) se obtiene nalmente v(t) = v0 1+


2kv0 t M0

Solucin al problema 14 o Para que el pndulo vuelva a su posicin inicial, el choque elstico en la parte inferior debe e o a ocurrir con momentum total nulo. Luego, despus de la colisin la masa M tendr una e o a velocidad v0 x. La velocidad v1 con que la masa m llega abajo, se obtiene de la relacin (conservacin de o o energ a): 1 2 mgL = mv1 , 2 de donde v1 = 2gL . Para que la colisin ocurra con momento lineal cero la rapidez de la masa M debe satisfacer o la relacin o mv1 = M v0 , de donde se obtiene para v0 el resultado v0 = m m v0 = M M 2gL .

Solucin al problema 15 o En todo instante la part cula #2 se mueva con la mitad de la velocidad de #1. Justo antes del choque la velocidad vi de la masa #1 viene dada por (conservacin de la energ o a): Mg o sea,
2 vi =

h 1 1 vi 2 = (M )vi + M 4 2 2 2 2gh . 5

5 2 = M vi , 8

172

Momento lineal y colisiones

Mientras ocurre el choque (o sea, durante un brev simo intervalo de tiempo [0, ]) la cuerda ejerce una gran fuerza sobre la masa #1 y #2 transmitiendo un cambio de momento lineal p: p =
0

(t) dt ,

donde (t) es la tensin de la cuerda en funcin del tiempo. Este impulso aumenta el o o momento lineal de la masa #1 en la magnitud p y disminuye el momento lineal de la masa #2 en el doble de esa misma magnitud. Si vf es la velocidad de la masa #1 justo despus de la colisin, entonces la ecuacin de conservacin del momento lineal para la e o o o part cula #1 (junto con la masa m) da la relacin o M vi = (M + m)vf + p . Para la M Despejando v2 se obtiene vf vi =M 2p 2 2

5M . 5M + 4m Para encontrar la velocidad v con que la masa #2 llega al suelo usamos nuevamente el principio de conservacin de la energ o a: v2 = v1 vf 1 1 2 (M + m)vf + M 2 2 2
2

+ Mg

3h 1 1 = (M + m)(2)2 + M v 2 . v 4 2 2

Despejando v y usando las expresiones para v2 y v1 se obtiene v 2 = gh 2M (5M + 3m) . (5M + 4m)2

Ejercicio: Verique que tanto para m = 0 como para m , la ultima ecuacin entrega o el valor correcto para v .

Solucin al problema 19 o El momento lineal inicial es cero, y como no hay fuerza externa actuando sobre el sistema, ser nulo siempre. Cuando m llega abajo ( a la parte horizontal), sean vm y vM las velocia dades de la masa m y de la cua, respectivamente. La conservacin del momento lineal y n o de la energ nos da las relaciones a mvm + M vM = 0 y 1 1 2 2 mgh = mvm + M vM . 2 2 de estas dos ecuaciones (con las incgnitas vm y vM ) se obtiene o vm = 2gh M . M +m

Despejando vm

6.5 Solucin a algunos de los problemas o

173

El signo + corresponde a la solucin del problema antes de la colisin de m con la pared o o mientras que la con el signo es la velocidad de m despus del rebote. La velocidad de la e cua es n m m vM = vm = 2gh . M M +m Solucin al problema 23 o La colisin entre las dos esferitas es un proceso no elstico, por consiguiente, para el proceso o a de colisin slo podemos usar la ley de conservacin del momento lineal. Sea v1 la velocidad o o o de las dos masas unidas justo despues de la colisin. La conservacin del momento lineal o o nos da la relacin o mv = (2m)v1 , o sea, v1 = v/2. Luego las dos masas permanecen unidas, el resorte se estira y el sistema queda en reposo cuando el resorte forma un ngulo con la normal (ver gura 6.17). a Una vez ocurrida la colisin, la energ se conserva. Aplicando la ley de conservacin de o a o energ al sistema que consiste de las dos esferitas y el resorte se obtiene a 1 1 2 (2m)v1 = k (L)2 , 2 2 donde L es el alargamiento que sufre el resorte. Este viene dado por L =
2 0

+(

tan )2

1 1 . cos

Despejamos v1 de las dos ultimas ecuaciones: v1 = k 2m


0

1 1 cos

Con cos = cos 60 = 1/2 se obtiene para v v = 2v1 =


0

2k . m

Solucin al problema 24 o Usando las relaciones que se encontraron en la seccin 6.2 para el choque elstico se deduce o a inmediatamente que, despus de la primera colisin, el carro A (de masa 2m) tendr la e o a velocidad 2m m v0 vA = v0 = 2m + m 3 mientras que la velocidad de B ser a vB = 2(2m) 4v0 v0 = . 2m + m 3

174 El carro B tardar un tiempo a

Momento lineal y colisiones

L 3L = . vB 4v0 para colisionar con el carro C. Esta colisin tambin es elstica. Despus de esta segunda o e a e colisin la velocidad de B ser o a m 2m vB 4v0 vB = vB = = . m + 2m 3 9 El signo negativo indica que B ahora se mueve hacia la izquierda, y por consiguiente, necesariamente debe colisionar denuevo con el carro A (que sigui movindose hacia la o e derecha con velocidad vA ). Cuando B colisiona con C, la separacin entre A y B es L = L vA t1 = L L/4 = 3L/4. o De ah en adelante B se mueve hacia la izquiera con velocidad vB = 4v0 /9, mientras que A sigue movindose hacia la derecha con velocidad vA = v0 /3. La rapidez relativa con que e se acercan es vr = v0 (4/9 + 1/3) = 7v0 /9. La distancia L ser cubierta en un tiempo a t1 = L 3L 9 27L = = . vr 4 7v0 28v0 El tiempo total que transcurre entre las dos colisiones de A con B es t1 + t2 , es decir, 12L ttot = t1 + t2 = . 7v0 t2 =

6.6.

Colisin de dos discos o

Un disco de radio R y masa m, que incide con velocidad v0 , colisiona elsticamente con un a disco de masa M y radio R, que inicialmente se encuentra en reposo. La colisin ocurre o con un parmetro de impacto b (ver gura 6.20). Encuentre el ngulo de dispersin del a a o disco incidente. Suponga que no hay friccin entre los discos y la supercie sobre la que se o deslizan, y que tampoco hay roce entre los bordes de los discos mientras estos colisionan. En el lado izquierdo de la gura 6.20 se muestra la situacin (vista por un observador para o el cual inicialmente la part cula M est en reposo) antes de la colisin y al lado derecho la a o situacin que se tiene despus de la colisin. o e o

6.6 Colisin de dos discos o Figura 6.20

175

Solucin anal o tica: Al no haber roce entre los bordes de los dos discos mientras colisionan, la fuerza de interaccin entre los discos necesariamente o ser perpendicular al per a metro, o sea, en la direccin radial. Sea z la direccin deo o nida por la velocidad del disco incidente. De la gura 6.21 se desprende que el disco M , que inicialmente est en reposo, desa pus de la colisin se mover formando un e o a a ngulo con respecto a z , donde viene dado por b sin = . (6.5) 2R

Figura 6.21 Sean vm y vM las rapideces de los dos discos despus de la colisin. La conservacin del e o o momento lineal tanto en la direccin y como en la direccin z nos da las ecuaciones o o M vM sin = m vm sin y mv0 = M vM cos + m vm cos . (6.7) El principio de la conservacin de la energ (recordemos que el choque es elstico) nos o a a entrega la relacin o 1 1 1 2 2 2 mv0 = mvm + M vM . (6.8) 2 2 2 En las ecuaciones (11.2), (11.3) y (11.4), las incgnitas son vm , vM y . o Despejemos vM de la ecuacin (11.2) y sustituymosla en las ecuaciones (11.3) y (11.4). De o a esta manera se obtienen las relaciones mv0 = mvm cos + mvm y
2 2 M mv0 = M mvm + mvm

(6.6)

sin sin sin sin

cos ,
2

(6.9)

(6.10)

respectivamente. Reordenando las dos ultimas ecuaciones, stas pueden escribirse de la e forma cos v0 = vm cos + sin , (6.11) sin y m sin2 2 2 v0 = vm 1 + . (6.12) M sin2

176

Momento lineal y colisiones

Elevando al cuadrado la penltima ecuacin y luego dividindola por la ecuacin (11.7), se u o e o obtiene cos 2 m sin2 cos + sin =1+ . (6.13) sin M sin2 A partir de esta ecuacin, usando relaciones trigonomtricas elementales se obtiene o e cos2 + 2 cos sin cos cos2 + sin2 sin sin2 = 1+ m sin2 M sin2 m sin2 , M sin2

= cos2 + sin2 +

2 cos sin

cos m sin2 cos2 = sin2 + . + sin2 sin M sin2 sin2

Multiplicando por sin2 y dividiendo por sin2 queda 2 cos sin cos m + cos2 = sin2 + , sin M

m sin(2) = cos(2) + , tan M de donde nalmente tan =


m M

sin(2) . cos(2)

(6.14)

La ultima ecuacin da el ngulo de dispersin de la part o a o cula incidente en funcin de , o a ngulo que a su vez est determinado si se conoce el parmetro de impacto b (ver ecuacin a a o (11.1)). Analicemos brevemente dos casos l mites: i) L mite M . Cuando la masa del blanco es extremadamente grande comparada con la masa del disco incidente, entonces el disco M no se mover y la colisin a o ser especular. Cuando la dispersin es a o especular, el ngulo de rebote del disa co m respecto a la normal es igual al a ngulo incidente. De la gura 6.21 se observa de inmediato que en ese caso la relacin entre el ngulo de dispersin y o a o el ngulo es a = 2 . (6.15)

Figura 6.22

6.6 Colisin de dos discos o

177

Demostremos que se obtiene el mismo resultado a partir de la ecuacin (6.14). Para M o sta queda e tan = tan(2) . Pero tan(2) = tan( 2) , luego tan = tan( 2) . De la ultima ecuacin se desprende inmediatamente el resultado (6.15). o ii) L mite M = m. Cuando M = m entonces la ecuacin (6.14) queda o tan = sin(2) 1 cos(2) 2 sin cos = 1 cos2 + sin2 = cot (6.17) (6.16)

(6.18)

De esta ultima relacin se desprende que o += . 2 (6.19)

Este es un resultado general: siempre que colisiona un objeto elsticamente con otro de a igual masa que inicialmente est en reposo, la suma de los ngulos de dispersin de ambos a a o objetos ser de 90 . a

Solucin grca o a Demostremos ahora el resultado dado por la ecuacin (6.14) usando un mtodo grco. o e a La gura 6.23 muestra los discos antes y despus de la colisin vistos desde dos sistemas e o de referencia distintos: el sistema del laboratorio (en que M inicialmente est en reposo) y a el sistema de coordenadas jo al centro de masas. A pesar de que el concepto de centro de masas se estudiar recien en el siguiente cap a tulo, para resolver el presente problema basta con saber que el sistema de referencia del centro de masas corresponde al referencial de un observador que se mueve con velocidad uniforme respecto al laboratorio y para el cual el momento lineal total antes de la colisin es nulo. Tal sistema de referencia es igual al del o laboratorio, un sistema de referencia inercial, es decir, en l tambin se cumplen las leyes e e de Newton.

178

Momento lineal y colisiones

Figura 6.23

Notacin: Para hacer ms transparente lo que sigue, a las magnitudes f o a sicas, cuando stas e estn medidas (observadas) desde el sistema de referencia del centro de masas, le agregae remos una prima. A las magnitudes despus de la colisin les agregaremos adicionalmente e o una tilde. El momento lineal se conserva para ambos observadores. En particular, para el observador en el sistema de referencia del centro de masas, el momento total de las part culas siempre es nulo (en efecto, sta ser, como veremos en el siguiente cap e a tulo, la denicin del sistema o de referencia del centro de masas). Resolvamos primero el problema para el observador del centro de masas. Tal como se mencion en el prrafo anterior, los momentos lineales de los discos, en todo instante, en o a particular antes y despus de la colisin, deben ser de igual magnitud pero apuntar en e o sentidos contrarios. Si la colisin es elstica entonces, adems, las magnitudes del momento o a a lineal, antes y despus de la colisin, deben ser iguales. O sea, medidos desde el CM, los e o momentos lineales de los dos discos, antes y despus de la colisin, tienen todos la misma e o magnitud. Si los cuatro vectores tienen la misma magnitud, las puntas de estos vectores deben estar ubicados todos sobre un c rculo (ver gura 6.24).

6.6 Colisin de dos discos o

179

Figura 6.24

Figura 6.25

En el sistema de referencia del centro de masas es fcil determinar los ngulos de dispersin a a o de los dos discos. Cuando no hay roce, la fuerza, y por consiguiente, el cambio de momento lineal de cada disco, debe ser perpendicular a la supercie de contacto. De la armacin o anterior es fcil deducir que la colisin (vista desde el CM) ser especular (ver gura 6.25). a o a De las guras 6.24 y 6.25 se deduce que el ngulo de dispersin de m viene dado por a o

= 2 ,

(6.20)

donde sin = b/(2R). Volvamos al sistema de referencia del laboratorio. Para pasar del sistema de referencia del centro de masas al del laboratorio, debemos sumar a todas las velocidades la velocidad del movimiento relativo entre los dos observadores (esto es, la velocidad del centro de masas). Para realizar esto grcamente debemos primero transformar la gura 6.24, que es a un grco de momentos lineales, a una de velocidades. Pero, para transformar momentos a lineales en velocidades basta con dividir a los vectores momento lineal correspondientes a cada part cula por su masa. Si los dos discos no tienen la misma masa, entonces los vectores de velocidad de las part culas ya no quedarn sobre el mismo c a rculo. Supongamos que M es mayor que m, entonces los vectores velocidad correspondientes a la masa M sern menores a que los del disco m y, en este caso, se obtiene una representacin grca como la mostrada o a en la gura 6.26).

180

Momento lineal y colisiones

Figura 6.26

Figura 6.27

Elijamos como unidad de magnitud para medir las rapideces a la rapidez que la part cula m tiene en el sistema de referencia del centro de masas, o sea, la rapidez (medida desde el sistema de referencia del centro de masas) de la part cula m, tanto antes como despus de e la colisin, es 1. La rapidez de la part o cula M , en esas unidades, es m/M . Como ya hemos mencionado, para pasar del sistema de referencia del centro de masas al sistema del laboratorio, debemos sumarle a todos los vectores velocidad del diagrama 6.26, la velocidad relativa de los dos observadores (que es vM ). En lugar de sumar un vector constante a todos los vectores de un grco resulta ms cmodo a a o simplemente mover el origen en la direccin contraria en esa misma magnitud. Trasladndo o a el origen en la cantida vM se obtiene la gura 6.27. Cuando los vectores velocidad se observan desde el sistema de centro de masas debe usarse el origen Ocm , mientras que si sto se realiza desde el sistema de referencia del laboratorio, hay que usar el origen Olab . e A partir de la gura 6.27 es fcil determinar el ngulo de dispersin . La gura 6.28 muestra a a o el detalle del tringulo relevante. Se tiene: a a = vm sin(2) = sin(2) m cos(2) M

b = vM vm cos(2) = y tan = a = b
m M

sin(2) . cos(2)

6.6 Colisin de dos discos o

181

Figura 6.28

Figura 6.29

Esta ultima ecuacin coincide con el resultado que hab o amos obtenido anteriormente en forma anal tica (ver ecuacin (6.14)). o Si M < m, entonces el punto Ocm cae fuera del c rculo original (ver gura 6.29). Note que en este caso el ngulo de dispersin siempre ser menor que un valor mximo dado por a o a a sin max = M . m

Problema: Considere una part cula de masa m y velocidad v0 que colisiona con otra part cula de masa m, inicialmente en reposo. La energ cintica de la part a e cula incidente es E0 = 2 mv0 /2. La part cula # 2 emerge de manera que su vector velocidad forma un ngulo de 45 a (medido en el sistema de referencia del laboratorio) con respecto a la direccin incidente. o Adems de esta informacin se sabe que el choque es inelstico, siendo Q = 0, 18 E0 la a o a energ que desaparece del movimiento relativo (indose a los grados de libertad internos a e del sistema, transformndose, por ejemplo, en calor). Encuentre grcamente los ngulos a a a (hay dos soluciones) en que emerge la part cula # 1.

182

Momento lineal y colisiones

Cap tulo 7

Torque, centro de masas y equilibrio


7.1. Producto vectorial

Para lo que sigue, necesitamos introducir una nueva operacin entre dos vectores, llamada o producto vectorial o producto cruz. Denicin: o Sean A y B dos vectores. Entonces denimos el vector C, que es el producto vectorial de A y B, por: C = A B |A| |B| sin C , (7.1) donde es el ngulo (ms pequeo) entre los dos vectores A y B, y C es un vector unitario a a n perpendicular al plano engendrado por los vectores A y B. Hay dos vectores unitarios que son perpendiculares al plano engendrado por los vectores A y B. Por convencin debe usarse el que se obtiene usando la regla de la mano derecha. o Regla de la mano derecha: Empue la mano y estire el dedo pulgar. Oriente los dedos n empuados de manera que apunten a lo largo del ngulo (desde A hacia B); entonces el n a pulgar indica la direccin y sentido del vector C. o De la denicin se desprende que el producto cruz de dos vectores es otro vector. Notemos o que la denicin del vector C es independiente de cualquier sistema de coordenadas. Es o inmediato que xx=yy =zz =0, x y = x = z , y y z = y = x z y z x = z = y . x

184

Torque, centro de masas y equilibrio

Una caracter stica importante del producto cruz es que no es conmutativo, sino anticonmutativo; en efecto, de la denicin se observa inmediatamente que: o A B = B A El producto cruz es distributivo respecto a la suma de vectores: A (B + C) = A B + A C y (A + B) C = A C + B C . Evaluemos el producto cruz entre los dos vectores A y B en trminos de sus coordenadas. e Sean A y B dos vectores A = (Ax , Ay , Az ) = Ax x + Ay y + Az z B = (Bx , By , Bz ) = Bx x + By y + Bz z , entonces se tiene A B = (Ax x + Ay y + Az z ) (Bx x + By y + Bz z ) = Ax Bx x x + Ax By x y + Ax Bz x z + Ay Bx y x + Ay By y y + +Ay Bz y z + Az Bx z x + Az By z y + Az Bz z z = (Ax By Ay Bx ) + (Ay Bz Az By ) + (Az Bx Ax Bz ) z x y Considere el paralelgramo engendrado o por dos vectores A y B (ver gura 7.1). El rea de tal paralelgramo viene dado a o por Area = |A B| . Figura 7.1 Ilustremos el uso del producto cruz con dos problemas. Problema 1: Sean P1 =(2,1,5), P2 =(5,2,8) y P3 =(4,8,2) las coordenadas de los vrtices e de un tringulo. Calcule su rea. a a Solucin: El vector que une los puntos P1 y P2 es o A = 3 + y + 3 , x z mientras que el vector que une los puntos P1 y P3 es B = 2 + 7 3 . x y z .

7.1 Producto vectorial

185

Ahora observe que el mdulo del producto vectorial de los vectores A y B es igual al doble o de rea del tringulo, por lo tanto a a Area del = = 1 |A B| 2 1 | 24 + 15 + 19| x y z 2

17, 04

Problema 2: Sean A y B dos vectores unitarios en el plano x, y , que forman a ngulos y con el eje x, respectiva mente (ver gura 7.2). Evale el producu to cruz de estos vectores de dos maneras, una vez usando la denicin y la segunda o vez usando la expresin en trminos de las o e coordenadas cartesianas, y de esta manera encuentre una expresin para sin( + ). o

Figura 7.2

Solucin: El ngulo entre los vectores A y B es + , luego o a |A B| = |A| |B| | sin( + )| = sin( + ) . Por otra parte |A B| = |(cos x sin y ) (cos x + sin y | = |(cos sin + sin cos ) z | = cos sin + sin cos . Igualando las dos expresiones anteriores concluimos que sin( + ) = cos sin + sin cos . Ejercicios: 1. Encuentre un vector unitario A que sea simultneamente perpendicular a los vectores a u = 2 + y z y v = x y + z . Cuntos vectores unitarios A existen con esta x a propiedad? Sea A = x + 3 2. Encuentre un vector en el plano x, y que sea perpendicular a z y A. Verique la expansin del producto vectorial triple: o A (B C) = B (A C) C (A B) por expansin directa en coordenadas cartesianas. o

2.

3.

186 4.

Torque, centro de masas y equilibrio Considere los vectores A = x + y , B = y + z y C = x z . a) b) Evale el producto escalar triple A (B C). u Evale A (B C). u

5.

Encuentre un vector que sea perpendicular al plano que pasa por los puntos P1 = (1, 1, 1), P2 = (1, 2, 3) y P3 = (2, 3, 1). Respuesta: C = (4 = 2 z ) con un nmero real no nulo. x y u

6.

Encuentre un vector que apunte a lo largo de la interseccin de los planos engendrados o por los vectores A = x+2, B = x3 y C = z +2, D = y +2 , respectivamente. y y x y z

7.2.

Torque

Considere un objeto (por ejemplo, una barra) que en cierto instante se encuentra en reposo. Que la fuerza total sobre la barra sea nula, y por lo tanto (usando la segunda ley de Newton) su aceleracin sea nula, no o signica que sta no empezar a moverse. e a Una situacin de ese tipo se muestra en o la gura 7.3. La fuerza total (es decir, la suma vectorial de la dos fuerzas aplicadas sobre la barra) es nula y efectivamente la barra como un todo no se trasladar; sin a embargo, las dos fuerzas paulatinamente harn que la barra rote. a Consideremos ahora la palanca mostrada en la gura 7.4. Ignoremos por un momento el peso de la palanca. Qu fuerza dee bemos hacer para mantener la palanca en equilibrio? Ya en la antigedad los griegos u conoc la respuesta: an F = Mg x . L

Figura 7.3

Figura 7.4

La ley que describe los resultados emp ricos de este tipo de situaciones, llamada ley de las palancas, se puede enunciar fcilmente si se introduce el concepto de torque. a

7.2 Torque Denicin: El torque que genera una fuerza F respecto a un punto P es o rF ,

187

donde r es el vector que va desde el punto P hasta el lugar donde se aplica la fuerza F .

El torque es la magnitud responsable de hacer girar a los objetos. El torque apunta en la direccin del eje de giro y en el sentido dado por la regla de la mano derecha (si los dedos o empuados indican el sentido de la rotacin entonces el pulgar extendido apunta a lo largo n o del eje de giro). Note que el torque que ejerce una fuerza depende de la posicin del punto Q donde sta o e se aplica y del P respecto al cual estamos evaluando el torque. Una fuerza F , respecto a puntos distintos, ejerce torques distintos. En el objeto mostrado en la gura 7.5 se aplica una fuerza en el punto Q. La magnitud del torque se puede evaluar multiplicando el tamao de la fuerza por el brazo. n El brazo es la distancia entre el punto P y recta que indica la direccin de F que pao sa por el punto Q. Con respecto al punto P1 el mdulo del torque producido por la o fuerza F es F a, donde F = |F | y a es el brazo. El vector apunta normal a la hoja de papel en direccin del lector. Respecto o al punto P2 el torque generado por la fuerza F es nulo, ya que el brazo es nulo.

Figura 7.5

El conocimiento emp rico que se tiene sobre palancas, y en general, sobre objetos en equilibrio (es decir, objetos que no aceleran ni comienzan a rotar) permite enunciar la siguiente ley (que complementa a la primera ley de Newton):

Si el torque neto ejercido por las fuerzas que actan sobre un cuerpo, respecto u a un punto P , es nulo, entonces el cuerpo no cambiar su estado rotacional (o a sea, si no estaba rotando en torno a P , no comenzar a rotar en torno a ese a punto y si estaba rotando con cierta velocidad angular, seguir rotando con la a misma velocidad angular).

Ejercicio: Demuestre que para la situacin mostrada en la gura 7.4, el torque neto, en o torno al punto 0, ejercido por las tres fuerzas que actan sobre la palanca, es nulo. (Ignore u el peso de la palanca.)

188

Torque, centro de masas y equilibrio

7.3.

Centro de masas

La evaluacin del torque debido al peso de o un objeto se simplica considerablemente si se introduce el concepto de centro de masas, que ya mencionado en el cap tulo anterior. Consideremos dos masas m1 y m2 , unidas por una barra de masa despreciable, dentro de un campo gravitacional g = g. z Evaluemos el torque neto debido al peso de las masas en torno al punto P . Tenemos: = r1 (m1 g) + r2 (m2 g) z z (m1 r1 + m2 r2 ) (M g) z = M

Figura 7.6

Sea M = m1 + m2 y denamos rcm = entonces = rcm (M g) . z O sea, una vez conocido el vector de posicin del centro de masas rcm , podemos evaluar el o torque debido a la fuerza de gravedad suponiendo que la masa total del objeto se encuentra en ese lugar. El concepto centro de masas ha aperecido en varias oportunidades. Denmoslo y analicea mos algunas de sus propiedades: Para N masas discretas {mj }j en los lugares {rj }j , la posicin del centro de masas viene o dada por N 1 rcm = mj rj , M
j=1

1 (m1 r1 + m2 r2 ) , M

con M = m1 + m2 + + mN . Para el caso de una lmina (en el plano x, y, con el origen a en ese mismo plano) de densidad supercial (x, y), la posicin del centro de masas viene o dada por 1 (x + y y ) (x, y) dx dy . x rcm = M lmina a Para un cuerpo slido de densidad (x, y, z), la posicin del centro de masas viene dada por o o rcm = 1 M r (x, y, z) dx dy dz .
cuerpo

7.3 Centro de masas

189

Los siguientes seis ejercicios se reeren a importantes propiedades del centro de masas. Ejercicios: 1. A pesar de que el vector centro de masas rcm dependa del origen que se elija para evaluarlo, la posicin del centro de masas es independiente de la eleccin del origen. o o Sea rcm el vector posicin del centro de o masas de un objeto evaluado usando un sistema de referencia cuyo origen es O y rcm el resultado que se obtiene usando otro sistema de coordenadas cuyo origen es O . Demuestre que rcm = r cm + a , donde a es el vector que une los dos or genes. 2. Figura 7.7

Considere tres masas m1 = m0 , m2 = 3m0 y m3 = 6m0 , ubicadas en los lugares r1 = x + 3 , y r2 = + 2 y r3 = 5 + 3 2 , x z x y z

respectivamente. Encuentre la posicin del centro de masas usando los dos procedio mientos siguientes: a) Usando la frmula o rcm = b)
i mi ri j

mj

Encontrando primero el centro de masas del subsistema formado por las part culas 1 y 2 solamente y luego encontrando el centro de masas de este subsistema con la part cula # 3. Formule en palabras una generalizacin (importante) de este resultado. o

c)

3.

Convnzase de que si un objeto tiene ejes y planos de simetr entonces el centro de e a, masas se ubica en tales planos y ejes. Por ejemplo, de acuerdo a este resultado, en una esfera, un cilindro recto, etc., el centro de masas se ubicar al centro de tales objetos. a Considere un cuerpo compuesto de N masas mj , situados en los lugares rj , con j = 1, 2, 3, . . . , N . Demuestre que la energ potencial de tal cuerpo, en un cama po gravitacional constante, se puede evaluar suponiendo que toda su masa M = m1 + m2 + + mN est concentrada en su centro de masas. a

4.

190 5.

Torque, centro de masas y equilibrio Considere un cuerpo compuesto de N masas mj , situados en los lugares rj , con j = 1, 2, 3, . . . , N . Demuestre que para evaluar el torque total respecto a un punto P debido a la fuerza de gravedad (constante), basta suponer que toda la masa del cuerpo est concentrada en el centro de masas. a Considere un sistema que consiste de dos masas m y M puntuales separadas por una distancia a. Demuestre que la posicin del centro de masas del sistema se ubica sobre o la recta que las une, encontrndose ms cercano a la masa mayor. a a

6.

7.4.

Evaluacin numrica del centro de masas de un semic o e rculo

Problema: Encontrar numricamente el centro de masas de una lmina semicircular de e a radio R y densidad supercial uniforme 0 . Solucin: Dividamos el semic o rculo en N franjas de ancho dz = R/N y luego aproximemos las franjas por rectngulos (ver gura 7.8). El centro de masa del rectngulo j se a a encontrar en el lugar a rj = j dz 1 dz 2 z= R (j 1/2) z N j = 1, . . . , N .

El rea del rectngulo j viene dada por a a Aj = (ancho) (largo) = dz 2 xj R = 2 N R = 2 N

R2 R2 (j 1)2 /N 2
2

N 2 (j 1)2

Figura 7.8

7.4 Evaluacin numrica del centro de masas o e

191

Encontremos ahora el centro de masas de los centros de masas de todos los rectngulos. Se a tiene:

Rcm = =
j

j rj

(masa de rectngulo j) a R(j 1/2) z 0 2 N


N

(masa del semic rculo) R N


2

N 2 (j 1)2

0 1 2

1 R2

4R N 3

N 2 (j 1)2 (j 1/2) z
j=1

= f (N )R , z

donde

f (N )

4 N 3

N 2 (j 1)2 (j 1/2)
j=1

El valor exacto para Rcm se obtiene para valores grandes de N . Para valores de N no demasiado grandes podemos evaluar f (N ) con una calculadora (hgalo a para N = 1 y N = 2 y compare su resultado con el de la tabla). Para valores grandes de N debemos hacer un pequeo programa y usar una computadora. n Un pequeo programa en BASIC que permite evaluar f (N ) (para N = 500) es: n

PI = 3.1415926 N = 500 S=0 FOR J = 1 TO N S = S + SQR(N*N (J 1)*(J 1)) * (J 0.5) NEXT J F = 4*S / (PI*N*N*N) PRINT N , F

192 N 1 2 3 5 10 20 50 100 200 500 1000 Exacto f (N ) 0.6366 0.5826 0.5344 0.4972 0.4642 0.4456 0.4334 0.4390 0.4268 0.4254 0.42490 0.4244 Error relativo % 50 35 26 17 9.4 5.0 2.1 1.1 0.56 0.24 0.12

Torque, centro de masas y equilibrio

Figura 7.9 Los resultados que arroja este programa para distintos valores de N , se presentan en la tabla adjunta. Recurriendo al clculo integral, es posible encontrar el resultado exacto, es a decir, el valor de f (); ste resulta ser 4/(3) = 0,4244 . . . En la gura 7.9 se muestra un e grco del error relativo entre el valor nmerico y el valor exacto en funcin de N . A partir a u o de N = 100 el error es menor que un 1 %. Nota: El mtodo numrico empleado aqu para resolver el problema no es el ms eciente. e e a La bondad del mtodo empleado radica en su simpleza. e Ejercicio: Use un procedimiento anlogo para calcular la posicin del centro de masas de a o una semiesfera de radio R y densidad de masa (uniforme) 0 .

7.5.

Equilibrio

Un cuerpo (objeto o sistema), que en cierto instante est en reposo, seguir en reposo si la a a fuerza neta que actan sobre l es nula y adems el torque neto de estas fuerzas (respecto a u e a cualquier punto), tambin es nulo. Un cuerpo que est en reposo y contina en ese estado e a u se dice que est en equilibrio. a Leyes de equilibrio: Para que un cuerpo est en equilibrio es necesario que se cumplan las siguientes e dos condiciones: i) La fuerza neta sobre el objeto debe ser nula. ii) El torque neto sobre el objeto debe ser nulo.

7.5 Equilibrio Consideremos un objeto (cuerpo r gido) formado por N masas {mj } ubicadas en los lugares {rj } (respecto a un origen O) y unidas por barras r gidas sin masas (ver gura 7.10). Sea Fj la fuerza externa que acta sobre cada una de las masas mj . u A continuacin, usando esta gura, deo mostraremos dos resultados importantes: Figura 7.10 Si la fuerza neta sobre un cuerpo es cero entonces el torque neto es independiente del punto respecto del cual se evala. En particular, si el torque es nulo repecto u a un punto, tambin lo ser respecto a cualquier otro punto. e a Demostracin: o

193

o Sean {rj } y {rj } los vectores de posicin de las masas {mj } respecto a un origen O y O, respectivamente. Sea adems R el vector que une los puntos O y O. Entonces a =
j

rj Fj (R + rj ) Fj
j

= =
j

R Fj +
j

r j Fj

= R
j

Fj + = R 0 + = .

El otro resultado importante es el siguiente: Si la fuerza neta Ftot que acta sobre un cuerpo de masa M no es nula, entonces u el punto del cuerpo que es acelerado de acuerdo a la segunda ley de Newton es el centro de masas. O sea, se tiene que: Ftot = M rcm . Demostracin: o En primer lugar notemos que las barras que unen las distintas masas en la gura 7.10 transmiten fuerzas. Sea Fij la fuerza que ejerce la masa mj sobre la masa mi . Debido al principio de accin y reaccin Fji = Fij . o o

194

Torque, centro de masas y equilibrio Fij .

La fuerza que ejercen todas las dems part a culas sobre la masa mi viene dada por Por lo tanto, la ecuacin de movimiento para la part o cula i viene dada por m i r i = Fi +
j

Fij .

Sumando todas las ecuaciones (o sea, sumando sobre i) se obtiene mi ri =


i i

Fi +
ij

Fij .

Debido al principio de accin y reaccin, la ultima suma (sobre i y j) es nula, luego o o M Pero M luego Ftot = M rcm . Debido a la importancia de este resultado lo reiteramos en palabras: La fuerza neta que acta sobre un objeto acelera al objeto como un todo. El lugar geomtrico que cumple u e con la segunda ley de Newton es el centro de masas. O sea, para analizar el movimiento traslacional, toda la masa se puede pensar como si estuviese concentrada en el centro de masas, siendo se tambin el lugar en que se aplica la fuerza neta. e e Corolario: Si la fuerza neta que acta sobre un cuerpo es nula, entonces el centro de masas u del cuerpo se traslada con velocidad uniforme (o nula). Observe que no es necesario especicar el punto respecto al cual se est evaluando el torque a neto, ya que, de acuerdo a los resultados expuestos anteriormente, si la fuerza neta es nula y el torque es nulo respecto a un punto, tambin lo ser con respecto a cualquier otro punto. e a Ilustremos el uso de las leyes del equilibrio resolviendo un problema. Problema: Una escalera de masa m y largo L se encuentra apoyada contra una pared lisa (o sea, no hay roce entre la escalera y la pared), formando un ngulo con ella. Una persona de masa a M se encuentra sobre la escalera. Cul es el m a nimo coeciente de roce esttico que debe a existir entre el suelo y la escalera para que la escalera no resbale, independientemente de la altura a la que se encuentra la persona? Solucin: o
i mi ri

=
i

Fi = Ftot .

i mi ri

= rcm ,

7.5 Equilibrio

195

Introduzcamos el sistema de coordenadas mostrado en la gura adjunta. Para que el sistema se encuentre en equilibrio, la fuerza total sobre la escalera debe ser nula. Hay cuatro fuerzas actuando sobre la escalera: i) El peso de la escalera mg; esta z fuerza se aplica en el centro de masas de la escalera. ii) El peso de la persona M g. z iii) La reaccin que ejerce la pared sobre o la escalera. Como la pared es lisa (no hay roce) tal fuerza es perpendicular a la pared: Fp = Fp x. iv) La reaccin del suelo sobre la escao lera. Esta es Fs = FN z + fr x, donde FN es la fuerza normal y fr es la fuerza de roce.

Figura 7.11

La condicin de que la fuerza total sea nula nos da la relacin: o o mg M g Fp x + FN z + fr x = 0 . z z De aqu se deducen la ecuaciones FN = (m + M )g y fr = Fp . (7.3) Evaluemos el torque total en torno al origen. Como la escalera est en equilibrio, el torque a neto debe ser nulo. Se tiene: M gx y + mg o sea, Fp = g(M x + m L sin ) 2 . L cos (7.4) L sin y Fp L cos y = 0 , 2 (7.2)

De las ecuaciones (7.3) y (7.4) se encuentra que la fuerza de roce viene dada por fr = g(2M x + mL sin ) . 2L cos

El valor mximo de la fuerza de roce se obtiene cuando la persona sube hasta la parte ms a a alta de la escalera (x = L sin ). En ese caso la fuerza de roce es fr = g M + m tan . 2

196

Torque, centro de masas y equilibrio

La fuerza de roce fr debe ser menor que el mximo posible, que es e FN , o sea, se debe a cumplir la relacin o m g M+ tan e FN = e (M + m)g . 2 De aqu se deduce que el menor valor posible que puede tener e para que la escalera no resbale es 2M + m min = tan . e 2(M + m)

7.6.
1.

Problemas
Al moverse una carga q con velocidad v en presencia de un campo magntico B, acta e u sobre la part cula una fuerza (la as llamada Fuerza de Lorentz) dada por F = q (v B) . Supongamos que para determinar la direccin y magnitud de un campo magntico o e constante, un investigador realiza dos experimentos, midiendo en cada uno de ellos la fuerza que acta sobre una carga: u a) b) Primero hace pasar la carga q a travs del campo magntico con velocidad v = e e v0 x. El investigador mide una fuerza F = F0 (2 4). z y Luego hace pasar la carga q con velocidad v = v0 z , midiendo una fuerza F = F0 ( 2). y x

A partir de estos resultados encuentre el campo magntico B (en funcin de v0 , F0 y e o q). Respuesta: B= F0 ( + 2 + 4) . x y z qv0

2.

Considere una part cula cuya carga elctrica y masa es q y m, respectivamente. La e part cula se mueve en un campo magntico homogneo B = B0 z . Si en el instante e e t = 0 la part cula se encuentra en el origen (r(0) = 0) y su velocidad en ese instante es v(0) = v0 x, encuentre el vector de posicin r(t) en funcin del tiempo. (La fuerza o o que el campo magntico ejerce sobre la part e cula viene dada por la fuerza de Lorentz ; ver problema anterior.) Indicacin: recuerde lo que sabe sobre el movimiento circular o uniforme. Demuestre que la posicin del centro de masas de una lmina triangular de densidad o a uniforme se ubica en el lugar donde se cortan las tres transversales de gravedad del tringulo. a

3.

7.6 Problemas 4.

197

En cinco de los seis vrtices de un hexgono regular hay una masa m0 . Encuentre la e a posicin del centro de masas. o Encuentre la posicin del centro de masas o de una lmina de densidad (de masa) unia forme 0 y que tiene la forma indicada en la gura adjunta. Figura 7.12

5.

6.

Encuentre la posicin del centro de masas o de un disco de densidad supercial 0 y que tiene un agujero circular como se indica en la gura adjunta. Respuesta: El centro de masas del disco con agujero queda al lado opuesto de la perforacin y a una distancia a = o r2 d/(R2 r2 ) del centro del disco de radio R.

Figura 7.13

7.

Considere una estructura formada por dos barras uniformes de largos a y b, unidas de modo que forman un ngulo recto y a que cuelga con hilo desde el cielo (ver gura adjunta). Determine el ngulo de a la estructura cuando ella se encuentra en equilibrio. Figura 7.14

8.

La gura muestra un letrero luminoso de masa m que cuelga de una barra (de masa despreciable) que se mantiene horizontal con la ayuda de una cuerda. Calcule la tensin de la cuerda y la fuerza ejercida o por la barra contra la pared. Figura 7.15

9.

Describa un procedimiento que permita determinar experimentalmente la posicin o del centro de masas de una lmina plana irregular con densidad desconocida (y no a necesariamente uniforme). (Observe que al colgar un cuerpo de un punto P y estar ste en equilibrio, el centro e de masas siempre debe estar sobre la normal que pasa por P .)

198 10.

Torque, centro de masas y equilibrio Una barra, cuya masa es de 10 Kg y tiene tres metros de largo, se dobla en 45 a 1 m de uno de los extremos y se cuelga como se indica en la gura adjunta. La estructura se encuentra en equilibrio gracias a una masa M que se cuelga en uno de los extremos.

a)

Encuentre la tensin T y el valor de o M . El equilibrio es estable o inestable? Conteste nuevamente las mismas preguntas de la parte a), pero asumiendo ahora que la barra al lado izquierdo, en lugar de estar doblada hacia abajo en 45 , est doblada a hacia arriba en 45 .

b)

Figura 7.16

11.

Considere una lmina triangular unifora me, de masa M = 5 Kg, que est sujeta a a una pared con una articulacin y colgao da del cielo con una cuerda, tal como se muestra en la gura adjunta. Encuentre la tensin T de la cuerda. o Figura 7.17

12.

Encuentre la posicin de equilibrio de una o varilla de largo L colocada dentro de un pocillo. Considere al pocillo como una semiesfera de radio R y asuma que entre ste e y la varilla no hay roce. Figura 7.18

13.

Se podr fomar una torre con ladrillos a (sueltos), uno encima de otro (ver gura), de manera que el ladrillo de ms a arriba est desplazado en ms de una e a unidad con respecto al de ms abajo, a sin que la torre se desplome ? Indicacin: Comience el anlisis con los o a ladrillos superiores. Figura 7.19

7.6 Problemas 14. Tres tambores del mismo radio estn a arrumbados como se indica en la gura adjunta. Encuentre el m nimo coeciente de roce esttico que debe existir a entre los tambores y tambin entre los e tambores y el suelo de manera que el sistema no se derrumbe.

199

Figura 7.20

15.

Un tringulo equiltero, de lado a = 10 cm y masa M = 10 kg se sujeta en forma a a r gida a una polea de radio R = 4 cm. El tringulo acta de contrapeso para mantener a u en equilibrio a una masa m = 1 kg que cuelga de un hilo enrollado en la polea (ver gura 7.18)

a)

Encuentre el valor del ngulo a que mantiene el sistema en equilibrio. ( es el ngulo entre la nora mal y la altura del tringulo.) a Cul es el mximo valor de m a a para el cual el sistema se mantendr en equilibrio? a

b)

Figura 7.21

16.

De una pieza metlica cuadrada, de a densidad supercial uniforme 0 , se recorta un tringulo issceles de manea o ra tal que la lmina resultante quede a en equilibrio en cualquier posicin si se o sujeta desde el punto P . Cul es la ala tura del tringulo? a

Figura 7.22

17.

Una barra de masa M y largo L, que puede pivotear libremente en torno a O, se mantiene en equilibrio con una masa m y una cuerda, tal como se muestra en la gura adjunta. Encuentre el ngulo para el caso a en que m/M = 0,5. El equilibrio es estable o inestable? Figura 7.23

200 18. Considere un puente cuyo armazn o consiste de 14 soportes de largo a. (En la gura se observan los 7 soportes de uno de los lados.) Asuma que los soportes slo transmiten o fuerzas a lo largo de ellos, o sea, en cada unin slo se transmiten fuero o zas y no torques. Encuentre la tensin adicional (al generado por el peo so del puente) que debe soportar cada soporte si por el centro del puente pasa un camin de peso W . o

Torque, centro de masas y equilibrio

Figura 7.24

Especique en cada caso si la tensin es de compresin o de traccin. o o o 19. Una cadena de masa M y largo L se encuentra apoyada (en equilibrio) sobre un cono recto cuyo ngulo ena tre la normal y el manto es . Encuentre la tensin de la cadena. o Indicacin: Aplique las leyes de o equilibrio a un pequeo trozo (inn nitesimal) de cadena. Figura 7.25 20. Un objeto formado por tres lminas a cuadradas de lada a, homogneas y e de igual densidad, descansa sobre una supercie horizontal apoyado en dos pivotes colocados en los vrtie ces del cuadrado inferior (ver gura 7.26). a) b) Encuentre la posicin del ceno tro de masas. Calcule la razn de la magnio tud de las fuerzas de reaccin o de cada pivote.

Figura 7.26

21.

Una regla T de masa M , largo a y barra transversal b posa sobre un plano horizontal pulido como se indica. Calcule las reacciones normales en cada punto de contacto con el suelo.

Figura 7.27

7.6 Problemas 22. Considere una semiesfera homognea de radio R. Demuestre e que el centro de masas de la semiesfera esta ubicado sobre el eje de simetr y a una distancia b = 3R/8 a de la base.

201

Figura 7.28

23.

Considere una semiesfera homognea de radio R que se e encuentre sobre un plano inclinado. Existe un roce esttico que evita a que la semiesfera se desliza por el plano. Determine el mximo ngulo a a de inclinacin que puede tener el o plano para que la semiesfera no se de vuelta.

Figura 7.29

24.

Considere una semiesfera de radio R, hecha de un material de densidad 0 , que se encuentra con la parte curva sobre una supercie horizontal (ver gura adjunta). El centro de masas de una semiesfera homognea e queda sobre el eje de simetr a una a distancia b = 3R/8 de la base.

Figura 7.30

a) Encuentre la magnitud y direccin del torque, respecto al punto de apoyo, ejero cido por la fuerza de gravedad cuando la semiesfera se ladea en un ngulo . a Observe que el torque que aparece trata de hacer volver a la semiesfera a su posicin de equilibrio (o sea, la posicin de equilibrio es estable). o o b) Coloquemos ahora un cilindro homogneo hecho del mismo material, del mismo e radio R y altura h, sobre el cilindro. Determine la posicin del centro de masas o del sistema compuesto. c) Describa en palabras la condicin que debe satisfacer la posicin del centro de o o masas para que la posicin de equilibrio del sistema compuesto siga siendo estao ble. d) Encuentre la altura l mite del cilindro para la cual el sistema compuesto pierde su estabilidad.

202 25. Considere una semiesfera de radio R, hecha de un material de densidad 0 , que se encuentra sobre una supercie horizontal y apoyada contra una pared tal como se muestra en la gura adjunta. El centro de masas de una semiesfera homognea quee da sobre el eje de simetr y a una disa tancia b = 3R/8 de la base. Suponga que, entre la semiesfera y el suelo el coeciente de roce esttico es = 3/16, mientras a que entre la pared y la semiesfera el roce es nulo.

Torque, centro de masas y equilibrio

Figura 7.31

a) Haga un diagrama de cuerpo libre para la semiesfera. b) Encuentre la magnitud y direccin del torque, respecto al punto de apoyo P , o ejercido por la fuerza de gravedad cuando la semiesfera est ladeada en un ngulo a a . c) Encuentre la fuerza de roce entre la semiesfera y el suelo. d) Encuentre el ngulo de inclinacin mximo max posible para que la esfera no a o a resbale. e) Coloquemos ahora un cilindro homogneo, hecho del mismo material, del mismo e radio R y de altura h sobre el cilindro. Determine la posicin del centro de masas o del sistema compuesto. (1 punto) f) Encuentre la altura l mite hmax del cilindro a partir de la cual, para h > hmax , el sistema compuesto se da vuelta (es decir, pierde su estabilidad).

26.

Una semiesfera homognea de masa M y e radio R se ha cortado en dos mitades. El sistema se dispone con las dos mitades, cara a cara, y con la supercie de corte vertical. A n de que las mitades no se separen, una cuerda sin roce y con masas iguales en sus extremos, es dispuesta como se indica en la gura. Determine las masas m nimas a atar en los extremos de la cuerda para que las mitades permanezcan juntas.

Figura 7.32

7.6 Problemas 27. En los extremos de una barra de masa despreciable se adhieren bolas de mas m y 2m, respectivamente. El sistema posa sobre un tiesto de fondo esfrico resbaloso, e de radio igual al largo de la barra. Calcule el ngulo que la barra forma con la a vertical. Un vaso cil ndrico (abierto por arriba), de radio basal a y altura b, hecho de un material de densidad supercial uniforme, posa sobre un plano inclinado y no resbala gracias a un tope jo en el plano. Demuestre que el centro de masas se ubica a lo largo del eje y a una distancia b2 /(a + 2b) de la base. Determine el ngulo de inclinacin a o mximo del plano de modo que el vaso no a vuelque. En la gura se muestra un cilindro de masa M y radio R, el cual se ata a la muralla mediante una cuerda. Alrededor de un calado que se le ha hecho al cilindro se enrolla una cuerda ideal. De la cuerda cuelga una masa m por determinar. Si el coeciente de roce entre el suelo y el cilindro es , determine la masa mxima a colgar a para que el cilindro no rote. Un semicilindro de radio R y peso W se encuentra en equilibrio esttico sobre un a plano horizontal, con un pequeo bloque n de peso Q sobre l. El bloque est ligado e a mediente un resorte ideal de largo natural a 0 = R y constante elstica k a un punto A en el borde (ver gura). Suponga que no hay roce entre la supercie del cilindro y la masa de peso Q. Determine el ngulo de a equilibrio. Considere conocida la distancia D a la que se encuentra el centro de masas del punto O. Analice con cuidado que pasa cuando Q es pequeo. n

203

Figura 7.33

28.

Figura 7.34

29.

Figura 7.35

30.

Figura 7.36

204

Torque, centro de masas y equilibrio

7.7.

Solucin a algunos de los problemas o

Solucin al problema 12 o Elijamos el origen y los ejes tal como se muestra en la gura adjunta. Sea el ngulo que la varilla forma con la a horizontal, o sea, el ngulo ABO es . a Por ser AOB un tringulo issceles, se a o tiene que el ngulo AOB tambin es . a e Figura 7.37 Como no hay roce entre las supercies de contacto, las fuerzas de reaccin debe ser pero pendiculares a las supercies de contacto. En otras palabras: la fuerza de reaccin F1 en o O ser a lo largo del radio OA, mientras que la fuerza de reaccin F2 en B ser perpena o a dicular a la varilla. Que la fuerza total horizontal sobre la varilla sea cero nos da la relacin o F1 cos(2) = F2 sin . La relacin correspondiente para la componente vertical es o M g = F2 cos + F1 sin(2) . (7.6) (7.5)

Para que la varilla est en equilibrio tambin el torque total (respecto a O) debe ser nulo. e e La fuerza F1 no ejerce torque (ya que su brazo es cero); el peso ejerce un torque g = M g (L/2) cos y , mientras que el torque generado por F2 es 2 = F2 2R cos y . De esta manera la condicin de que el torque total sea nulo nos da la relacin o o Mg L cos = 2F2 R cos . 2 (7.7)

Tenemos tres ecuaciones con tres incgnitas. De la ultima ecuacin se deduce inmediatao o mente que M gL F2 = . (7.8) 4R De la ecuacin (7.5) se encuentra o F1 = F2 Usando (7.8) y (7.9) en (7.6), se tiene Mg = o sea, 4R 2 cos (1 cos2 ) cos = cos + = . 21 L 2 cos 2 cos2 1 M gL sin 2 sin cos + 4R 2 cos2 1 , sin . 2 cos2 1 (7.9)

7.7 Solucin a algunos de los problemas o

205

La ultima relacin es una ecuacin de segundo grado para cos ; resolvindola se encuentra o o e nalmente 1 + 1 + 82 cos = . 4 Solucin al problema 14 o La gura adjunta muestra las fuerzas que actan sobre los tambores (las echas con u l neas llenas son fuerzas que actan sobre u el tambor inferior, mientras que las echas con l neas segmentadas corresponden a fuerzas que actan sobre el tambor superior). Obu serve que no hay una fuerza horizontal entre los dos tambores inferiores. Observe tambin e que el ngulo entre la horizontal y la reca ta que une a los centros de un tambor inferior con tambor superior es de 60 , luego el sin = 3/2 y cos = 1/2.

Figura 7.38

La unicas fuerzas que producen un torque sobre el tambor inferior (respecto a su centro) son las dos fuerzas de roce. Como el torque total sobre el tambor inferior debe ser nulo se deduce que ambas fuerzas de roce deben tener la misma magnitud; llammosla fr . e La fuerza neta vertical sobre uno de los tambores inferiores debe ser nula; esto nos da la relacin o 3 1 F1 M g F2 fr = 0 . 2 2 La relacin correspondiente a las fuerzas horizontales es o 1 3 F2 = 0 . fr + fr 2 2 Como 2F1 debe ser igual al peso total de los tres tambores se tiene 3 F1 = M g . 2 Tenemos tres ecuaciones con tres incgnitas (en realidad la tercera ecuacin ya es la solucin o o o de una de las incgnitas). Para las otras dos incgnitas fr y F2 se encuentra o o 3 fr = 1 Mg 2 y 1 F2 = M g . 2 Si 1 es el coeciente de roce esttico entre el tambor inferior y el suelo, y 2 es el coeciente a de roce entre los tambores inferior y superior, entonces, para que el sistema no se derrumbe, debe cumplirse fr 1 F1 y fr 2 F2 .

206

Torque, centro de masas y equilibrio

De estas desigualdades se deduce, nalmente, que 2 3 1 y 3

2 2

3.

Solucin al problema 19 o El radio del c rculo que forma la cadena es R= L . 2

Consideremos un trozo de cadena de largo innitesimal R d. Debido a la curvatura, la tensin T ejerce sobre el pequeo trozo de o n cadena una fuerza neta FT hacia el centro O (ver gura 7.39a): FT = 2T sin d 2 r = T d r . Figura 7.39a

No hay roce entre la cadena y el cono, luego la fuerza que el cono ejerce sobre la cadena es perpendicular al manto. Sea FN la magnitud de esta fuerza. De la la gura 7.39b se desprende que FN = FN cos r + FN sin z . Por ultimo, la otra fuerza que acta sobre el u trozo de cadena, debido a la gravedad, es Fg = d Mg z . 2 Figura 7.39b d Mg z = 0 . 2

Como el trozo de cuerda est en reposo, la a suma de las tres fuerzas debe ser nula, es decir,

FT + FN + Fg = T d r + FN cos r + FN sin z + Igualando las componentes se obtienen las relaciones T d = FN cos y Mg d = FN sin . 2

7.7 Solucin a algunos de los problemas o Despejando la tensin se encuentra, nalmente o Mg . 2 tan

207

T =

Solucin al problema 22 o Para evaluar la posicin del centro de o masas de la semiesfera, la colocamos con la cara plana sobre el plano x y, haciendo coincidir el eje con z , y luego la rebanamos en tajadas de ancho dz (ver gura). Evaluemos primero la masa de la rebanada que se encuentra a la altura z. Su masa es 0 (R2 z 2 ) dz, donde 0 es la densidad de masa de la semiesfera. El centro de masa de esta rebanada por supuesto que queda sobre el eje z a la altura z.

Figura 7.40

El centro de masas de la semiesfera ser la suma de los centros de masas de cada rebanada a pesada con la masa de de cada rebanada, es decir, 1 M 0 M
R

zcm = =

z0 (R2 z 2 ) dz =
0

0 M

(zR2 z 3 ) dz
0

1 2 2 1 4 R z z 2 4

R 0

0 1 4 = R . M 4

Pero M = 20 R3 /3, luego zcm = 3R/8. Solucin al problema 24 o

a) La masa de la semiesfera es M = 20 R3 /3. El torque en torno al punto de contacto P viene dado por = M gb sin x , donde x es un vector unitario que, para la situacin mostrada en la gura adjunta, o apunta hacia el lector.

208

Torque, centro de masas y equilibrio

Figura 7.41a

Figura 7.41.b

b) Al colocar sobre la semiesfera un cilindro de altura h la posicin del centro de masas o es 5 1 h M R + Mc R + , zcm = M + Mc 8 2 donde Mc = R2 h 0 es la masa del cilindro. Reemplazando las masas de los cuerpos se obtiene 1 5 2 h2 zcm = 2 R + hR + . 12 2 3R + h c) Mientras el la posicin del centro de masas del sistema compuesto se encuentre por o debajo del centro del semicirculo (punto A), el equilibrio ser estable. (Es fcil convena a cesrse de que el torque que aparece al ladear el sistema trata de restituir al cuerpo a su posicin de equilibrio). Por lo contrario, si el centro de masas del sistema compuesto o se encuentra por encima del punto A, el equilibrio ser inestable. a d) La altura l mite h0 se obtiene cuando zcm = R. Se tiene zcm = R = 1 2 3 R + h0 5 2 h2 R + h0 R + 0 12 2 .

Despejando la altura l mite se encuentra que sta viene dada por e R h0 = . 2

Solucin al problema 25 o

a) El diagrama de cuerpo libre se muestra en la gura adjunta.

7.7 Solucin a algunos de los problemas o

209

b) La masa de la semiesfera es M = 20 R3 /3. El torque en torno al punto de contacto P viene dado por = M gb sin x , donde x es un vector unitario que, para la situacin mostrada en la gura adjunta, o apunta hacia el lector.

Figura 7.42a

Figura 7.42.b

c) La fuerza Fp que ejerce la pared sobre la semiesfera es en la direccin horizontal + o y y su magnitud es tal que el torque total respecto a P es nulo. Luego M gh sin 3 y = M g sin y . R 8 Como la fuerza horizontal total debe ser nula, y la unica otra fuerza horizontal es la fuerza de roce, se tiene Fp = Fr = Fp = M gh sin 3 y = M g sin y . R 8

d) La fuerza de roce no debe sobrepasar el valor M g, o sea, 3 3 M g sin M g = M g 8 16 1 . 2 El ngulo cr a tico es, por lo tanto, max = 30 . sin e) Al colocar sobre la semiesfera un cilindro de altura h la posicin del centro de masas o es 1 5 h M R + Mc R + , scm = M + Mc 8 2 donde Mc = R2 h 0 es la masa del cilindro. Reemplazando las masas de los cuerpos se obtiene 1 5 2 h2 scm = 2 R + hR + . 12 2 3R + h de donde

210

Torque, centro de masas y equilibrio

Figura 7.43a

Figura 7.43.b

f) La altura l mite hmax se obtiene cuando scm = R. Se tiene scm = R = 1 2 3 R + hmax 5 2 h2 R + hmax R + max 12 2 .

Despejando la altura l mite se encuentra que sta viene dada por e R hmax = . 2 Solucin al problema 30 o Las guras 7.44a y 7.44b muestran los diagramas de cuerpo libre de la masa Q y el semicilindro, respectivamente. Fr = kx es la fuerza ejercida por el resorte, W el peso del semicilindro y Q el peso del bloque que se encuentra a una distancia x de O. La distancia d entre el centro de masas y O la supondremos conocida.

Figura 7.44a

Figura 7.44.b

Debido a que ambos objetos estn en equilibrio se debe tener que la fuerza total sobre a cada uno de ellos debe ser nula, y tambin el torque total sobre el semicilindro (en torno a e cualquier origen).

7.7 Solucin a algunos de los problemas o Para el bloque Q se obtiene la relacin o Ftot = (Fr cos + Fr sin ) Q + (N1 cos z N1 sin x) = 0 . x z z

211

Igualando las componentes de los vectores de la ultima igualdad y usando el hecho que Fr = kx, se obtienen las ecuaciones, kr cos = N1 sin y kx sin = Q N1 cos . La fuerza total que acta sobre el semicilindro (que tambin debe ser nula) es u e W z + N z + (Fr cos Fr sin ) + (N1 cos z + N1 sin x) = 0 . x z Esto nos da las relaciones N1 cos kx sin W + N = 0 y N1 sin kx cos = 0 . Esta ultima ecuacin no da informacin nueva ya que coincide con (7.10). Por ultimo, o o evaluando el torque total (en torno al punto O) que acta sobre el semicilindro, se obtiene: u N1 x W d sin = 0 . (7.13) (7.12) (7.11) (7.10)

De las cuatro equaciones (7.10), (7.11), 11.28) y (7.13), con las cuatro incgnitas N , N1 , x o y , podemos despejar cos . Realizando el lgebra, se obtiene, a cos = W dk . Q2

Que pasa cuando Q es pequeo? Es claro que la solucin obtenida slo tiene sentido si n o o Q2 > W kd. Al analizar el problema (hgalo!) con ms cuidado se encuentra que = 0 (y, a a por lo tanto, x = 0) tambin (para todos los valores de Q) es una solucin de este problema e o 2 < W kd, la solucin = 0 es la unica, de equilibrio. Tambin se encuentra que para 0 Q e o siendo estable. Para W dk < Q2 , hay tres soluciones: = 0 y = Acos (W dk/Q2 ); siendo la primera de stas inestable, y estables las otras dos. e Para comprender mejor lo que est ocurriendo es util analizar el problema tambin desa e de el punto de vista de la energ potencial. Deniendo el origen de la energia potencial a gravitacioneal cuando = 0, se encuentra que 1 U () = W d (1 cos ) Qx sin + kx2 . 2 El primer trmino al lado derecho es el cambio de la energ potencial gravitacional del e a semicilindro, el segundo el cambio de la energ potencial gravitacional de la masa Q y el a tercero la energ potencial del resorte. Con kx = Q sin queda a U () = W d(1 cos ) Q2 sin2 . 2k

212

Torque, centro de masas y equilibrio

Para hacer un estudio grco de esta relacin introducimos el parmetro Q2 /(kW d) y a o a denimos U () U ()/(W d); de esta manera la ultima ecuacin queda de la forma o U () = (1 cos ) sin2 . 2

La gura 7.45 muestra el grco para = 0; 0, 5; 1,0 y 2. Para 0 < < 1, el grco tiene a a un slo minimo, para > 1 el grco tiene dos m o a nimos (en 0 ) y un mximo (en = 0). a Para encontrar 0 debemos evaluar la derivada de U () respecto a e igualarla a cero: dU () = sin 2 sin cos = 0 , d 2 o sea, sin (1 cos ) = 0 . Esta ecuacin se satisface si sin = 0 o o (1 cos ) = 0. La primera de estas condiciones nos da la solucin = 0 o mientrs que la segunda entrega las soluciones 0 = Acos (1/), soluciones que existe slo si 1. Para = 2, se o obtiene 0 = 60 .

Figura 7.45

Cap tulo 8

Momento angular
8.1. Momento angular de una part cula

Consideremos una part cula de masa m y cuya posicin (respecto a algn sistema de refeo u rencia inercial) viene dada por el vector r. Sea F la fuerza neta que acta sobre la part u cula. a Entonces, de acuerdo a la 2 ley de Newton, la ecuacin de movimiento es o F = dp . dt

Tomando el producto cruz con el vector r se obtiene rF =r Observemos que d dr dp dp (r p) = p+r =r . (8.2) dt dt dt dt La ultima igualdad se deduce del hecho que los vectores dr/dt = v y p son paralelos. Usando (8.2) en (8.1) se obtiene d = r F = (r p) . dt Denimos el momento angular de una part cula por rp, entonces d . dt Igual que en el caso del torque, el momento angular de una part cula depende del origen que se use para evaluarlo. Si el torque que acta sobre una part u cula, medido respecto a cierto origen es nulo, entonces el momento angular de la part cula, respecto al mismo origen, no variar en el tiempo, es decir, se conservar. a a = dp . dt (8.1)

214

Momento angular

Evaluemos el momento angular de una part cula en movimiento. Supongamos que una part cula de masa m se mueve en el plano x ,y y sean r(t), (t) las coordenadas polares del vector de posicin r(t). La posicin de la part o o cula vendr dada por a r = r , r donde r = cos x + sin y . Derivando obtenemos la velocidad v = rr+rr . Figura 8.1

Pero d r = (cos x + sin y ) = sin() x + cos() y , dt luego v = r r + r . De esta manera, para el momento angular de la part cula se encuentra la expresin o = r p = m r r v = mrr r r + mr2 r = mr2 z , donde z es el vector unitario perpendicular al plano (x, y) (cuya direccin en que apunta se o encuentra usando la regla de la mano derecha). Observe que si la part cula se aleja en direccin radial (o sea, = 0 y r = 0) entonces el o momento angular es nulo. Slo si el ngulo del vector de posicin cambia a medida que o a o transcurre el tiempo, el momento angular es no nulo. El momento angular de una part cula est relacionado con el aspecto rotacional de su movimiento! a Ejemplo: Consideremos una part cula que se mantiene en un movimiento circular uniforme (con velocidad angular 0 ) mediante un hilo.

8.2 Momento angular de varias part Iculas Sea R el radio de c rculo. El momento angular de la part cula (respecto al centro de la circunferencia) viene dado por = mR2 0 z . La direccin en que apunta es a lo laro go del eje de giro, y en el sentido dado por la regla de la mano derecha (los dedos empuados indicando el sentido de la n rotacin; el pulgar extendido da el sentido o del momento angular). Figura 8.2

215

2 El hilo ejerce una fuerza sobre la part cula (la fuerza centr peta dada por mR0 r), pero esta fuerza no ejerce un torque respecto al origen ya que F y r son paralelos. Debido a que el torque es nulo, el momento angular de la part cula se conserva (o sea, a medida que transcurre el tiempo no cambia la magnitud ni la orientacin del vector ). o

8.2.

Momento angular de varias part culas

Consideremos ahora N masas {mj } ubicados en los lugares {rj }. Sean {Fj } la fuerza externa que acta sobre cada part u cula y {fji } la fuerza que la masa i ejerce sobre la masa j. Por supuesto que debido al tercer principio de Newton, fji = fij . Supongamos adems que la a fuerza que una part cula i ejerce sobre otra part cula j es a lo largo de la l nea que las une (o sea, que la interaccin entre las part o culas es central). La ecuacin de movimiento (2a ley de Newton) para cada part o cula es Fj +
i

fji =

dpj . dt

Tomando el producto cruz con el vector rj se obtiene rj Fj +


i

fji

= rj

dpj . dt

Por la misma razn discutida en la seccin anterior o o rj dpj d = (rj pj ) . dt dt d d (rj pj ) = dt dt

Usando esta relacin y sumando sobre j, se obtiene o r j Fj +


j ji

rj fji =
j

rj pj .
j

Pero ri fij + rj fji = (ri rj ) fij = 0 ,

216

Momento angular

ya que (ri rj ) es paralelo a fij . Luego, la doble suma ji rj Fji es nula. De esta manera, usando las deniciones de momento angular y torque, se obtiene j =
j

d dt

j j

(8.3)

Sea

L
j

el torque y el momento angular total del sistema de part culas, entonces la ecuacin (8.3) o queda dL =L. (8.4) = dt En palabras: Si el torque total que acta sobre un sistema (respecto a un punto P ) es u nulo, entonces el momento angular del sistema (respecto al mismo punto) no cambiar. Lo a anterior se conoce como la ley de conservacin del momento angular. Las fuerzas internas o de un sistema pueden cambiar el momento angular de las part culas que lo componen, pero no pueden modicar el vector momento angular total. Ilustremos el uso de la ley de conservacin de momento angular con algunos ejemplos o Ejemplo 1 Demuestre que un planeta, que se mueve alrededor del sol, barre reas iguales en tiempos a iguales, es decir, dA/dt =constante. Coloquemos el origen de nuestro sistema de coordenadas en el lugar donde est el sol. La a fuerza que el sol ejerce sobre los planetas es a lo largo de la direccin radial, por lo tanto, la o fuerza atractiva de gravitacin no ejerce torque sobre el planeta. De lo anterior se desprende o que el momento angular del planeta debe ser en todos los instantes el mismo. Cul es el rea A que barre el planeta a a en un tiempo t ? La respuesta es A = t t 1 | r (v t) | = |r p| = . 2 2m 2m

Como = | | se conserva a lo largo de la trayectoria, se deduce que el rea barria da en un tiempo t es independiente del punto de la trayectoria que se considere. Figura 8.3

8.2 Momento angular de varias part Iculas Ejemplo 2 Considere una masa M colgada de una varilla r gida, de masa despreciable y de largo L, que puede girar libremente en torno al punto O (ver gura adjunta). En el instante t = 0 la masa M explota y una parte M/2 sale disparada con velocidad v en una direccin que forma un ngulo con reso a pecto a la horizontal. Encuentre la energ a cintica de la parte que qued adosada a la e o varilla en el instante inmediatamente posterior a la explosin. o Figura 8.4

217

Sobre el sistema (la varilla con la masa colgando) actan las siguientes fuerzas: i) el peso u M g, ii) una fuerza F0 que ejerce el eje de giro sobre la varilla y iii) fuerzas originadas z por la explosin. En el instante t = 0 el peso no ejerce un torque sobre el sistema respecto a o 0 ya que en ese instante los vectores r y M g son paralelos. La fuerza F0 tampoco ejerce z un torque ya que el brazo para esta fuerza es nulo. Las fuerzas originadas por la explosin o son fuerzas internas y por consiguiente no modican el momento angular total del sistema. Concluimos que el momento angular total antes y justo despus de la explosin deben ser e o iguales. Inicialmente el momento angular es cero. Despus de la explosin el momento angular del e o fragmento que sale disparado es
1

M Lv cos y . 2

Si la velocidad angular de la varilla en el instante posterior a la explosin es 0 , el momento o angular de la masa que qued adosada a la varilla es o
2

M 2 L 0 y . 2
1

Como la suma de los dos momentos angulares

debe ser nula, se tiene que

M 2 M Lv cos = L 0 . 2 2 v cos . L Finalmente, conociendo la velocidad angular 0 podemos evaluar la energ cintica del a e fragmento que qued adosado a la varilla, en el instante inmediatamente posterior a la o explosin: o 1M 2 2 M 2 K= L 0 = v cos2 . 2 2 4 0 = Despejando 0 se encuentra

218

Momento angular

Antes de analizar un tercer ejemplo debemos demostrar una proposicin imo portante. Consideremos nuevamente N part culas con masas {mj } ubicadas en los lugares {rj } y con velocidades {vj }. Sean Rcm y Vcm la posicin y velocidad o del ventro de masas. Denotemos por o rj y vj los vectores de posicin y velocidad de la part cula mj respecto al centro de masas. Entonces L=
j j

Figura 8.5 =
j

rj pj =
j

mj ( rj vj ) .

Por otra parte rj = Rcm + rj y vj = Vcm + vj . Sustituyendo estas relaciones en la ecuacin anterior se obtiene o L =
j

mj Rcm + rj

Vcm + vj
j

=
j

mj Rcm Vcm + M

mj rj M

Vcm + M Rcm

mj vj M

+
j

mj rj vj

Pero
j

mj rj M

= Rcm

y = Vcm M son la posicin y velocidad del centro de masas medidas desde el centro de masas luego o ambas sumatorias son nulas. De esta manera la ecuacin anterior queda o Proposicin: o L = M Rcm Vcm +
j j

mj vj

= Rcm Pcm + L .

8.2 Momento angular de varias part Iculas

219

En palabras: El momento angular respecto a cualquier punto O es igual al momento angular debido a la traslacin del sistema como un todo, es decir, el movimiento del centro de masas o con toda la masa concentrada en ese lugar, ms el momento angular (rotacional intr a nseco) del sistema visto desde el centro de masas. Ejemplo 3 Considere dos part culas de masa m unidas por una barra de masa despreciable y largo L. Una tercera part cula, tambin e de masa m, colisiona con las anteriores, quedando adosada a la # 2. Si la velocidad incidente de la masa # 3 es v0 , y sta e incide como se muestra en la gura 8.6, encuentre la posicin de la masa # 1 en o funcin del tiempo. o Resolveremos el problema de dos maneras. Primero elijeremos el sistema de coordenadas de manera que el eje x coincida con la recta a lo largo de la cual se mueve el centro de masas del sistema (ver gura 8.7). Si t = 0 corresponde al instante en que ocurre la colisin, entonces la posicin del centro o o de masas del sistema total (es decir, de las tres masas), tanto antes como despus de la e colisin, vendr dado por o a rcm (t) = Posterior a la colisin, la barra con masas o 2m y m en sus extremos, rotar con cierta a velocidad angular 0 en torno al centro de masas. Podemos evaluar 0 usando la ley de conservacin del momento angular. o Antes de la colisin el momento angular o del sistema es Li = L mv0 3 z . Figura 8.7 v0 tx . 3

Figura 8.6

Despus de la colisin, para el sistema de e o referencia que estamos usando, la varilla con las masas slo tiene un momento ano gular intr nseco: Lf = 2L 2L m 0 3 3 + L (2m) 3 0 L 3

z=

2 m0 L2 z . 3

220 Usando la ley de conservacin del momento angular se encuentra que o 0 = v0 . 2L

Momento angular

Volveremos a resolver el problema pero eligiendo ahora un sistema de coordenadas jo en el laboratorio y con el origen coincidiendo con la la posicin de la part o cula # 2 antes de la colisin. Nuevamente elegimos el eje x a lo largo de la velocidad de la part o cula incidente y el eje y a lo largo de la direccin que tiene la barra antes de la colisin. o o En este sistema de coordenadas, el momento angular del sistema, antes de la colisin, es o nulo. Despus de a la colisin, el momento angular total de la barra con las tres masas, e o tambin deber ser nulo. El momento angular de este sistema complejo que se aleja, se e a puede evaluar usando la proposicin recin demostrada. Consta de dos partes: el momento o e angular del centro de masas y el momento angular rotacional intr nseco. Como el centro de masas se mueve con velocidad v0 /3, la masa total es 3m y el brazo (distancia entre el origen y la tangente de la trayectoria del centro de masas) es L/3, el momento angular del centro de masas ser a Rcm Pcm = L v0 (3m) z. 3 3

El momento angular intr nseco, igual que en el caso anterior, viene dado por 2 L = m0 L2 z . 3 La condicin que la suma de los dos momentos angulares anteriores sea nula, nos da la o misma relacin que ya hab o amos encontrado: 0 = Para la posicin de la o r1 (t) = v0 . 2L

masa #1 se obtiene la expresin o 2L y 3 v0 2L tx + [cos(0 t) y sin(0 t) x] 3 3 para t < 0 para t > 0

8.3.
1.

Problemas
Consideremos un satlite articial, de masa m, que gira en torno a la tierra a lo e largo de una rbita el o ptica y yas distancias mxima y m a nima a la supercie de la tierra son 2555 km y 352 km, respectivamente. La velocidad mxima del satlite es a e de 29737 km/h. El radio terrestre es igual a 6382 km. Cules sern las velocidades a a del satlite en el perigeo (rmin ) y apogeo (rmax ), respectivamente? e

8.3 Problemas 2. Una bala de masa m y velocidad v pasa a travs de la lenteja de un pndulo de e e masa M , y emerge con velocidad v/2. La lenteja del pndulo est colgada de una e a cuerda de longitud . Cul debe ser el a valor de v para que la lenteja del pndulo e describa un c rculo completo? Cmo se o modica el problema si, en lugar de una cuerda, la lenteja est colgada de una vaa rilla r gida sin masa? Figura 8.8 3. Una part cula de masa m y velocidad v0 incide sobre una barra de largo L y masa despreciable, que en cada uno de los extremos tiene una masa m, tal como se indica en la gura. Suponga que el choque entre las esferas # 1 y # 2 es elstico y a central (frontal). Se mover la part a cula # 1 despus del choque? Si su respuesta es e armativa evalue su direccin y magnitud. o Una masa m1 se deja caer desde una altura h sobre un balanc (ver gura 8.10). El n balanc puede girar libremente en torno n a O en el sentido contrario al reloj. Sobre el otro extremo del balanc hay una man sa m2 . Al chocar la masa m1 contra el balanc sta queda adosada a l. Qu fracn, e e e cin de la energ total inicial se disipa o a en la colisin? Desprecie la masa del bao lanc n. Considere dos masas m, unidas por una varilla de largo L. Esta varilla est sola dada en su centro a otra varilla, formando un ngulo . El sistema anterior roa ta con una velocidad angular 0 en torno a la segunda varilla (ver gura adjunta). En cierto instante la soldadura se rompe, desacoplndose el movimiento de las dos a varillas. Describa, de ah en adelante, el movimiento de la varilla con las dos masas.

221

Figura 8.9

4.

Figura 8.10

5.

Figura 8.11

222 6. Una masa m realiza un movimiento circular, con radio R0 , sobre una mesa (sin friccin), atada a un hilo (ver gura 8.12). Sea o 0 la velocidad angular inicial. Cul es el a trabajo que debe realizarse (tirando del hilo) para achicar el radio de giro desde R0 a R0 /2 ?
2 2 Respuesta: W = 3m0 R0 /2 .

Momento angular

Figura 8.12

7.

Considere una varilla r gida, pero de masa despreciable, cuyo largo es L y que tiene dos masas m, una adosada en uno de los extremos y la otra al centro (ver gura). La varilla puede girar libremente en el plano vertical alrededor de un eje que pasa por el extremo en que no tiene una masa adosada. Todo el sistema se encuentra en un campo gravitacional constante g = g. Suponga que z este sistema inicialmente se encuentra en reposo en su posicin de equilibrio ineso table. Una leve perturbacin hace que el o sistema salga de su posicin de equilibrio o y paulatinamente comienza a caer. a) b)

Figura 8.13

Encuentre la velocidad angular = y la aceleracin angular = de la varilla o cuando sta forme un ngulo con la vertical. e a Encuentre la fuerza que la varilla ejerce sobre el eje cuando la varilla pasa por la horizontal (es decir, cuando = /2).

8.

Dos masas m unidas por un hilo de largo L, caen con el hilo horizontal partiendo desde el reposo. Despus de caer una e distancia h, una de ellas choca elsticaa mente con una viga. a) Determine la velocidad angular con que girarn las masas en torno a a su centro de masas despus de la e colisin. o Encuentre la tensin a la que eso tar sometido del hilo despus de a e que ha ocurrido la colisin. o

b)

Figura 8.14

8.3 Problemas 9. Considere un pndulo cnico (es decir, e o una masa m colgada de un hilo ideal de largo L), que gira en c rculos formando un ngulo 0 con la vertical. a a) Con qu velocidad angular gie rar si el hilo se acorta lentamente a hasta llegar a L/2? Que trabajo debe realizarse para acortar el hilo en esa magnitud?

223

b)

Figura 8.15

10.

Un alambre (de masa despreciable) de largo 2L se dobla al centro de manera que forma un ngulo . En cada extremo a el alambre tiene una masa m. Este dispositivo se cuelga de un eje tal como se muestra en la gura adjunta. Calcule el per odo de oscilacin del sistema para o pequeas oscilaciones en torno a su posin cin de equilibrio estable. Verique que o la expresin general, en los l o mites = 0 y = , da los resultados esperados.

Figura 8.16

Para resolver este problema suponga que el sistema esta oscilando (con pequeas n oscilaciones) y evale para un instante arbitrario el torque y el momento angular. u Luego, usando la ecuacin (8.4) demuestre que la variable (t) satisface la ecuacin o o diferencial de un oscilador armnico. o

11.

Considere una varilla de largo L que tiene dos masas M adosadas tal como se muestra en la gura. Un masa m que incide con velocidad v0 , choca con el pndulo quedando adosada a l a una e e distancia h del eje. Determine el impulso trasmitido por el eje al pndulo durante e la colisin. A qu altura debe impactar o e m para que el impulso transmitido por el eje sea nulo? (en ese caso el eje no se percata de la colisin). o

Figura 8.17

224

Momento angular

8.4.

Solucin a algunos de los problemas o

Solucin al problema 6 o Debido al principio de conservacin de la energ la energ cintica que tiene el sistema o a, a e cuando la varilla forma un ngulo con la normal debe ser igual al cambio de energ a a potencial, o sea, K = U , con 1 K = m 2 y U = mg(L L cos ) + mg De esta manera se deduce que 2 () = 12 g (1 cos ) . 5 L 12 g sin . 5 L L L cos 2 2 3 = mgL (1 cos ) . 2 L 2
2

1 5 + m(L)2 = mL2 2 2 8

Derivando esta relacin encontramos la aceleracin angular, en efecto, o o 2 = Pero = , luego 6g sin . 5L Demostremos que el mismo resultado se puede obtener usando la ecuacin de movimiento o = d /dt. Cuando la varilla forma un ngulo con la normal, el torque respecto a un origen a ubicado en el eje es L 3 = mgL sin + mg sin = mgL sin . 2 2 Para el momento angular tenemos == = m(L)L + m L 2 L 5 = mL2 . 2 4

Reemplazando estas expresiones en la ecuacin de movimiento se obtiene o 3 5 mgL sin = mL2 , 2 4 de donde, nuevamente 6g sin . 5L Supongamos ahora que la varilla est pasando por la horizontal (es decir, = /2). En ese a instante el centro de masas (que est ubicado a una distancia 3L/4 del eje) acelera con una a aceleracin o == 3 acm = |at | |ac | = L 2 z x z 4 3 L 4 x= 9 g( + 2) . z x 10

8.4 Solucin a algunos de los problemas o

225

(Observe que la componente z de la aceleracin de la part o cula m que est en el extremo a de la varilla, cuando sta pasa por la horizontal, es 6g/5, o sea, mayor que g; convnzase de e e que as deb ser). La fuerza neta que acta sobre la varilla (cuando pasa por la horizontal) a u es Ftot = Feje 2mg . z Pero Ftot = (2m)acm , luego Feje 2mg = 2m z de donde se deduce que 1 Feje = mg ( 18) . z x 5 Solucin al problema 8 o z Al chocar con la viga la velocidad de la masa m ser v0 = 2gh. El choque con la a viga es elstico y el hilo que une ambas masas (que no puede ejercer fuerzas transversales a a su orientacin) no interviene para nada en ese proceso. Luego la masa m rebotar con o a la velocidad 2mg. La otra masa no modica su velocidad mientras ocurre el choque. z Por lo tanto, justo despus de la colisin, la velocidad del centro de masas (respecto a un e o observador junto a la viga) ser nula. a Para un observador junto a la viga (en el lugar donde ocurrir la colisin), el momento a o angular antes de la colisin es o i = Lm 2gh . Despus de la colisin ser e o a
f

9 g( + 2) , z x 10

= Lcm +

donde Lcm es el momento angular debido a la traslacin del centro de masas y f es el o momento angular observado desde el centro de masas. Ya que justo despus de la colisin e o el centro de masas est en reposo Lcm = 0. Denotemos por 0 la velocidad angular del hilo a despus de la colisin, entonces e o = 2m L 0 2 L mL2 = 0 . 2 2

Como el impulso que ejerce la viga no cambia el momento angular del sistema respecto al punto en que se aplica esa fuerza de percusin, se tiene que el momento angular debe o conservarse. Luego mL2 0 = Lm 2gh , 2 de donde 0 = 8gh . L

226

Momento angular

En este problema el mismo resultado tambin se puede obtener usando la conservacin de e o la energ Despus de la colisin, como el centro de masas est en reposo, toda la energ a. e o a a cintica se debe a la rotacin, siendo sta e o e 1 Kr = 2 m 2 L 0 2
2

1 2 = mL2 0 4

Por otra parte, el cambio de energ potencial es a U = 2mgh . Igualando ambas expresiones obtenemos nuevamente que 8gh . 0 = L Despus de ciertos momentos de reexcin, es claro que la ca de las dos masas en un e o da campo gravitatorio constante, no afecta la tensin del hilo. Por lo tanto, la tensin de la o o cuerda se debe slo al movimiento rotacional de las dos masas. El radio de giro de ellas es o L/2. La magnitud de la fuerza centr peta (que es igual a la tensin del hilo) es o
2 Fcent = m0

L h = 4mg . 2 L

Solucin al problema 10 o Denotemos por al ngulo que el pndulo hace respecto a su posicin de equilibrio (ver a e o gura 8.18). El momento angular del pndulo ser e a = 2Lm(L ) x , donde es la velocidad angular del pndue lo (siendo positiva cuando gira en la direccin contraria a los punteros del reloj). o Derivando respecto al tiempo se deduce que d = 2mL2 x . dt Figura 8.18 El torque de la fuerza gravitacional (respecto a un origen en el eje) es = mgL sin(/2 ) x mgL sin(/2 + ) x = 2mgL cos(/2) sin x

8.4 Solucin a algunos de los problemas o

227

Sustituyendo las dos relaciones anteriores en la ecuacin de movimiento = d /dt encono tramos 2mgL cos(/2) sin = 2mL2 . Para pequeas oscilaciones en torno de la posicin de equilibrio podemos usar la aproximan o cin sin o . De esta manera obtenemos + g cos(/2) L =0.

Esta es la ecuacin de movimiento de un oscilador armnico cuyo per o o odo es T = 2 g cos L 2


1

228

Momento angular

Cap tulo 9

Rotacin de un cuerpo r o gido


En este cap tulo estudiaremos rotaciones de un cuerpo slido. No consideraremos el caso o general, que ser materia de cursos ms avanzados, sino que analizaremos slo el caso en a a o que el movimiento rotacional del slido es en torno a un eje jo (o que al menos no cambie o su orientacin a medida que transcurre el tiempo). o

9.1.

Las ecuaciones bsicas a

Consideremos un cuerpo slido, que gira con velocidad angular 0 en torno a un eje jo que o elegiremos como el eje z . El origen lo elegimos en algn lugar sobre el eje. Es usual denir u un vector velocidad angular por 0 = 0 z , donde 0 = |0 | y el signo se elige usando la regla de la mano derecha: si los dedos curvados indican la direccin de rotacin, entonces el pulgar muestra la direccin en que apunta 0 . o o o Para ser concretos, supongamos que el slio do consta de N masas mj , (j = 1, 2, . . . , N ), ubicadas en los puntos rj , unidas por varillas r gidas sin masa (ver gura 9.1). El vector posicin de cada part o cula se puede descomponer como sigue: rj = rj + zj z . La magnitud rj = |rj | es la distancia de la masa mj al eje de giro. La velocidad de cada masa viene dada por vj = 0 rj . Figura 9.1 Para la rapidez de la masa j se obtiene vj = rj 0 .

230

Rotacin de un cuerpo r o gido

Evaluemos la energ cintica del slido y tambin la componente del momento angular que a e o e apunta a lo largo del eje de rotacin. o La energ cintica (debido a la rotacin del slido) viene dada por la suma de las energ a e o o as cinticas de cada una de las masas, o sea: e K = 1 2 mj vj 2 j=1 N 1 2 2 = mj rj 0 2
j=1 N

Observe que las coordenadas zj de las distintas masas no intervienen en la expresin para o la energ cintica. Algo similar ocurre al evaluar la componente z del momento angular a e (hagmoslo aqu para una part a cula): = mr v = mr (0 r ) = m0 r2 mr (0 r ) = m0 r2 z mr 0 z = m0 r2 z m(z z + r ) 0 z = m0 (r2 z 2 ) z m0 zr
2 = m0 r z m0 zr .

Para la componente z del momento angular se tiene entonces


z 2 = mr 0 .

La componente z del momento angular de todas las part culas que componen el slido es, o por lo tanto,
N

Lz =
j=1

2 mj rj 0 .

N 2 En dos ocasiones ya nos ha aparecido la expresin o cij=1 mj rj . Es util denir expl tamente este concepto: Deniremos el momento de inercia del slido en torno a un eje o por N

I
j=1

2 mj rj .

De esta manera la energ cintica y la componente del momento angular paralela al eje de a e rotacin vienen dadas por o 1 2 K = I0 2 y Lz = I 0 .

9.1 Las ecuaciones bsicas a

231

Si sobre un sistema aplicamos un torque en la direccin z, entonces cambiar la componente o a z de su momento angular de acuerdo a la relacin o z = dLz d0 =I = I 0 , dt dt

o sea, conociendo el torque podemos evaluar su aceleracin angular. o En el cap tulo anterior demostramos que el momento angular de un slido que se mueve en o el espacio, respecto a un origen O se puede escribir como una suma de dos contribuciones: i) el momento angular debido a la traslacin del sistema como un todo, es decir, el movimiento o del centro de masas con toda la masa concentrada en ese lugar y, ii) el momento angular (rotacional intr nseco) del sistema, L , visto desde el centro de masas, es decir, L = Rcm Pcm + L . Mostraremos a continuacin que se tiene una expresin anloga para la energ cintica. Sea o o a a e vj la velocidad de la part cula j medida desde un sistema de referencia O, vj la velocidad de la misma part cula pero vista desde el sistema de referencia jo al centro de masas y Vcm la velocidad del centro de masas. Entonces se tiene que vj = vj + Vcm . La energ cintica (para el observador O) es a e 1 K= 2
N 2 mj vj . j=1

A partir de las dos ultimas ecuaciones se encuentra que K = 1 2 1 2 1 2 1 2


N

mj vj vj
j=1 N

mj (vj + Vcm ) (vj + Vcm )


j=1 N N 2

mj vj
j=1 N

+
j=1

mj Vcm vj
N j=1

1 + 2

N 2 mj Vcm j=1

mj vj
j=1

+ Vcm

1 2 mj vj + M Vcm 2

(9.1)

El primer trmino al lado derecho de la ultima ecuacin es la energ cintica del slido e o a e o vista desde el centro de masas, o sea, corresponde a la energ cintica debido a la rotacin a e o

232

Rotacin de un cuerpo r o gido

intr nseca del slido. Como vimos al inicio de la presente seccin, esta energ cintica la o o a e podemos escribir de la forma 1 K = 2
N j=1

1 2 mj vj2 = I0 . 2

El segundo trmino del lado derecho de la ecuacin (11.16) es nulo ya que ( mj vj )/M e o es la velocidad del centro de masas vista desde el centro de masas. Por ultimo, el tercer trmino del lado derecho de la ecuacin (11.16) es la energ cintica de traslacin del slido e o a e o o como un todo. Concluimos que la ecuacin (11.16) se puede escribir de la forma o 1 1 2 2 K = M Vcm + I0 . 2 2

9.2.

Momento de inercia

De vital importancia para describir las rotaciones de un slido es el concepto de momento o de inercia. Para un slido constituido de N masas discretas (unidas r o gidamente con varillas sin peso), el momento de inercia viene dado por
N

I=
j=1

2 mj rj .

Para distribuciones de masa continua, la expresin anterior debe sustituirse por una con o integrales que adecuadamente describa la situacin. Por ejemplo: si un cuerpo slido viene o o descrito por una densidad de masa (x, y, z) = (r ), entonces el momento de inercia en torno al eje z viene dado por I=
Slido o

(x2 + y 2 ) (r ) dx dy dz .

En la ultima expresin (x2 + y 2 ) es el cuadrado de la distancia al eje de la masa del volumen o d3 r = dx dy dz ubicado en el lugar r. Evaluemos algunos momentos de inercia importantes:

Ejemplo 1: Evaluemos el momento de inercia de una varilla de largo L y masa M en torno a un eje que pasa perpendicularmente por uno de sus extremos (ver gura 9.2). Figura 9.2

9.2 Momento de inercia

233

La densidad lineal de la varilla es = M/L. El trozo de varilla de largo dx que se encuentra a una distancia x del eje tiene una masa igual a dx y su contribucin al momento de inercia o 2 dx. Sumando todas las contribuciones desde x = 0 hasta x = L se obtiene es x
L

I=
0

x2 dx =

1 3 x 3

=
0

L3 , 3

o sea I= M L2 . 3

Ejemplo 2: Evaluemos el momento de inercia de un anillo de radio R y masa M en torno a un eje que pasa perpendicularmente por el centro (ver gura 9.3). Como toda la masa del anillo est a la a distancia R del eje, el momento de inercia es simplemente I = M R2 . Figura 9.3

Ejemplo 3: Evaluemos el momento de inercia de un disco uniforme de radio R y masa M en torno a un eje que pasa perpendicularmente por el centro (ver gura 9.4). La densidad supercial del disco viene dada por = M/(R2 ). Para encontrar el momento de inercia subdividiremos el disco en anillos innitesimales. El momento de inercia dI de un anillo de radio r y ancho dr viene dado por (ver ejemplo anterior) dI = (masa del anillo) r2 . Pero la masa de tal anillo es (masa del anillo) = 2r dr , luego dI = 2 r3 dr . Figura 9.4

234

Rotacin de un cuerpo r o gido

Sumando la contribucin de todos los anillos desde r = 0 hasta r = R se encuentra o


R

I=

dI =
0

2 r3 dr = 2

r4 4

=
0

4 R . 2

Sustituyendo la expresin para se obtiene nalmente o M R2 . 2

I=

Ejemplo 4: Encontremos el momento de inercia de una esfera uniforme de radio R y masa M alrededor de un eje que pasa por el centro. La densidad de masa de la esfera viene dada por M 0 = 4 3 . 3 R Para encontrar el momento de inercia de una esfera supongamos que ella est consa tituida por numerosos discos innitesimales de grosor dz (ver gura 9.5). El radio del disco innitesimal que se encuentra a una altura z viene dado por R2 z 2 . El a rea de tal disco es por lo tanto A = (R2 z 2 ) .

Figura 9.5

Para la masa dM (que es el volumen del disco innitesimal multiplicado por su densidad) se obtiene dM = 0 (R2 z 2 ) dz . La contribucin de tal disco al momento de inercia de la esfera es (ver ejemplo 3) o

1 0 dI = dM (R2 z 2 ) = (R2 z 2 )2 dz . 2 2

9.2 Momento de inercia Sumando la contribucin de todos los discos desde z = R hasta z = R se encuentra o
R

235

I=

dI =
R

dI = 0 2 0 2
R

0 2

(R2 z 2 )2 dz
R

= =

(R4 2R2 z 2 + z 4 ) dz
R

2 1 R4 z R2 z 3 + z 5 3 5

R R

2 1 0 2 R5 R5 + R5 = 2 3 5 8 = 0 R 5 15 Sustituyendo la expresin para 0 se obtiene nalmente o 2 I = M R2 . 5 Ejemplo 5: Evaluemos el momento de inercia de una esfera hueca de radio interno Ri y radio externo Re , hecha de un material de densidad uniforme 0 , para un eje que pasa por el centro. Es fcil resolver este problema si se observa que la esfera hueca se puede pensar como dos a esferas concntricas sobrepuestas: una de radio Re con densidad 0 y otra de radio Ri con e densidad negativa 0 . El momento de inercia de esta superposicin (que coincide con la de la esfera hueca) viene o dado por (ver ejemplo 4) I= 8 8 8 5 5 5 5 0 Re + (0 ) Ri = 0 (Re Ri ) . 15 15 15

Expresemos el resultado tambin en trminos de la masa de la esfera hueca, que es: e e 4 3 3 M = (Re Ri ) 0 . 3 Sustituyendo esta relacin en la expresin para el momento de inercia se encuentra o o
5 2 (R5 Ri ) I= M e 3 . 3 5 (Re Ri )

Para Ri 0, se recupera, tal como debe ser, el resultado del ejemplo anterior. Ejemplo 6: Evaluemos el momento de inercia de una cscara esfrica de radio R y masa M , para un a e eje que pasa por el centro.

236

Rotacin de un cuerpo r o gido

Para resolver este problema usamos el resultado del ejemplo anterior, evalundolo en el a l mite Ri Re . Para encontrar este l mite pongamos Ri = Re con muy pequeo. Se n tiene:
3 3 3 Re Ri = Re (Re )3 3

3 Re

3 Re

Re Re

3 3 Re Re 1 3 2 3Re

De la misma manera se encuentra que


5 5 4 Re Ri = 5Re .

Reemplazando estos resultados en la expresin para el momento de inercia, e igualando Re o con R, se encuentra (para el momento de inercia de una cscara esfrica) a e 2 I = M R2 . 3 Ejemplo 7: Evaluemos el momento de inercia de un anillo de radio r y masa M en torno a un eje que coincide con un dimetro del anillo a (ver gura 9.6). Para resolver este problema subdividamos el anillo en numerosos sectores angulares innitesimales. La densidad lineal del anillo es = M/(2R). La masa del anillo del sector comprendido entre y + d es r d. Su contribucin al momento de o inercia del anillo es dI = R d (R sin )2 . Sumando la contribucin de todos los sectores (desde = 0 hasta = 2) se obtiene o
2

Figura 9.6

I= Pero

dI =
0 2

R3 sin2 d .

sin2 d = ,
0

luego 1 I = R3 = M R2 . 2

9.3 Problemas

237

9.3.
1.

Problemas
(Teorema de Steiner o teorema de los ejes paralelos) Demuestre que el momento de inercia I para las rotaciones de un cuerpo slido alrededor de un eje L es o I = I0 + M R2 , donde I0 es el momento de inercia para rotaciones del slido alrededor del eje paralelo o a L que pasa por el centro de masas y R es la distancia de separacin de los dos ejes. o

2.

Encuentre el momento de inercia de las supercies, de densidad supercial uniforme 0 , mostrados en la gura 9.7 y en torno a los ejes ah indicados.

Figura 9.7

3.

Considere un sistema de dos masas m1 y m2 , separadas por una distancia r. Demuestre que el momento de inercia con respecto al eje que pasa por el centro de masas en forma perpendicular a la l nea que los une, viene dado por r2 , donde = m1 m2 /(m1 +m2 ) es la masa reducida del sistema.

4.

Encuentre el momento de inercia de los alambres, de densidad lineal uniforme 0 , mostrados en la gura 9.8 y en torno a los ejes ah indicados.

238

Rotacin de un cuerpo r o gido

Figura 9.8

5.

La molcula de metano consiste de un e a tomo de carbono localizado al centro de un tetraedro regular cuyos vrtices estn e a ocupados por 4 tomos de hidrgeno. La a o distancia C H es de 1.08 . (1 = A A 108 cm). Cul es el momento de inercia a de la molcula de metano para una rotae cin alrededor de un eje C H? Las masas o de los tomos de hidrgeno y carbono son: a o 27 Kg y m =19.9 1027 Kg, mH =1.68 10 C respectivamente.

Figura 9.9

6.

Una esfera sube rodando un plano inclinado en 30 . Cuando la esfera se encuentra al pie del plano, su centro de masas se traslada con una velocidad de 5 m/s. Hasta dnde subir la esfera por el plano inclinado? Cunto tiempo tardar en regresar al o a a a punto de partida?

7.

Se enrolla una cuerda alrededor de la plataforma de un carrusel de radio R = 2 m para echarlo andar. Durante 10 s se tira de la cuerda con una fuerza de 200N. Durante ese tiempo el carrusel da una vuelta completa. Cul es el momento de inercia a del carrusel? Respuesta: I =3183 kg m2 . Figura 9.10

9.3 Problemas 8. Los dos discos mostrados en la gura adjunta tienen masas m y radios R iguales. El disco superior puede rotar libremente alrededor de su eje. Una cuerda est enroa llada alrededor de ambos discos. Encuentre: a) b) c) La aceleracin del centro de masas o del disco inferior. La tensin de la cuerda. o La aceleracin angular de cada disco o alrededor de su centro de masas. Figura 9.11

239

9.

Una esfera de densidad uniforme 0 y radio r rueda sin deslizarse a lo largo de una v que posee una vuelta circular de radio a R (ver gura 9.12). La esfera inicia su movimiento partiendo, del reposo, desde una altura h. Cul es la m a nima altura h requerida para que la esfera no deje la v a? Cul ser la altura h si la bola en lugar a a de rodar se desliza resbalando? Respuesta: h = 27 (R r)/10 .

Figura 9.12

10.

Una bola de palitroque, de radio R y masa M , se lanza de manera que inicialmente resbale (sin que ruede) con velocidad v0 . Si el coeciente de roce entre el suelo y la bola es , qu distancia recorrer la bola antes de que ruede sin resbalar? Cul es e a a su velocidad nal? Respuesta: vf = 5v0 /7 .

11.

Considere dos poleas (discos) de masas m1 , m2 y radios R1 , R2 , respectivamente. Con estas poleas se realiza el montaje mostrado en la gura adjunta (la cuerda est enrollada en torno a la polea # 2). a Encuentre la aceleracin de la masa M . o Respuesta: M + m1 a = g . M + 3 m1 + 2m2 2 Figura 9.13 Una varilla de largo L y masa M puede rotar libremente alrededor de un pivote A. Una bala de masa m y velocidad v impacta contra la varilla en un punto P alejado una distancia a desde el pivote, quedando incrustada en ella.

12.

240

Rotacin de un cuerpo r o gido

a)

Encuentre el momento angular alrededor del punto A inmediatamente antes y despus de la colisin. e o Determine el momento lineal del sistema inmediatemente antes y despus de la colisin. e o Cul es el valor Q de la colisin, a o es decir, cunta energ es disipada a a durante el proceso?

b)

c)

Figura 9.14

13.

Dos nios, cada uno de masa M , estn sentados en los extremos de una barra horin a zontal de largo L y masa m. La barra gira inicialmente con una velocidad angular 0 alrededor de un eje vertical que pasa por su centro. a) Cul ser la velocidad angular si caa a da nio se mueve una distancia d han cia el centro de la barra (sin tocar el suelo)? En cunto cambiar la energ a a a cintica de rotacin del sistema? e o Figura 9.15

b)

14.

Una esfera, un disco y un aro, hechos de materiales homogneos, tienen el mismo e radio R y la misma masa M . Los tres objetos se dejan libres desde la parte superior de un plano inclinado. Los tres objetos parten desde el reposo y ruedan sin resbalar. El plano tiene un largo L y su inclinacin respecto a la horizontal es . o a) b) c) Cules son sus velocidades al llegar al pie del plano inclinado? a Encuentre la fuerza de roce fr en cada caso. Cunto tarda cada uno de los objetos en llegar a la parte inferior? a

15.

Un aro circular de radio R oscila en torno a un eje horizontal que pasa por A (ver gura). El eje es normal al plano del aro. a) Cul ser el largo de un pndulo a a e simple con igual per odo de oscilacin que el del aro? (Haga el anlisis o a slo para pequeas oscilaciones.) o n b) Se desea que el aro de una vuelta completa alrededor de A. Cul es a la m nima velocidad angular que debe poseer el aro, en la parte inferior, para que esto sea posible?

Figura 9.16

9.3 Problemas 16.

241

Un aro circular de masa m y radio R descansa sobre una supercie horizontal sin roce (ver gura, vista desde arriba). Contra el aro se dispara tangencialmente una bala con velocidad v0 , cuya masa tambin es m. La bala queda incrustada en el aro. e a) b) Describa el movimiento del sistema despus del choque. e Cul es la velocidad del centro de a masas del sistema antes y despus e del choque? Cul es el momento angular del sisa tema respecto a su centro de masas antes del choque? Cul es la velocidad angular con a que gira el sistema despus del choe que? Cunta energ cintica se pierde a a e en el choque?

c)

d)

Figura 9.17

e)

17.

Un aro de masa M y radio r, rueda sin resbalar por la supercie interior de una cinta circular ja de radio R (ver gura 9.18). Encuentre el per odo de este movimiento para pequeas oscilaciones alrededor de la n vertical. Respuesta: T = (2)/0 , con
2 0 =

g . 2(R r)

Figura 9.18

18.

Considere la mquina de Atwood mostraa da en la gura adjunta. La polea consta de un disco uniforme de masa m (que coincide con el valor de la masa ms pequea a n colgada de la mquina) y radio R. El moa mento de inercia para rotaciones en torno al eje de un disco es I = mR2 /2. El roce entre la cuerda y la polea hace que esta ultima gire mientras las masas estn e en movimiento. Suponga que la cuerda no tiene masas y que no desliza sobre la polea. La masa 2m parte del reposo desde una altura h. Figura 9.19

242

Rotacin de un cuerpo r o gido a) Usando el teorema de conservacin de la energ encuentre la velocidad de la o a, masa 2m cuando sta llega al suelo. e b) Encuentre la tensin de la cuerda a ambos lados de la mquina de Atwood. Es o a decir, encuentre 1 y 2 en funcin de m, g y R. (Cuando el momento de inercia o de la polea no se puede despreciar (lo que es el caso del presente problema) entonces la tensin de la cuerda no es la misma a ambos lados de la polea.) o c) Encuentre la tensin de la cuerda que sujeta la polea mientras las masas estn o a en movimiento. d) Encuentre la tensin de la cuerda que sujeta la polea despus de que la masa o e 2m lleg al suelo (y todas las componentes de la mquina de Atwood estn en a a a reposo).

19.

Considere dos poleas jas unidas por una correa (o cadena) de transmisin tal como o se muestra en la gura adjunta. Una masa M colgada por una cuerda enrollada en la polea #1 pone en movimiento el sistema. Suponga que las poleas son discos de radio R y tienen una masa tambin igual a M (es decir, el momento de inercia de e las dos poleas coinciden, tenindose I = M R2 /2). Note que una correa (o cadena) de e transmisin slo puede transmitir una fuerza de traccin. Para el presente problema o o o slo la parte superior de la correa transmite una fuerza entre las poleas. o

Figura 9.20 a) Encuentre la tensin T de la cuerda. o b) Encuentre la aceleracin angular de la polea #1. o c) Usando la ley de conservacin de la energ encuentre la velocidad v que tiene o a, la masa M despus de haber bajado una distancia h. (La masa M parte desde e el reposo).

9.3 Problemas 20. Una barra uniforme de largo L apoyada contra la pared comienza a resbalar (sin roce). Inicialmente el ngulo que forma a con la pared es 0 . Encuentre la altura z para la cual el extremo A de la barra se separa de la pared vertical. Respuesta: z = 2 L cos 0 . 3 Figura 9.21

243

21.

Una carretilla de hilo, formada de dos discos y un cilindro de las dimensiones indicadas en la gura 9.22a, se tira del hilo que tiene enrollado tal como se muestra en la gura 9.22b. Encuentre la aceleracin de la carretilla de hilo si sta rueda sin resbalar. o e

Figura 9.22

22.

Considere un automvil de masa M , cuya geometr se muestra en la gura adjunta, o a y que inicialmente se mueve con velocidad v0 hatx. Suponga que en cierto instante el automvil frena bloqueando las dos ruedas delanteras. Encuentre la distancia que o el automvil alcanza a recorrer durante el frenado si el coeciente de roce cinemtico o a entre el pavimento y las ruedas es c . Asuma que durante el frenado, las ruedas traseras, en todo instante, estn en contacto con el pavimento, situacin que generalmente a o se da en la prctica. a Bajo ciertas condiciones extremas de frenado, el automvil podr elevarse por la o a parte trasera y tumbarse. Encuentre la condicin que debe satisfacerse para que el o automvil quede, en todo instante, con las cuatro ruedas sobre el pavimento. o Haga tambin un anlisis del proceso de aceleracin del automvil. e a o o

244

Rotacin de un cuerpo r o gido

Figura 9.23 23. Considere una varilla r gida (de masa despreciable), que en cada uno de sus extremos tiene adosada una masa m. La varilla se desplaza inicialmente sin rotar sobre el plano (x, y), con la velocidad del centro de masas v = v0 x y con la varilla orientada de manera de formar un ngulo con el eje x, (ver a gura). En cierto lugar una de las masas choca elsticamente con una pared a r gida, tal como se muestra en la gura. Despus de la colisin (el centro de e o Figura 9.24 masas de) la varilla con las masas se trasladar uniformemente, rotando sia multneamente con velocidad angular a constante. Desprecie el roce entre las masas m y el plano y suponga tambin que la pared e est pulida, es decir, no hay fuerzas de roce entre la masa m y la pared cuando entran a en contacto. a) b) c) Determine la velocidad angular de la varilla despus de la colisin. e o Encuentre el impulso transmitido al sistema por la pared durante la colisin. o Verique que el resultado obtenido en la parte b) da los resultados correctos en los l mites = 0 y = /2.

24.

Uno de los extremos de un resorte ideal de constante elstica k se ja a una paa red. El otro extremo se ja al eje de una rueda cil ndrica de radio R y masa M . El resorte se comprime una distancia a, manteniendo su posicin horizontal. o Figura 9.25

9.3 Problemas

245

Estando el cilindro en contacto con el suelo (supercie rugosa) se suelta ste del e reposo. Calcule la velocidad angular del cilindro cuando la elongacin del resorte es o nula. Suponga que el cilindro no resbala.

25.

Cuatro bolitas idnticas, de masa m cada una, se unen mediante varillas de masa e despreciable de largo a de tal forma que las bolas queden ubicadas en los vtices de e un cuadrado.

a)

Calcule el momento de inercia con respecto a un eje a lo largo de la diagonal del cuadrado. Calcule el momento de inercia con respecto a un eje paralelo a uno de los lados y que pasa por el centro de ste. e Si los dos cuadrados se hacen rotar, cada uno entorno a los ejes descritos anteriormente y con la misma velocidad angular, determine cual de ellos (y en que porcentaje) tiene mayor energ cintica. a e

b)

c)

Figura 9.26

26.

Un bloque rectangular y un cilindro se unen con una varilla como se muestra en la gura. El cilindro rueda sin resbalar mienmtras que el bloque desliza sobre el piso rugoso (c ). Si la masa del cilindro y del bloque es la misma (M/2) y el radio del cilindro es R, calcule el tramo recorrido por el sistema desde aquel instante en que el sistema se desplaza con rapidez v0 . Compare con el resultado que obtendr si a el cilindro se mantiene fuera de contacto con el piso. Figura 9.27

246 27. Considere un cubo de arista L y masa que se desliza von velocidad v0 sobre un plano horizontal (sin roce). En cierto lugar el cubo se encuentra con un tope. a) Demuestre que el momento de inercia del cubo respecto a un eje de rotacin que coincide con una de sus o aristas es I = 2M L2 /3. Cual es la m nima velocidad que debe tener el cubo para que se vuelque?

Rotacin de un cuerpo r o gido

Figura 9.28

b)

28.

Una ardilla de masa m corre (aceleradamente) dentro de un cilindro hueco de radio R y masa M . La ardilla en ningn mou mento resbala y el cilindro posa sobre un plano rugoso horizontal (sobre el cual rueda sin resbalar). A consecuencia de su movimiento acelerado la ardilla se mantiene siempre a una altura h del suelo. Determinar la aceleracin con que se traslada o el centro de masas del cilindro (que es la misma con que se traslada la ardilla).

Figura 9.29

29.

Un disco de radio R y masa M , inicialmente en reposo, puede girar libremente alrededor de un eje vertical jo. Sobre ese disco se coloca otro disco, de radio r y masa m que inicialmente rota con una velocidad angular 0 . Debido al roce entre los dos discos el segundo disco eventualmente quedar en reposo respecto al primero. Si a la separacin entre los centros entonces es o D, encuentre la velocidad angular nal con que girarn los dos disco entorno al a eje.

Figura 9.30

9.3 Problemas 30. Una varilla de masa M y longitud L cuelga con uno de sus extremos jo al techo. La varilla puede rotar libremente entorno a este punto. Sobre el piso un cuerpo pequeo de masa m choca elsticamente con n a el extremo inferior de la varilla. a) Determine la velocidad angular de la varilla inmediatamente despus de la e colisin. o Determine la masa de la varilla si a consecuencia del choque la masa incidente queda detenida. Figura 9.31

247

b)

31.

Una cuerda se enrolla entorno a un cilindro. El cilindro se ubica sobre un plano horizontal rugoso () y en contacto con una pared vertical del mismo material del piso (ver gura). La cuerda se tira con una fuerza F hacia abajo. Calcular la razn o entre las fuerzas normales experimentadas en el suelo y la pared, respectivamente.

Figura 9.32

32.

Considere la conguracin experimental o mostrada en la gura adjunta. Suponga que no hay roce entre la carretilla de hilo y el plano inclinado. Suponiendo conocidos el momento de inercia I de la carretilla para rotaciones alrededor de su eje, los radios R y r, el ngulo de inclinacin del a o plano inclinado, encuentre la aceleracin a o del eje de la carretilla. Un mono se encuentra sobre una plataforma que puede rotar (sin roce) alrededor de un eje. Inicialmente la plataforma y el mono se encuentran en reposo. Qu debe e hacer el mono para alcanzar los pltanos a que estn al otro lado. No hay nada a la a mano del mono y supondremos que con slo soplar no es posible poner en movio miento la plataforma.

Figura 9.33

33.

Figura 9.34

248 34. Considere un pndulo (f e sico) formado por una varilla de largo R y masa M en cuyo extremo est adosada una esfera de radio a R y masa 2M . El pndulo cuelga de uno e de los extremos de la varilla. a) Determine el momento de inercia del pndulo para rotaciones entorno al e punto de suspensin. o b) Determine el per odo de este pndue lo para pequeas oscilaciones. n

Rotacin de un cuerpo r o gido

Figura 9.35

c) Determine la velocidad angular 0 debe darse al pndulo para que ste logre e e llegar justo a la posicin invertida. o 35. Una rueda de bicicleta se sostiene del eje con un hilo (amarrado a un solo lado). El punto de amarre se ubica a D = 20 cm del centro de la rueda. El neumtico y la a llanta pesan M = 4 kg y tienen un radio R = 30 cm. La rueda se hace girar a 10 rev/s. El eje se orienta (inicialmente) de manera horizontal. a) Demuestre que el eje de la rueda se mantendr en posicin horizontal a o y que esta realizar un movimiento a circular (coincidiendo el eje de este movimiento con el hilo). Este movimiento se llama precesin. o Encuentre la velocidad angular de precesin. o

Figura 9.36

b) 36.

Un cubo, de lado h y masa m, est colocado sobre una cinta transportadora en la a orientacin que se muestra en la gura adjunta. El coeciente de roce esttico entre o a el bloque y la cinta es e = 0, 5 y el coeciente de roce cintico es c = 0, 4. En t = 0, e la cinta comienza a moverse, aumentando linealmente su aceleracin. En t = 20 s su o 2 x y en t = 60 alcanza 9 m/s2 x. Entre t = 0 y t = 60 s, aceleracin es a = 3 m/s o se observa que el bloque se desplaza en la direccin x manteniendo su orientacin o o original (es decir, sin rotar con respecto a su centro de masa).

Figura 9.37

9.4 Solucin a algunos de los problemas o a) b) c)

249

Calcule la fuerza de roce que acta sobre el bloque cuando la aceleracin de la u o cinta transportadora es 3 m/s2 . Haga un grco de la aceleracin en funcin del tiempo. a o o Considere un sistema de coordenadas solidario al bloque, con origen en su centro de masa y con el eje x paralelo a la direccin en que se mueve la cinta. Cul o a es la coordenada x del punto donde acta la fuerza normal efectiva cuando la u aceleracin de la cinta transportadora es 6 m/s2 x ? o

9.4.

Solucin a algunos de los problemas o

Solucin al problema 8 o Denamos el eje z apuntando hacia arriba y concentrmonos primeramente en el disco e inferior. Sea a = a (con a > 0) la aceleracin lineal del disco inferior. Para tal disco, z o usando la segunda ley de Newton, se encuentra la ecuacin de movimiento o Ftot = mg + T z = ma , z o sea, mg T = ma . (9.2) Sea = la aceleracin angular del disco inferior. El torque ejercido por el peso (respecto al o centro de masas del disco) es nulo mientras que el torque ejercido por la cuerda es = T R. Se tiene d d = TR = = (I) = I = I . dt dt Usando el valor del momento de inercia de un disco I = mR2 /2, se encuentra 1 T = mR . 2 (9.3)

Siendo los dos discos iguales y siendo que el torque sobre el disco superior (respecto a su centro de masas) es igual al del disco inferior, se concluye que ambos discos se desenrollarn a con la misma aceleracin y velocidad angular. o La aceleracin lineal a y la aceleracin angular (que es la misma para ambos discos) no o o son magnitudes independientes, sino que estn correlacionadas. En efecto, se tiene que a 2R = a . (9.4)

A partir de las tres ecuaciones (9.2), (9.3) y (9.4), con las tres incgnitas T , a y , se o encuentra a = 4g/5 , T = mg/5 y = 2g/(5R) .

250 Solucin al problema 15 o

Rotacin de un cuerpo r o gido

Para encontrar las ecuaciones de movimiento de un problema en que la energ se conserva, a en muchas ocasiones el mtodo ms fcil consiste en escribir una expresin para la energ e a a o a y derivar sta respecto al tiempo. e Por ejemplo, para una masa que cae bajo el efecto de la gravedad: la energ total viene a dada por 1 E = mgz + mz 2 . 2 Derivando esta expresin respecto al tiempo se encuentra o 1 0 = mg z + m2z z , 2 o sea, la ecuacin de movimiento para la ca libre z = g. o da Usemos esta idea para resolver el presente problema. Si es el ngulo de desviacin del anillo a o es su velocidad angular, la energ total del sistema vendr dada respecto a la normal y a a por 1 (9.5) E = mgR(1 cos ) + I 2 . 2 Aqu m es la masa del aro e I es el momento de inercia respecto al punto de suspencin A. El o primer trmino al lado derecho representa el cambio de energ potencial del aro (respecto a e a su posicin de equilibrio) mientras que el segundo es la energ cintica rotacional en torno o a e a A. Derivando (9.5) respecto al tiempo se obtiene 0 = mgR sin + I , o sea, mgR sin . + I Usando el teorema de Steiner y el resultado del ejemplo 2 de la segunda seccin, deducimos o 2 . Usando esta relacin, y la que el momento de inercia del aro, en torno a A, es I = 2mR o aproximacin sin para ngulos pequeos, se obtiene o a n g + =0. 2R Esta ecuacin de movimiento para el aro corresponde a la de un oscilador armnico y o o coincide con la de un pndulo de largo L = 2R. El per e odo de oscilacin es o T = 2 2R . g

Para que el aro d una vuelta completa la energ cintica en la parte inferior debe coincidir e a e con 2mgR, que es la diferencia de energ potencial que el aro debe sobreponer. Si denotamos a por 0 a la velocidad angular en el m nimo, se tiene

9.4 Solucin a algunos de los problemas o

251

1 2 I = 2mgR , 2 0 o sea, 0 = 2g . R

Solucin al problema 18 o

a) El cambio de energ potencial debe ser igual a la energ cintica nal. La masa 2m a a e baja una distancia h mientras que la masa m sube una distancia h. Luego U = mgh . Sea v0 la rapidez nal de la masa 2m, justo antes de chocar con el suelo. Entonces la energ cintica, en ese instante, es a e 1 1 1 2 2 2 K = mv0 + (2m)v0 + I0 , 2 2 2 donde 0 = v0 /R es la velocidad angular nal de la polea. Usando el valor I = mR2 /2 para el momento de inercia se encuentra que
2 K = 2mv0 .

Igualando K con U se encuentra


2 v0 =

gh . 2

b) Los diagramas de cuerpo libre de las dos masas nos dan las ecuaciones de movimiento 1 mg = ma y (2m)g 2 = (2m)a , donde a es la aceleracin (hacia arriba) de la masa m. Sea 0 la aceleracin angular o o de la polea. El hecho de que la cuerda no resbale sobre la polea nos da la relacin o 0 R = a . Finalmente, evaluando el torque total que acta sobre la polea se encuentra la ecuacin u o de movimiento para la rotacin o 2 R 1 R = I 0 .

252

Rotacin de un cuerpo r o gido Tenemos cuatro ecuaciones para las cuatro incognitas 1 , 2 , a y 0 . Resolviendo este sistema de ecuaciones se encuentra 2 a= g, 7 9 1 = mg 7 y 2 = 10 mg . 7

c) Mientras las masas estn cayendo, como la polea no se desplaza, la fuerza total sobre a ella debe ser nula. Por lo tanto, la tensin de la cuerda que sujeta la polea debe ser o = 1 + 2 = 19 mg . 7

d) Consideremos la situacin que se tiene cuando la masa 2m ha tocado el suelo y todo o est detenido. El torque total sobre la polea es nulo y, por lo tanto, la tensin de la a o cuerda que pasa por la polea debe ser la misma a ambos lados, siendo su valor mg. Conclu mos que la tensin de la cuerda que sujeta la polea, en este caso, es o = 2mg .

Solucin al problema 19 o Slo la parte superior de la correa de transmisin transmite fuerza. Denotemos sta por F . o o e La ecuacin de movimiento para la rotacin de la polea #2 es o o F R = I 2 . Para la polea #1 sta es e R = I 1 . 2 El diagrama de cuerpo libre para la masa M nos da la ecuacin o TR F Mg T = Ma , donde a es la aceleracin (hacia abajo) de la masa M . Los ngulos de rotacin de ambas o a o poleas no son independientes sino que estn relacionados por a 1 R = 2 R . 2

Derivando dos veces respecto al tiempo y cancelando el radio R se obtiene 2 = 22 .

9.4 Solucin a algunos de los problemas o

253

Finalmente, tambin la aceleracin a est relacionada con 1 . En efecto, a = R1 . De e o a 1 y 2 . De esta las ecuaciones anteriores podemos despejar las cinco incgnitas a, T , F , o manera se encuentra 8 a = R 1 = g, 13 5 T = Mg 13 y 2 F = Mg . 13 Despus de bajar una distancia h la energ potencial disminuye en U = M gh. Esta e a energ debe transformarse en energ cintica. a a e Si la velocidad de la masa M es v, entonces las velocidades angulares de las poleas #1 son 1 = v/R y 2 = v/(2R), respectivamente. La energ cintica es, por lo tanto, a e K = = 1 M v2 + 2 1 M v2 + 2 1 2 1 2 I + I 2 1 2 2 1 1 13 2 2 M R2 (1 + 2 ) = M v 2 . 2 2 16

Igualando esto con la diferencia de energ potencial se encuentra para v la expresin a o v= 16 gh . 13

Solucin al problema 21 o El momento de inercia de la carretilla, para rotaciones alrededor de su eje de simetr es a, I=2 1 m(2R)2 2 1 + (2m)R2 = 5mR2 . 2 (9.6)

El diagrama de cuerpo libre para la masa M nos da la ecuacin o M g T = M aM , (9.7)

donde T es la tensin de la cuerda y aM la aceleracin (hacia abajo) de la masa M . Aplicando o o la segunda ley de Newton para el movimiento traslacional horizontal de la carretilla se encuentra la ecuacin o T fr = (4m)aC , (9.8) donde fr es el roce esttico entre la carretilla y la mesa y aC la aceleracin de la carretilla. a o (Este roce es el responsable de hacer que la carretilla ruede sin resbalar). Sea la aceleracin o angular de la carretilla. El torque neto sobre la carretilla debe ser igual al momento de inercia multiplicado por , o sea, fr (2R) T R = I . (9.9)

254

Rotacin de un cuerpo r o gido

Por supuesto que hay una estrecha relacin entre y aC , que viene dada por el hecho que o la carretilla rueda sin resbalar. Esta relacin es o 2R = aC . (9.10)

Tambin existe una relacin que vincula AM , aC y : La cuerda es inextensible y por lo e o tanto la diferencia entre la aceleracin de la carretilla y la masa M debe coincidir con la o aceleracin con que la cuerda se enrolla, es decir, o aC aM = R . (9.11)

Las ecuaciones (9.6), (9.7), (9.8), (9.9), (9.10) y (9.11) resuelven el problema. Para la aceleracin de la carretilla se obtiene o ac = g 2M . M + 21m

Solucin al problema 22 o Introduzcamos el sistema de coordenadas mostrado en la gura 9.23. (El eje y apunta hacia el interior del papel). Las distintas fuerzas que actan sobre el automvil durante u o el frenado son: Las fuerzas F1 = F1 z y F2 = F2 z que el pavimento ejerce sobre las ruedas delanteras y traseras, respectivamente; el peso Fp = M g y la fuerza de roce z fr = +c F1 x . Como el automvil no se eleva, la fuerza total en la direccin z debe ser nula, es decir, o o 0 = (F1 + F2 M g) z . (9.12)

Por otra parte, como el automvil durante el frenado tampoco gira (en torno al eje y ), el o torque total (respecto al centro de masas) debe ser nulo. Los torques que ejercen las cuatro fuerzas son: 1 = +F1 d y 2 = F2 b y p = 0 y r = fr h = F1 c h . y y La suma de estos torques debe ser nulo, condicin que nos entrega la relacin o o F1 d F2 b F1 c h = 0 . De las ecuaciones (9.12) y (9.13) se pueden despejar F1 y F2 , obtenindose e F1 = M g b b + d c h (9.13)

9.4 Solucin a algunos de los problemas o y F2 = M g d c h . b + d c h

255

Las ecuaciones anteriores dejan de ser vlidas si c h > d, ya que en ese caso, la fuerza F2 a se vuelve negativa, lo que signica que las ruedas traseras dejan de estar en contacto con el suelo. En otras palabras, las ecuaciones anteriores son vlidas mientras d > c h, relacin a o que favorece un diseo (del automvil) en que el centro de masas se ubica en la parte trasera n o y cerca del suelo. Conociendo F1 podemos calcular la fuerza de roce fr (que es la fuerza responsable de la (des)aceleracin del automvil). Para la la aceleracin se obtiene o o o a= fr F1 c bc x= x=g x = a0 x . M M b + d c h

Finalmente, la distancia D que recorre el automvil durante su frenado es o D=


2 v0 2 b + d c h = v0 . 2a0 2gbc

Observe que en este problema la fuerza neta sobre el automvil no es nula y, por lo tanto, o el torque neto respecto a otro origen no es nulo aun cuando lo sea respecto al centro de masas. Conrme la aseveracin anterior evaluando el torque respecto al punto de contacto o de la rueda delantera con el suelo. Reiteramos: Para determinar si cambiar el estado rotacional de un cuerpo acelerado, debe a evaluarse el torque total respecto al centro de masas (ver tambin cap e tulo siguiente). Analicemos ahora el proceso de aceleracin. Sea sta a = a0 x. Supongamos que el motor o e ejerce la fuerza sobre las ruedas traseras y que estas no resbalan. En ese caso la fuerza de roce (esttica) fr = fr x actuar sobre las ruedas traseras y en direccin ). Mientras a a o x el automvil est con las cuatro ruedas sobre el suelo, el torque total respecto al centro de o a masas debe ser nulo, o sea, F1 d F2 b + fr h = 0 . La fuerza que acelera el automvil es la fuerza de roce, es decir, o fr = M a . En la direccin vertical la suma de todas las fuerzas sobre el automvil debe ser nula: o o F1 + F2 = M g . Las tres ultimas ecuaciones permiten determinar F1 y F2 : F1 = M F2 = M gb a0 h , b+d gd + a0 h . b+d

256

Rotacin de un cuerpo r o gido

Observe que F1 es mayor que cero slo mientras a0 < gb/h. Para aceleraciones mayores, las o ruedas delanteras del automvil pierden contacto con el suelo. La aceleracin maxima (si o o el roce esttico lo permite) viene dada por a b . h

amax = g

Solucin al problema 29 o El momento angular total debe conservarse. Inicialmente el momento angular es 1 2 mr 2

Li = I1 0 =

0 .

El momento angular una vez que el segundo disco este en reposo respecto al primero es Lf = I , donde I es el momento de inercia de la conguracin nal para rotaciones alrededor del eje o jo. Se tiene 1 I = I1 + I2 = M R2 + 2 1 2 mr + mD2 2 ,

donde para I2 hemos usado el teorema de Steiner. Igualando los dos momentos angulares se deduce 1 2 1 mr 0 = M R2 + 2 2 o sea, = mr2 0 . mr2 + M R2 + 2mD2 1 2 mr + mD2 2

Solucin al problema 35 o En la gura 9.38 la rueda gira en el plano (x, z), movindose la parte superior de la rueda e hacia el lector. El momento angular debido al giro de la rueda alrededor de su eje, por lo tanto, apunta en la direccin y . o

9.4 Solucin a algunos de los problemas o Suponiendo que toda la masa de la rueda est concentrado en la periferia, su moa mento de inercia para rotaciones alrededor de su eje ser a I0 = M R2 . El momento angular asociado a este movimiento (para el instante mostrado en la gura 9.38), respecto al origen O, ser a L0 = I0 y , donde es la velocidad angular con que gira la rueda alrededor de su eje (esto es, = 2 10 s1 ).

257

Figura 9.38

El torque total repecto a O es = M gD x . Pero, por otra parte, = dL , dt

luego el cambio de momento angular (para la situacin mostrada en la gura 9.38) ser o a dL = M gD x . dt Observe que L y dL son perpendiculadt res (ver tambin gura 9.39). Como L y e dL estn en el plano (x, y), el vector L a seguir en ese plano, pero cambiar su a a orientacin. En otras palabras, el eje de o la rueda girar en el plano (x, y), mana tenindose horizontal. e Figura 9.39 Para deducir la velocidad angular con la cual el eje de la rueda gira alrededor del eje z , recordemos algunos aspectos del movimiento circular uniforme: Sea r un vector de largo R que gira en el plano (x, y) con velocidad angular uniforme. En ese caso v = dr/dt es siempre perpendicular a r. Si conocemos el radio de giro y la rapidez con que gira podemos determinar la celocidad angular: = v |dr/dt| = . R |r|

258

Rotacin de un cuerpo r o gido

La situacin en el presente problema es anloga. Tenemos un vector L que gira en el plano o a (x, y). Conocemos L = |L| y la rapidez |dL/dt|, luego podemos encontrar la velocidad angular de presesin con la que gira L: o = |dL/dt| |L| = M gD gD = 2 . M R2 R

Tanto ms rpido gira la rueda, tanto ms lento es la precesin. a a a o

Cap tulo 10

Fuerzas cticias
Las fuerzas cticias son fuerzas que deben incluirse en la descripcin de un sistema f o sico cuando la observacin se realiza desde un sistema de referencia no inercial y, a pesar de o ello, se insiste en usar las leyes de Newton. O sea, la introduccin de fuerzas cticias hace o posible la descripcin de un sistema f o sico usando, por ejemplo, un sistema de referencia uniformemente acelerado o un sistema de referencia jo a un cuerpo que rota uniformemente. En lo que sigue trataremos slo estos dos casos. o

10.1.

Referencial uniformemente acelerado

Sea S : (, y , z ) un sistema de referencia inercial y S : ( , y , z ) un sistema de referencia x x que acelera con aceleracin constante a0 respecto a S. El vector que une los or o genes O y O de ambos sistemas de referencia es 1 R(t) = R0 + V0 t + a0 t2 . 2 Sean r (t) y r (t) los vectores de posicin de una masa m en los sistemas de referencia S y o S , respectivamente. La relacin entre r y r es o r =R+r . Derivando dos veces respecto al tiempo se obtiene r = R + r = a0 + r , o sea, mr = mr ma0 . (10.1) Sea F la fuerza real neta que acta sobre la masa m, es decir, la fuerza que genera la u aceleracin r de la masa m observada desde un sistema de referencia inercial. En otras o palabras F = mr .

260

Fuerzas cticias

Si se insiste en usar la segunda ley de Newton, pero con las magnitudes observadas desde un sistema de referencia acelerado, se tiene F = mr , pero la fuerza F ahora ya no es F sino que, de acuerdo con la ecuacin (10.1), o F = F ma0 . El trmino ma0 = Fct es la fuerza cticia que hay que agregar a la fuerza real F para e poder seguir usando la segunda ley de Newton desde un sistema acelerado con aceleracin o a. Observe que esta fuerza cticia acta como un campo gravitacional constante (adicional al u campo gravitacional g que pudiese estar presente). Ejemplo: Consideremos un pndulo sobre un e carro que acelera con aceleracin constante o a = a0 x (ver gura 10.1). Encontremos el a ngulo entre la normal y la posicin de o equilibrio del pndulo. e Resolveremos el problema de dos maneras: i) usando primero el sistema de referencia inercial del observador O y ii) el sistema de referencia acelerado jo a O .

Figura 10.1

i) En el sistema de referencia inercial el diagrama de cuerpo libre de la masa m se muestra en la gura 10.2a. La fuerza neta que acta sobre la masa m es u F = + Fg = cos z + sin x mg z . En el sistema de referencia inercial la part cula acelera con una aceleracin a = a x, luego o F = cos z + sin x mg z = ma . x Igualando las componentes de esta ecuacin vectorial se obtiene o cos = mg y sin = ma . Dividiendo la segunda ecuacin por la primera se deduce nalmente que o tan = a . g

10.1 Referencial uniformemente acelerado

261

Figura 10.2a

Figura 10.2b

ii) Para un observador sobre el carro la masa m no se mueve. Por eso, para O la fuerza neta sobre la masa m debe ser nula. El diagrama de cuerpo libre en este caso se muestra en la gura 10.2b. Adems de la fuerza ejercida por la tensin del hilo y de la gravedad, a o debemos agregar la fuerza cticia Fct = ma x. Tenemos F = + Fg + Fct = 0 , o sea 0 = cos z + sin x mg z ma x . Nuevamente, igualando las componentes de esta ecuacin vectorial se deduce que o cos = mg y sin = ma , o sea, las mismas relaciones encontradas en la parte i). Para el observador O sobre el carro, tambin podr e amos haber simplemente considerado un campo gravitacional efectivo (ver gura 10.3). ge = g a = g z a x . Es evidente que el ngulo que ge hace con la a normal cumple con la relacin tan = a/g . o Si el pndulo realiza pequeas oscilaciones en e n torno a su posicin de equilibrio la frecuencia o angular de las oscilaciones ser a = donde es el largo del pndulo. e ge = g 2 + a2 ,

Figura 10.3

262

Fuerzas cticias

10.2.

Referencial en rotacin uniforme o

Sea S : (, y , z ) un sistema de referencia x inercial y S : ( , y , z ) un sistema de refex rencia que coincide con el sistema S en cierto instante (por ejemplo, en t = 0), pero que rota con velocidad angular = z constante en torno al eje z (ver gura 10.4). Sea A un vector con componentes Ax , Ay , Az en el sistema de referencia S y componentes Ax , Ay , Az en el sistema de referencia S , o sea, A(t) = Ax (t) x + Ay (t) y + Az (t) z . y A(t) = Ax (t) x + Ay (t) y + Az (t) z . Figura 10.4

Los vectores unitarios del sistema de referencia inercial x, y , z son jos, sin embargo, los vectores unitarios del sistema de referencia rotatorio x , y , z rotan, tenindose e x = cos(t) x + sin(t) y y = sin(t) x + cos(t) y z =z. Derivando estos vectores respecto al tiempo se encuentra x = sin(t) x + cos(t) y = y y = cos(t) x sin(t) y = x z =0. Evaluemos la derivada del vector A en ambos sistemas de referencia. Por una parte se tiene A = Ax x + Ay y + Az z , y por otra parte A = Ax x + Ax + Ay y + Ay + Az z + Az x y z = (Ax x + Ay y + Az z ) + (Ax y Ay x ) . Usando las relaciones anteriores y el hecho que A = z Ax (t) x + Ay (t) y + Az (t) z = Ax y Ay x ,

10.2 Referencial en rotacin uniforme o podemos escribir Ax x + Ay y + Az z = Ax x + Ay y + Az z o sea dA dt =


S

263

+A,

dA dt

+A.
S

(10.2)

En la ultima expresin los ejes unitarios no aparecen expl o citamente, por consiguiente, es una expresin que tiene una validez general (es decir, no slo para rotaciones en torno al eje o o z ). La ecuacin (10.2) relaciona la derivada temporal de cualquier vector en el sistema de o referencia inercial S con la derivada temporal de ese mismo vector, pero observada desde un sistema de referencia S que rota con velocidad angular respecto a S. Apliquemos la ecuacin (10.2) al vector posicin r de una part o o cula de masa m. Se tiene dr dt o sea, vS = vS + r . Ac vS es la velocidad de la part a cula m observada por el observador inercial S y vS es la velocidad de la misma part cula, pero observada desde el sistema de referencia rotatorio. Apliquemos nuevamente la ecuacin (10.2), pero ahora al vector vS . Se tiene o dvS dt =
S

=
S

dr dt

+r ,
S

dvS dt

+ vS .
S

(10.3)

Usando la ecuacin (10.3), se obtiene o dvS dt =


S

d(vS + r ) dt dvS dt dvS dt +


S

+ (v S + r)
S

= =

d( r ) dt

+ vS + ( r)
S

+ 2 vS + r
S

El lado izquierdo de la ultima ecuacin es la aceleracin de la part o o cula observada por el observador inercial S, denotmosla por aS . El primer trmino al lado derecho es la e e aceleracin de la misma part o cula pero observada desde el sistema de referencia rotacional S , denotmosla por aS . De esta manera obtenemos e maS = maS 2m vS m r . (10.4)

Sea F la fuerza real neta que acta sobre la masa m, es decir, la fuerza que genera la u aceleracin aS de la masa m observada desde un sistema de referencia inercial. En otras o palabras F = maS .

264

Fuerzas cticias

Si se insiste en usar la segunda ley de Newton, pero con las magnitudes observadas desde un sistema de referencia acelerado, se tiene F = maS , pero la fuerza F ahora ya no es F sino que, de acuerdo con la ecuacin (10.4), o

F = F 2m vS m r

Los trminos e Fct = 2m vS m r (10.5)

son la fuerza cticia que hay que agregar a la fuerza real F para poder seguir usando la segunda ley de Newton desde un sistema de referencia que rota respecto a un sistema de referencia inercial con velocidad angular . El primer trmino de la fuerza cticia dada por la ecuacin (10.5) es la as llamada fuerza e o de Coriolis FCoriolis = 2m vS mientras el segundo trmino se llama fuerza centr e fuga

Fcent = m r

Lo interesante de la fuerza de Coriolis es que ella slo aparece si, en el sistema de referencia o rotacional S , la masa se mueve, y en ese caso, es perpendicular a la direccin de movimiento. o Cuando m est en reposo (es decir, vS = 0) entonces la unica fuerza cticia que hay que a agregar a la fuerza que se observa en un sistema inercial, es la fuerza centr fuga. Cuando realizamos experimentos en la tierra (laboratorio) siempre asumimos que un sistema jo al laboratorio representa un sistema de referencia inercial. Sin embargo, la rotacin de o la tierra en torno a su propio eje (con una frecuencia = 2/(24 3600) = 7, 27 105 s1 hace que el sistema de referencia no sea inercial y que, en la prctica, debamos en ocasiones a agregar la fuerza cticia (10.5) para obtener una descripcin correcta del sistema. La fuerza o de Coriolis es responsable de muchos efectos (a veces sorprendentes) que se observan a nuestro alrededor. Por ejemplo, es la responsable de la rotacin de los ciclones y de las o corrientes marinas o del giro del plano de oscilacin de un pndulo. o e

10.3 Problemas

265

10.3.
1.

Problemas

Demuestre que la fuerza centr fuga que acta sobre una masa m (si sta es obu e servada desde un sistema de referencia rotacional que gira con velocidad angular respecto a un sistema de referencia inercial) viene dada por
2 Fcentr fuga = m ,

donde es la distancia entre el eje de rotacin y la masa m y es un vector unitario o que apunta el eje hacia la masa m y es perpendicular al eje de giro (ver gura 10.5). Observe que la fuerza centr fuga tiene la misma magnitud y direccin de la fuerza o centr peta slo que apunta en el sentido o opuesto. 2.

Figura 10.5

En un parque de diversiones, los participantes se sostienen contra la pared de un cilindro giratorio mientras el suelo se hunde. El radio del cilindro es R = 3m y el coeciente de roce entre las personas y la pared del cilindro es e = 0, 4. Determine el nmero m u nimo de revoluciones por minuto que se requiere para que el juego funcione. Haga el clculo de dos maneras distintas: i) usando un sistema de referencia inercial a y ii) usando un sistema de referencia solidario al cilindro.

3.

Considere el efecto de la rotacin terrestre sobre el movimiento de un proyectil que o se lanza desde la supercie terrestre con velocidad v0 . Suponga que el alcance del proyectil es tal que en todo instante se mueve en un campo gravitacional constante, es decir, Fg = mg. a) Demuestre que la velocidad del proyectil viene dada por v = v0 + g t 2 r . Todas las magnitudes estn medidas respecto a un observador solidario con la a tierra. Ac r es el vector posicin del proyectil medido desde el punto de lanzaa o miento y es el vector velocidad angular de la tierra. Al resolver el problema no se debe incluir la fuerza centr fuga ya que sta est ine a cluida en el valor local de g que se est usando. Al rotar la tierra no slo se a o modica la magnitud g sino que tambin su direccin. La fuerza centr e o fuga incluso modica la forma de la tierra; de hecho, la normal a la supercie terrestre usualmente no pasa por el centro de la tierra. b) Demuestre que, al despreciar trminos del orden 2 , para la aceleracin se obtiene e o la ecuacin o a = g 2 g t 2 v0 .

266

Fuerzas cticias (Nuevamente todas las magnitudes medidas desde un sistema de referencia solidario a la tierra). Integre la ultima ecuacin y demuestre que o r(t) = v0 t + 1 2 1 g t g t3 v0 t2 . 2 3

4.

Desde un edicio de altura h = 100 m situado en el Ecuador terrestre, se suelta una piedra. Debido a la rotacin terrestre, o la piedra no caer a lo largo de la normal a sino que se desviar levemente de ella. Una a vez que llega al suelo, encuentre la magnitud y direccin de la desviacin. Desprecie o o efectos debido al roce viscoso con el aire. Indicacin: use el resultado obtenido en el o problema anterior. Respuesta: La desviacin es hacia el este o y es de magnitud 2 h 3 2h g Figura 10.6 2, 19 cm .

5.

Desde el Ecuador se lanza un proyectil con velocidad v0 = 500 m/s en la direccin o esteoeste, con un ngulo de elevacin = 10 . Encuentre como cambia el tiempo que a o el proyectil tarda en volver a chocar con la tierra y el alcance del proyectil debido a la rotacin terrestre. Para resolver este problema no hay que incluir la fuerza centr o fuga ya que el efecto de ella ya se incluy en el vector la aceleracin de gravedad g, que o o supondremos constante en magnitud y direccin sobre toda la trayectoria. Ignore o cualquier efecto debido al roce con el aire y desprecie correcciones del orden 2 . Respuesta: El alcance disminuye en D =
3 4v0 sin g2

1 sin cos2 3

62, 9 m .

Qu pasa si en lugar de dispararlo de este a oeste se dispara de oeste a este o de sur e a norte? 6. Considere un canal de ancho a ubicado sobre la tierra a una latitud > 0. Por el canal uye agua con una velocidad v0 . Demuestre que el nivel del agua al lado derecho del canal es superior a la del lado izquierdo en una cantidad h = 2av0 sin , g

10.3 Problemas donde es la velocidad angular de la tierra.

267

7.

Un balde con agua gira en torno a su eje de simetr con velocidad angular . Dea bido a la rotacin, la supercie del agua o no ser plana. Encuentre su forma. a

Figura 10.7 8. Considere un pndulo cnico de largo , cuya masa gira en el plano horizontal en un e o c rculo de radio R. Si se ignora la fuerza de Coriolis, la frecuencia angular del pndulo e cnico es 0 = g/ , siendo esta independiente del sentido del giro. Demuestre que o al incluir la fuerza de Coriolis, las frecuencias en ambos sentidos ya no son iguales, tenindose e g 1 = + ( sin )2 sin y 2 = g + ( sin )2 sin ,

donde es la latitud del lugar en que se encuentra el pndulo. e 9. (Pndulo de Foucault) e Al superponer (sumar) las dos soluciones de un pndulo cnico correspondientes al e o mismo radio de giro, pero rotando en sentidos opuestos, se obtiene la solucin de un o pndulo simple. e a) b) Demuestre lo anterior en forma expl cita para un pndulo cnico ignorando la e o fuerza de Coriolis. Al realizar el mismo clculo, pero ahora incluyendo el efecto de Coriolis (ver a problema anterior), se encuentra que debido a la rotacin terrestre, el plano de o oscilacin del pndulo simple no se mantendr invariante sino que girar paulatio e a a namente. Demuestre que la velocidad angular con que gira el plano de oscilacin o del pndulo viene dado por F = sin , donde es la velocidad angular de la e tierra en torno a su propio eje y es la latitud del lugar en que se encuentra el pndulo. e Foucault fue el primero en demostrar experimentalmente, con un pndulo muy e largo, que el plazo de oscilacin efectivamente gira a medida que transcurre el o tiempo. Observe que en el Ecuador el plano de oscilacin no gira, mientras que o en los polos da una vuelta completa en 24 horas (despus de 6 horas el plano de e ). oscilacin habr girado en 90 o a

268

Fuerzas cticias

10.

Considere una cua de masa M y ngun a lo de elevacin que puede deslizarse o sobre un plano horizontal sin roce. Sobre el plano inclinado se encuentra otra masa m, que a su vez tambin puede e deslizarse sobre el plano sin roce. Encuentre la aceleracin del plano inclinao do M .

Figura 10.8

10.4.

Solucin a algunos de los problemas o

Solucin al problema 3 o Como la tierra est girando en torno a su propio eje, la fuerza efectiva que acta sobre el a u proyectil ser a Fe = m g 2 m v . (La fuerza centr fuga Fcentrifuga = m (r) no debe incluirse ya que su efecto est consia derado en el valor local de la aceleracin de gravedad g). Todas las magnitudes en la ecuacin o o anterior se reeren a variables medidas en un sistema jo a la Tierra, es decir, respecto a un sistema de referencia que rota con velocidad angular respecto a un sistema de referencia inercial. ( es la velocidad angular de la rotacin de la Tierra alrededor de su propio eje). o Al incluir la fuerza efectiva podemos seguir usando la segunda ley de Newton. Se tiene Fe = m o sea, m dv = m g dt 2 m dr . Integrando (sumando) desde un instante inicial a uno nal se obtiene
f f f

dv dr = mg 2m , dt dt

dv =
i i

g dt 2
i

dr .

Sea ti = 0, tf = t, vi = v0 , vf = v, ri = 0 y rf = r. Entonces, evaluando las integrales de la ultima ecuacin se obtiene o v v0 = g t 2 r . (10.6) Derivando esta ecuacin respecto al tiempo se encuentra la aceleracin o o a = g 2 v . Reemplazemos v en esta ecuacin por la expresin dada por (10.6), entonces o o a = g 2 ( v0 + g t 2 r ) = g 2 v0 2 g t + o(2 )

10.4 Solucin a algunos de los problemas o

269

Integrando estas expresiones respecto al tiempo se encuentra la velocidad y luego la posicin o en funcin de t: o
f f f f

a dt =
i i

g dt 2 v0
i

dt 2 g
i

t dt

v v0 = g t 2 v0 t g t2
f f f f f

v dt =
i i

v0 dt +
i

g t dt 2 v0
i

t dt g
i

t2 dt

r (t) = v0 t +

1 2 1 g t ( v0 ) t2 ( g ) t3 . 2 3

(10.7)

Solucin al problema 5 o Denamos los ejes del sistema referencia solidario con la Tierra tal como se muestra en la gura adjunta. El proyectil se dispara desde P en la direccin esteoeste con un ngulo o a de elevacin , luego la velocidad inicial del o proyectil viene dada por v0 = v0 cos y + v0 sin x . Los vectores correspondientes a la aceleracin o de gravedad (local) y la velocidad angular de la Tierra vienen dados por g = g g Figura 10.9 y =z. Reemplazando estas relaciones en (10.7) se obtiene la posicin del proyectil a medida que o transcurre el tiempo (medido desde el lugar de lanzamiento): 1 1 r(t) = v0 t cos y + v0 t sin x gt2 x + g t3 y v0 t2 cos x v0 t2 sin y . 2 3 Sea t el instante en que el proyectil vuelve a caer sobre la Tierra. En ese instante se tiene que r (t ) = D y ,

270

Fuerzas cticias

donde D es el alcance del proyectil. Evaluando r (t) en el instante t e igualando el resultado con la expresin anterior, se puede despejar t y D obtenindose o e 2 v0 sin t = g + 2 v0 sin

2v0 sin g

2v0 g

sin cos + o(2 ) .

(Estamos despreciando todas las correcciones del orden 2 ). Para D se obtiene la expresin o D = vo t cos 1 3 t g + v0 t2 sin . 3

Sustituyendo en esta ecuacin la expresin para t se encuentra (despreciando nuevamente o o todos los trminos de orden o(2 )) que e D=
2 2v0 v3 sin cos + 4 0 2 sin g g

1 sin2 cos2 3

El primer trmino del lado derecho de la ultima ecuacin es el alcance del proyectil si se e o ignora la fuerza de Coriolis; el segundo trmino es la correccin (a primer orden en ), que e o sufre el alcance debido a la rotacin terrestre. o Solucin al problema 9 o a) La posicin en funcin del tiempo de un o o pndulo cnico de largo e o que recorre un circulo de radio R viene dada por r(t) = R cos(t) x + R sin(t) y , con = g/ . Esta solucin corresponde a o un pndulo cnico que gira en la direccin e o o contraria al reloj. Una solucin que gira en el o mismo sentido que el reloj viene dada por r2 (t) = R cos(t) x + R sin(t) y . Figura 10.10 Al ignorar la rotacin de la Tierra (es decir, al despreciar la fuerza de Coriolis) las frecuencias o angulares para ambos sentidos es la misma. Al sumar las dos soluciones se obtiene la proyeccin sobre el plano x y de la solucin o o correspondiente a un pndulo lineal. En efecto: e (t) = r1 (t) + r2 (t) = 2R cos(t) x . El lado derecho corresponde al movimiento de un oscilador a lo largo del eje x con amplitud 2R (esto es, la proyeccin de la posicin del pndulo sobre el plano horizontal). o o e

10.4 Solucin a algunos de los problemas o b) Al incluir el efecto de la fuerza de Coriolis, los vectores posicin (en el plano x y) de o los pndulos cnicos, a medida que transcurre e o el tiempo, vienen dados por r1 (t) = R cos(1 t) x + R sin(1 t) y , y r2 (t) = R cos(2 t) x + R sin(2 t) y , con 1 = y 2 = g + ( sin )2 sin , Figura 10.11 g + ( sin )2 sin

271

donde es la latitud del lugar en que se encuentra el pndulo (ver problema 10.8). Al e sumar las dos soluciones y usar las relaciones 1 cos(1 t) + cos(2 t) = 2 cos (1 2 ) 2 y 1 sin(1 t) + sin(2 t) = 2 cos (1 2 ) 2 se encuentra g

cos

1 (1 + 2 ) 2 1 (1 + 2 ) 2

sin

(t) = r1 (t) + r2 (t) = 2R cos

2 sin2 +

t [cos(t sin ) x sin(t sin ) y ] .

La expresin delante del parntesis cuadrado corresponde a un movimiento oscilatorio de o e amplitud 2R y con esencialmente la frecuencia g/ . El trmino en parntesis cuadrado e e es un vector unitario que indica la direccin de oscilacin. Observe, sin embargo, que ese o o vector unitario rota lentamente en el plano x y a medida que transcurre el tiempo. O sea, la direccin de oscilacin de este oscilador (el plano de oscilacin del pndulo) rotar a o o o e a medida que avanza el tiempo, siendo la velocidad angular de rotacin de este movimiento o F = sin .

272 Solucin al problema 10 o

Fuerzas cticias

Resolvamos el problema usando un sistema solidario a la cua. Sea a = a la aceleracin n x o de la cua. Al gracar los diagramas de cuerpo libre en el sistema de referencia acelerado, n debemos agregar las fuerzas cticias. Los diagramas de cuerpo libre de la masa m y de la cua se muestran en la gura siguiente. n

Figura 10.12a

Figura 10.12b

En el sistema solidario a la cua, la aceleracin de la masa m es am = ax x a2 z , donde n o az = tan , (10.8) ax y la aceleracin de la cua M es nula. De los diagramas de cuerpo libre se deducen las o n ecuaciones de movimiento mg z + FN cos z + FN sin x + ma x = m am y FN sin x + M a x M g z + FN z = 0 . De la segunda ecuacin se encuentra que FN = M a/ sin , y de la primera se obtienen las o componentes az y ax de la aceleracin de m: o az = mg FN cos ax = ma + FN sin . Usando estas relaciones y (10.8), se encuentra tan = de donde, nalmente, a= mg FN cos , ma + FN sin

mg . M cotg + (m + M ) tan

Cap tulo 11

Gravitacin o
El presente cap tulo tratar algunos aspectos de la teor de gravitacin de Newton. Coa a o menzaremos este cap tulo con un complemento matemtico sobre elipses. a

11.1.

Elipses

Consideremos dos puntos, f1 y f2 ubicados en un plano. Consideremos adicionalmente un tercer punto P (en el mismo plano), y denotemos por r y r a las distancias de este punto y f1 y f2 , respectivamente. Por denicin, una elipse es el lugar geomtrico de todos los o e puntos del plano para los cuales r + r = 2a, en que 2a es una constante (mayor que la separacin entre f1 y f2 ). o Introduzcamos un sistema de coordenadas cartesiano, con el origen en el centro de la gura geomtrica y el e eje x a lo largo de la recta que une los dos focos f1 y f2 . Es claro que el semieje mayor de la elipse es a. Sea 2c la distancia entre los dos focos, entonces a2 c2 . Figura 11.1

semieje menor = b =

11.1.1.

Elipse en coordenadas cartesianas

De la gura 11.1 se deduce que 2a = r + r = = (c + x)2 + y 2 + (c x)2 + y 2 (c2 + x2 + y 2 ) 2cx .

(c2 + x2 + y 2 ) + 2cx +

274 Elevando al cuadrado se obtiene 2(c2 + x2 + y 2 ) + 2 o sea, 2a2 (c2 + x2 + y 2 ) = = (c2 + x2 + y 2 ) + 2cx (c2 + x2 + y 2 ) + 2cx

Gravitacin o

(c2 + x2 + y 2 ) 2cx = 4a2 ,

(c2 + x2 + y 2 ) 2cx

(c2 + x2 + y 2 )2 4c2 x2 .

Elevando nuevamente al cuadrado queda 4a4 4a2 (c2 + x2 + y 2 ) + (c2 + x2 + y 2 )2 = (c2 + x2 + y 2 )2 4c2 x2 , o sea, c2 x2 = a4 a2 c2 a2 x2 a2 y 2 . Usando la relacin c2 = a2 b2 se deduce nalmente o x2 y 2 + 2 =1. a2 b (11.1)

11.1.2.

Elipse en coordenadas polares

Otra representacin util y comn de la elipse es en trminos de coordenadas polares (r, ), o u e tomando como origen a uno de los focos (ver gura 11.2). 1 Se tiene que x = r cos c , y = r sin y y 2 = r2 (1 cos2 ) . Sustituyendo estas expresiones en la ecuacin de la elipse en coordenao das cartesianas dada por la ecuacin o (11.1), se obtiene Figura 11.2 b2 (r cos c)2 + a2 r2 (1 cos2 ) = a2 b2 , b2 r2 cos2 2b2 rc cos + b2 c2 + a2 r2 a2 r2 cos2 = a2 b2 , o sea, a2 r2 = (a2 c2 ) b2 + 2b2 cr cos + (a2 b2 ) r2 cos2 = b4 + 2b2 = b2 + a2 b2 r cos + (a2 b2 ) r2 cos2 a2 b2 r cos
2

11.1 Elipses Extrayendo la ra se deduce que z, ar = b2 + Introduzcamos los parmetros a r0 y b2 c = 1 2 (excentricidad) . a a Con estas deniciones la ecuacin (11.2) se puede escribir de la forma o 1 1 = (1 cos ) . r r0 b2 a a2 b2 r cos .

275

(11.2)

(11.3)

(11.4)

(11.5)

Resumen: Las dos formas ms usuales para representar una elipse son: a i) Coordenadas cartesianas x, y (con el origen al centro): x2 y 2 + 2 =1, a2 b (11.6)

donde los parmetros a y b representan a los semiejes mayor y menor, respectivaa mente. ii) Coordenadas polares r, (con el origen en uno de los focos): 1 1 = (1 cos ) . r r0 Las ecuaciones (11.3) y (11.4) relacionan los parmetros a, b con r0 , . a Veamos algunas propiedades adicionales de las elipses. Coloquemos el origen en uno de los focos y sea r1 la distancia m nima (perigeo) y r2 la distancia mxima (apogeo) entre el origen y la elipse. (No confundir el perigeo y apogeo a con los semiejes menor y mayor de la elipse!) Se tienen las siguientes relaciones: r1 = a c y r2 = a + c , de donde se deduce que r1 + r2 = 2 a (11.7)

276 y r1 r2 = a2 c2 = b2 . En otras palabras, el semieje mayor es el promedio aritmtico e a= y el semieje menor el promedio geomtrico e b= del apogeo y perigeo. La excentricidad, en trminos de r1 y r2 , es e = r 2 r1 . r1 + r2 r1 r 2 r1 + r2 2

Gravitacin o

Ejercicio: Conociendo el rea de un c a rculo y las ecuaciones en coordenadas cartesianas de un c rculo y de una elipse, demuestre que el rea A de una elipse viene dada por a A = ab , donde a y b son sus semiejes. Mostraremos nalmente que si un rayo (por ejemplo, de luz) emerge de un foco y ste se e reeja especularmente en la elipse, entonces tal rayo pasar necesariamente por el otro foco. a Para demostrar esta propiedad consideremos dos puntos P y P sobre la elipse, innitesimalmente cercanos. Sean R1 y R1 los rayos que unen los puntos P y P con el foco f1 y R2 y R2 los rayos que unen los puntos P y P con el foco f2 (ver gura 11.3). Como R1 + R2 = R1 + R2 = 2a, se deduce que P A = P B. Los tringulos P AP y P BP son a equivalentes, luego los ngulos y son iguales. Pero = , luego = . Esta ultima a relacin implica que los rayos R1 y R1 corresponden a rayos reejados en forma especular o por la elipse.

Figura 11.3

11.2 Las leyes de Kepler Ejercicio: Considere la funcin siguiente (en coordenadas polares): o 1 =1 r Graque r() para = 0, 0.5, 1.0 y 1.5. cos .

277

11.2.

Las leyes de Kepler

Basndose en precisas mediciones de la posicin de los planetas realizadas por Tycho Brahe, a o Johannes Kepler (15711630) pudo establecer las siguientes leyes para describir el movimiento de los planetas alrededor del Sol: 1. Los planetas se mueven alrededor del Sol en rbitas el o pticas, estando el Sol en uno de los focos. Cada planeta barre reas iguales en tiempos iguales. a El cuadrado del per odo de un planeta alrededor del Sol es proporcional al cubo del semieje major de su trayectoria.

2. 3.

Estas leyes emp ricas se conocen como Leyes de Kepler. Comentarios: a) Por lo que sabemos, la primera ley de Kepler no puede ser completamente correcta. Si dos cuerpos son libres excepto por la fuerza de interaccin que existe entre ellos, o entonces stos deben moverse de manera que el centro de masas se mantenga en reposo e (o en movimiento uniforme). O sea, el Sol necesariamente tambin tiene que estar e movindose. Sin embargo, siendo la masa del Sol muy superior a la de los planetas, e el centro de masas Solplaneta esencialmente coincide con la posicin del Sol y en ese o caso resulta ser una muy buena aproximacin suponer que el Sol est en reposo en o a uno de los focos de la elipse. (La masa del Sol es de 1, 99 1030 kg mientras que la de la Tierra es de 5, 98 1024 kg.) b) Tal como se demostr en el ejemplo 1 de la seccin 9.2, el hecho de que un planeta o o barre reas iguales en tiempos iguales es equivalente a decir que el momento angular a no var en funcin del tiempo. Esto a su vez implica que el torque ejercido por el a o Sol sobre el planeta es nulo, lo que a su vez implica que la fuerza entre los dos cuerpos debe ser a lo largo de la l nea que los une. En otras palabras, la segunda ley de Kepler implica que la fuerza entre el Sol y el planeta debe ser radial. La fuerza gravitacional, por lo tanto, es de la forma F (r ) = f (r) r . c) La tabla adjunta muestra algunos de los parmetros para los planetas pertenecientes a al sistema solar.

278

Gravitacin o

Planeta Mercurio Venus Tierra Marte Jpiter u Saturno Urano Neptuno Plutn o

Semieje mayor u.a. 0,387 0,723 1,000 1,523 5,202 9,554 19,218 30,109 39,60

Per odo [s] 7, 60 106 1, 94 107 3, 16 107 5, 94 107 3, 74 108 9, 30 108 2, 66 109 5, 20 109 7, 82 109

Excentricidad 0,205 0,006 0,016 0,093 0,048 0,055 0,046 0,008 0,246

Inclinacin o 7 00 3 23 1 51 1 18 2 29 0 46 1 46 17 07

Masa [kg] 3, 28 1023 4, 83 1024 5, 98 1024 6, 37 1023 1, 90 1027 5, 67 1026 8, 80 1025 1, 03 1026 5, 4 1024

Los astrnomos, para medir distancias, frecuentemente usan la unidad astronmica o o u.a.. Una u.a. es igual al semieje mayor de la rbita terrestre. 1 u.a.= 1, 495 1011 m. o La excentricidad = (r1 r2 )/(r1 + r2 ) de la mayor de los planetas es bastante a pequea, siendo sus rbitas, por lo tanto, casi circulares. (Al dibujar una elipse con n o una excentricidad = 0, 05 es dif cil, slo mirndola, darse cuenta de que diere de o a un c rculo.) Si asumimos que, en primera aproximacin, las rbitas de los planetas son circulares, entono o ces, a partir de la tercera ley de Kepler, podemos encontrar una expresin para la magnitud o de la fuerza gravitacional. En efecto: La tercera ley de Kepler dice que T 2 = Kr3 , donde K es una constante (la misma para todos los planetas) y r es el radio de la rbita o circular. El radio r de la rbita, la velocidad v y el per o odo T estn relacionados por a T = 2r . v

Por otra parte la magnitud de la fuerza gravitacional debe coincidir con la fuerza centr peta, o sea, mv 2 f (r) = , r donde m es la masa del planeta. Usando estas tres ecuaciones, despejando v y T , se encuentra 4 2 m 1 f (r) = . K r2 La constante K, de acuerdo a la tercera ley de Kepler, no depende de ninguna propiedad de los planetas, pero si podr depender de alguna propiedad del Sol. Es ms o menos evidente, a a por razones de simetr que si la fuerza gravitacional depende de la masa m del planeta, a, entonces debe tener la misma dependencia de la masa del Sol M . Esta ultima observacin o sugiere escribir 4 2 = GM , K

11.2 Las leyes de Kepler

279

donde G es una nueva constante que ahora es independiente de las masas del Sol y del planeta. De esta manera se deduce que la fuerza gravitacional entre dos masas m y M , separadas por una distancia r, es F (r ) = G La ultima ecuacin precisamente es la o ley de gravitacin universal de Newton o siendo G la asi llamada constante gravitacional. Usando una balanza de torsin (ver gura 11.4), Cavendish en el o ao 1798 midi G en el laboratorio. n o El dispositivo experimental fue el siguiente: dos masas m se colocan en los extremos de una barra r gida y se cuelgan desde el centro de un alambre (hilo de torsin) que se sujeta rmemente del o cielo. El sistema puede girar libremente en el plano horizontal, pero tiene una orientacin para la cual est en equilio a brio. mM r. r2

Figura 11.4

Si las masas m se sacan del equilibrio, por ejemplo, dndoles una pequea velocidad de a n rotacin, entonces el sistema comenzar a oscilar tenindose un pndulo de torsin. A partir o a e e o del per odo de este movimiento oscilatorio se puede deducir la constante de restitucin de o torsin (o sea, el torque que ejerce el hilo de torsin cuando este se gira en una magnitud o o ). La orientacin del pndulo se dedecta con un rayo de luz que es reejado por un pequeo o e n espejo adosado a la varilla. El experimento mismo se realiza en dos etapas. Primero se colocan dos masas M en las posiciones A y se registra la orientacin de la varilla. Luego se rotan las masa M a las o posiciones B y se vuelve a registrar la orientacin de la varilla. A partir de la variacin de o o la orientacin de la varilla en estas dos mediciones se puede deducir la fuerza entre m y M . o El experimento es bastante dif ya que la fuerza de atraccin de las dos masas es muy cil o pequea. Por ejemplo, una masa m = 20 kg es atra por otra de masa M = 150 kg con n da una fuerza de slo 2, 3,5 105 g N (esto es, el peso de una masa de 23 milligramos) cuando o la separacin de sus centros es de 30 cm. o El valor actualmente aceptado para el valor de G es: G = (6, 673 0, 003) 1011 m3 . kg s2

Ejercicio: Conocidos el per odo y el radio de la rbita terrestre alrededor del Sol y usando o el valor de G recin dado, determine la masa del Sol. e

280

Gravitacin o

11.3.

Satlites e

En esta seccin analizaremos algunos aspectos del movimiento de satlites que orbitan o e gravitacionalmente entorno a un objeto masivo. Algo es un satlite si su masa m es mucho e menor que la masa M del objeto alrededor del cual orbita. En particular, los resultados de la presente seccin podrn aplicarse al movimiento de los planetas alrededor del Sol, de o a las lunas alrededor de los planetas y de los satlites articales alrededor de la Tierra. Las e trayectorias de todos estos objetos corresponden a elipses (y c rculos), o sea, a cnicas con o excentricidades 0 < 1. En la seccin 11.1 se relacionaron los parmetros r0 y de una elipse con el semieje mayor o a a y el semieje menor b y tambin con el perigeo r1 = rmin y el apogeo r2 = rmax de e ella. A continuacin estudiaremos la dependencia de estos parmetros de las constantes de o a movimiento de la rbita; espec o camente del momento angular orbital y de la energ a total E. La condicin m o M implica que la masa mayor esencialmente se matendr en reposo en a uno de los focos de la elipse, lugar en que ubicaremos el origen. La fuerza sobre la part cula m (el satlite) viene dada por e mM r. r2 Esta fuerza, que es conservativa, da origen a una energ potencial a F (r) = G U (r) = G mM . r

Si en cierto instante la posicin y veloo cidad de la part cula m es r y v, entonces la energ total del sistema ser a a 1 mM E = K + U = mv 2 G . 2 r

Figura 11.1

Sean P1 y P2 los puntos correspondientes al perigeo y apogeo, respectivamente (ver gura 11.5). Como la energ total se conserva sta debe ser igual a E tanto en el apogeo como a e en el perigeo, o sea, 1 mM 1 mM 2 2 = mv2 G . (11.8) E = mv1 G 2 r1 2 r2 Como la fuerza que acta sobre m es central se tiene que tambin el momento angular u e (respecto al origen) se conserva. Evaluando el momento angular en el apogeo y perigeo, se encuentra = mr1 v1 = mr2 v2 . (11.9)

11.3 Satlites e De esta ecuacin se obtiene o v1 = y v2 = Sustituyendo esto en (11.8) se encuentra E= y E= De estas dos ecuaciones se deduce
2 2mEr1 = 2

281

mr1 .

mr2

1 2 mM 2 G r 2m r1 1 1 2 mM . 2 G r 2m r2 2

(11.10)

(11.11)

2Gm2 M r1 2Gm2 M r2 .

y
2 2mEr2 = 2

Restndolas se obtiene a E(r2 r1 )(r2 + r1 ) = GmM (r2 r1 ) E = G mM mM = G , r1 + r2 2a

o sea, el semieje mayor de la rbita viene determinado slo por la energ E: o o a a = G mM . 2E (11.12)

Encontremos ahora una relacin entre b (el semieje menor de la rbita) y el momento o o angular y energ del satlite. Esta se obtiene restando las ecuaciones (11.11) de (11.10): a e
2 2 2mr1 2 2

2 2mr2

mM mM +G =0 r1 r2 1 1 r1 r2

1 1 2 r2 r1 2
2

= 2Gm2 M

2 2 r2 r1 r1 r2 2 2 r 2 = 2Gm M r r r1 2 1 2 2

r2 + r1 = 2Gm2 M r1 r2
2

2a = 2Gm2 M , b2

282 o sea, b2 =
2 a 2 = . Gm2 M 2Em

Gravitacin o

(11.13)

La importancia de las ecuaciones (11.12) y (11.13) radica en que son stas las que relacionan e las constantes de movimiento de la rbita con su forma geomtrica. o e Otra relacin importante se obtiene usando la segunda Ley de Kepler: como el momento o angular se conserva se tiene que la part cula m barre reas iguales en tiempos iguales. La a cantidad de rea que el satlite barre en un intervalo de tiempo dt lo podemos evaluar a e cuando ste se encuentra en el apogeo: e 1 dA = r2 v2 dt = dt . 2 2m De esta relacin se deduce que el rea total, A = ab, la part o a cula m lo barre en un tiempo T dado por ab = es decir, T2 = Usando (11.13) se obtiene, nalmente, T2 = 4 2 3 a , GM (11.14) 4m2 2 a2 b2
2

2m

T ,

resultado que no es otro que la tercera ley de Kepler. Pero observe que ahora conocemos la constante de proporcionalidad entre T 2 y a3 . Deseamos recalcar que las ecuaciones (11.13) y (11.14) son slo vlidas en el l o a mite m Cuando las dos masas son del mismo orden las ecuaciones deben corregirse). Ilustremos el uso de las ecuaciones anteriores resolviendo un problema: Problema: Si la Tierra conservara su energ total, pero perdiera la mitad de su cantidad de momento a angular respecto al Sol (por ejemplo, cambindole bruscamente la direccin de su velocidad), a o a) Qu tanto se acercar al Sol? e a b) Cul ser la distancia de alejamiento mxima (apogeo) de la Tierra en su nueva a a a o rbita? c) Cul ser el largo del ao en ese caso? a a n d) Cul tendr que ser el ngulo en que se var bruscamente la direccin de la Tierra? a a a a o M.

11.4 Potencial efectivo Suponga que inicialmente la rbita es circular. o

283

Solucin: o Sea R el radio de la rbita circular en torno al Sol (estamos suponiendo que el centro de o masas del sistema coincide con la posicin del Sol). La rbita circular es un caso particular o o de elipse para la cual a = b = R. Sean a y b los semiejes mayor y menor de la elipse nal (despus de haberle cambiado bruscamente su direccin). e o Como la energ de la Tierra no cambia, se tiene que a = a = R. De la ecuacin (11.13) se a o deduce que el semieje menor disminuir a la mitad, o sea, b = b/2 = R/2. Para el perigeo a y apogeo se encuentra r1 = a y r2 = a + Como el per odo slo depende de a o (tercera ley de Kepler) y a no cambia, se encuentra que el ao de la n Tierra en su nueva rbita seguir o a siendo de 365 d as. La magnitud del momento angular viene dada por la magnitud del momento lineal por el brazo. Como el momento lineal no cambia, para disminuir el momento angular a la mitad debemos disminuir el brazo a la mitad. De la gura 11.6 se desprende inmediatamente que el ngulo dea . be ser = 60 a2 b2 =R R2 R2 /4 2 3 =R 2

2+ 3 a2b2 = R . 2

Figura 11.6

11.4.

Potencial efectivo

Sea r el vector posicin de m. La fuerza que acta sobre m es o u F (r) = G y la energ potencial asociada a esta fuerza es a U (r) = G mM . r (11.16) mM r, r2 (11.15)

284

Gravitacin o

A continuacin mostraremos que cuando la energ potencial es central, es decir, slo deo a o pende de la magnitud del vector r y no de su direccin, entonces el problema de determinar o la trayectoria del satlite se puede reducir a un problema unidimensional. e Es evidente que si la energ potencial de una part a cula es central, entonces el campo de fuerzas generador del potencial, es radial. Por otra parte, una fuerza radial no ejerce torque respecto al origen. Si el torque respecto al origen es nulo, el momento angular de la part cula no puede alterarse, luego el momento angular (para una part cula movindose en e un potencial central) es una constante de movimiento (igual que, por ejemplo, la energ a total). Que el momento angular sea una constante de movimiento signica que ni la magnitud ni la direccin del momento angular cambian a medida que transcurre el tiempo. o Tambin sabemos que la velocidad de la part e cula es siempre perpendicular al momento angular. Como la direccin del momento angular no cambia se concluye que el movimiento o de la part cula necesariamente debe transcurrir en un plano. Debido a la importancia de este resultado lo volvemos a remarcar: En un potencial central la fuerza siempre es solamente radial. Un potencial central no ejerce un torque sobre la part cula respecto al origen, lo que a su vez implica que el momento angular de la part cula respecto a tal origen nunca var a (es una constante de movimiento). Una consecuencia de lo anterior es que el movimiento de una part cula en un potencial radial siempre trancurre en un plano. El papel que juega el momento angular como constante de movimiento es similar al papel de constante de movimiento que juega la energ total; ambos son magnitudes que, de alguna a manera, estn determinadas por las condiciones iniciales del problema. Supongamos, por a consiguiente, que conocemos la energ total E, la magnitud del momento angular del a sistema y el plano en el que transcurre el movimiento. Descompongamos la velocidad v(r ) de la part cula, cuando ella se encuentra en el lugar r, en una componente radial y una componente perpendicular a r (ver gura 11.7): v(r ) = vr r + v . (11.17)

Ac r y son vectores unitarios; el primero a en la direccin radial, el segundo perpendicuo lar a r, pero en el plano de la trayectoria.

Figura 11.7 El mdulo del momento angular de la part o cula es = r m v ,

11.4 Potencial efectivo

285

o sea, una vez conocido la velocidad v queda determinada por la distancia r de la part cula al centro: . (11.18) mr De lo anterior concluimos que el problema queda esencialmente resuelto si logramos establecer cmo var la distancia r en funcin del tiempo. Una vez conocido r(t), no slo se o a o o conoce la velocidad radial vr = r, sino que tambin, usando la ecuacin (11.18), la velocidad e o v . A su vez, conocida la velocidad en funcin del tiempo podemos, integrando, obtener la o posicin r (t) de la part o cula en todo instante. De la discusin anterior se desprende que debemos centrar la atencin en resolver el movio o miento radial de la part cula de masa m. Sabemos que la energ total es la suma de la energ cintica y la energ potencial: a a e a E =K +U . Por otra parte, la energ cintica viene dada por a e 1 1 2 2 K = m v 2 = m vr + v 2 2 , (11.20) (11.19) v (r) =

que, usando r = vr y la ecuacin (11.18), queda de la forma o 1 1 K = m v2 = m 2 2 r2 +


2

m2 r2

(11.21)

Sustituyendo este resultado en (11.19) se obtiene 1 E = m r2 + 2


2

2mr2

+ U (r)

(11.22)

Esta es una ecuacin que slo depende del radio r, de la velocidad radial r y constantes de o o movimiento. Esta ecuacin, por lo tanto, se puede reinterpretar como la correspondiente al o problema de una part cula de masa m y energ total E, que se mueve en una dimensin a o (dada por la coordenada r) en un potencial dado por
2

Ue (r) = U (r) +

2mr2

(11.23)

De esta manera hemos reducido el problema original a un problema unidimensional: el de una part cula de masa m afectada por una fuerza Fe (r) =
2 dUe (r) mM = G 2 + . dr r mr3

(11.24)

Ue (r) se llama el potencial efectivo, mientras que 2 /(2mr2 ) es el as llamado potencial centr fugo. Cuando el momento angular es cero, el potencial efectivo coincide con el potencial original.

286 La ecuacin (11.22) podemos escribirla como o 1 E = mr2 + Ue (r) . 2 Derivndola respecto al tiempo se obtiene a dUe r , dr

Gravitacin o

0 = mrr +

(11.25)

de donde, dividiendo por r y usando (11.24), encontramos una ecuacin de movimiento o para r(t): m r = G
2 mM + . r2 mr3

(11.26)

Es esta la ecuacin que resolveremos en la prxima seccin para encontrar las trayectorias o o o de los satlites. e

De acuerdo a lo discutido ms arriba, para establecer cmo se comporta el vector r, debemos a o resolver el problema de una part cula de masa m movindose en el potencial dado por la e ecuacin (11.23): o Ue (r) =
2 GM m + . r 2mr2

(11.27)

La energ total E = K +U y el momento angular son constantes de movimiento, es decir, a para un problema f sico en particular, tienen valores jos bien determinados.

a) Caso

= 0.

Consideremos brevemente el caso de momento angular = 0. Este valor para el momento angular implica que la part cula nunca tiene una velocidad tangencial, o sea, la part cula siempre se mueve a lo largo de la recta que une M con m. La gura 11.8 muestra el potencial Ue (r) que, en este caso, coincide con U (r). Si la energ de la part a cula es E < 0, entonces la mxima distancia a la que se puede a alejar es r0 .

11.4 Potencial efectivo

287

Figura 11.8 : Potencial efectivo para

= 0.

Si la part cula en un instante est en r0 , entonces su energ total E coincide con a a la energ potencial U (r0 ) y la energ cintica, por lo tanto, es nula la part a a e cula se encuentra en reposo. Sin embargo, sobre la part cula acta una fuerza ya que la u pendiente de la energ potencial en r0 no es nula. De hecho, la pendiente es positiva, a luego sobre m acta una fuerza negativa que la hace acelerar hacia el origen. A medida u que transcurre el tiempo la distancia entre m y M disminuir progresivamente. La a energ cintica de m (y por consiguiente el mdulo de la velocidad radial), como a e o tambin la fuerza atractiva irn aumentando. Finalmente, despus de transcurrido un e a e tiempo nito, la masa m llegar al origen, teniendo una energ cintica innita. (No a a e debemos preocuparnos demasiado por este innito que apareci. Obviamente ningn o u potencial f sico es de la forma GmM/r hasta r = 0. Todos los objetos f sicos tienen un tamao, y a ms tardar cuando la distancia entre los objetos es menor que la suma n a de sus radios, la interaccin cambia de carcter.) o a

b) Caso

= 0.

La gura 11.9 muestra el potencial U (r), el potencial centr fugo 2 /(2mr2 ) y el potencial efectivo Ue (r) para dos valores del momento angular ( 1 > 2 ). Es este ultimo, el potencial efectivo, l que es util para analizar el comportamiento de la variable e r = |r | en funcin del tiempo. o

288

Gravitacin o

Figura 11.9: Potencial centr fugo y efectivo para dos valores del momento angular distintos (no nulos).

Consideremos nuevamente una part cula con energ E < 0 (ver gura 11.10). La a part cula, en este caso, est restringida a moverse entre r1 y r2 . (Para el caso del a movimiento de la tierra alrededor del sol el vector r recorre una trayectoria el ptica y las magnitudes r1 y r2 corresponden a la distancia m nima y mxima de esa elipse). a La energ cintica de la part a e cula es K = E U (r), siendo Kr = E Ue (r) la energ cintica radial y Kt = Ue (r) U (r) la energ cintica tangencial. Cuando la a e a e part cula se encuentra en el perigeo r1 o en el apogeo r2 , ella no tiene energ cintica a e radial, pero s tiene una energ cintica tangencial. Note que la energ cintica a e a e tangencial es mayor en el perigeo que en el apogeo.

11.5 Trayectorias de los satlites e

289

Figura 11.10: Potencial efectivo para

= 0.

Si la energ es positiva (E > 0), entonces la part a cula no est ligada; la part a cula se acerca hasta una distancia m nima y luego se aleja indenidamente, para no volver. (Estas trayectorias corresponden a las soluciones hiperblicas del problema; los coo metas que provienen de fuera del sistema solar son un ejemplo de tales trayectorias.) Cuando la energ total es exactamente cero, la trayectoria tampoco es acotada y la a trayectoria, como veremos en la siguiente seccin, corresponde a una parbola. o a Cuando la energ total es igual al m a nimo del potencial efectivo, entonces la part cula no tiene energ cintica radial, pero s una energ cintica tangencial; tal trayectoria a e a e corresponde a la solucin circular del problema gravitatorio. o

11.5.

Trayectorias de los satlites e

En esta seccin analizaremos las posibles trayectorias de un satlite de masa m cuando o e es atra gravitacionalmente por un cuerpo masivo de masa M de acuerdo a la ley de do gravitacin universal. o Del analisis hecho en la seccin anterior (ver ecuacin 11.26) se desprende que debemos o o estudiar la ecuacin de movimiento o m = G r
2 mM + . 2 r mr3

(11.28)

290

Gravitacin o

El momento angular es una constante de movimiento y viene determinada por las condiciones iniciales del problema, es decir, para un problema en particular no cambia su valor a medida que transcurre el tiempo. Sabemos que al menos algunas de las soluciones de esta ecuacin deben ser elipses. En o coordenadas polares la ecuacin de una elipse es o 1 1 (1 cos ) . = r r0 Esto sugiere que, en lugar de analizar una ecuacin para r en funcin de t, ser mejor buscar o o a una ecuacin para w = w() 1/r en funcin de , pues tal ecuacin probablemente sea o o o simple. Realicemos esos cambios de variable. Tenemos r= Pero d d 1 1 dw() 1 dw d r= = 2 = 2 . dt dt w w dt w d dt d w2 == = , dt mr2 m r= dw . m d

luego

Derivando nuevamente respecto al tiempo se obtiene r= d dw dt d = d dw d d


2 d d2 w d d2 w = = 2 w2 . dt m d2 dt m d2

Sustituyendo esta expresin en (11.28) y usando que w = 1/r, se obtiene o


2

o sea,

w2

2 d2 w = GmM w2 + w3 , d2 m

d2 w 1 +w = . d2 r0 En la ecuacin anterior se introdujo r0 denido por o


2

(11.29)

r0

GM m2

Tal como sospechbamos, la ecuacin diferencial para w en funcin de , efectivamente es a o o muy simple. Si el lado derecho fuese nulo, la solucin ser o a w() = A cos() , donde A es una constante. (Elegir la funcin seno en lugar coseno slo redene el lugar o o desde el cual estamos midiendo los ngulos.) Es claro que al agregarle la constante 1/r0 a

11.6 El campo y potencial gravitacional

291

a la ultima expresin, se encuentra una solucin de (11.29). Por ultimo, si en lugar de A o o introducimos dado por = A r0 , entonces la expresin para r() = 1/w() queda de la forma o 1 1 (1 cos() ) . = r() r0 (11.30)

Hemos encontrado una expresin para la distancia r en funcin del ngulo polar. Como es o o a usual para ecuaciones diferenciales de segundo grado, la solucin general tiene dos constano tes (en este caso r0 y ) que deben determirse a partir de las condiciones iniciales. La ecuacin (11.30), de acuerdo al valor de psilon, corresponde a las distintas secciones o e cnicas: o =0 0< <1 =1 1< corresponde a un c rculo corresponde a una elipse corresponde a una parbola a corresponde a una hiprbola e

11.6.

El campo y potencial gravitacional

La ley de gravitacin universal de Newton nos da la fuerza de atraccin debido a la gravedad o o que acta entre dos masas, en efecto, u F (r ) = G mM r r2

es la fuerza que acta sobre la masa m debido a la masa M donde r es la separacin entre u o las masas y r es un vector unitario que apunta de M a m. Es conveniente introducir el concepto de campo gravitacional para describir el efecto que una masa M introduce en su entorno. Denimos el campo gravitacional de la masa M por g(r ) G M r. r2

Si en el lugar r de este campo gravitacional generado por la masa M colocamos una masa m, entonces sobre m actuar una fuerza a F = mg(r ) . Un campo es un ente que est denido en todos los puntos del espacio que nos interesa. a Como g es un vector, el campo gravitacional se dice que es un campo vectorial. No slo existen campos vectoriales, tambin existen campos escalares, tensoriales, etc.. Un o e ejemplo de un campo escalar es la temperatura. Supongamos que en todos los puntos de una pieza puedo medir la temperatura T , entonces T (r ) dene un campo escalar (el campo

292

Gravitacin o

de temperatura) de la pieza. Que un campo sea vectorial signica que en cada punto no slo tiene una magnitud sino que tambin una direccin. o e o El campo gravitacional satisface el principio de superposicin: si distintas masas m1 , m2 , o . . . ,mN ubicados en los lugares r1 , r2 , . . . ,rN generan campos gravitacionales g1 , g2 , . . . ,gN , entonces el campo gravitacional en el punto r ser a g(r ) =
j

gj (r rj ) = G
j

mj (r rj ) . |r rj |3

Cuando se tiene un campo de fuerzas conservativo, resulta muy conveniente introducir el concepto de energ potencial. El campo gravitacional es conservativo y en un cap a tulo anterior demostramos que la energ potencial de dos masas M y m, separadas por una a distancia r, viene dado por mM U (r) = G . r La constante aditiva arbitraria se ha elegido de manera que la energ potencial sea nula a cuando las dos masas estn separadas por una distancia innita (r ). La relacin a o anterior sugiere denir otro campo, un campo escalar, por (r) = G M . r

(r ) se llama el potencial gravitatorio de la masa M . Si se coloca una masa m en el potencial gravitatorio de una masa M , entonces la energ potencial del sistema es a U (r) = m(r) , donde r es la separacin entre las masas m y M . o El potencial gravitacional tambin satisface el principio de superposicin: si distintas masas e o m1 , m2 , . . . ,mN ubicados en los lugares r1 , r2 , . . . ,rN generan potenciales gravitacionales 1 , 2 , . . . ,N , entonces el potencial gravitacional total en el punto r ser a (r ) =
j

j (r rj ) = G
j

mj . |r rj |

11.6 El campo y potencial gravitacional Ilustremos los conceptos anteriores con dos problemas. Problema 1: Considere dos masas, de valor m y 2m, que se encuentran separados por una distancia 2a (ver gura adjunta). a) Calcule el valor de la energ potencial a gravitacional de una masa m ubicada en el punto medio entre las dos. Figura 11.11 b) Cul es la fuerza gravitacional ejercida sobre m ? a c) Cul es la velocidad de escape ve ? Depender sta de la direccin ? a ae o Solucin o La energ potencial de una masa m ubicada en el punto central es a U = m = G La fuerza que acta sobre m es u F = m g = m G 2m mm m x + G 2 x = G 2 x . a2 a a 2mm 3mm mm G = G . a a a

293

La velocidad de escape es la m nima velocidad que debe darse a la part cula para que se aleje llegando hasta innito (se escape del sistema). Para que la masa m escape su energ a debe ser no negativa. La m nima energ que debe tener es por lo tanto E = 0, o sea, la a energ cintica debe ser de igual magnitud que la energ potencial pero de signo contrario: a e a 1 3mm 2 K = m ve = G . 2 a Finalmente, para la velocidad de escape, que no depende de la direccin, se encuentra o ve = 6Gm . a

294 Problema 2:

Gravitacin o

Considere dos part culas, de masas m y M , que inicialmente estn en reposo y separadas a por una distancia muy grande (innita). Demuestre que en cualquier instante su velocidad relativa de acercamiento atribuible a la atraccin gravitacional es o v= donde D es la distancia que las separa. Solucin o Situmonos en el sistema del centro de masas. Si en cierto instante las velocidades de las e dos masas (observadas desde el centro de masas) son vm y vM , estas cumplen con mvm = M vM . Para la energ cintica de ambas part a e culas se obtiene 1 1 m 1 2 2 2 K = mvm + M vM = mvm 1 + 2 2 2 M . 2G(m + M ) , D

Inicialmente tanto la energ potencial (la separacin es grande) como la cintica son nua o e las. Como la energ total se conserva, esta deber ser nula en todo instante. Cuando las a a part culas estn separadas por una distancia D la energ potencial es a a U = G mM , D

y, por lo tanto, la energ cintica debe ser de la misma magnitud pero de signo contrario: a e 1 m 2 K = mvm 1 + 2 M Despejando vm se obtiene
2 vm =

=G

mM . D

2GM 2 . D(m + M )

La velocidad relativa es igual a v = vm + vM , luego v= 2GM 2 m + D(m + M ) M 2GM 2 = D(m + M ) 2G(m + M ) . D

11.7 El caso elctrico: la ley de Coulomb e

295

11.7.

El caso elctrico: la ley de Coulomb e

Resulta conveniente, en este punto, mostrar brevemente la gran analog existente entre la a fuerza de gravedad entre dos masas y la fuerza elctrica entre dos cargas q1 y q2 . e Caso gravitatorio La fuerza gravitatoria sobre la masa # 2, debido a la masa # 1 viene dada por la ley universal de gravitacin de Newton: o F = G m1 m2 r r2 Caso electrosttico a La fuerza electrosttica sobre la carga a # 2, debido a la carga # 1 viene dada por la ley de Coulomb: F = 1 q1 q2 r 4 0 r2

donde r es un vector unitario que apunta de la part cula # 1 a la # 2. El campo y potencial gravitacional asociado a una masa M es g(r ) = G y M . r La relacin entre g y es: o (r ) = G
r

donde r es un vector unitario que apunta de la part cula # 1 a la # 2. El campo y potencial electrosttico asoa ciado a una carga Q es E(r ) = y (r ) = 1 Q . 4 0 r 1 Q r 4 0 r2

M r r2

La relacin entre E y es: o


r

(r ) = (r0 )
r0

g(r ) dr .

(r ) = (r0 )
r0

E(r ) dr .

Al colocar una masa m en el campo gravitacional de una masa M , entonces la fuerza sobre m y su energ potencial viea nen dadas por F (r ) = mg(r ) y U (r) = m(r) .

Al colocar una carga q en el campo gravitacional de una carga Q, entonces la fuerza sobre q y su energ potencial viea nen dadas por F (r ) = q E(r ) y U (r) = q(r) .

Observaciones: a) Las ecuaciones electrostticas se han dado en el sistema de unidades MKS. La unidad a de la carga en este sistema de unidades es el Coulomb [C]. b) En la electrosttica el factor 1/(4 0 ) juega el rol que G juega en la gravitacin. a o

296

Gravitacin o

c) Una diferencia entre la gravitacin y la electrosttica es que la fuerza de gravitacin o a o entre dos masas iguales es atractiva mientras que la de dos cargas iguales es repulsiva. Esto se reeja en un cambio de signo en algunas de las ecuaciones. d) Otra diferencia importante e interesante es que en la electrosttica las cargas pueden a ser positivas y negativas. Este hecho da origen a numerosas situaciones y fenmenos o interesantes que no pueden darse en el caso gravitatorio (por ejemplo, el concepto de dipolo puntual).

11.8.

Campo gravitacional de una cscara esfrica a e

Ejercicio: Demuestre que d dx b a + bx = , 2 a + bx

donde a y b son constantes. Como corolario de este ejercicio se tiene que


x2

x1

1 2 dx = a + bx b a + bx

x2

=
x1

2 [ a + bx2 b

a + bx1 ] .

Considere una cscara esfrica de radio R a e de densidad de masa uniforme. Si la masa total de la cscara es M , entonces su a densidad supercial de masa ser a = M . 4R2

Deseamos encontrar el potencial gravitacional en el punto P debido a la cscaa ra esfrica. Para ello primero evaluamos e la contribucin de un anillo de ancho R d o que forma un ngulo theta con la l a nea que une P con el centro de la esfera. La masa del anillo es dm = (2R sin ) R d . Figura 11.12

11.8 Campo gravitacional de una cscara esfrica a e

297

Todos los puntos del anillo estn a la misma distancia D = (R sin )2 + (r R cos )2 del a punto P , luego la contribucin del anillo al potencial gravitatorio es o d = G (R sin )2 + (r R cos )2 sin d = 2GR2 . 2 + r 2 2rR cos R

dm = 2GR2 D

sin d

Sumando la contribucin de todos los anillos obtenemos el potencial total: o =


0

d = 2GR2
0

R2

sin d . + r2 2rR cos

La integral la podemos evaluar con un cambio de variable. Denotemos cos por s. Entonces s = cos ds = sin , d o sea sin d = ds . Observando que si = 0 entonces s = 1 y si = , entonces s = 1, podemos reescribir la integral: 1 sin d ds I= = . 2 + r 2 2rR cos 2 + r 2 2rRs R R 0 1 Usando ahora el resultado del ejercicio propuesto al iniciar la presente seccin (con a = o R2 + r2 , b = 2rR, x1 = 1 y x2 = 1), se obtiene
1

I =
1

R2

ds + r2 2rRs R2 + r2 + (2Rr) 1

= =

2 R2 + r2 + (2rR)(1) 2rR 1 [(R + r) |R r|] rR

Distinguiendo con cuidado los dos casos r > R y R < r, se obtiene que 2 si r < R R I= . 2 si r > R r De esta manera, para el potencial de una cscara esfrica de radio R y masa M se encuentra a e la expresin o G M para R > r R . (r) = G M para r > R r Algunas observaciones importantes respecto a este resultado:

298

Gravitacin o

a) Para r > R, o sea, cuando el punto P se encuentra fuera de la cscara esfrica, el a e potencial gravitacional que sta ejerce es idntica a la que se hubiese obtenido si toda e e la masa de la cscara se ubica en el origen. a b) Si el potencial afuera es el mismo al de una masa puntual, entonces tambin el campo e gravitacional lo ser, o sea, la fuerza que la cscara esfrica ejercer sobre una masa a a e a m (si sta se encuentra en el exterior) ser e a F = G mM r. r2

c) En el interior de la cscara esfrica el potencial gravitacional es constante (no dea e pende de la posicin). Esto signica que el campo gravitacional ah es nulo, o sea, si o colocamos una masa m en el interior, sobre ella la cscara esfrica no ejercer ninguna a e a fuerza gravitacional. Resumen: Para una cscara esfrica de radio R y masa M se tiene a e GM r 2
r (para r > R) (para r < R) (para r > R)

g(r ) =

(r ) =

0 GM r

GM (para r R) R La gura 11.13 muestra un grco de la intensidad del campo gravitacional y el potencial a para una cscara esfrica. a e

Figura 11.13

11.8 Campo gravitacional de una cscara esfrica a e

299

A continuacin presentaremos una demostracin alternativa de que una cscara esfrica no o o a e ejerce ningn campo gravitacional al interior de ella. u Algunos preliminares: Al intersectar un a ngulo (innitesimal) d con un c rculo de radio r, el largo del arco es ds = r d (ver gura 11.14). Si tal arco se inclina en un a ngulo , el largo del arco (l nea segmentada) ahora es ds = rd/ cos . Estos conceptos se pueden generalizar para ngulos slidos en tres dimensiones. Paa o ra ellos tomemos una esfera de radio r y consideremos un rea A (que puede tea ner forma irregular) sobre la supercie. Si unimos todos los puntos del per metro del a rea con el centro obtenemos un ngulo a slido. o En forma anloga a lo que ocurre para a a ngulos en un plano, denimos el ngua lo slido por A/r2 . El ngulo slido o a o completo (en el espacio tridimensional) es, por lo tanto, 4. A la inversa, si tenemos un ngulo slido innitesimal d e intera o ceptamos este con una esfera de radio r, el a rea denido por la interseccin ser dA = o a r2 d. Si el rea la inclinamos en un ngua a 2 d/ cos . lo , su rea ser dA = r a a

Figura 11.14

Figura 11.15

Volviendo a la cscara esfrica de radio R a e y masa M , evaluemos el campo gravitacional en un punto P que se encuentra en su interior (ver gura 11.16). Consideremos un ngulo slido d desde a o el punto P y hacia los dos lados. El ngua lo slido intersecta a la cscara esfrica en o a e los lugares A y B (ver gura 11.16). La inclinacinde las reas en A y B es en el miso a mo ngulo ya que ABO es un tringulo a a issceles. El rea que el ngulo slido ino a a o 2 tersecta en A es r1 d/ cos mientras que 2 en B es r2 d/ cos . Sea la densidad supercial de masa de la cscara esfrica. a e

Figura 11.16

300 El campo gravitacional que el rea A genera en P es a gA =


2 r1 d cos

Gravitacin o

1 d 2 n = cos n . r1

Ac n es un vector unitario que apunta de P a A. En forma anloga se obtiene que el campo a a gravitacional que el rea B genera en P es a gB =
2 r2 d cos

1 d n 2 ( ) = cos n. r2

Se observa que el campo gravitacional de las reas A y B se cancela exactamente. Lo mismo a ocurre con cualquier otro ngulo slido. De la discusin anterior se concluye que al interior, a o o el campo gravitacional generado por una cscara es necesariamente nulo. a

11.9.

Campo gravitacional de una esfrica slida e o

Evaluemos el campo gravitacional en algn punto al exterior de una esfera slida de masa M u o y radio R. Para ello es conveniente pensar que la esfera est compuesta de muchas cscaras a a esfricas (como una cebolla). Ya sabemos que el campo gravitacional de cada cscara es el e a mismo que el que se obtiene al concentrar toda la masa de la cscara en el centro. La suma a de todas las cscaras (que es la esfera slida), por lo tanto, generar un campo gravitacional a o a igual al de una masa (equivalente a la suma de las masas de todas las cscaras) concentrada a en el centro. O sea, si el punto P est al exterior de la esfera, a una distancia r del centro, a el campo gravitacional ser a M g(r) = G 2 r . r Un razonamiento anlogo permite encontrar el potencial gravitacional (para un punto en a el exterior): (r) = GM/r. Si el punto P se encuentra al interior de la esfera se puede proceder de una manera parecida. Dividamos nuevamente la esfera en numerosas cscaras esfricas. Todas a e las cscaras esfricas con un radio mayor a e que r no contribuyen al campo gravitacional en P (pues P est al interior de ellas). a Las capas con radio menor que r las podemos concentrar en el centro. El campo gravitacional por lo tanto ser a m r, r2 donde m es la masa de las cscaras interiores, esto es, a g(r) = G m=M r3 . R3 Figura 11.17

11.9 Campo gravitacional de una esfrica slida e o

301

De manera anloga se procede para encontrar el potencial gravitatorio al interior de una a esfera slida. o Resumen: Para una esfera slida homognea, de radio R y masa M , se tiene o e GM r (para r > R) r2 g(r ) = GM r r (para r < R) R3 GM (para r > R) (r ) =
r

GM 2R

r2 3 R2

(para r R)

La gura 11.18 muestra un grco de la intensidad del campo gravitacional y el potencial a para una cscara esfrica. a e

Figura 11.18

11.9.1.

Densidad media de la Tierra

Problema: Determinar la densidad media de la Tierra suponiendo concocidos el valor de la constante de la gravitacin G, la aceleracin de la gravedad g y el radio terrestre R. o o Solucin: Sobre una masa m colocada en la supercie de la Tierra acta una fuerza igual o u a mg. Por otra parte, el campo gravitacional de la Tierra para un punto sobre su supercie se puede evaluar suponiendo que toda la masa de la Tierra est en el centro, es decir, a |g(R)| = GM , R2

donde M es la masa total de la tierra. Conociendo el campo garvitacional podemos evaluar la fuerza que acta sobre una masa m; esta es u m|g(R)| = GM m . R2

302 Igualando las dos expresiones para la fuerza se obtiene para g la expresin o g= GM . R2

Gravitacin o

Esta ecuacin relaciona la aceleracin de la gravedad g=9,81 m/s2 con la constante gravio o tacional G y la masa y radio terrestre M y R, respectivamente. La densidad media de la Tierra viene dada por = = M 3M 3g = = V 4R3 4GR 3 981, 0 4 6, 67 108 6, 4 108 5, 5
g . cm3

(Para referencia: la densidad del hierro, que es el principal elemento del ncleo terrestre, es u de 7,86 g/cm3 .)

11.10.
1.

Problemas

Jpiter tiene doce lunas conocidas, cuatro de las cuales fueron descubiertas por Gau lileo. Estos son los radios y per odos de las primeras cuatro:

R (Km) 180.000 422.000 671.000 1.072.000

T (d as) 0,498 1,769 3,551 7,155

a) Obedecen estos cuatro satlites a la tercera ley de Kepler? e b) Con estos datos y el valor de G, encuentre la masa de Jpiter. u c) El dimetro de Jpiter es de 142.900 Km. Cul es su densidad media? a u a 2. Las rbitas de dos satlites terrestres A y B son el o e pticas, siendo R y 4R, respectivamente, sus ejes mayores. a) Cul es el cuociente entre sus energ mecnicas totales? a as a b) Cul es el cuociente entre sus per a odos? c) Qu puede decirse del cuociente entre sus momentos angulares? e Un satlite articial recorre una trayectoria circular 320 Km por encima de la supere cie terrestre. a) Cul ser su velocidad? a a b) Con qu frecuencia girar alrededor de la Tierra? e a

3.

11.10 Problemas 4. Dos satlites articiales de igual mae sa orbitan alrededor de un planeta. S1 se mueve en una rbita circuo lar de radio 108 [m] y per odo de 5 d as, S2 se mueve en una rbita o 8 [m] y el ptica de radios rmin = 10 rmax = 2 108 [m]. a) A partir de los datos para la rbio ta circular, calcule la masa del planeta. b) Encuentre el per odo de S2 .

303

Figura 11.19

c) Cul satlite es ms veloz al pasar por B? Cul tiene mayor energ a e a a a? d) Calcule el cuociente entre las velocidades que S2 tiene en B y en A. e) Qu maniobra deber ordenar el puesto de mando para poner S2 en la rbita de e a o S1 ? 5. Un satlite es geoestacionario si para un observador jo sobre la Tierra este no se e mueve. La rbita de tal satlite necesariamente deber coincidir con el plano del o e a Ecuador terrestre. Determine el radio de la rbita del satlite. o e Respuesta: R 6. 42,000 km.

A qu distancia de la Tierra debe colocarse un cuerpo en la l e nea dirigida hacia el Sol de tal manera que la atraccin gravitacional solar contrarreste a la atraccin de o o 7 Km de distancia y su masa es de M = 3, 24 105 la Tierra? El Sol est a 15 10 a S MT (MT es la masa de la Tierra = 5, 97 1024 Kg). Analice el problema incluyendo el efecto introducido por la rotacin de la Tierra alrededor del Sol (o sea, tanto la o Tierra como el cuerpo giran alrededor del Sol una vez al ao). n Para encontrar la solucin (aproximada) use el hecho de que MT /MS o Respuesta: MT 3MS 1.

es la solucin que se tiene si el cuerpo se encuentra entre la Tierra y el Sol. Hay dos o soluciones adicionales si se permite que r sea mayor que R. Convnzase de sto y e e encuntrelas. e

7.

Dos satlites A y B giran alrededor de la Tierra en la misma rbita circular (de radio e o R), pero uno al lado opuesto de la Tierra respecto al otro. Se desea interceptar el satlite B con un proyectil lanzado desde A. Estudie el problema e indique velocidad e y direccin en que debe lanzarse el proyectil para lograr su objetivo. D al menos 3 o e soluciones distintas.

304 8.

Gravitacin o El perigeo, punto ms prximo de la rbita de un satlite, se halla a 320 Km de la a o o e supercie terrestre, y el apogeo, punto ms alejado, a 2400 Km. a a) Cul es el semieje mayor de la rbita del satlite? a o e b) Cul es la excentricidad de su rbita? a o c) Si el satlite tiene una masa de 15 Kg, cul es su energ total? e a a d) Cul es su velocidad en el apogeo? a e) Cul es su velocidad en el perigeo? a f) Cul es su momento angular? a

9.

Una pequea masa m cae hacia el Sol partiendo del reposo desde una distancia igual n al radio de la rbita terrestre. Determine el tiempo de ca usando slo las leyes de o da o Kepler.

10.

Se dice que la rbita de un satlite es heliosincrnica si pasa diariamente por los o e o mismos lugares a la misma hora. Suponiendo que la altura m nima de un satlite es e de 200 km por sobre la supercie terrestre, encuentre los radios de todas las orbitas heliosincrnicas circulares. o

11.

Considere la parbola y = ax2 . a a) Encuentre su foco. b) Traslade el sistema de coordenadas de manera que el nuevo origen coincida con el foco calculado en la parte a). c) Introduzca coordenadas polares (r, ), midiendo desde el m nimo de la parbola a y demuestre que la ecuacin de la parbola queda de la forma o a 1 1 = (1 r s con = 1 y s algn valor constante. u

cos )

12.

Un satlite gira sobre el Ecuador, en el mismo sentido que la Tierra, en una trayectoria e el ptica con rmin = 200 Km y rmax = 500 Km. Se desea poner este satlite en e o rbita geoestacionaria. El cohete del satlite es capaz de acelerarlo con una aceleracin e o a = 50 m/s2 . En qu instantes y durante cunto tiempo se deben prender los motores e a para lograr el propsito? o

13.

Calcule el per odo de rotacin de la Luna entorno a la Tierra sabiendo que el radio o de su rbita es 60,3 veces el radio de la Tierra. o

11.10 Problemas 14. Un cometa de masa m se dirige (cae) radialmente hacia el sol. Observaciones astronmicas permiten estableo cer que la energ mecnica a a total del cometa es nula, es decir, E = 0. El cometa se estrella contra Venus, cuya masa es m. Supongamos adems a que la trayectoria de Venus es circular, de radio R0 . A consecuencia del choque, el cometa y Venus forman un solo astro que llamaremos Vennus. a) b) c) d) e) f)

305

Figura 11.20

Calcule la rapidez v0 y el per odo de Venus antes de la colisin. o Calcule la energ mecnica de Venus en su rbita antes de chocar con el cometa. a a o Calcule la velocidad radial y el momento angular de Vennus inmediatamente despus de la colisin. e o Determine la energ mecnica de Vennus y exprsela en trminos de m, y v0 . a a e e Demuestre que la rbita de Vennus es el o ptica y determine el semieje mayor de la rbita. o Determine si el ao para los venusianos se ha acortado o alargado a causa del n choque con el cometa.

Respuesta parte f): La razn del per o odo de Vennus y Venus es T 1+ = T 1 + 4


3/2

15.

Un proyectil de masa m se lanza tangencialmente a la supercie de la Tierra. Suponiendo que no hay resistencia del aire (como en la Luna), calcular la rapidez v0 con que el proyectil debeser lanzado para que orbite en forma circular y razante a la Tierra. Compare esta velocidad con la velocidad de escape. Calcule el momento angular y la energ del proyectil en esta situacin. a o El proyectil del problema anterior ahora es lanzado horizontalmente a la supercie de la Tierra con una rapidez v0 , con 1, < 2. Calcule la distancia radial del perigeo y apogeo y la excentricidad de la bita. o

16.

306 17. Dos part culas de igual masa se unen mediante una cuerda ideal de longitud h. El par es atra gravitacionalmente por un planeta de do masa M . La distancia entre el planeta y la part cula mas cercana es R, con h R. a) Despreciando la fuerza de atraccin eno tre las dos part culas, calcule la tensin o de la cuerda si ellas caen sobre el planeta con la cuerda estirada y dispuesta radialmente. Ahora tome en cuenta la atraccin grao vitacional entre las dos masas. Demuestre que para que la cuerda no est tene sa la masa de cada part cula debe ser m = M (h/R)3 .

Gravitacin o

b)

Figura 11.21

18.

Marte tiene un dimetro medio de 6.800 Km, la masa de Marte es 0.107 MT . a a) Cul es la densidad media de Marte comparada con la de la Tierra? a b) Cul es el valor de g en Marte? a c) Cul es la velocidad de escape en Marte? a La estrella enana Sirio B tiene un radio que es 1/50 del radio solar, a pesar de tener aproximadamente la misma masa que el sol. a) Cul es el valor de g en la supercie de Sirio B? b) Cul ser la densidad media a a a de Sirio B? Una esfera uniforme de densidad o y radio R1 tiene una cavidad esfrica de radio R2 . e Encuentre el potencial en el punto r (ver gura 11.22). Cul ser el peso suyo si el radio de la Tierra a a doblara su valor, a) mantenindose la masa de la Tierra igual e a la actual? b) mantenindose la densidad promedio de la e Tierra igual a la actual?

19.

20.

21.

Figura 11.22

22.

Dos esferas de plomo, de 1 m de radio, estn en contacto. a a) Cul es la magnitud de la fuerza de atraccin mutua? a o b) Qu velocidad tendr en el instante de contacto si partieran de posiciones muy e an separadas en el espacio y cayeran una contra la otra? (plomo = 11,3 g/cm3 ). Un satlite de masa m = 5 kg es lanzado a una rbita circular cuyo per e o odo es de 120 minutos. Ignore la rotacin de la Tierra y tambin cualquier efecto del roce viscoso o e

23.

11.10 Problemas del satlite con el aire. e

307

a) b)

Calcule la razn ewntre el radio de la rbita del satlite y el radio terrestre. o o e Calcule la energ m a nima requerida para poner al satlite en dicha bita. De e o esta energ qu fraccin se uso para subirlo y que fraccin para darle la a e o o velocidad requerida?

24.

Una nave csmica se dirige hacia la Luo na a lo largo de una trayectoria parablica que casi toca a la supercie luo nar. En el momento de mxima aproa ximacin un motor de frenado, en un o breve lapso, modica la velocidad de la nave de manera que esta entre en una o rbita circular alrededor de la Luna. Encuentre la velocidad de la nave justo antes y despus del frenado. La masa y e el radio lunar son M = 7, 34 1022 kg y R = 1, 74 106 m, respectivamente. Figura 11.23

25.

Se taladra un tnel liso y recto a travs de un planeta esfrico cuya densidad de u e e masa o es constante. El tnel pasa por el centro del planeta y es perpendicular al u eje de rotacin del mismo. El planeta rota con una velocidad angular determinada de o modo que los objetos dentro del tnel no tienen aceleracin relativa al tnel. Hallar u o u la relacin entre o y para que esto sea cierto. o

26.

Demuestre que en un conducto excavado en la Tierra, siguiendo una cuerda cualquiera, (no siguiendo necesariamente un dimetro), el movimiento de un objeto ser armnico a a o simple. (Desprecie efectos de roce y de la rotacin de la Tierra). Encuentre el per o odo del movimiento.

27.

Discuta el origen de las mareas. Por qu se presentan dos mareas y no una sola? e

308 28. Encuentre la rbita de una part o cula que se mueve en un campo central que genera una fuerza F (r) = Gm1 m2 2 + 3 . r2 r

Gravitacin o

Si es pequeo con respecto al mon mento angular muestre que la rbio ta corresponde a una elipse cuya orientacin precesa lentamente. Eno cuentre el ngulo en que cama bia la orientacin del semieje mao yor en un per odo. (Indicacin: Reo pita lo hecho en la seccin 11.5 pero o con la nueva expresin para F ). o 29.

Figura 11.24

a) Demuestre que para escapar de la atmsfera de un planeta una condicin que debe o o cumplir una molcula es que tenga una velocidad tal que v > 2GM/r, siendo M la e masa del planeta y r la distancia de la molcula al centro del planeta. e b) Determine la velocidad de escape para una part cula atmosfrica a 1000 Km sobre e la supercie de la Tierra. c) Haga lo mismo para la Luna. Considere una esfera de radio a y hecha de materia homognea y que al interior tiene e una cavidad esfrica concntrica de radio b. e e a) b) c) d) Haga un grco de la fuerza de gravitacin F ejercida por la esfera sobre una a o part cula de masa m, localizada a una distancia r del centro de la esfera. Haga un grco de la energ potencial gravitacional U (r) de la masa m en a a funcin de r. o Demuestre que en el l mite b a vuelve a obtener el resultado correspondiente a una cscara esfrica. a e Demuestre que en el l mite b 0 vuelve a obtener el resultado correspondiente a una esfera slida. o

30.

31.

Considere un recipiente (cascarn) o semiesfrico. Demuestre que en e cualquier punto del plano que pasa por el borde del recipiente (a modo de tapa, regin punteada), el campo o gravitatorio es perpendicular a dicho plano. Figura 11.25

11.10 Problemas 32.

309

Considere una semiesfera slida de masa M y radio R, ja. Cul es el trabajo que o a se debe realizar para llevar una pequea masa m desde el centro de la base hasta el n innito? Una part cula de masa m se encuentra situada sobre el eje de simetr de un anillo a de masa M y radio R, a la distancia d del plano del anillo. a) b) c) Encontrar la fuerza con que el anillo atrae a la masa m, en trminos de G, M , e m, R y d. Discuta en particular los casos d = 0 y d , y convnzase de que se obtiene e lo que uno espera intuitivamente. Si la masa m se mueve a lo largo del eje y parte del innito (desde una distancia muy grande) con velocidad nula, con que velocidad pasar por el centro del a anillo? (Suponga que el anillo no se mueve, es decir, hay una fuerza externa que lo mantiene jo).

33.

34.

Considre un satlite de masa m que gira en una rbita circular de radio R an torno e o a un planeta de masa M m. a) b) Determine la velocidad del satlite. e Suponga que el satlite es interceptado por un proyectil, tambin de masa m, que e e se desplaza radialmente hacia el planeta. Sea v1 = v1 r la velocidad del proyectil justo antes del impacto. Suponga que el choque es completamente inelstico ( a es decir, el proyectil y el satlite forman un solo cuerpoo despus de la colisin). e e o Encuentre la velocidad que tiene el proyectilsatlite justo despus del choque. e e Determine la m nima rapidez v1 que debe tener el proyectil justo antes del choque para que el cuerpo proyectilsatlite logre escapar del campo gravitacional del e planeta.

c)

35.

Considere un casquete esfrico jo, e muy delgado, de densidad uniforme, radio R y masa M , que posee dos oricions que lo perforan en posiciones diametralmente opuestas. Una masa puntual m se encuentra inicialmente en reposo a una distancia 3R de su centro sobre la l nea que une las perforaciones. Encuentre el tiempo que tarda la masa m en cruzar el casquete de un extremo al otro.

Figura 11.26

310 36. Dos cscaras esfricas del mismo raa e dio R y masa M se encuentran separados (sus centros) por una distancia r. a) Haga un grco esquemtico a e de la energ potencial gravia tatoria a lo largo de la recta A, B. Si una part cula de masa m se suelta en reposo desde el centro de la esfera #1, con qu velocidad llega a la supere cie de la esfera #2? Figura 11.27

Gravitacin o

b)

(c) Cul es la mxima separacin r a la que pueden estar las esferas para que la a a o masa m llegue a la supercie de la cscara esfrica #2? a e

11.11.

Solucin a algunos de los problemas o

Solucin al problema 4 o (a) La fuerza gravitacional debe coincidir con la fuerza centr peta. Sea M la masa del planeta y m la masa del satlite, entonces e G Mm mv 2 r= r, 2 R R

donde v = 2R/T es la velocidad del satlite en su rbita circular. De las ecuaciones e o anteriores se deduce que M= 4 2 R3 4 2 1024 = kg GT 2 6, 67 1011 (5 24 3600)2 3, 2 1024 kg .

(b) De acuerdo a la tercera ley de Kepler T1 = T2 a1 a2


3/2

donde a1 y a2 son los semiejes mayores de las trayectorias el pticas de los satlites. e De acuerdo al enunciado a1 = 108 [m] y a2 = (rmin + rmax )/2 = 1, 5 108 [m], luego a1 /a2 = 2/3. Para T2 se obtiene T2 = T1 a2 a1
3/2

= 5 (1, 5)1 , 5 d as

1, 19 d . as

11.11 Solucin a algunos de los problemas o

311

(c) La energ total y el semieje mayor de un satlite estn relacionados por la ecuacin a e a o E = G mM . 2a

La energ es inversamente proporcional al semieje mayor, pero observe que debido a al signo menos, la rbita que tiene el mayor a tiene tambin mayor energ (es menos o e a negativa). Conclu mos que el satlite S2 es el que tiene mayor energ En el punto B e a. ambos satlites tienen la misma energ potencial, luego la energ cintica de S2 es e a a e mayor que la de S1 . El satlite S2 es mas rpido que S1 cuando pasan por B. e a (d) Se tiene que (todo evaluado en el punto B) E2 a1 2 = = . E1 a2 3 Por otra parte, para la rbita circular U = 2K1 , o sea, o E1 = U + K 1 = U Se tiene U Mm U = = G . 2 2 2R2

E2 U + K2 2 = = . E1 U/2 3

De aqu se deduce que K2 = 2U/3. Luego K2 4 v2 2U/3 = = 2 . = 2 K1 U/2 3 v1 (e) Para pasar de la bita el o ptica a la circular, el satlite S2 debe, cuando se encuentra e pasando por el punto B, prender los motores y frenar hasta bajar la velocidad de v2 a v1 .

312 Solucin al problema 9 o

Gravitacin o

La trayectoria de la masa m que cae hacia el Sol es el l mite de una elipse en que el semieje menor b tiende a cero. En ese caso la trayectoria es una l nea recta estando el Sol en uno de los extremos. El semieje mayor de tal elipse es a = R/2, donde R = aT es el radio (igual al semieje mayor) de la trayectoria circular de la Tierra. De acuerdo a la tercera ley de Kepler Tm = TT a aT
3/2

= 23/2 = 0, 3536 .

El tiempo t0 que demora la masa m en llegar al sol es la mitad del per odo de su rbita, o o sea, Tm TT t0 = = 0, 3536 64, 5 d . as 2 2 Solucin al problema 14 o (a) Sea M la masa del Sol, entonces, igualando la fuerza gravitacional con la fuerza centr peta GM m mv 2 r = 0r 2 R0 R0 se obtiene
2 v0 =

GM . R0

(b) La energ menica de Venus (antes de la colisin) es a a o Ei = GM m GM m 2 1 2 + mv0 = 2 . R0 2 2R0

(c) Como el cometa (cuando est lejos) se mueve radialmente hacia el sol, no tiene moa mento angular (respecto al origen en el Sol). Luego el momento angular de Vennus es el mismo que el de Venus L = R0 mv0 . Esto nos permite encontrar la componente de la velocidad de Vennus justo despus e de la colisin. El momento angular justo despues de la colisin es o o L = R0 (m + m)v . Como el momento angular se conserva se deduce que v = v0 . 1+

La conservacin del momento lineal en la direccin radial hay que darse cuenta que la o o interaccin entre Venus y el cometa son fuerzas internas y, por lo tanto, para calcular o

11.11 Solucin a algunos de los problemas o

313

la velocidad del cometa podemos ignorar el efecto introducido por la interaccin entre o el cometa y Venus. El cometa tiene energ nula, luego, a K = U = + GM m 1 2 = mvC . R0 2

(vC es la velocidad del cometa justo antes de la colisin ignorando el efecto introducido o por Venus). Se deduce que 2GM 2 vC = . R0 Aplicamos ahora la conservacin del momento lineal a lo largo de la direccin radial o o mvC = (m + m)vr , donde vr es la velocidad de Vennus justo despus de la colisin. Se deduce que e o vr = vC . 1+

(d) La energ mecnica de Vennus (la evaluamos justo despus del choque) es a a e Ef = U +K = = (1 + ) = GM m 2R0 GM m(1 + ) 1 2 2 + m(1 + )(v + vr ) R0 2 1+ .
2

GM m 1 + (1 + ) 2 R0 2 1 + 4 1+ = Ei

1 (1 + )2

GM m R0

1 + 4 1+

(e) la rbita de Vennus obviamente ya no es un c o rculo. Como la energ es negativa debe, a por lo tanto, ser el ptica. Se tiene que af af Ei = = . Ef ai R0 Aqu ai y af son los semiejes mayores de las rbitas de Venus y Vennus, respectiva o mente. Se deduce que 1+ Ei af = R0 = R0 . Ef 1 + 4 (f) Usando la tercera ley de Kepler podemos calcular la razn del per o odo de Vennus y Venus: af 3/2 T 1 + 3/2 = = . T r0 1 + 4

314 Solucin al problema 17 o

Gravitacin o

(a) La fuerza neta que acta sobre la part u cula ms cercana, llammosla #1, es a e F1 = GM m r + Tr . R2

La fuerza neta que acta sobre la otra part u cula (#2) es F2 = GM m r Tr . (R + h)2

Como el hilo que las une es inextensible, ambas part culas aceleran con la misma aceleracin a = a. Se tiene o r F1 = ma = r y F2 = ma = r GM m r Tr . (R + h)2 GM m r + Tr . R2

Despejando la tensin del hilo T se encuentra que o T = Con h GM m 2 1 1 2 R (R + h)2 .

R se obtiene la expresin o T = GM mh . R3

(b) Para que la cuerda no quede tensa la fuerza gravitacional entre las part culas debe coincidir con T , o sea, Gmm T = . h2 Despejando m de las dos ultimas ecuaciones se encuentra m=M h R
3

Solucin al problema 32 o El trabajo que debemos realizar es independiente del camino que elijamos para llevar la masa m del centro de la basa de la semiesfera hasta el innito. Elijamos un camino

11.11 Solucin a algunos de los problemas o recto paralelo a la base. Sea ste el e eje x y denotemos por y al eje que coincide con el eje de simetr de la a semiesfera (ver gura). Sea F (x) la fuerza gravitacional que acta sobre la masa m cuando esu ta se encuentra sobre el eje x en la posicin x. Esta fuerza tendr dos o a componentes F (x) = Fx (x) + Fy (x) . x y Figura 11.28

315

La componente y de la fuerza no nos interesa pues al evaluar el trabajo con la expresin o

W =
0

F (x) (dx x)

se observa que tal componente es perpendicular al desplazamiento y, por lo tanto, no gura al evaluar el producto punto. Para evaluar Fx (x) coloquemos una segunda semiesfera idntica en la parte inferior (ver gura l e nea punteada). Por simetr la fuerza que ejerce a esta segunda esfera es F (x) = Fx (x) Fy (x) , x y o sea, la componente x es la misma, pero la componente y cambia de signo. El efecto de las dos semiesferas es F + F = 2Fx (x) . x Pero, por otra parte, las dos semiesferas forman una esfera completa de masa 2M , y para ese caso sabemos que la fuerza es (ver seccin 11.9) o G(2M )m x (para x > R) 2 mg(x ) =
x

G(2M )m x R3

(para x < R)

Igualando las dos expresiones se encuentra que GM m x2 Fx (x) = GM m R3

(para x > R)

(para x < R)

Esto es exactamente el mismo resultado que se tendr para una esfera slida completa de a o masa M y radio R. Podemos entonces usar los resultados que se encontraron en la seccin o 11.9 para la esfera slida. El trabajo lo podemos evaluar usando el potencial gravitatorio o . El potencial gravitatorio (con el cero del potencial en el innito) al centro de una esfera slida es o 3GM (0) = . 2R

316 Luego el trabajo para llevar la masa m del origen al innito es W =m 3GM . 2R

Gravitacin o

Solucin al problema 35 o Sobre la part cula m no se ejerce ninguna fuerza cuando se encuentra al interior del casquete. Si v0 es la velocidad con que llega a la supercie, entonces el tiempo de traves es a t0 = 2R/v0 . Para calcular la velocidad v0 hacemos uso del hecho de que la energ debe a conservarse. La energ de m en el punto de partida es a Ei = mientras que cuando llega a la supercie es Ef = GM m 1 2 + mv0 . R 2 GM m , 3R

Igualando las dos expresiones se deduce que


2 v0 =

4GM . 3R

Al interior del casquete no hay fuerzas sobre la masa m y, por lo tanto, su velocidad se mantiene contante. Para el tiempo de traves se obtiene a t0 = 3R3 . GM

Cap tulo 12

Fluidos
12.1. Conceptos Preliminares
pasador

placa
Figura 12.1

La densidad de una sustancia es la razn entre su masa y volumen: = m/V . La densidad o del agua, a 4 C, es 1.00 g/cm3 (es el valor mximo de la densidad del agua). a Los uidos se dividen en dos subclases: los l quidos y los gases. Los l quidos se caracterizan por ocupar un volumen denido independiente del volumen del recipiente que lo contiene. Un gas, por otra parte, tiende a expandirse y a ocupar todo el volumen del recipiente que lo contiene. La compresibilidad del uido es otra propiedad marcadamente distinta en los l quidos y en los gases. Un l quido es bastante incompresible y en la gran mayor de las a aplicaciones se puede suponer que su densidad es constante. Lo opuesto es cierto para los gases. Estos son sustancias muy compresibles y generalmente no se puede suponer que su densidad sea constante. A pesar de que los uidos estn constituidos por molculas, en le presente cap a e tulo se tratan como un medio continuo. El uso de los aspectos macroscpicos de un uido est juso a ticado cuando el camino libre medio (es decir, la distancia media que alcanza a recorrer una molcula del uido antes de colisionar con otra) es mucho menor que las distancias e involucradas del sistema bajo consideracin. o

Un uido es una substancia incapaz de soportar fuerzas de cizalla. Es sta la propiee dad que distingue a un slido de un uido. o En la gura 12.1 se muestra una placa, la cual se intenta deslizar hacia la derecha mediante la aplicacin de una fuerza F . o Un pasador slido evita que esto ocurra. o Sin embargo, cuando el pasador es sustituido por un l quido o un gas, la placa comenzar a deslizarse (aun para fuerzas F a pequeas). El uido no es capaz de ejern cer una fuerza de cizalla para mantener el equilibrio.

318

Fluidos

Sea F una fuerza que acta en forma perpendicular sobre un rea A. Se dene la presin u a o P por la relacin o F P A Considere un uido en reposo (por ejemplo, un vaso de agua, una piscina o una lago). Al sumergir un objeto en el uido, ste ejercer una fuerza sobre las supercies del objeto. La e a fuerza por unidad de rea (o presin) que ejerce un uido sobre los objetos (o supercies) a o con las que est en contacto, tiene varias propiedades importantes: a a) La fuerza que un uido en reposo ejerce sobre una supercie es siempre perpendicular a ella. Esto est directamente relacionado con el hecho de que un uido es incapaz de a ejercer una fuerza de cizalla. b) Un uido, en un punto particular, ejerce la misma presin en todas las direcciones o (Principio de Pascal ). En otras palabras, la presin es una magnitud escalar. Si suo mergimos en el uido un cubo innitesimal, la fuerza sobre todas las caras del cubito ser la misma, siendo su magnitud F = P A. Aqu A es el rea de una de las caras del a a cubito y P es la presin del uido en el lugar donde se encuentra el cubo (estamos o despreciando variaciones de la presin sobre distancias del tamao del cubito). o n c) Los lugares isobricos (de igual presin) en un uido en reposo (y de densidad consa o tante) son los planos horizontales. En la gura 12.2, en los puntos A, B, C, D y E la presin es la misma. Tambin la presin es igual en los puntos F , G, H e I. La o e o presin es mayor en puntos ubicados a mayor profundidad. En el punto J la presin o o es menor que en el punto F .

aire F G B

J H E

lquido

Figura 12.2

12.2.

La presin atmosfrica P0 o e

La presin en la supercie de un uido que se encuentra en un recipiente abierto a la o atmsfera no es nula, sino igual a la presin atmosfrica. Esta ultima se debe a que estamos o o e inmersos en un uido (compresible) constituido por el aire. La atmsfera de la Tierra ejerce o una presin sobre todos los objetos con los que est en contacto, en particular sobre otros o a

12.2 La presin atmosfrica P0 o e

319

uidos. La presin atmosfrica sobre la supercie terrestre la denotaremos por P0 , y es igual o e a la presin ejercida por el peso de toda la columna de aire que est por encima. o a P0 no es despreciable o insignicante como algunas personas suelen creer. Por el contrario, la presin atmosfrica juega un papel importante en numerosos aparatos y mquinas de la o e a vida diaria. Antes de continuar digamos algo sobre las unidades de la presin: o En el sistema SI, la unidad de presin es el Pascal: 1 Pa = 1 N/m2 . A 105 Pa se le suele o llamar bar , o sea 1 bar = 105 Pa. Observe que 1 bar es aproximadamente la presin que o 2 . En efecto, ejerce una masa de 1 kg si sta est apoyada sobre un rea de 1 cm e a a 1 Kg/cm2 = 9,81 N = 0,981 105 N/m2 = 0,981 bar. 2 0,0001 m

Tambin observe que 1 kg es la masa de una columna de agua de 10 m de altura y 1 cm2 e de seccin transversal. o Otra unidad frecuentemente usada para medir la presin es la atmsfera (atm). 1 atm o o corresponde a la presin promedio que ejerce la atmsfera terrestre a nivel del mar. Expeo o rimentalmente se encuentra que sta es aproximadamente 1,013 105 N/m2 = 1.013 bar. Se e dene la atmsfera estndar por o a 1atm = 1,0135 105 N/m2 = 1,0135bar . O sea, y esto es util recordar, 1 atm es aproximadamente igual a un bar e igual a la presin o que ejerce el peso de una masa de 1 kg sobre 1 cm2 , que a su vez es igual a la presin o adicional ejercida por una columna de agua a 10 metros de altura. La palma de una mano tiene un rea de aproximadamente 100 cm2 , luego la fuerza que a ejerce la atmsfera sobre la palma extendida es aproximadamente igual a la que ejercer o a una masa de 100 kg apoyada sobre ella. La fuerza sobre la palma es balanceada por una fuerza igual y contraria aplicada sobre el dorso de la mano. Considere un tubo de 1 m de largo y seccin o transversal A, cerrado por uno de los extremos. Llenemos el tubo con mercurio y coloquemos el tubo, con el extremo abierto hacia abajo, en un recipiente con mercurio. Observaremos que el nivel de mercurio se situar a a aproximadamente 760 mm del nivel del recipiente (ver gura 12.3). El extremo superior del tubo queda al vac o. Apliquemos la segunda ley de Newton a la columna de mercurio (que sobresale de la supercie del l quido en el recipiente). Cules a son las fuerzas que actan sobre ella? u

vaco presin 0

h = 760 mm presin P0

Hg

Figura 12.3

320

Fluidos

Hay slo dos: por una parte est la presin que el uido que est en el recipiente ejerce o a o a sobre el mercurio que est en el tubo: tal fuerza es F1 = P0 A; por otra, est el peso del a z a mercurio al interior de la columna, F2 = AhHg g. Como el uido est en reposo la fuerza z a neta debe ser nula, o sea P0 A = AhHg g . La densidad del mercurio es Hg = 13,6 g/cm3 . Con esto obtenemos par P0 el valor P0 1,014 105 Pa = 1 atm .

La fuerza que eleva l mercurio al interior del tubo es la presin atmosfrica! El dispositivo o e que acabamos de describir es un barmetro de mercurio. La altura de la columna de mercurio o mide la presin atmosfrica. La presin atmosfrica promedio a nivel del mar corresponde o e o e a 760 mm de mercurio. Al repetir el mismo experimento, pero con una columna de agua, la altura ser 13.6 vea ces mayor (recuerde que la densidad del mercurio es 13.6 g/cm3 y la del agua 1 g/cm3 ). Multiplicando los 76 cm por 13.6 se obtienen 10.34 m. Este dato es muy importante, ya que interviene en varias aplicaciones tecnolgicas. Por ejemplo, al intentar elevar agua de o un pozo (cuya supercie est en contacto con el aire que nos rodea) succionando por el a extremo superior de un tubo largo, slo se tendr xito si el nivel de agua no est a ms de o ae a a 10.34 metros de profundidad (en la prctica esta altura es menor ya que el agua comienza a a hervir bastante antes de llegar a los 10.34 metros).

12.3.

Principio de Arqu medes


^ z P0 A

Al sumergirnos en un uido, la presin auo menta. Evaluemos este aumento de presin o para un uido incompresible (l quido) de densidad . para ello consideremos el uido contenido en un paralelep pedo imaginario de altura h y rea A. Una de las caras de rea a a A la ubicamos de manera que coincida con la supercie del l quido mientras que la otra queda a una profundidad h (ver gura 12.4). Por lo dicho en la seccin anterior, la presin o o P = P (h) es slo una funcin de la profundio o dad h.

Figura 12.4

Es claro que las fuerzas netas horizontales ejercidas por el uido externo sobre el paralelep pedo son nulas, de lo contrario el uido del paralelep pedo acelerar lo que estar a a en contradiccin con la suposicin de que el uido se encuentra en reposo. o o Las fuerzas que actan sobre el paralelep u pedo en la direccin vertical son: i) la fuerza o que el aire ejerce sobre la cara superior, que es F1 = P0 A, ii) la fuerza que el uido z (exterior) ejerce sobre la cara inferior, que es F2 = P (h)A y iii) la fuerza debida al peso z del paralelep pedo con su uido. Esta fuerza de gravedad es F3 = (Ah)g. Como el z

12.3 Principio de Arqu medes paralelep pedo est en equilibrio, la fuerza total debe ser nula, es decir, a 0 = F1 + F2 + F3 = (P0 A + P (h)A Ahg) . z De la ecuacin anterior se deduce que o P (h) = P0 + gh ,

321

donde P0 es la presin atmosfrica que acta sobre la supercie del uido. Observe que el o e u aumento de la presin con la profundidad es igual a la presin ejercida por el peso de la o o columna del uido que se encuentra por encima. Estamos en condiciones de demostrar el Principio de Arqu medes:

Al sumergir un cuerpo parcial o totalmente en un uido aparece una fuerza llamada empuje que acta sobre el cuerpo y apunta en la direccin opuesta a u o la gravedad. La magnitud del empuje es Fe = gV , donde es la densidad del uido y V es el volumen del uido que fue desplazado por el cuerpo.

Para demostrar este principio observe primeramente que la fuerza que el l quido ejerce sobre cada parte de la supercie del cuerpo sumergido o parcialmente sumergido es independiente del material de que est hecho. a Por lo tanto, en lo que a empuje respecta, podemos reemplazar la parte sumergida del cuerpo A por un l quido igual al l quido que lo rodea (ver gura 12.5). Si es la densidad del l quido y Vs el volumen de la parte sumergida del cuerpo A, entonces el peso del cuerpo B es W = Vs g. Por supuesz to que el cuerpo B estar en equilibrio, por a consiguiente la fuerza de empuje que el l quido exterior ejerce sobre B debe exactamente contrarrestar el peso. Luego la fuerza de empuje es Fe = Vs g. z

^ z

Figura 12.5

Ms an, el cuerpo B est en equilibrio neutro (es decir, dentro del l a u a quido lo podemos trasladar a cualquier punto y orientarlo en cualquier direccin, quedando en reposo), luego o la fuerza de empuje debe estar actuando como si estuviera aplicada en el centro de gravedad de B. Esto es un dato de importancia para analizar el equilibrio de objetos otantes o sumergidos.

322 Ejemplo: Considere tres cubos del mismo tamao, adheridos tal como se muestra en la n gura 12.6. El material del cual estn hechos a los dos cubos A y B es 1 = 0,5 g/cm3 , mientras que el cubo C est hecho de un material a de densidad 2 = 2 g/cm3 . Observe que la densidad media de los tres cubos es igual a la del agua ( = 1 g/cm3 ) y, por lo tanto, al sumergirlo en agua, la fuerza de empuje exactamente cancela el peso. Cul ser la a a orientacin de equilibrio estable que el objeo to adquirir cuando est otando rodeado a a de agua?

Fluidos

B A C

Figura 12.6

Las unicas fuerzas que estn actuando sobre el objeto son el peso W y el empuje Fe . Ya a sabemos que ambas fuerzas tienen la misma magnitud y apuntan en direcciones opuestas y, por lo tanto, la fuerza neta sobre el objeto es nula. Pero para que se encuentre en equilibrio tambin el torque neto debe ser nulo. Esto se logra slo si ambas fuerzas son e o colineales (actan a lo largo de la misma recta). Encontremos los puntos en que actan las u u dos fuerzas. La gravedad acta en el centro de masas. El u B A centro de masas de los cubos A y B se encuentra en a y el centro de masas de C se a encuentra en b. El centro de masas del objeL 2 to completo se encontrar sobre la recta que a une a con b. Como el cubo C tiene el doble de masa de los dos cubos A + B juntos, el 1 centro de masas del objeto completo se ubicar ms cerca de b que de a. En la gura 12.7 a a C b hemos designado el centro de masas del objeto completo con el nmero 1. Se tiene que u b, 1 = a, b/3. L La fuerza de empuje, por otra parte, acta en u el centro de masas que se obtiene al sustituir Figura 12.7 los tres cubos por agua (en la gura lo hemos designado con el nmero 2). u Nuevamente el centro de masas de los cubos A+B se encuentra en a, mientras que el de C se encuentra en b. El centro de masas de los centros de masas nuevamente se encontrar sobre a la recta a, b. Pero ahora los cubos A + B pesan el doble de lo que pesa C, luego el centro de masas ahora estar ms cerca de a que de b. De hecho, el centro de masas cuando los a a tres cubos estn hechos de agua debe estar sobre el plano de simetr indicado en la gura a a con una l nea punteada. En resumen, la fuerza de gravedad acta en 1 y el empuje acta en 2. Para que no haya u u torque sobre el sistema la recta a, b debe orientarse a lo largo de la vertical. Concluimos que

12.4 La frmula baromtrica o e

323

el ngulo de la gura 12.6 debe coincidir con el de la gura 12.7. Se deduce inmediatamente a que tan = 1/2. Convnzase de que el equilibrio es estable cuando el punto 2 est sobre el e a punto 1 e inestable cuando 1 est sobre 2. a

12.4.

La frmula baromtrica o e

Considere N molculas de un gas connadas en un volumen V y a una temperatura T . Si e la ecuacin de los gases ideales es aplicable se tiene que o P V = N kB T . Aqu P es la presin del gas y kB = 1,38 1016 erg/K es la constante de Boltzmann. Sea o m la masa de cada molcula, entonces e P = N m kB T kB T = , V m m

donde es la densidad de masa del gas. De esta relacin se deduce que, mientras la temo peratura se mantenga constante, la presin de un gas es proporcional a su densidad. En o particular, si 0 y P0 son la densidad y presin de la atmsfera al nivel del mar (z = 0) y o o (z)y P (z) son las mismas magnitudes, pero a una altura z (por sobre el nivel del mar), entonces P0 0 = . P (z) (z) Por otra parte (ver gura 12.8), la presin a o una altura z es la misma que la que hay a una altura z es la misma que la que hay a una altura z + dz ms la presin ejercida por a o el peso del gas que hay entre las alturas z y z + dz, o sea, P (z) = P (z + dz) + (z)g dz . Esta ecuacin se puede reescribir de la forma o dP g0 = (z)g = P (z) . dz P0 (12.1) Figura 12.8
^ z z+dz g z

Esta es la ecuacin diferencial que gobierna el comportamiento de la presin atmosfrica o o e (a temperatura constante). Para resolver esta ecuacin debemos antes discutir la funcin o o exponencial . La funcin exponencial o La ecuacin diferencial del tipo o df (t) = f (t) , f(t) = dt (12.2)

324

Fluidos

donde es una constante (real o compleja), aparece frecuentemente en las ciencias naturales (y tambin en las ciencias econmicas). Es muy importante discutir y analizar sus soluciones. e o Una ecuacin diferencial es una ecuacin que involucra una funcin y sus derivadas (primera, o o o segunda, etc.). La derivada de ms alto orden que aparece en la ecuacin dene el orden a o de la ecuacin diferencial. La ecuacin diferencial (12.2) es de primer orden. o o Nos interesa encontrar la solucin ms general de (12.2). Un resultado importante de la o a teor de ecuaciones diferencial (y que no demostraremos aqu es que la solucin general a ) o de una ecuacin diferencial de orden n tiene n constantes arbitrarias. En otras palabras, o sabremos que tenemos la solucin general de la ecuacin (12.2) si sta tiene una constante o o e que se puede elegir arbitrariamente. Una vez que se ha encontrado la solucin general, la o constante arbitraria se elige de manera que la solucin corresponda a la solucin del proo o blema planteado (o sea, cumpla con las condiciones iniciales). Ejemplo: Consideremos la ecuacin diferencial z = a0 . Esta es una ecuacin diferencial de o o segundo orden. La solucin general es z(t) = z0 + v0 t + a0 t2 /2. La solucin general tiene dos o o constantes arbitrarias z0 y v0 , las que deben elegirse de manera que la solucin corresponda o a la situacin f o sica concreta que se est considerando. a Denamos la funcin exp(t) mediante la serie o exp(t) = 1 + t t2 t3 + + + . 1! 2! 3! (12.3)

Es evidente que su derivada es igual a la funcin, es decir, o d exp(t) = exp(t) . dt Ejercicio: Demuestre que la funcin f (t) = A exp(t), donde A es una constante arbitraria, o es la solucin general de la ecuacin o o f(t) = f (t) . Como consecuencia del ejercicio anterior concluimos que la solucin general de la ecuacin o o (12.1) es g0 P (z) = A exp z , P0 donde la constante arbitraria A se determina exigiendo que la presin en z = 0 sea P0 . Esto o nos da la condicin A = P0 . De esta manera obtenemos la frmula baromtrica o o e P (z) = P0 exp g0 z P0 .

Reiteramos que este resultado, que nos da la presin baromtrica en funcin de la altura, o e o es slo aproximadamente correcto ya que, contrariamente a nuestra suposicin, la tempeo o ratura de la atmsfera normalmente disminuye a medida que uno se eleva. o

12.4 La frmula baromtrica o e

325

Ejercicio: Demuestre que la funcin f (t) = exp(1 t) exp(2 t) es una solucin de la ecuacin o o o diferencial f(t) = (1 + 2 )f (t) . Por consiguiente, f (t) = exp(1 t) exp(2 t) debe poder escribirse de la forma f (t) = A exp((1 + 2 )t). Demuestre que en ese caso A = 1, o sea exp(1 t) exp(2 t) = exp((1 + 2 )t) . Observe que esta relacin justica la introduccin de la notacin o o o exp(t) = et . La funcin et = exp(t) se llama funcin exponencial . o o Ejercicio: Evaluando la serie (12.3) para t = 1, demuestre que e = 2,718 . . . Problemas (relacionados con la funcin exponencial) o 1. Suponiendo que la atmsfera tiene una temperatura constante, determine la presin o o atmosfrica a 10 km de altura. (La densidad del aire, en la vecindad de la supercie e terrestre, a 20 C, es aproximadamente 0 = 1,29 kg/m3 .) Considere un cilindro de radio R sobre el cual se apoya una cuerda. Sea e el coeciente de roce esttico entre la cuerda y el cilindro. Suponga que en uno de los extremos a de la cuerda est colgando una masa M . Cul es la m a a nima masa que debe colgarse en el otro extremo para que la cuerda no resbale? Respuesta: m = M ee . 3. La cantidad de ncleos de un elemento radiactivo que decae en un intervalo [t, t ] u es proporcional al nmero de ncleos no deca u u dos que se ten inicialmente (en el a instante t). Demuestre que la armacin anterior implica que o N (t) = N0 et , donde N (t) es el nmero de ncleos en el instante t que no ha deca u u do, N0 la misma magnitud pero en el instante t = 0 y es una constante positiva (la as llamada constante de desintegracin). o Para el caso en que = 0,01 s1 , determine el tiempo que debe transcurrir para que decaiga la mitad de los ncleos. u 4. Suponga que cierto banco (en el pa de las maravillas) para intereses a una tasa de s 100 % anual sobre los depsitos, y ms an, los paga en forma continua, sumando los o a u intereses al capital depositado. Si una persona deposita $1000, cunto le devolver el a a banco al cabo de un ao? n Respuesta: $ 2 718.28. . . = e 1000. (12.4)

2.

326

Fluidos

12.5.

Tensin supercial o

Entre dos molculas de un uido actan fuerzas. Estas fuerzas, llamadas fuerzas de van e u der Waals o fuerzas cohesivas son de origen elctrico. Una de las caracter e sticas de estas fuerzas es que su alcance es muy pequeo (rpidamente se desvanecen cuando la distancia n a entre las molculas es dos o tres veces su tamao); otra caracter e n stica es que mientras las molculas no se traslapan, la fuerza es atractiva. e El efecto neto de las fuerzas de cohesin soo bre una molcula que est en el interior del e a l quido es nulo, pero no as para una molcula e que se encuentra en la supercie (ver gura 12.9). Para poner una molcula en la supere cie hay que realizar un trabajo. O sea, la existencia de una supercie en un uido introduce una energ potencial. Esta energ a a es proporcional a la supercie y se tiene que dW = dA .

Figura 12.9

Aqu es una constante que depende del uido y se llama tensin supercial y dA es un o elemento (innitesimal) de supercie. En realidad la tensin supercial depende de las dos o substancia que estn en contacto. La siguiente tabla da valores de la tensin supercial a o para algunos casos. Substancia Agua Agua Agua Hg Hg Alcohol met lico Glicerol C3 H8 O3 Solucin jabonosa o En contacto con aire aire aire vac o aire aire aire aire Temp. C 0 20 80 20 20 20 20 20 [N/m] 0.0756 0.07275 0.0626 0.475 0.436 0.0227 0.0634 0,025
F

Para medir la tensin supercial se pueo de usar el dispositivo mostrado en la gura 12.10. Un alambre movible, inicialmente sumergido, se tira lentamente, extrayndolo del e l quido (con una pel cula del l quido adosada). Midiendo la fuerza F se puede deducir . En efecto, al mover el alambre movible a una altura h a h+dh, el trabajo que se realiza es dW = F dh.

Figura 12.10

12.5 Tensin supercial o

327

Por otra parte, la supercie de la pel cula aumenta en dA = 2L dh (el factor 2 se debe a que la pel cula tiene una supercie a cada lado). Se tiene

dW F dh F = = . dA 2L dh 2L

Problema: Deseamos encontrar la diferencia de presin entre el interior y exterior de una o pompa de jabn de radio R = 1 cm. o Si, soplando con una pajita, aumentamos el radio de la pompa de R a R + dR, entonces la supercie aumenta en dA = 2 (4(R + dr)2 4R2 ) = 16R dR .

El factor 2 nuevamente se debe a que hay que considerar tanto la supercie interior como exterior de la pompa. El cambio de energ debido al aumento de la supercie es por lo a tanto dW = dA = 16R dR .

Por otra parte, podemos evaluar el trabajo directamente, multiplicando el desplazamiento dR por la fuerza P 4R2 , es decir, dW = P 4R2 dR .

Igualando las dos ultimas expresiones se encuentra la diferencia de presin o 4 . R

P =

Con = 0,025 N/m y R = 0,01 m se obtiene P = 10 N/m2 . Si se deja de soplar por la pajita, la pompa se desina. Observe que la presin al interior de una pompa de jabn es mayor tanto ms pequeo o o a n es su radio. De esta observacin se deduce que al juntarse una pompa de jabn grande o o con una pequea, la pequea inar a la ms grande. De esta manera la pompa grande n n a a aumentar su tamao mientras que la pequea disminuir: en otras palabras, la ms grande a n n a a absorber a la ms pequea. a a n

328

Fluidos

12.6.

Capilaridad

Por otra parte, si la fuerza de adhesin es mucho menor que la fuerza de cohesin, entonces o o el l quido tender a concentrarse, adquiriendo una forma compacta tipo gota (ver gura a 12.11 b). Como resultado de esta competencia entre las distintas fuerzas de adhesin y cohesin, o o se forma un ngulo de contacto bien caracter a stico entre el l quido y el slido. Experio mentalmente se determina que este ngulo de contacto para las substancias aguavidrio es a aproximadamente 0 , mientras que para mercuriovidrio = 140 . Considere la l nea a lo largo de la cual conviven las tres fases. Conocemos la magnitud y la direccin de la fuerza sobre proveniente de la tensin supercial del l o o quido. Por el principio de accin y reaccin, el slido ejercer sobre el l o o o a quido una fuerza de la misma magnitud pero en direccin opuesta. Esta fuerza es la que hace subir un uido por un o capilar. Consideremos un tubo jo, de dimetro ina terior muy pequeo 2r y con un extremo inn merso verticalmente en un l quido cuya tensin supercial es . El largo de la l o nea en este caso es 2r. La fuerza que el tubo ejerce sobre el l quido a travs de la tensin e o supercial es F = (2r) cos , donde es el ngulo de contacto del l a quido con el material del tubo. Esta fuerza debe compensarse exactamente con el peso del l quido (que est por sobre el nivel exterior). a
2r

La fuerza entre molculas de dos substancias e distintas se llama fuerza de adhesin. Cono sideremos una pequea cantidad de l n quido (medio #2) en contacto con una supercie slida plana(medio #3) y ambos en contacto o con un gas (medio #1) (ver gura 12.11). Sea {i,j }, con i, j = 1, 2, 3 las tensiones superciales para las distintas interfases de la gura 12.11. Si la fuerza de adhesin (entre el l o quido y el slido) es mucho mayor que la fuerza de o cohesin (entre las molculas del l o e quido), entonces el l quido tender a esparcirse sobre el a slido (ver gura 12.6a). En este caso se dice o que el l quido moja al slido. o

Figura 12.11

Figura 12.12

12.7 Fluidos en movimiento El peso del l quido que subi por el tubo capilar es o Fg = 0 (r2 h)g ,

329

donde 0 es la densidad del l quido. Igualando las dos fuerzas se obtiene para la altura mxima h a la que sube el l a quido la expresin o h= 2 cos . 0 gr

Ejemplo: Los xilemas que trasportan los nutrientes en una plante t picamente tienen un radio de 103 cm. Evaluemos la altura mxima a la que podrn llegar los nutrientes. Sua a pondremos que el ngulo de contacto = 0 y para la densidad y tensin supercial del a o l quido usaremos la del agua. Usando la frmula expuesta ms arriba se encuentra que h o a 1,5 m. La capilaridad es efectivamente uno de los mecanismos que las plantas usan para elevar la savia, sin embargo, no puede ser el mecanismo responsable para elevar el agua de las ra ces hasta la punta de los rboles grandes (cuya altura puede superar los 100 metros), ya que para ello los xilemas a tendr que tener un dimetro 100 veces menor. an a

12.7.

Fluidos en movimiento

Consideraciones preliminares Los uidos en movimiento se pueden clasicar con respecto a varios aspectos. Uno de ellos es la compresibilidad. La hidrodinmica se preocupa de estudiar el ujo de uidos a incompresibles, mientras que la aerodinmica analiza los ujos de uidos compresibles. a Notamos, sin embargo, que incluso los gases pueden aproximadamente como incompresibles mientras su velocidad no supere a la tercera parte de la velocidad del sonido. Otro aspecto clasicatorio se introduce respecto al roce interno. Se tiene el ujo de un uido ideal si se ignoran todos los efectos debido al roce interno (es decir, se ignora la viscosidad del uido). En caso contrario se estar considerando ujos de l a quidos y gases reales. La trayectoria de un pequeo elemento de n uido dene una l nea de corriente o l nea 2 A2 de ujo. A su vez todo un haz de l neas de 1 v2 ujo dene un tubo de ujo (ver gura 12.13) tambin podemos clasicar los uidos en moe A1 vimiento con respecto al comportamiento de v1 sus l neas de corriente. Si stas no var a e an medida que transcurre el tiempo se tiene un Figura 12.13 ujo estacionario o ujo laminar ; en caso contrario, el ujo es turbulento. En un ujo laminar, dos l neas de corriente cercanas entre s en cierto lugar, se mantendrn a cercanas en todas partes. Tambin dos l e neas de corriente del uido nunca se cruzan. Cuando el ujo es turbulento entonces elementos de uido que inicialmente estn innitea simalmente cerca pueden llegar a estar separados por distancias macroscpicas a medida o

330

Fluidos

que transcurre el tiempo. El ujo del uido en este caso es catico y se forman remolinos o errticos (llamadas tambin corrientes parsitas). a e a

Flujo laminar

Flujo turbulento

Figura 12.14 La disipacin de energ es mucho mayor cuando el ujo es turbulento que cuando es o a laminar. Ecuacin de continuidad o Consideremos un tubo de ujo como, por ejemplo, el que se muestra en la gura 12.7. Sean A1 , 1 y v1 el rea transversal del tubo, la densidad y velocidad del uido en la entrada a del tubo y A2 , 2 y v2 las mismas magnitudes pero a la salida del tubo. Para un ujo estacionario, la cantidad de uido que ingresa por el tubo durante un intervalo de tiempo dt debe coincidir con la que emerge en ese mismo intervalo por el otro extremo, luego 1 A1 v1 dt = 2 A2 v2 dt , relacin a la que se denomina ecuacin de continuidad . Cuando el ujo es incompresible, o o la densidad no cambia (o sea, 1 = 2 ), luego, para uidos incompresibles, la ecuacin de o continuidad es A1 v1 = A2 v2 . Ecuacin de Bernoulli o En lo que sigue consideraremos el ujo estacionario de un uido ideal incompresible. Sean P1 y P2 las presiones a la entrada y salida de un tubo de ujo, respectivamente. Evaluemos el trabajo neto en el punto de entrada realizado por la presin sobre el uido que est al o a interior del tubo. En un tiempo dt la seccin transversal inicial avanza una distancia v1 dt, o siendo el trabajo sobre el uido W1 = F1 v1 dt = P1 A1 v1 dt . Por otra parte, el uido que emerge del tubo realiza un trabajo igual a W2 = F2 v2 dt = P2 A2 v2 dt . La diferencia es el trabajo neto realizado sobre el uido: dW = W1 W2 = (P1 A1 v1 P2 A2 v2 ) dt .

12.8 Aplicaciones del principio de Bernoulli

331

Este trabajo neto hecho sobre el uido debe ser igual al cambio de energ (potencial y a cintica) del uido: e dW = dU + dK . Si z1 es la altura del uido a la entrada del tubo y z2 la altura a la salida, el cambio de energ potencial es a dU = (A2 v2 dt)z2 g (A1 v1 dt)z1 g . El cambio de energ cintica es a e 1 1 2 2 dK = (A2 v2 dt)v2 (A1 v1 dt)v1 . 2 2 De las ecuaciones anteriores se obtiene (P1 A1 v1 P2 A2 v2 ) dt = [(A2 v2 dt)z2 g (A1 v1 dt)z1 g] 1 1 2 2 + (A2 v2 dt)v2 (A1 v1 dt)v1 . 2 2 Usando la ecuacin de continuidad, se encuentra o 1 2 2 P1 P2 = g(z2 z1 ) v2 v1 2 o sea, para cualquier punto a lo largo de un tubo de ujo, 1 P + gz + v 2 = constante . 2 Esta ultima relacin, consecuencia directa del teorema de conservacin de la energ se o o a, conoce con el nombre de ecuacin de Bernoulli . Es importante recalcar que la ecuacin de o o Bernoulli recin deducida es slo vlida para uidos ideales, o sea aplicable slo a situaciones e o a o en las cuales la viscosidad es despreciable. ,

12.8.

Aplicaciones del principio de Bernoulli

Supondremos impl citamente que en todos los casos analizados en la presente seccin que o el uido bajo consideracin es ideal y que el ujo es estacionario. En la prctica los resultao a dos obtenidos aqu sern slo una primera aproximacin al problema estudiado. Para una a o o descripcin ms precisa es necesario incluir en el formalismo los efectos introducidos por la o a viscosidad.

332 Problema 1: Un tambor de altura h y rea a A, parado y abierto por la tapa superior (es decir, en contacto con la atmsfera), se o encuentra lleno de agua. Asuma que en la parte inferior del manto se abre un tapn o de seccin transversal a. Cunto tiempo o a tardar en vaciarse el tambor? a Solucin: Apliquemos la ecuacin de Bero o noulli en los puntos 1 y 2, en la parte superior del uido en el tambor y una vez que ha emergido del tambor (gura 12.14). En ambos lugares la presin del uido es igual a la o presin atmosfrica P0 . o e

Fluidos

A 1

Figura 12.15

Elijamos el origen del eje vertical en la base del tambor. De acuerdo a la ecuacin de o Bernoulli se tiene 1 P0 + gh + 0 = P0 + 0 + v 2 , 2 donde v es la velocidad del uido a la salida del tambor. La velocidad, por lo tanto, es v= 2gh .

Esta ultima relacin se llama teorema de Torricelli . Observe que la velocidad del uido es o la misma que la que adquiere un objeto cuando cae una distancia h. Supongamos ahora que en cierto instante el uido dentro del tambor est a una altura z. a El volumen de uido que emerge en un tiempo dt es av dt, lo que hace bajar el nivel del tambor en dz = av dt/A. Tenemos que o, escribindolo de otra forma, e dz a = A z 2g dt . dz a a = v= dt A A 2gz ,

Integrando la ultima ecuacin desde que se comienza a evacuar el tambor hasta que est vac o e o, se obtiene: a dz = A z z=h 0 a 2z 1/2 = A h a 2 h= A
z=0 t=T

2g
t=0 T

dt

2gt
0

2gT .

El tiempo que demora en evacuarse el tambor es T = 2A a h . 2g

12.8 Aplicaciones del principio de Bernoulli Problema 2: Considere un sifn consistente o de un tubo con un dimetro constante de a 10 cm, con el cual se extrae agua de una represa. Con las alturas mostradas en la gura 2.15, evale el ujo que pasa por el tubo. u Solucin: Apliquemos la ecuacin de Bero o noulli en los puntos 1 y 2. Se tiene que 1 P0 + g(h2 h1 ) + 0 = P0 + 0 + v 2 , 2
4 ^ z h 1= 2 m 1 3 agua 2 O h 2= 6 m

333

Figura 12.16

donde v es la velocidad del agua al interior del tubo. Como el uido es incompresible y el dimetro del tubo no cambia, la velocidad para un uido ideal al interior del tubo en todos a los lugares es la misma. Para la velocidad v se obtiene v= 2g(h2 h1 ) .

El volumen de agua que pasa por el tubo en un tiempo dt es dV = Av dt , donde A es la seccin transversal del tubo. Sustituyendo los valores del enunciado se obtiene o dV = (0,05)2 dt 2 9,81 4 m3 /s 70 litros/s .

Cul es la presin en el punto 3 (al interior del tubo, a la altura del nivel de agua del a o tranque)? Para responder esta interrogante aplicamos la ecuacin de Bernoulli en los puntos 2 y 3. o Tenemos 1 1 P0 + 0 + v 2 = P3 + g(h2 h1 ) + v 2 . 2 2 Ac P3 es la presin del agua en el punto 3. Se obtiene a o P3 = P0 g(h2 h1 ) . Una columna de agua de 10 metros corresponde a aproximadamente la presin atmosfrica o e P0 . Por lo tanto, g(h2 h1 ) = 0,4P0 . Luego P3 0,6P0 . Anlogamente, para la presin en el punto 4 se obtiene a o P4 = P0 gh2 0,4P0 .

Observe que h2 no puede sobrepasar los 10 metros, ya que de lo contrario la columna de agua se corta.

334 Otras aplicaciones i) Atomizador: Al pasar una corriente de aire por encima de un tubo abierto, se reduce la presin al interior del tubo. Si la veloo cidad del aire es v, la presin P justo o encima del tubo es 1 P = P0 v 2 . 2 La disminucin de presin provoca que o o el l quido suba por el tubo. Una vez que el l quido llega a estar en contacto con la corriente de aire, ste se atomiza. Ese te principio es usado en las botellas de perfume y en los aspersores de pintura. ii) Tubo de Venturi: Al hacer pasar un l quido por una tuber estrechada, en el lugar consa treido baja la presin. La disminucin n o o de la presin permite determinar la veo locidad del uido. Apliquemos la ecuacin de Bernoulli en o los puntos 1 y 2 (gura 12.18).

Fluidos

aire P0

Figura 12.17

P1

P2 2

A1

A2

Figura 12.18

Si la tuber es horizontal (o sea, no hay cambios en la energ potencial del uido) se tiene a a que 1 2 1 2 P1 + v1 = P2 + v2 . 2 2 Por otra parte, la ecuacin de continuidad nos da la relacin o o A1 v1 = A2 v2 . De las ecuaciones anteriores se deduce que v2 = A1 2(P1 P2 ) . A2 A2 1 2

Si el ujo es sucientemente alto, el tubo de Venturi puede usarse para bombear. Por ejemplo, los extractores de saliva usados por los dentistas se basan en este principio.

12.9 *Viscosidad iii) Efecto Magnus: Consideremos un cilindro (o una esfera) en un uido en movimiento. Si el cilindro rota en torno a un eje perpendicular a la corriente del uido, y adems hay roce viscoso ena tre le cilindro y el uido, entonces el cilindro arrastrar al uido haciendo que las velocidaa des del uido a ambos lados del cilindro no sean iguales. En el caso mostrado en la gura adjunta, la velocidad es mayor arriba que abajo.
F
1

335

Figura 12.18

De acuerdo a la ecuacin de Bernoulli, la presin en el lugar 1 sern inferior que en el lado 2 o o a (P1 < P2 ). Esta diferencia de presin genera una fuerza neta sobre el cilindro hacia arriba. o Es este efecto, llamado efecto Magnus, el responsable de los as llamados efectos que pueden observarse en numerosos juegos de pelota. Justamente para aumentar el efecto las pelotas no deben ser completamente suaves en la supercie (pelusas en la pelota de tenis). iv) Bomba de chorro (jet) de agua: Por una tobera inyectora P se hace ingresar agua a alta velocidad en una cmara. De esta a manera se genera una disminucin de la preo sin en la vecindad de P , lo que a su vez pero mite aspirar el aire de un recipiente. El l mite inferior a que puede bombear este dispositivo (usando agua y a temperatura ambiente) es de aproximadamente P 2,7 104 Pa (la 1/40 ava parte de la presin atmosfrica) o e
agua

aire P

Figura 12.20

12.9.

*Viscosidad

Entre las distintas molculas de un uido actan fuerzas de adhesin. Por esta razn, cuando e u o o uyen y distintas partes del uido se mueven con velocidades relativas, aparecen fuerzas de roce interno, tambin llamada viscosidad . A pesar de que los uidos no maniestan e resistencia a fuerzas de cizalla, la viscosidad hace que s presenten cierta resistencia al deslizamiento. Otra consecuencia de la viscosidad es que la velocidad del uido que est en contacto con a una supercie (de un slido) es nula (con respecto a la supercie). o En esta seccin slo analizaremos casos en que el ujo es laminar. o o

336 Consideremos dos placas paralelas de rea A, a separadas por una distancia D y con un uido entre ellas. Una de las placas la mantenemos ja y la otra se mueve (paralelamente) con velocidad v0 (ver gura 12.21). El uido en contacto con la placa superior se mueve con velocidad v0 , mientras que el que est en a contacto con la placa inferior est en reposo. a
^ z rea A v0

Fluidos

Fr

en reposo

Figura 12.21 Newton experimentalmente encontr que para muchos uidos la fuerza que se debe realizar o para mantener la placa en movimiento es Fr = A dv v0 = A , D dz

o sea, es proporcional al rea A y al gradiente (derivada) de la velocidad. La constante de a proporcionalidad es la viscosidad dinmica. Los uidos que cumplen con esta relacin se a o llaman uidos newtonianos. La siguiente tabla da la viscosidad para algunas substancias: Fluido Agua Agua Agua Alcohol et lico Glicerina Glicerina Aire Aire Aire Helio Temp. C 0 20 100 20 0 20 -31.6 20 230 20 viscosidad [Ns/m2 ] 1,79 103 1,00 103 0,28 103 1,2 103 12.11 1.49 1,54 105 1,83 105 2,64 105 1,94 105

(Otra unidad usada para medir la viscosidad es el poise [P]: 1 [P] = 10 [Ns/m2 ].) De la tabla se observa que la viscosidad es mucho mayor para los l quidos que para los gases. Tambin se e observa una fuerte dependencia de la temperatura. Para los l quidos la viscosidad disminuye al aumentar la temperatura, mientras que para los gases aumenta. Flujo laminar en tubos El efecto de la viscosidad en el ujo de uidos por tubos de seccin redonda es de gran o importancia en muchas aplicaciones. Consideremos aqu un caso: el ujo estacionario de un l quido newtoniano por un tubo horizontal de largo L y radio R. Sean P1 y P2 las presiones del l quido en los dos extremos del tubo y determinemos el perl de velocidad v(r) del uido al interior del tubo y el ujo por unidad de tiempo.

12.9 *Viscosidad

337

Sea v(r) la velocidad del uido al interior del tubo. Sabemos que v(R) = 0, o sea, el uido en contacto con el tubo est en reposo. Consideremos ahora el uido encerrado al interior a de un cilindro de radio r (ver gura 12.22). Llamemos A al uido interior y B al uido que est ubicado a distancia mayores que r. El rea de contacto del uido A con B es 2rL. a a
Fluido A Fluido B

P 1

R L

P 2

Figura 12.22

La fuerza que B ejerce sobre A es, por lo tanto, Fr = (2rL) dv(r) x. dr

Observe que dv/dr es negativo y, por lo tanto, la fuerza que el uido exterior ejerce sobre A es contraria a la direccin del uido. Como el ujo es estacionario, la fuerza total sobre o el uido A debe ser nula, o sea, la fuerza ejercida por las presiones P1 y P2 sobre el cilindro interno debe cancelar exactamente a la fuerza Fr debida a la viscosidad: P1 r2 x P2 r2 x + Fr = 0 . De esta manera se deduce que dv P1 P2 = r. dr 2L Integrando sobre r y jando la constante de integracin de manera que v(R) = 0 se eno cuentra el perl de velocidades al interior del tubo (ecuacin de Poiseuille): o v(r) = P1 P2 2 (R r2 ) . 4L

Este perl es de forma parablica. o Conocido el perl de velocidades podemos evaluar el ujo dV /dt (la cantidad de uido que atraviesa la seccin transversal del tubo por unidad de tiempo). La cantidad de uido que o pasa entre dos cilindros concntricos de radios r y r + dr en un tiempo dt es (2r dr)v(r) dt. e Sumando sobre todos los cilindros (integrando sobre r) se obtiene la cantidad de uido dV que pasa por el tubo en un tiempo dt:
R

dV =
0

(2r dr)v(r) dt .
R

Se obtiene dV P1 P2 = 2 dt 4L
0

r(R2 r2 ) dr =

P1 P2 R4 . 8L

338

Fluidos

Observe que la cantidad de agua que se puede hacer pasar por un tubo aumenta dramtia camente cuando se aumenta su dimetro. Aumentar la diferencia de presin en un factor 2 a o aumenta el ujo en ese mismo factor; aumentar el dimetro en un factor 2 (sin aumentar a la diferencia de presin) aumenta el ujo en un factor 16. o Tambin podemos escribir la ultima ecuacin como sigue: e o P = P1 P2 = 8L dV , R4 dt

o sea, la prdida de presin al pasar un ujo dV /dt por un tubo es proporcional a su largo e o L y a la viscosidad e inversamente proporcional a la cuarta potencia de R. Flujo laminar alrededor de una esfera Usando matemticas ms avanzadas se puede evaluar la fuerza de roce Fr debido a la a a viscosidad que acta sobre una esfera de radio R cuando sta se mueve respecto a un uido u e con velocidad v0 . Si el ujo es laminar la fuerza es (ley de Stokes) Fr = 6rv0 . Esta ecuacin, midiendo la velocidad terminal de esferas cayendo en el uido, permite o determinar su coeciente de viscosidad.

12.10.
1.

Problemas

El rey Hiern de Siracusa pidi a Arqu o o medes que examinara una corona maciza que hab ordenado hacer de oro puro. La corona pesaba 10 kg en el aire y 9.375 kg a sumergida en agua. Arqu medes concluy que la corona no era de puro oro. Asumiendo o que en su interior conten plata, cunto oro ten la corona de Hiern? La densidad a a a o 3 ; la de la plata, 10.5 g/cm3 . del oro es 19.3 g/cm Considere un vaso de agua lleno hasta el borde, con un trozo de hielo otando en l. Por supuesto que el hielo, al otar, sobrepasar por encima del borde del vaso. A e a medida que el hielo se derrite. Se derramar el vaso? a Suponga ahora que en el mismo vaso ota un pequeo barco de juguete hecho de n latn. Suponga adems que el barquito tiene un pequeo oricio por el cual penetra o a n agua, haciendo que el barquito lentamente se llene de agua. Durante este proceso, o sea mientras el barco se llena de agua pero an no se hunde, el nivel del agua del vaso u baja, queda a igual altura o sube? Cuando nalmente el barquito se hunde, que pasa con el nivel del agua?

2.

3.

Considere un cilindro de masa M , rea A y altura h, que ota parado en un l a quido de densidad 0 .

12.10 Problemas a) Hasta qu alguna estar sumergido e a el cilindro en el l quido? b) Si el recipiente que contiene le l quido es muy grande (por ejemplo, un lago), qu trabajo debe realizarse e para sacar el cilindro del l quido? c) Var la respuesta si el recipiente a que contiene el l quido es un tambor cil ndrico de rea A0 ? a

339

Figura 12.23

4.

Considere una varilla de madera muy liviana, de largo L, seccin transversal A y o densidad , que se hace otar en el agua (designe la densidad del agua por 0 ). a) Convnzase de que no es posible que e la varilla ote parada. b) Para lograr que la varilla ote parada, agregumosle una masa puntual e m en el extremo inferior. Cul es la a m nima masa m que debe agregarse para lograr el objetivo?

Figura 12.24

5.

Figura 12.25

Considere un vaso comunicante de 2 cm2 de seccin transversal que contiene mercuo rio ( = 13,6 g/cm3 ). A un lado se echan 360 gramos de glicerina ( = 1,2 g/cm3 ) y en el otro 1/4 de litro de alcohol ( = 0,8 g/cm3 ). Encuentre el desnivel d que glicerina existe entre los niveles superiores de la glicerina y el alcohol. Haga un grco cuaa litativo de la presin hidrosttica en o a funcin de la profundidad para cada uno o de los dos brazos del vaso comunicante (graque las dos curvas en el mismo gra co).

alcohol

mercurio

340 6. Considere un cilindro de seccin A y altuo ra h que se encuentra otando en la interfase de dos uidos de densidades 1 y 2 , respectivamente (1 > 2 ). Encuentre la densidad del cilindro si ste se encuentra e sumergido en el uido 1 en una magnitud d.

Fluidos

h
1

Figura 12.26 7. Qu volumen de helio se requiere si debe elevarse un globo con una carga de 800 kg e (incluido el peso del globo vac o)? Las densidades del aire y del helio, a la presin de o una atmsfera, son aire = 1,29 kg/m3 y He = 0,18 kg/m3 , respectivamente. o Una varilla de largo L y densidad 1 ota en un l quido de densidad 0 (0 > 1 ). Un extremo de la varilla se amarra a un hilo a una profundidad h (ver gura adjunta). a) Encuentre el ngulo . a b) Cul es el m a nimo valor de h para el cual la varilla se mantiene en posicin vertical? o c) Si A es la seccin transversal de la o varilla, encuentre la tensin del hilo. o 9.

8.

aire L h

fluido

hilo

Considere las tres mediciones mostradas en la gura adjunta:


P1

P2

Figura 12.28

P1 es el peso de un recipiente con agua con un objeto sumergido en l. e P2 es el peso cuando el objeto est sumergido en el agua, pero colgado de una a cuerda sin que toque el fondo del recipiente. P3 es el peso del recipiente con agua. Encuentre la densidad promedio del objeto.

Figura 12.27

P3

12.10 Problemas 10.

341

En un canal horizontal, de ancho b, uye agua con velocidad v, siendo el nivel de agua h. Asuma que en cierto lugar el canal se ensancha en una pequea cantidad db. n Demuestre que el nivel del agua cambiar en a dh = hv 2 db . b(gh v 2 )
S1 h m1 L

Note que si v 2 < gh el nivel del agua sube. 11. Un corcho cil ndrico de masa m1 y seccin o transversal S1 ota en un l quido de densidad . El corcho est conectado por medio a de una cuerda sin masa, de largo L, a un cilindro de aluminio de masa m2 y seccin transversal S2 . El cilindro de alumio nio puede deslizarse sin roce por un oricio hermtico en el fondo del tiesto. Calcular e la profundidad h a la que debe hallarse la base del corcho para que el sistema de los dos cilindros est en equilibrio. La presin e o atmosfrica, juega algn rol? e u Un prado es regado con un regador hechizo que consiste en una botella plstia ca, con numerosos agujeros de 1 mm de dimetro, acostada sobre el prado y coa nectada aun a manguera. Asuma que una bomba de agua se encarga de generar un ujo de agua constante de 0.2 litros por segundo. Cuntos agujeros debe tener la a botella para que el agua llegue a mojar el prado a 8 metros de distancia de la botella? Cul es la presin al interior de la a o manguera si sta tiene una seccin transe o versal de 4 cm2 ? Un tubo de largo L, lleno de agua, gira en el plano horizontal en torno a un eje vertical que pasa por uno de sus extremos. En el extremo junto al eje, el tubo est abiera to, coincidiendo por lo tanto la presin del o uido con la presin atmosfrica. El tubo o e gira con velocidad angular constante . Si en el otro extremo, en cierto instante, se abre un pequeo oricio, con qu velocin e dad emerger el agua del tubo? (Especia que la rapidez y direccin de la velocio dad.)

m2

S2

Figura 12.29
Vista lateral

12.

Vista frontal

Figura 12.30

13.

L
Figura 12.31

342 14. Para abastecer de agua a una casa de dos pisos se recurre a un hidropack. Este sistema consiste en una depsito subterrneo, o a 5m una bomba y un cilindro con agua y aire. La bomba inyecta agua a presin al cilindro, que en o su parte superior queda con ai1m re comprimido. Un medidor de depsito presin detiene la bomba cuano do la presi n del cilindro alcanza o el valor deseado (el mismo medibomba dor vuelve a encender la bomba cuando la presin baja de cierto o Figura 12.32 nivel).

Fluidos

llave

aire comprimido

Si el nivel del agua en el cilindro se sita 1 metro por debajo del suelo, calcule la u presin necesaria en el aire comprimido para que una llave de 1 cm2 de seccin, a una o o altura de 5 metros sobre el suelo, entregue un caudal de 12 litros por minuto. (La seccin transversal del cilindro es grande respecto a la de la llave.) Tambin encuentre o e la presin del aire al interior del cilindro. o

15.

La fuerza de sustentacin de un avin moo o por metro derno es del orden de 1000N cuadrado de ala. Suponiendo que el aire es un uido ideal y que la velocidad del aire por debajo del ala es de 100 m/s, cul a debe ser la velocidad requerida por sobre el ala para tener la sustentacin deseada? o (La densidad del aire es 1.3 kg/m3 .)

Figura 12.33

16.

Un bombero lanza agua con su manguera hacia un incendio formando un ngulo de a 45 con la horizontal. El agua que emerge del pistn penetra horizontalmente por una o ventana del tercer piso que se encuentra a una altura h = 10 metros. La manguera que transporta el agua desde el carro bomba tiene un dimetro D de 6 cm y concluye a en un pistn cuya abertura tiene un dimetro d de 1.5 cm. o a

a) Cuntos litros de agua emergen del pistn por minuto? a o b) Cul es la presin P que debe soportar la manguera (en atmsferas)? a o o

12.10 Problemas 17. Considere la tuber que lleva el agua a de una represa hacia una turbina. Suponga que la bocatoma se encuentra a 10 metros bajo el nivel de las aguas y que la turbina se encuentra 80 metros por debajo de ese nivel. Al inicio, es decir a la salida de la represa, la tuber a tiene un dimetro de 40 cm. Suponga a que el uido se comporta como un uido ideal. a) Cul es el dimetro mximo que a a a puede tener la tuber en su exa tremo inferior para que no se produzcan cortes de la columna de agua al interior de la tuber a?

343

10 m

80 m

Figura 12.34

b) Cul ser la cantidad de agua que pasar en ese caso por la tuber y cul la a a a a a velocidad del agua emergente? c) Si el proceso de generacin de energ elctrica usando la presente turbina fuese o a e 100 % eciente, cul ser la potencia de esta central? Esto corresponde al a a consumo promedio de cuntas casas? a d) Haga un grco cualitativo de la presin al interior de la tuber en funcin de a o a o la altura. Cmo cambia esta presin si la seccin de la tuber en el punto o o o a, emergente, se disminuye a la mitad? A la centsima parte? e 18. Considere una tuber de una calefaccin. En el stano su dimetro es de 4.0 cm y en a o o a el segundo piso, 5 metros ms arriba, la tuber tiene un dimetro de slo 2.6 cm. Si a a a o en el stano una bomba se encarga de bombear el agua con una velocidad de 0.5 m/s o bajo una presin de 3.0 atmsferas, cul ser la rapidez de ujo y la presin en el o o a a o segundo piso? Suponga que el nivel de un l quido (agua) en un tambor tiene una altura h. A una altura b se hace una pequea n perforacin lateral que permite que o el agua emerja horizontalmente. A qu altura debe hacerse la perforacin e o para que el alcance d del agua se mximo? a Respuesta: b = h/2. 20.

19.

Figura 12.35

En un torrente de agua se sumerge un tubo doblado, tal como se muestra en la gura adjunta. La velocidad de la corriente con respecto al tubo es v = 2,5 m/s. La parte superior del tubo se encuentra a h0 = 12 cm sobre el nivel del agua del torrente y tiene un pequeo agujero. n

344 A qu altura h subir el chorro de agua que sale por el agujero? e a

Fluidos

h h0 v

Figura 12.36

21.

Considere una masa esfrica homognea en equilibrio hidrosttico. Sea RT el radio y e e a 0 la densidad de masa. a) Muestre que la presin a una distancia r del centro viene dada por o p= 2 2 G R2 r 2 3 0 .

b) Evale la presin al centro de la Tierra. RT = 6,3 108 cm y densidad uniforme u o promedio 0 = 5,5 g/cm3 . 22. En un baln el gas en su interior se encuentra a una presin P . Demuestre que la o o velocidad con que escapa el gas, al abrir la vlvula, es a 2(P P0 ) ,

v=

donde es la densidad del gas y P0 la presin atmosfrica. (Esta ecuacin se conoce o e o por ley de Bunsen.) 23. Considere una prensa hidrulica (ver gura 12.37). Sean R1 = 25 cm y R2 = 150 cm a los radios de los mbolos de bombeo y de presin, respectivamente. e o Si de la palanca que acta sobre el mbolo de bombeo se tira con una fuerza F1 = u e 100 [N] (ver gura), qu fuerza ejercer el mbolo de presin sobre el objeto S? e a e o 24. Se quiere confeccionar aluminio poroso (algo as como queso suizo) que se mantenga en suspensin en agua. Determine la razn entre el volumen de los poros y el volumen o o del aluminio poroso. (La densidad del aluminio slido es = 2700 kg/m3 .) o

12.10 Problemas
F2 L mbolo de presin S mbolo de bombeo L F1 estanque de reserva

345

vlvula de retorno

vlvula

vlvula

Figura 12.37

25.

Considere un cuerpo l quido de densidad uniforme 0 , que se mantiene unido debido a la gravedad y que gira con una velocidad angular 0 . Si bien el cuerpo es esfrico si e 0 = 0, cuando 0 = 0 (pero no demasiado grande), el cuerpo adquiere la forma de un esferoide oblato. Demuestre que si la desviacin de la esfericidad es pequea, o n entonces
2 R1 R2 3 0 = , R 8 0 G

R1

O R2

donde R R1 R2 . Evale (R1 R2 )/R u para la Tierra y comprelo con el valor a experimental, que es 1/298,4.

Figura 12.38

26.

Considere la situacin mostrada en la o gura 12.39. Un cilindro de radio R y largo L evita que el agua de cierto recipiente se rebase. El cilindro se puede mover libremente. La densidad del cilindro es tal que, cuando el agua llega a la parte superior del cilindro, la posicin del cilindro o es la mostrada en la gura. Encuentre la fuerza que ejerce el agua sobre el cilindro. Encuentre la densidad del material del que est hecho el cilindro. a

^ z ^ x

cilindro

R agua
0

Figura 12.39

346 27.

Fluidos Considere una caja de dimensiones a, b y h, llena de agua. Todos los lados de la caja estn rmemente unidos entre s exa , cepto uno de los lados laterales (de dimensin b h). Evale la magnitud de la fuerza o u exterior m nima con que debe presionarse ese lado contra el resto de la caja para que el agua no escurra. Si la fuerza se aplica en un solo lugar, encuentre la posicin en o la que debe aplicarla.

agua F h b a

Figura 12.40

28.

Un mol de aire en condiciones normales (a nivel del mar y a 20 C de temperatura) ocupa un volumen de 22.4 litros. Estime la densidad del aire si gran parte de l e est constituido por nitrgeno. (Resp.: 1,28 kg/m3 .) a o Cul es el m a nimo volumen que debe tener un globo de helio ( = 0,18 kg/m3 ) para levantar un veh culo de 1200 kg?

29.

Dos globos esfricos inados con aire, ame bos de radio R, se unen mediante una cuerda de longitud L. Los dos globos se mantienen bajo el agua con el punto medio de la cuerda jo al fondo. Calcular la fuerza de contacto entre los globos.

L /2

L /2

Figura 12.41

30.

Figura 12.42

Una varilla yace en el fondo de un recipiente con agua formando un ngulo de a con la vertical. La varilla es de seccin 60 o uniforme y est formada por dos pedazos a iguales en longitud pero de distinta densidad. La densidad de una de las porciones de la varilla es la mitad de la del agua. Determine la densidad de la otra porcin. o

30 o

12.10 Problemas
a

347

31.

Considere un bloque de hielo ( = 920 kg/m3 ) en forma de L, formado de tres cubos de 25 cm por lado. Mediante un peso se desea sumergir el hielo en agua como se indica en la gura. Determine la masa del peso y la ubicacin en el hielo o donde deber adherirse de modo que el a hielo se mantenga justo sumergido lo ms a estable posible. Considere un sistema de vasos comunicantes formado por dos tubos de seccin o transversal de 50 cm2 que estn unidos por a un tubito corto de seccin transversal muy o pequea (o sea, para efectos de este pron blema podemos despreciar la cantidad de uido que se encontrar en el tubito). Inia cialmente en este sistema de vasos comunicantes se encuentran dos litros de agua.

hielo

Figura 12.43

32.

Figura 12.44 a) Encuentre la altura en que se encontrarn las interfases entre los l a quidos y el aire en cada uno de los tubos si en uno de los tubos se le agregan 2 litros de un l quido cuya densidad es = 0,8 g/cm3 . b) Para la situacin descrita en la parte a), encuentre la presin en el fondo de los o o vasos comunicantes. c) Encuentre la altura en que se encontrarn las interfases entre los l a quidos y el aire en cada uno de los tubos si en uno de los tubos, en lugar de 2, se le agregan 3 litros de un l quido cuya densidad es = 0,8 g/cm3 . 33. Un tubo horizontal por el que uye l quido de densidad 0 a razn de Q m3 /s, se o bifurca en dos ramas en el plano vertical, una superior y otra inferior, de secciones transversales a1 = a2 = a, abiertas a la atmsfera (ver gura 12.45). Si la distancia o entre las ramas es h, determinar: a) Las cantidades q1 y q2 de l quido (en m3 /s) que uyen por ambas ramas. b) La condicin que debe cumplir Q para que haya ujo en la rama superior. o

34.

Una gotita de agua de 1 mm de radio se pulveriza en gotitas de 104 mm de radio. En qu factor aumenta la energ supercial (debido a la tensin supercial)? e a o

348
q1 Q h g

Fluidos

q2

Figura 12.45

35.

La gura 12.46 muestra un tubo de Pitot, instrumento que se usa para medir la velocidad del aire. Si el l quido que indica el nivel es agua y h = 12 cm, encuentre la velocidad del aire. La densidad del aire es aire = 1,25 kg/m3 . Respuesta: v0 = 43,4 m/s = 156 km/h.

aire agua

Figura 12.46

36.

Considere dos placas planas de vidrio, separadas por una distancia de 0,1 mm, con un extremo sumergidas en agua en forma vertical. Qu distancia se elevar el agua e a entre las placas debido a la capilaridad?

37.

Encuentre la velocidad terminal que adquiere una esfera de cobre de 0,5 cm de dimetro, cuando cae en agua (Cu = 8,92 g/cm3 ). En qu factor disminuye la a e velocidad terminal si el dimetro se achica en un factor 10? a

38.

Considere un oleoducto de 5 km y 50 cm de dimetro por el cual se desea bombear a 1 m3 por segundo. Si uno de los extremos est abierto a la presin atmosfrica, a o e qu presin P1 debe existir en el otro extremo? Suponga que la densidad del petrleo e o o es = 950 kg/m3 y el coeciente de viscosidad es aproximadamente = 0,2 Pa s. Cul es la potencia dW/dt (energ por unidad de tiempo) disipada por la friccin a a o interna originada por la viscosidad? Respuesta: P1 7,5 atm; dW/dt 650 kW.

12.11 Solucin a algunos de los problemas o

349

12.11.

Solucin a algunos de los problemas o


rea A a h 0 fluido

Solucin al problema 8 o El largo a de la parte de la varilla sumergida es a = h/ sen . La fuerza de empuje se aplica en el lugar a/2 y la fuerza de gravedad en el lugar L/2 (medidos desde O). Sea A la seccin transversal de la varilla. o Entonces la fuerza de empuje viene dada por Fe = 0 Aag = 0 A z h g . z sen
1

Fe aire

Fg O T

hilo

Figura 12.47 La fuerza de gravedad es Fg = 1 LAg . z El torque ejercido por ambas fuerzas respecto a O debe ser nulo, o sea, a L Fe cos = Fg cos . 2 2 Simplicando se obtiene Fe a = Fg L . Sustituyendo las expresiones par Fe y Fg se deduce que 0 Aa2 g = 1 AL2 g , h2 = 1 L2 . sen2 Despejando se encuentra nalmente que 0 sen = 0 h . 1 L o sea

Si el lado derecho de la ultima ecuacin es mayor o igual a uno, la varilla se mantendr en o a posicin vertical. El m o nimo valor de h para que la varilla est en posicin vertical es e o hmin = L 1 . 0

La tensin del hilo se obtiene exigiendo que la fuerza total sea nula. De esta manera se o obtiene que T = Fe Fg = 0 A = ALg1 donde M es la masa de la varilla. h g 1 LAg sen 0 0 1 = Mg 1 1 1

350 Solucin al problema 16 o

Fluidos

a) Si v es velocidad con que emerge el agua del pistn, la velocidad hacia arriba la o ser v/ 2. El agua alcanza a subir una altura h, luego su velocidad es a v = 2 gh = 20 m/s . La cantidad de agua V que emerge del pistn en t = 60 segundos es o V = vt d 2
2

1 = 20 60 3,14 (0,015)2 m3 = 212 litros . 4

b) Usemos el teorema de Bernoulli para comparar el ujo del agua justo a la salida del pistn con el ujo en la manguera justo detrs del pistn. No hay cambio en la o a o energ potencial. Como la seccin transversal de la manguera es 16 veces mayor que a o la abertura del pistn, la velocidad del agua en la manguera ser 16 veces menor que o a la velocidad emergente v. A la salida del pistn la presin es la presin atmosfrica, o o o e que ignoraremos en el presente clculo, ya que slo estamos interesados en la presin a o o adicional p que debe soportar la manguera debido al agua que uye en su interior. Se tiene 1 v 2 1 p + 0 = 0 v 2 . 2 16 2 Ignorando la energ cintica del agua al interior de la manguera (convnzase de que a e e modica el resultado nal en menos de un 0.5 %), se obtiene 1 1 kg m2 kg p = 0 v 2 = 1000 3 400 2 = 2 105 , 2 2 m s m s2 lo que corresponde a aproximadamente 2 atmsferas. o Solucin al problema 27 o Elijamos el eje z a lo largo de la vertical, con el origen al fondo de la caja sobre la tapa mvil. o La presin a una altura z es P (z) = 0 g(h z). Dividamos la tapa en franjas horizontales o de largo b y ancho (altura) dz. La fuerza que ejerce el uido sobre la franja que est a la a altura z es dF = P (z)b dz . Sumando (integrando) la fuerza que el l quido ejerce sobre cada una de las franjas se obtiene la fuerza total h h 1 F = P (z)b dz = 0 gb (h z) dz = 0 bgh2 . 2 0 0 Para encontrar a qu altura h0 debemos aplicar esta fuerza sobre la tapa, evaluemos el e torque que ejerce el uido sobre la tapa respecto al origen. El torque que el uido ejerce sobre la franja que est a la altura z es a d = zP (z)b dz .

12.11 Solucin a algunos de los problemas o

351

Sumando (integrando) el torque que el l quido ejerce sobre cada una de las franjas se obtiene el torque total
h h

=
0

zP (z)b dz = 0 gb
0

1 z(h z) dz = 0 bgh3 . 6

Para que la tapa est en equilibrio el torque que ejerce la fuerza total externa F debe e coincidir en magnitud con , es decir, F h0 = , o sea 1 1 0 bgh2 h0 = 0 bgh3 . 2 6 De esta ecuacin se deduce nalmente que h0 = h/3. o Solucin al problema 33 o La relacin de Bernoulli se puede aplicar entre los puntos A y B1 y tambin entre A y B2 . o e Por transitividad, la relacin de Bernoulli tambin es vlida entre los puntos B1 y B2 . Se o e a tiene 1 2 1 2 P1 + gh1 + v1 = P2 + gh2 + v2 . 2 2 Pero P1 = P2 = P0 (la presin atmosfrica), h1 = 0 y h2 = h, luego o e 1 2 1 2 v1 = gh + v2 . 2 2

q1 B1 Q A h B2 q2
Figura 12.48

^ z g

Los ujos que circulan por la rama superior e inferior vienen dados por q1 = av1 y q2 = av2 , respectivamente. Tambin se tiene que Q = q1 + q2 . De las relaciones anteriores se deduce e que Q2 2a2 gh q1 = 2Q y Q2 + 2a2 gh q2 = . 2Q

352 Para que circule l quido por la rama superior se debe tener que Q > a 2gh .

Fluidos

Cap tulo 13

Oscilador Armnico o
13.1.
2 La ecuacin diferencial x(t) + 0 x(t) = 0 o

La ecuacin diferencial que gobierna el comportamiento de un oscilador armnico simple o o es 2 x(t) + 0 x(t) = 0 . (13.1) Esta es una ecuacin diferencial lineal de segundo orden. Comenzaremos este cap o tulo exponiendo algunos resultados generales relativos a este tipo de ecuaciones, resultados que sern de gran utilidad para nuestros propsitos. a o Sean x1 (t) y x2 (t) dos soluciones cualesquiera de cierta ecuacin diferencial. o Tal ecuacin diferencial es lineal si x1 (t) + x2 (t) tambin es solucin, donde o e o y son constantes (reales o complejas) arbitrarias. Ejercicio: Demuestre que la ecuacin diferencial del oscilador armnico es lineal. o o El orden de la derivada ms alta da el orden de la ecuacin diferencial. La a o solucin general de una ecuacin diferencial de orden n tiene n constantes aro o bitrarias (que luego deben ser determinadas usando las condiciones de borde). La ecuacin diferencial del oscilador armnico es de segundo orden, por lo tanto, la solucin o o o general tiene dos constantes arbitrarias. Sean x1 (t) y x2 (t) dos soluciones cualesquiera (distintas) de (13.1). Como la ecuacin diferencial (13.1) es lineal, se tiene que la funcin o o xg (t) = x1 (t) + x2 (t), con y constantes arbitrarias, tambin es solucin. Pero observe e o que la solucin xg (t) tiene dos constantes arbitrarias y, por lo tanto, debe ser una solucin o o general del problema. En otras palabras, todas las posibles soluciones de (13.1) deben ser de la forma xg (t); las distintas soluciones se diferencian slo por los valores de y . o En el lenguaje tcnico se dice que las soluciones de la ecuacin diferencial (13.1) forman un e o espacio vectorial de 2 dimensiones, siendo x1 y x2 dos vectores particulares de ese espacio. Los dos vectores x1 (t) y x2 (t) (si uno de ellos no es mltiplo del otro) forman una base del u espacio vectorial. Cualquier otro vector (o sea, solucin de (13.1)) es una combinacin lineal o o de los vectores base, es decir, es de la forma x1 (t) + x2 (t).

354 Sabemos que las funciones x1 (t) = cos(0 t) y x2 (t) = sen(0 t)

Oscilador Armnico o

(13.2) (13.3)

son dos soluciones particulares de (13.1). Estas dos funciones (y de hecho as se hace frecuen temente) pueden ser tomadas como los dos vectores base del espacio vectorial formado por las soluciones de (13.1). Cualquier otra solucin x(t) de la ecuacin diferencial del oscilador o o armnico puede escribirse de la forma o x(t) = a cos(0 t) + b sen(0 t) . Las constantes a y b se determinan a partir de las condiciones iniciales. Observe que no es necesario elegir las funciones (13.2) y (13.3) como vectores base del espacio vectorial; de hecho, cualquier otro par de soluciones (mientras una no sea mltiplo u de la otra) tambin habr servido. Lo interesante es que las funciones (13.2) y (13.3) no e a son las funciones ms convenientes para usar como base. Existe un par de soluciones de a (13.1) que, si se usan como base, simplican notoriamente los clculos. En lo que sigue de a esta seccin introduciremos esta nueva base, estudiaremos algunas de sus propiedades y la o relacionaremos con la base dada por las funciones (13.2) y (13.3).

Consideremos la funcin o z(t) = et = exp(t) . Al derivar z(t) dos veces se obtiene z(t) = et y z (t) = 2 et = 2 z(t) . Observe que esta ultima ecuacin se puede escribir de la forma o z (t) 2 z(t) = 0 . Esta ecuacin es idntica a la del oscilador armnico si se identica o e o
2 2 = 0 ,

lo que es equivalente a la relacin o con i = i0 , 1. Observe que acabamos de demostrar que las funciones x1 (t) = ei0 t y x2 (t) = ei0 t (13.5) (13.4)

2 13.1 La ecuacin diferencial x(t) + 0 x(t) = 0 o

355

son dos soluciones particulares de la ecuacin diferencial del oscilador armnico, o sea, de o o (13.1). Resulta que stas son las funciones ms convenientes para generar todas las dems e a a soluciones de (13.1). Cualquier solucin x(t) de (13.1) se puede escribir de la forma o x(t) = ei0 t + ei0 t , donde las constantes y se determinan a partir de las condiciones iniciales. (Las constantes y , generalmente, resultan ser nmeros complejos). u Determinemos las relaciones entre las dos bases. Como cos(0 t) es solucin de (13.1) debe o poder escribirse de la forma cos(0 t) = c1 ei0 t + c2 ei0 t . Determinemos las constantes c1 y c2 . Derivando (13.6) se encuentra que 0 sen(0 t) = i0 c1 ei0 t i0 c2 ei0 t , o sea, sen(0 t) = i c1 ei0 t c2 ei0 t Evaluando (13.6) y (13.7) para t = 0 se obtiene 1 = c1 + c2 y 0 = i(c1 c2 ) . De estas relaciones se deduce que c1 = c2 = 1/2. De esta manera hemos demostrado que cos(0 t) = y 1 i0 t e ei0 t . (13.9) 2i Tambin podemos escribir exp(i0 t) y exp(i0 t) en funcin de cos(0 t) y sen(0 t). Usando e o las relaciones anteriores no es dif demostrar que cil sen(0 t) = ei0 t = cos(0 t) + i sen(0 t) y ei0 t = cos(0 t) i sen(0 t) . (13.11) Por ultimo, sustituyendo en (13.10) 0 t por encontramos una de las ms bellas ecuaciones a de la matemtica a ei + 1 = 0 , relacin que combina de manera simple los ms importantes nmeros de esa ciencia: 0, 1, o a u , e e i = 1. Ejercicio: Demuestre que el mdulo de los nmeros complejos exp(i0 t) y exp(i0 t) es o u uno, es decir, demuestre que |ei0 t | = |ei0 t | = 1 . (13.10) 1 i0 t e + ei0 t 2 (13.8) . (13.7) (13.6)

356

Oscilador Armnico o

13.2.

El oscilador armnico simple o


2 x(t) + 0 x(t) = 0 ,

Cada vez que la ecuacin dinmica de un sistema tiene la forma o a

estaremos en presencia de un oscilador armnico. o Ejemplo: Consideremos un pndulo de largo R. Elijamos el origen en el punto de suspene sin. El momento angular y el torque (en torno al origen) vienen dados por o

l = mR(R) y = Rmg sen . Por otra parte = luego mR2 = Rmg sen . Esta relacin se puede escribir de la forma o g sen = 0 . + R dl = mR2 , dt

m
Figura 13.1

2 Denotando g/R por 0 y restringindonos a pequeos ngulos, de manera que podamos e n a usar la aproximacin sen , se obtiene o 2 + 0 = 0 .

(13.12)

La constante 0 est relacionada con el per a odo T del movimiento por la relacin 0 T = 2. o Conocer la ecuacin dinmica de un sistema permite, en principio, conocer la evolucin o a o temporal del mismo. Para encontrar la solucin expl o cita del problema se procede generalmente de la siguiente manera: i) se busca la solucin general de la ecuacin dinmica; o o a ii)las constantes arbitrarias de la ecuacin general se determinan exigiendo que la solucin o o cumpla con las condiciones de borde (iniciales) del problema. Ilustremos el procedimiento con nuestro ejemplo concreto. Supongamos que en el instante t = 0 el ngulo y la velocidad angular del pndulo son 0 y , respectivamente. Deseamos a e encontrar una expresin expl o cita para (t). Resolveremos este problema de dos maneras: a) Sabemos que la solucin general de (13.12) puede escribirse de la forma o (t) = a cos(0 t) + b sen(0 t) . Determinaremos las constantes a y b. Para ello derivemos primero la ultima ecuacin o respecto al tiempo. Se obtiene (t) = a 0 sen(0 t) + b 0 cos(0 t) .

13.2 El oscilador armnico simple o

357

Evaluando las dos ultimas ecuaciones para t = 0, y usando las condiciones iniciales, se obtiene (0) = a = 0 y (0) = b0 = . La solucin expl o cita se obtiene sustituyendo los valores de a y b, que se deducen de estas relaciones, en la solucin general: o (t) = 0 cos(0 t) + sen(0 t) . 0

b) Como vimos en la seccin anterior, en lugar de cos(0 t) y sen(0 t) tambin podemos o e usar las soluciones particulares exp(i0 t) y exp(i0 t) como base. O sea, otra forma de escribir la solucin general de (13.12) es o (t) = exp(i0 t) + exp(i0 t) . Determinaremos las constantes y . Para ello, nuevamente, derivemos la solucin o general: (t) = i0 exp(i0 t) i0 exp(i0 t) . Evaluando estas dos ultimas ecuaciones para t = 0, y usando las condiciones iniciales, se obtiene (0) = 0 = + y (0) = = i0 i0 . Despejando y de estas dos relaciones: 1 2 1 = = 2 = 0 0 + i 0 0 i , .

Sustituyendo estos valores en la solucin general se obtiene o (t) = 1 2 0 i 0 exp(i0 t) + 1 2 0 + i 0 exp(i0 t)

Demostremos ahora que las expresiones encontradas en las partes a) y b) son equivalentes. En efecto, reordenando los trminos de la solucin encontrada en la parte b) se encuentra e o que (t) = 0 1 1 (exp(i0 t) + exp(i0 t)) i (exp(i0 t) + exp(i0 t)) 2 0 2 ei0 t + ei0 t ei0 t ei0 t = 0 + 2 0 2i = 0 cos(0 t) + sen(0 t) . 0

358

Oscilador Armnico o

Los dos procedimientos dan exactamente el mismo resultado. En el presente ejemplo, el segundo mtodo result ser ms engorroso, ms largo y menos transparente y ciertamente e o a a no se observa ninguna ventaja al haber introducido la base con exponenciales complejas. Sin embargo, en las secciones siguientes, al estudiar problemas levemente ms complejos, a la ventaja de usar las exponenciales complejas en lugar del seno y coseno resultar ms a a evidente.

13.3.

El oscilador armnico atenuado o

fr = x(t)

(con > 0) .

Figura 13.2

Usando la segunda ley de Newton se deduce que la posicin x(t) satisface la siguiente o ecuacin diferencial o m(t) = k x(t) x(t) . x Introduciendo las constantes 0 k/m y /2m se encuentra que la relacin dinmica o a para este oscilador armnico con roce es o
2 x + 2 x + 0 x = 0 .

(13.13)

Esta es la ecuacin diferencial del oscilador armnico atenuado. o o Ejercicio: Demuestre que la ecuacin diferencial (13.13) es lineal. o Deseamos encontrar la solucin general de la ecuacin (13.13). Sabemos que, si encontramos o o dos soluciones particulares distintas de (13.13) (denotmoslas por x1 (t) y x2 (t)), entonces e la solucin general vendr dada por o a x(t) = x1 (t) + x2 (t) , donde las constantes y se eligen de manera que la solucin satisfaga las condiciones o iniciales. Procederemos de acuerdo al siguiente esquema: primero encontraremos la solucin general o de (13.13) y luego determinaremos las constantes arbitrarias de la solucin general de o

^ x

Ejemplo: Consideremos una masa m adosada a un resorte de constante de restitucin o k. Supongamos que la masa m slo se pueo de desplazar a lo largo del eje x. Sea x(t) la posicin de m, siendo x = 0 la posicin o o de equilibrio. Supongamos adems que sobre a el sistema acta una fuerza de roce que es u proporcional a la velocidad x (pero de signo contrario), o sea

13.3 El oscilador armnico atenuado o

359

manera de obtener la solucin particular que, en t = 0, satisface las siguientes condiciones o iniciales: x(0) = x0 y x(0) = v0 = 0 . Ansatz (o hiptesis de trabajo): Busquemos soluciones del tipo x(t) = et , donde es una o constante por determinar. Derivando el Ansatz dos veces se obtiene x(t) = et , y x(t) = 2 et . Sustituimos estas relaciones en (13.13),
2 2 et + 2 et + 0 et = 0 ,

o sea,
2 2 + 2 + 0 = 0 .

Resolviendo esta ecuacin de segundo grado para se encuentra o = Debemos distinguir tres casos: i) Caso > 0 (oscilador armnico supercr o tico). En este caso la ecuacin (13.14) nos entrega dos soluciones distintas de la ecuacin o o diferencial, stas son e 2 + 2 0 t x1 (t) = e y
2 2 0 t 2 2 0 .

(13.14)

x2 (t) = e La solucin general, por lo tanto, es o x(t) = e

2 + 2 0 t

+e

2 2 0 t

Determinando y de manera que la solucin general anterior cumpla con las cono diciones iniciales x(0) = x0 y x(0) = 0, se encuentra x0 x(t) = 2 1+
2 2 0

2 + 2 0 t

2 2 0

2 2 0 t

360 La gura 13.3 muestra cualitativamente el comportamiento del oscilador en este caso. En el caso supercr tico la friccin es muy o grande y la masa m no oscila. Imag nese una bolita colgada de un resorte sumergida en un frasco con miel.
x (t) x0

Oscilador Armnico o

0 t

Figura 13.3 b) Caso < 0 (oscilador armnico subcr o tico). En este caso la ecuacin (13.14) tambin nos da dos soluciones distintas: o e 2 +i 0 2 t x1 (t) = e = et eit y
2 i 0 2 t

x2 (t) = e con

= et eit ,

La solucin general viene dada por o

2 0 2 .

x(t) = et eit + eit

Evaluando y de manera que la solucin cumpla las condiciones de borde x(0) = x0 o y x(0) = 0, se encuentra x(t) = x0 t e 2 1+ i eit + 1 sen(t) .
x (t) x0

it e i (13.15)

= x0 et cos(t) +

La gura 13.3 muestra cualitativamente el comportamiento del oscilador en este caso. En el caso subcr tico la friccin es relativao mente pequea y la masa m oscila. Note que n a medida que transcurre el tiempo la amplitud de la oscilacin decae exponencialmente. o

0 t

Figura 13.4 c) Caso = 0 (oscilador armnico cr o tico). Este caso es levemente ms complicado, ya que la ecuacin (13.14) nos da slo una a o o solucin: o x1 (t) = et .

13.4 El oscilador armnico forzado o

361

Debemos, de alguna manera, encontrar otra solucin para poder construir la solucin o o general. Ejercicio: Demuestre que la otra solucin de la ecuacin diferencial o o x + 2 x + 2 x = 0 es la funcin o x2 (t) = t et . Usando el resultado del ejercicio se encuentra que, para un oscilador armnico ateo nuado cr tico, la solucin general viene dada por o x(t) = ( + t) et . Para que la solucin cumpla con las condiciones de borde se determina que sta viene o e dada por x(t) = x0 (1 + t) et . (13.16) Observe que, independiente de las condiciones iniciales, el oscilador armnico atenuado pauo latinamente siempre se acercar a su posicin de equilibrio, es decir, para t , siempre a o x(t) 0. Ejercicio: Demuestre que la solucin (13.16) tambin se puede obtener a partir de (13.15) o e poniendo 0 = + y realizando el l mite 0.

13.4.

El oscilador armnico forzado o

Agregumosle al oscilador armnico atenuado una fuerza armnica externa Fe de una free o o cuencia , es decir, Fe = F0 cos(t) . Situaciones de este tipo se dan con gran frecuencia en la naturaleza. La ecuacin diferencial para el oscilador en este caso es o
2 x + 2 x + 0 x =

F0 cos(t) . m

(13.17)

Ejemplo: Demuestre que la ecuacin diferencial anterior no es lineal, o sea, la suma de dos o soluciones ya no sigue siendo solucin. o Si el lado derecho es nulo (o sea, F0 = 0), entonces la ecuacin coincide con la analizada o en la seccin anterior. En este caso conocemos la solucin general. Denotemos esta solucin o o o general (de la ecuacin homognea) por xh (t). Tal solucin tendr dos constantes arbitrarias. o e o a Sea xp (t) una solucin particular cualquiera de (13.17), entonces la solucin general ser o o a x(t) = xh (t) + xp (t) .

362

Oscilador Armnico o

Efectivamente, es fcil demostrar que x(t) es solucin de (13.17). De que es la solucin a o o general se desprende del hecho de que sta, por ser la ecuacin diferencial de segundo e o orden, debe tener dos constantes arbitrarias, las que x(t) efectivamente tiene (las de la funcin xh (t)). o En general, la solucin x(t) tiene un comportamiento complejo. Sin embargo, para tiempos o grandes (t ) la solucin xh (t) siempre desaparece, quedando slo la solucin particular o o o xp (t). Observe que xp (t) es independiente de las condiciones iniciales. Todas las soluciones del problema, para t , terminarn siendo idnticas. Cuando esto ocurre, se dice que se a e ha llegado al estado estacionario. Las oscilaciones iniciales del oscilador, que son altamente irregulares, y que si dependen de las condiciones iniciales, se llama es transiente. Para muchos problemas prcticos la solucin que interesa es la del estado estacionario. a o En lo que sigue encontraremos la solucin xp (t) que es la correspondiente al estado estacioo nario. Por ser algebraicamente mucho ms simple, usaremos extensivamente las funciones a exponenciales complejas. La fuerza externa la reemplazaremos por la expresin o Fe = F0 eit . En otras palabras, en lo que sigue encontraremos la solucin estacionaria de la ecuacin o o diferencial F0 it 2 x + 2 x + 0 x = e . (13.18) m Observe que la parte real de Fe corresponde a la fuerza externa Fe = F0 cos(t), luego, al tomar la parte real de esta ecuacin diferencial, obtenemos la ecuacin (13.17); y a su vez, o o la parte real de x(t) corresponder a la solucin estacionaria de (13.17). a o Hagamos el siguiente Ansatz: x(t) = A eit , o sea, analicemos si (13.18) puede tener una solucin de este tipo. Aqu A es una constante o que eventualmente habr que determinar. Derivamos x(t) respecto al tiempo: a x(t) = iA eit y x(t) = 2 A eit . Sustituyendo esto en (13.18) se obtiene
2 2 A eit + 2iA eit + 0 A eit =

F0 it e , m F0 2 2 A + 2iA + 0 A = , m

o sea, nuestro Ansatz es una solucin slo si o o A=


2 (0

F0 /m . 2 ) + 2i

Observe que A es un nmero complejo. u

13.4 El oscilador armnico forzado o Cualquier nmero complejo A se puede escribir de la forma u A = AR + iAI = |A| ei = |A| cos + i|A| sen ,

363

donde |A| es el mdulo y la fase del nmero complejo. Conociendo la parte real e imagio u naria de A se pueden encontrar el mdulo y la fase usando las relaciones o |A| = y AI . AR Usando las expresiones anteriores para el nmero complejo A se encuentra que ste puede u e escribirse de la forma tan = A= F0 /m
2 (2 0 )2 + 4 2 2

A2 + A2 R I

exp i arctan

2 2 0

Hemos encontrado una solucin particular de (13.18): o x(t) = F0 /m (2


2 0 )2

4 2 2

exp it + i arctan

2 2 0

La solucin estacionaria de (13.17) es la parte real de x(t), o sea, o x(t) = con |A| = y tan = 2 2 2 . 0 F0 /m (2
2 0 )2

4 2 2

cos t + arctan

2 2 0

= |A| cos(t + ) ,

F0 /m
2 (2 0 )2 + 4 2 2

Observe que la solucin estacionaria, o sea, despus de que el transiente ha desaparecido, o e oscila con la misma frecuencia con que la fuerza externa est forzando el sistema. Observe, a sin embargo, que las dos oscilaciones (la de la fuerza externa y la de la respuesta del sistema) no van es fase, sino que estn desfasados en . La amplitud con que oscila el sistema en el a estado estacionario viene dada por |A|. Resonancias Analicemos con ms detalle la amplitud con que oscila un oscilador armnico forzado en su a o estado estacionario. La gura 13.5a muestra la amplitud |A| en funcin de la frecuencia o con que se est forzando el oscilador. Las distintas curvas corresponden a distintos parmea a tros del coeciente de roce 2/0 . cuando el roce es pequeo, la amplitud llega a ser n muy grande cuando la frecuencia con que se fuerza el oscilador es parecida a la frecuencia natural del oscilador 0 . Estas grandes respuestas de un sistema de est mulos pequeos se n

364 conoce con el nombre de resonancias.

Oscilador Armnico o

Para pequeo, la amplitud mxima de la resonancia viene dada por n a |A|max = F0 . 2m0

La friccin , aun cuando es pequea, no puede despreciarse. De lo contrario se obtienen o n resultados absurdos; la amplitud del oscilador se ir incrementando indenidamente. a Figura 13.5a Figura 13.5b La gura 13.5b muestra el comportamiento de la fase . Observemos que si la frecuencia con que se fuerza el sistema es mucho menor que la frecuencia natural del sistema 0 , entonces el est mulo y la respuesta del sistema esencialmente estarn en fase; cuando a 0 , las dos magnitudes estarn completamente desfasadas. Cuando el sistema entra en a resonancia 0 , el desfase entre el est mulo y la respuesta del sistema es de 90

13.5.

Osciladores armnicos acoplados o


Figura 13.7 Sean x1 y x2 los desplazamientos de las dos masas respecto a sus posiciones de equilibrio. Las ecuaciones de movimiento para estas masas son: M x1 = kx1 + k(x2 x1 ) y M x2 = kx2 k(x2 x1 ) . (13.20) (Note que (x2 x1 ) es el alargamiento neto del resorte central respecto al largo que tiene cuando el sistema est en equilibrio.) Las ecuaciones (13.19) y (13.20) son dos ecuaciones a diferenciales de segundo orden acopladas (la segunda derivada de x1 depende no slo de o x1 , sino que tambin de x2 , y lo mismo ocurre para la segunda derivada de x2 ). Sumando e y restando las dos ecuaciones diferenciales obtenemos M (1 + x2 ) = k(x1 + x2 ) x y M (2 x1 ) = 3k(x2 x1 ) . x Denamos dos nuevas variables 1 y 2 por 1 = x1 + x2 (13.19)

Considere la conguracin mostrada en la o gura 13.7. Las masas estn restringidas a moa verse a lo largo del eje x. Analicemos la forma en la cual oscila este sistema.

^ x

13.5 Osciladores armnicos acoplados o y 2 = x2 x1 .

365

Con estas deniciones las dos ultimas ecuaciones diferenciales se pueden escribir de la forma M 1 = k1 y M 2 = k2 . Observe que estas ecuaciones ya no estn acopladas y que cada una de ellas corresponde a a la de un oscilador armnico simple. Las soluciones generales vienen dadas por o 1 (t) = A cos(1 t) + B sen(1 t) y 2 (t) = C cos(2 t) + D sen(2 t) , con 1 y 2 3k = 3 1 . M k M

Conociendo 1 y 2 en funcin del tiempo tambin conocemos el comportamiento de x1 y o e x2 : 1 x1 (t) = (1 (t) + 2 (t)) 2 A B C D cos(1 t) + sen(1 t) + cos(2 t) + sen(2 t) = 2 2 2 2 y 1 x2 (t) = (1 (t) 2 (t)) 2 A B C D = cos(1 t) + sen(1 t) cos(2 t) sen(2 t) . 2 2 2 2 Esta solucin general tiene cuatro constantes arbitrarias (A, B, C y D), las que se detero minan exigiendo que la solucin cumpla con las cuatro condiciones iniciales (la posicin y o o velocidad de cada una de las masas). Por ejemplo, si en t = 0, x1 (0) = x2 (0) = 0, x1 (0) = v0 y x2 = 0, entonces las constantes resultan ser A = C = 0, B = v0 /1 y D = v0 /2 . Debido a que la razn entre las frecuencias 1 y 2 no es un nmero racional, el sistema, o u en general, no manifestar un comportamiento peridico. a o

366 Modos normales

Oscilador Armnico o

Si en el problema anterior hacemos oscilar el sistema de manera que C = D = 0, entonces la posicin de ambas masas vendr dada por o a x1 (t) = x2 (t) = A B cos(1 t) + sen(1 t) . 2 2

Observe que en ese caso ambas masas oscilan juntas (en fase) y que el movimiento de cada una de ellas es armnico (con per o odo T1 = 2/1 ). Algo parecido ocurre cuando el sistema oscila de manera que A = B = 0. En este caso x1 (t) = x2 (t) = C D cos(2 t) + sen(2 t) . 2 2

Nuevamente ambas masas oscilan juntas, pero en sentido opuestos (en contrafase) y el movimiento de cada una de ellas es armnico (con per o odo T2 = 2/2 ). Estos modos de oscilacin armnicos del sistema se conocen con el nombre de modos normales. o o Un concepto util en la discusin de sistemas ms complejos es el de grados de libertad. o a Los grados de libertad de un sistema son el nmero de variables que se requieren para u describir el sistema. Por ejemplo: una masa m restringida a moverse a lo largo de una recta tiene un grado de libertad. La misma part cula, si su movimiento est connado a un plano, a tendr dos grados de libertad. Un sistema consistente de tres part a culas que pueden moverse en un plano, tiene 6 grados de libertad. Dos part culas en el espacio tridimensional unidas por una barra r gida poseen 5 grados de libertad. A continuacin resumiremos, sin demostracin, algunas caracter o o sticas generales que presentan todos los sistemas consistentes de osciladores armnicos acoplados. o i) Un sistema de osciladores armnicos acoplados de N grados de libertad se describe o con N funciones {xj (t)}. Las ecuaciones dinmicas son ecuaciones diferenciales de a segundo orden y generalmente estn acopladas. a ii) Siempre es posible introducir nuevas variables {j (t)} cuyas ecuaciones diferenciales son de la forma 2 j + j j = 0 , o sea, corresponden a osciladores armnicos simples. Las variables j (t) son combio naciones lineales de las variables {xj (t)}. Los mtodos generales para encontrar estas e nuevas variables sern materia de cursos ms avanzados. Sin embarga, en muchas a a situaciones simples no es dif encontrarlos por simple inspeccin. cil o iii) Algunas de las frecuencias j pueden ser nulas, en cuyo caso la ecuacin diferencial o j = 0. Los modos normales de frecuencia nula corresponden a la es simplemente traslacin o rotacin del sistema como un todo. o o iv) Cada una de estas nuevas variables da origen a un modo normal. Un sistema con N grados de libertad tiene N modos normales (algunos de ellos pueden tener frecuencia nula).

13.5 Osciladores armnicos acoplados o

367

v) La solucin de las ecuaciones diferenciales para las variables j (t) son inmediatas. En o total se tendrn 2N constantes arbitrarias. a vi) Siempre es posible despejar xj (t) en funcin de las funciones {j (t)} (en el lenguao je tcnico, el movimiento del sistema, en general, es una suma superposicin de e o los distintos modos normales). De esta manera se encuentra la solucin general del o problema. Las constantes arbitrarias se determinan exigiendo que la solucin cumo pla con las condiciones iniciales. Hay 2N condiciones iniciales: xj (0) y xj (0) para los j = 1, 2, . . . , N grados de libertad. vii) Cuando slo se excita un unico modo normal, todas las part o culas se movern armnia o camente y con la misma frecuencia. Cuando se excitan dos o ms modos normales es a forma simultnea, las part a culas no se movern armnicamente y el movimiento, en a o general, ni siquiera ser peridico. a o viii) Frecuentemente, en sistemas no demasiado complejos, es posible no slo identicar o algunos o todos los modos normales, sino que tambin encontrar las frecuencias rese pectivas por simple inspeccin del problema. o
m

Ejemplo: Consideremos la conguracin o mostrada en la gura 13.8. Las tres masas slo pueden moverse a lo largo del anillo de o radio R. Los resortes, todos con constante de restitucin k, tambin siempre se deforman a o e lo largo de la circunferencia. Encontraremos todos los modos normales con sus frecuencias.

k R

Figura 13.8 El sistema tiene tres grados de libertad y, por lo tanto, existirn tres modos normales. Uno a de ellos tiene frecuencia cero y corresponde a una rotacin (r o gida) uniforme de las tres masas a lo largo del anillo. Es evidente que otro modo normal de oscilacin del sistema es el mostrado en la gura 13.9: o una de las tres part culas queda quieta y las otras dos se mueven en sentidos opuestos.
1

2 t =0 3 T/ 4 T/2 3T / 4

Figura 13.9 No es dif encontrar la frecuencia angular de este modo. De los resortes que unen las cil part culas que se mueven, uno se acorta en una magnitud a = R y el otro se alarga en 2a;

368

Oscilador Armnico o

la fuerza neta sobre la part cula es, por lo tanto, 3ka. Para la frecuencia de este modo de vibracin se obtiene = 3k/m. o Otro modo normal se encuentra si la part cula 2 se mantiene quieta y las part culas 1 y 3 oscilan movindose en direcciones opuestas (ver gura 13.10). Por supuesto que este modo e de oscilacin tiene la misma frecuencia que el modo anterior (en el lenguaje tcnico se dice o e que los dos modos son degenerados).
1

2 t =0 3 T/ 4 T/2 3T / 4

Figura 13.10

Pareciera que existe un cuarto modo, en que la part cula 3 se mantiene quieta y las part culas 1 y 2 oscilan, movindose en direcciones opuestas (ver gura 13.11). Efectivamente este e tambin es un modo normal, pero no es uno distinto; en efecto, la superposicin de los e o modos mostrados en las guras 13.9 y 13.10 generan el modo mostrado en la gura 13.11. En el lenguaje tcnico se dice que el modo de la gura 13.11 no es un modo independiente e sino que es una combinacin lineal de los modos normales mostrados en las guras 13.9 y o 13.10.
1

2 t =0

3 T/ 4 T/2 3T / 4

Figura 13.11

13.6.

Modos normales de una cuerda

Consideremos una cuerda de largo L, sin masa, bajo la tensin , que al centro tiene adosada o una masa m y analicemos el movimiento transversal de la masa en ausencia de gravedad. Sea u(t) el desplazamiento transversal de la masa en funcin del tiempo. En todo momento o supondremos que el ngulo de la cuerdo con la horizontal es pequeo, es decir, que a n u(t) = tan L/2 .

13.6

Modos normales de una cuerda

369

m L
Figura 13.11

Adems supondremos que la tensin no var debido a la pequea elongacin que sufre a o a n o la cuerda cuando est deformada. La fuerza transversal neta sobre la masa m debida a la a tensin de la cuerda es o F = 2 sen 2 2 u 4 = u. L/2 L

La segunda ley de Newton nos da la relacin o m = u o sea,


2 u + 1 u = 0 ,

4 u, L

con

4 . Lm Concluimos que la masa m oscilar armnicamente con frecuencia 1 . a o


2 1 =

Consideremos ahora dos masas m adosadas a la cuerda en forma equiespaciada. Este sistema ahora tiene dos grados de libertad y, por lo tanto, tendr dos modos normales de oscilacin: a o uno en que las dos part culas oscilan en fase y otro en que oscilan en contrafase (ver gura 13.12). En el modo 1, la fuerza transversal que acta sobre cada masa es u F = sen = = u 3 = u. L/3 L

El desplazamiento de cada masa satisfacer la ecuacin de movimiento (segunda ley de a o Newton) 3 m = u , u L que es la ecuacin de un oscilador armnico con frecuencia angular o o
2 1 =

3 . Lm u 2u 9 = u. L/3 L/3 L

En el modo 2, la fuerza transversal que acta sobre cada masa es u F = sen sen = =


(13.21) (13.22)

370

Oscilador Armnico o

L /3 u

m u L /3 L m
Figura 13.12

L /3

La ecuacin de movimiento de cada masa (segunda ley de Newton) en este caso es o 9 u. L Nuevamente es la ecuacin de un oscilador armnico, pero ahora con la frecuencia angular o o m = u 9 . (13.23) Lm Generalicemos los resultados anteriores y consideremos N masas m adosadas en forma equiespaciada a la cuerda. Denamos el eje x a lo largo de la cuerda cuando est en su a posicin de equilibrio y elijamos el cero coincidiendo con el extremo izquierdo de la cuerda o (el otro extremo de la cuerda estar en x = L). La posicin longitudinal de la masa j ser a o a
2 2 =

xj = j

L . N +1

El sistema tiene N grados de libertad y por lo tanto existirn N modos normales. En lo a que sigue encontraremos los N modos normales con sus frecuencias respectivas. Para ello introduzcamos las N funciones y (x, t) = u(t) sen x , (13.25) L con = 1, 2, 3, . . . , N . Consideremos un particular (por ejemplo = 3) y desplacemos las N part culas transversalmente en una distancia uj (t) = y (xj , t). La gura 13.6 muestra esquemticamente la a situacin que se tiene en este caso. o Encontraremos la ecuacin de movimiento de la part o cula j. Los ngulos que la cuerda al a lado izquierdo y derecho de la part cula j forman con la horizontal son uj uj1 L/(N + 1)

modo 1

modo 2

(13.24)

13.6

Modos normales de una cuerda

371

y(x)

=3 u2 x2 m L N +1
Figura 13.13

y respectivamente (ver gura 13.14).

uj+1 uj , L/(N + 1)

L N +1 m u j 1 x j 1

uj

uj + 1

xj
Figura 13.14

xj+ 1

La fuerza transversal neta que acta sobre la part u cula j es F = sen + sen ( ) (2uj uj+1 uj1 ) N +1 . L

La ecuacin de movimiento para la part o cula j es, por lo tanto, m = u Pero uj (t) = y (xj , t), luego uj = u(t) sen xj = u(t) sen L j N +1 , (N + 1) (2uj uj+1 uj1 ) . L

uj+1 + uj1 = u(t) sen = 2 sen

(j + 1) + sen N +1 j cos N +1 N +1

(j 1) N +1

^ x

372 y 2uj uj+1 uj1 = 2u(t) 1 cos N +1 sen

Oscilador Armnico o

j N +1

Con estas relaciones la ecuacin de movimiento para la part o cula j queda m = u o sea,
2 u + u = 0

2 (N + 1) L

1 cos

N +1

u,

con
2 =

2 (N + 1) mL

1 cos

N +1

(13.26)

Observe que sta resulta ser la de un oscilador armnico y que es independiente de j, o e o sea, todas las masas oscilarn armnicamente con la misma frecuencia. En otras palabras, a o el movimiento ser el de un modo normal de vibracin del sistema. Haciendo variar se a o obtienen los distintos modos de vibracin. o Ejercicio: Demuestre que la ecuacin (13.26), para N = 1 (y = 1) coincide con (13.21) o y que para N = 2 (con = 1 y 2) coincide con (13.22) y (13.23), respectivamente. Ejercicio: Demuestre que para enteros > N no se obtienen nuevos modos de oscilacin. o A continuacin estudiaremos el caso de una cuerda de largo L, pero con una densidad lineal o de masa uniforme . La masa de tal cuerda es L. Para obtener la cuerda con masa tomaremos el l mite N y m 0 de manera que la masa total de la cuerda sea L, o sea, L y m0 tal que N m = L

En este l mite, para las frecuencias , se tiene


2 =

2 N 2 L(N m)

1 1

1 22 2 N2 .

22 , L2

o sea, = L

Esta ultima ecuacin da las frecuencia de los modos normales de una cuerda de largo L, o densidad lineal y bajo tensin . Hay innitos modos normales, todos ellos mltiplos o u enteros de una frecuencia fundamental 1 = L .

13.7 Problemas

373

13.7.
1.

Problemas

La aceleracin de la gravedad var con la posicin sobre la Tierra debido a su rotacin o a o o y a que el globo terrqueo no es exactamente esfrico. Esto fue descubierto por pria e mera vez en el siglo XVII, cuando se observ que un reloj pendular, cuidadosamente o ajustado para marcar el tiempo correcto en Par perd alrededor de 90 s por d s, a a cerca del Ecuador. a) Demuestre que una pequea variacin de g produce una pequea modicacin n o n o del per odo del pndulo T dado por e T 1 g = . T 2 g b) Cunto deber variar g para lograr explicar la diferencia del per a a odo de un pndulo entre Par y el Ecuador? e s

2.

Una masa de 2 kg se sujeta a un resorte de constante de fuerza k = 10 N/m que descansa sobre una supercie horizontal lisa. Otra masa de 1 kg se desliza a lo largo de la supercie hacia la primera a 6 m/s. a) Hallar la amplitud de la oscilacin si las masas realizan un choque perfectamente o inelstico y ambas quedan adosadas al resorte. Cul es el per a a odo de oscilacin? o b) Hallar la amplitud y per odo de la oscilacin si el choque es perfectamente elstio a co. c) En cada caso encuentre una expresin para la posicin x de la masa sujeta al o o resorte en funcin del tiempo, admitiendo que el choque se produce en el instante o t = 0.

Figura 13.15 a) Encuentre el nuevo punto de equilibrio x0 . b) Con qu per e odo oscilar la masa m alrededor de x0 ? a c) Encuentre la energ cintica y el potencial en funcin del tiempo. (Especique a e o claramente los or genes usados para especicar las energ potenciales.) as d) Encuentre la velocidad mxima que llegar a tener la masa m mientras oscila. a a

l0

x0

3.

Un resorte de constante de fuerza k = 100 N/m cuelga verticalmente de un soporte. En su extremo inferior (que se encuentra a una distancia l0 del techo) se engancha una masa de 0.5 kg, que luego (en el instante t = 0) se suelta, desde el reposo. La masa comenzar a oscilar en torno a un a nuevo punto de equilibrio x0 .

374

Oscilador Armnico o

4.

En una cuenca esfrica de radio r se dese liza una masa m1 una pequea distancia n s1 , siendo s1 r. Una segunda masa m2 se desplaza en sentido opuesto hasta una distancia s2 = 3s1 (tambin s2 r). e a) Si las masas se dejan libres en el mismo instante y resbalan sin roce, en dnde se encontrarn? o a b) Si la colisin es elstica, cundo o a a volvern las masas nuevamente a esa tar en reposo y en qu lugar? e

m1

s1

s2

m2

Figura 13.16

5.

Un bloque de madera se desliza sobre una supercie horizontal lisa. El bloque est sua jeto a un resorte que oscila con per odo de 0.3 s. Un segundo bloque descansa en su parte superior. El coeciente de roce esttico entre los dos bloques es s = 0,25. a a) Si la amplitud de oscilacin es 1 cm, se deslizar el bloque situado encima? o a b) Cul es la mayor amplitud de oscilacin para la cual no se deslizar el bloque a o a de encima?

6.

Una variable x(t) se comporta armnicamente. Si en t = 0, la posicin, la velocidad o o y aceleracin vienen dadas por x(0) = 1 cm, v(0) = 2 cm/s y a(0) = 4 cm/s2 , o respectivamente. Encuentre la posicin x(t) y la velocidad v(t) para t = 6 s. o La gura 13.17 muestra un tubo de seccin constante A y forma de U, abierto o a la atmsfera. El tubo est lleno hasta o a el nivel indicado por una l nea a trazos con un l quido incompresible que uye a travs del tubo con un rozamiento despree ciable. La longitud total de la columna de l quido es L. Demuestre que si se hace descender la supercie del l quido en uno de los brazos de la U y luego se deja libre, el nivel del uido oscilar armnicamente a o alrededor de su posicin de equilibrio con o un per odo dado por T = 2 L/2g.

7.

Figura 13.17

8.

Encuentre (aproximadamente) el menor valor de la frecuencia angular que podr a tener un oscilador armnico x(t), si lo que se conoce es que x(0) = 0, v(1 s) = 2 cm/s o y a(2 s) = 4 cm/s2 . Suponga que una variable x(t) var armnicamente con una frecuencia angular 0 = a o 1 . 2s

9.

13.7 Problemas

375

a) Encuentre la posicin x y la velocidad v en el instante t = 3 s si x(0) = 1 cm y o x(1 s) = 1 cm. b) Repita lo mismo pero con las condiciones de borde x(1 s) = 1 cm y v(1 s) = 4 cm/s. c) Repita lo mismo pero ahora con las condiciones de borde x(0) = 2 cm y v(2 s) = 4 cm/s.

10.

Se cuelga una masa M de un resorte y se pone en movimiento oscilatorio vertical, con una amplitud de 7 cm. La frecuencia de las oscilaciones es de 4 Hz. Al llegar M a la posicin ms baja, se le coloca encima una pequea piedrecita. Supongamos que la o a n masa de la piedrecita es tan pequea que no tiene mayor efecto sobre la oscilacin. n o

a) A qu distancia por encima de la posicin de equilibrio perder contacto la e o a piedrecita con la masa M ? b) Cul es la velocidad de la piedrecita cuando se separa de la masa M ? a

11.

Figura 13.18

12.

Considere una variable x(t) que satisface la ecuacin de un oscilador armnico ateo o nuado. Suponga que 0 = 1 rad/s y que se tienen las siguientes condiciones iniciales: x(0) = 2 cm, v(0) = 0 cm/s.

a) Encuentre la solucin si = 2,20 . Graque la solucin en el intervalo 0 < t < o o 20 s. b) Repita lo mismo de la parte (a), pero con = 0 . c) Repita lo anterior, pero ahora con = 0,50 . d) Repita lo de las partes (a), (b) y (c), con las condiciones iniciales x(0) = 0 cm y v(0) = 50 cm/s.

Un pndulo simple de 50 cm de largo cuele ga del techo de un vagn que se acelera o con una aceleracin a = 7 m/s2 en direco cin horizontal. Encuentre el per o odo del pndulo para pequeas oscilaciones en tore n no a su posicin de equilibrio. o

L m a

376

Oscilador Armnico o

m 1 = mg1 (1 2 ) m 2 = mg2 (1 2 ) La constante acopla los dos osciladores armnicos. Si = 0 (o sea, si el acoplao miento se hace cero) cada pndulo oscila e independientemente del otro.

1 l 2

Figura 13.19 a) Introduzca las nuevas variables 1 (t) = 1 (t) + 2 (t) 2 (t) = 1 (t) 2 (t) y demuestre que stas var armnicamente con las frecuencias e an o 0 = g/ y 1 =
2 0 + ,

respectivamente, donde = 2/(m ). b) Demuestre que la solucin general se puede escribir de la forma o 1 1 (t) = [A cos(0 t + ) + B cos(1 t + )] 2 1 2 (t) = [A cos(0 t + ) B cos(1 t + )] 2 Las constantes A, B, y se determinan con las condiciones de borde. c) Sea 0 = 1 rad/s y = 0,1 s2 . Encuentre la solucin para el caso en que o 1 (0) = 2 (0) = 0. Graque 1 (t) y 2 (t). 1 (0) = 0 , 2 (0) = d) Repita lo anterior, pero para el caso en que 1 (0) = 2 (0) = 0 y 1 (0) = 2 (0) = 0. e) Repita lo anterior, pero para el caso en que 1 (0) = 2 (0) = 0 y 1 (0) = 2 (0) = 0. f) Para el caso (c) el movimiento de cada pndulo consiste en un movimiento oscie latorio cuya amplitud tambin var peridicamente. Sea la frecuencia angular e a o de la variacin peridica de la amplitud. Encuentre . o o 14. Pndulo f e sico: Considere un objeto de masa M , que puede oscilar alrededor de un eje que lo atraviesa. Sea I el momento de inercia para rotaciones alrededor de ese eje

13.

Considere dos pndulos idnticos acoplae e dos. Las ecuaciones de movimiento en ese caso vienen dadas por:

13.7 Problemas

377

y la distancia entre el eje y el centro de masas del objeto. Encuentre el per odo T para pequeas oscilaciones alrededor de su posicin de equilibrio. Demuestre que un n o pndulo simple equivalente, es decir, uno que tenga el mismo per e odo, tiene un largo
0

I . m
m

15.

Considere la conguracin mostrada en la o gura 13.20. Las cuatro masas slo pueden o moverse a lo largo del anillo de radio R. (Los resortes tambin siempre se deforman e a lo largo de la circunferencia.) Encuentre la frecuencia de los modos normales de oscilacin. o

k R

k k m

Figura 13.20 16. Considere una masa m resbalando sin roce (en presencia de la aceleracin de gravedad o g y ) a lo largo de un perl de la forma y(x) = x3 x2 , con = 1 m2 y = 3/2 m1 . Graque y(x). Si la masa realiza pequeas oscilaciones n en torno al m nimo local, encuentre el per odo T de tal movimiento. 17. Una masa de 2 kg oscila colgada de un resorte de constante de restitucin k = 400 o N/m. La constante de amortiguamiento es = 1 s1 . El sistema es forzado por una fuerza sinusoidal de amplitud F0 = 10 N y frecuencia angular = 10 rad/s. a) Cul es la amplitud de las oscilaciones en el rgimen estacionario? a e b) Si se var la frecuencia de la fuerza impulsora, a qu frecuencia se producir la a e a resonancia? c) Encuentre la amplitud de las vibraciones en la resonancia. 18. Considere una masa m = 50 g que oscila sujeta a un resorte de constante de restitucin o k. Suponga que hay algn dispositivo que atena las oscilaciones con una fuerza que u u es proporcional a la velocidad (o sea, estamos en presencia de un oscilador armnico o atenuado). Con un cronmetro se mide el per o odo de oscilacin; ste resulta ser o e igual a 2.1 s. a) Cunto valen 0 y ? a b) En cunto disminuir la amplitud mxima de oscilacin entre dos ciclos consea a a o cutivos?

378 19.

Oscilador Armnico o Una masa m = 1 kg cuelga de un resorte de constante de restitucin k = 200 N/m. La o constante de amortiguamiento es = 1 s1 . En el instante t = 0 comienza a actuar sobre la masa una fuerza F = F0 sin(t), con F0 = 2 N y = 10 s1 . a) Si x(0) = 0 y v(0) = 0, encuentre x(t) para t = 1 s, t = 100 s y t = 1000 s. b) Encuentre la energ disipada en un ciclo cuando el oscilador se encuentra en el a rgimen estacionario. e

20.

Una masa m descansa sobre una mesa horizontal lisa (sin roce). El movimiento de la masa est restringido a desplazamientos a lo largo del eje x. Sobre la masa acta una a u fuerza F (t) = F0 sin(t). x a) Encuentre la aceleracin a(t) y la velocidad v(t) de la masa, si en el instante o t = 0 se encontraba detenida. b) Encuentre la posicin x(t) si adems se sabe que x(0) = 0. Demuestre que el o a movimiento es armnico con una amplitud A = F0 /(m 2 ). o c) La masa ahora se sujeta adicionalmente a un resorte de constante de restitucin k. (La orientacin del resorte tambin es a lo largo del eje x). Compare el o o e movimiento que tiene ahora con el que ten cuando no estaba unida al resorte. a

= donde es una constante (que depende del largo, dimetro y material de que est hea a cho el alambre). Figura 13.21

Para este problema suponga que = 1250 g cm2 /s2 . Si las esferas son de aluminio (Al = 2, 7 g/cm3 ), qu dimetro deben tener las esferas para que el per e a odo sea exactamente de un segundo? (El momento de inercia de una esfera slida de masa M y o radio R para una rotacin alrededor de un eje que pasa por su centro es I = 2mR2 /5). o 22. Una masa de m = 0.5 kg, despus de caer una distancia h = 5 m, se adosa a un e resorte (largo) de constante k = 2 kg/s2 . El sistema resultante viene gobernado por la ecuacin de movimiento o

21.

(Pndulo de torsin) e o Suponga que un extremo de un alambre metlico est rmemente adosado del ciea a lo de una pieza y del otro cuelgan dos esferas slidas tal como se mustran en la gura o adjunta. Al girar las esferas con el alambre en un ngulo (alrededor del eje formaa do por el alambre), el alambre ejercer un a torque que har que las esferas retornen a a la posicin de equilibrio. El torque que o ejerce el alambre es

13.7 Problemas

379

2 z (t) + 0 z(t) + 20 z(t) = 0

o sea, corresponde a un oscilador armnico amortiguado cr o tico. La magnitud z(t) mide la posicin de la masa m respecto al punto de equilibrio y 0 = k/m es la o frecuencia natural del sistema. La solucin general est dada por la relacin o a o z(t) = (A + Bt)e0 t donde A y B son constantes que se ajustan con las condiciones iniciales. (Para los clculos numricos que siguen, use para la aceleracin de gravedad el valor a e o g = 10 m/s2 )

a) Determine A y B usando las condiciones iniciales. b) Sea t0 el instante en que el resorte tiene su mxima compresin. Evale a o u t0 . (Elija el cero del tiempo en el instante en que la masa colisiona con el resorte). c) Haga un grco esquemtico de la a a funcin z(t). o d) Cul ser la energ total disipada a a a por el amortiguador?

Figura 13.22 23. Considere dos cilindros que giran rpidaa mente en sentidos contrarios tal como se muestra en la gura adjunta. Sobre estos cilindros se coloca un tabln de masa M y o densidad uniforme. Sea d la distancia entre los dos cilindros y sea el coeciente de roce cinemtico entre el tabln y los cia o lindros. Demuestre que el movimiento del tabln es armnico. Encuentre el per o o odo del movimiento.

M d

Figura 13.23

24.

Considere dos masas m1 y m2 unidas por un resorte de largo natural o y constante de restitucin k. Supongamos que el movimiento de ambas masas est restringido a o a

resorte amortiguado

380 lo largo de la recta que los une.

Oscilador Armnico o

Sean x1 (t) y x2 (t) las posiciones de las masas m1 y m2 , respectivamente. a) Demuestre que x1 (t) y x2 (t) satisfacen las ecuaciones diferenciales acopladas

m1 x1 (t) = k[x2 (t) x1 (t) m2 x2 (t) = k[x2 (t) x1 (t) b) Denamos las variables 0 (t) y 1 (t) por

0] 0]

m1 x1 (t) + m2 x2 (t) m1 + m2 1 (t) = x2 (t) x1 (t) 0 0 (t) = Demuestre que las variables 0 (t) y 1 (t) satisfacen las ecuaciones diferenciales (desacopladas)

0 = 0 1 + 2 1 = 0 con 2 = k m1 + m2 m1 m2

c) Demuestre que la solucin ms general del problema se puede escribir de la forma o a m2 ( m1 + m2 m1 x2 (t) = A + Bt + ( m1 + m2

x1 (t) = A + Bt

0 0

+ C cos(t) + D sin(t)) + C cos(t) + D sin(t))

d) Denamos 0 y por 0 = k/m y = m2 /m1 . Exprese en trminos de 0 e y . Cunto vale en trminos de 0 si ? Coincide esto con lo que a e usted intu Cuanto vale en trminos de 0 si = 1? a? a e e) Sea 0 = 8 cm y 0 = 1 rad/s y = 1 (o sea, m1 = m2 ). Encuentre la solucin o que satisface las siguientes condiciones iniciales: x1 (0) = 0, x2 (0) = 10 cm y v1 (0) = v2 (0) = 0. Graque x1 (t) y x2 (t) en un mismo grco para el intervalo a 0 < t < 15 s. f) Repita lo mismo de la parte (e) pero para las condiciones iniciales x1 (0) = 0, x2 (0) = 8 cm, y v1 (0) = 4 cm/s y v2 (0) = 0.

13.7 Problemas g) Repita la parte (f) pero con = 10. h) Repita la parte (f) pero con = 0,1.
m

381

m x

25.

Figura 13.24

Sean x1 (t), x2 (t) y x3 (t) las posiciones de las tres masas en funcin del tiempo. o a) Demuestre que x1 (t), x2 (t) y x3 (t) satisfacen las ecuaciones diferenciales acopladas

m1 = k(x2 x1 x m3 = k(x2 x3 + x b) Intoduzca las nuevas variables denidas por

0)

m2 = k(x3 + x1 2x2 ) x
0)

0 = (x1 + x2 + x3 )

, 1 = (x1 x3 ) +

y 2 = (x1 2x2 + x3 ).

Demuestre que estas nuevas variables satisfacen las ecuaciones diferenciales desacopladas
2 2 , 1 + 1 1 = 0 y 2 + 2 2 = 0, k/m y 2 = 31 . Interprete!

0 = 0

con 1 =

c) Demuestre que la solucin general al problema se puede escribir de la forma o

x1 (t) = A + Bt + C cos(1 t + 1 ) + D cos(2 t + 2 ) x2 (t) = A + Bt 2D cos(2 t + 2 ) x1 (t) = A + Bt C cos(1 t + 1 ) + D cos(2 t + 2 ) +

Las constantes A, B, C, D, 1 y 2 se eligen de manera que la solucin satisfaga o las condiciones de borde. Convnzase de que, en general, las condiciones de borde e determinan a las seis constantes. d) Suponga que 0 = 5 cm y 1 = 1 rad/s. Encuentre la solucin que satisface o las siguientes condiciones iniciales: x1 (0) = 8 cm, x2 (0) = 0, x3 (0) = 8 cm, v1 (0) = v2 (0) = v3 (0) = 0. Graque en un mismo grco x1 (t), x2 (t) y x3 (t) en a el intervalo 0 < t < 15 s.

Considere tres part culas de masa m que slo pueden moverse a lo largo del eje x o y estn unidas por resortes de largo naa tural 0 y constantes de restitucin k (ver o gura).

382

Oscilador Armnico o e) Repita lo mismo que la parte (d), con las condiciones iniciales x1 (0) = 4 cm, x2 (0) = 2 cm, x3 (0) = 6 cm y v1 (0) = v2 (0) = v3 (0) = 0 f) Repita lo mismo que la parte (d), con las condiciones iniciales x1 (0) = 8 cm, x2 (0) = 0 cm, x3 (0) = 5 cm y v1 (0) = v2 (0) = v3 (0) = 0 g) Repita lo mismo que la parte (d), con las condiciones iniciales x1 (0) = 5 cm, x2 (0) = 0 cm, x3 (0) = 5 cm, v1 (0) = v2 (0) = 0 y v3 (0) = 3 cm/s.

26.

Considere un resorte, de constante de restitucin k, que conecta dos masas, M y m o restringidas a moverse a lo largo del eje x. Encuentre la frecuencia de oscilacin de o tal sistema.

27.

Suponga que la energ potencial de cierta molcula diatmica viene razonablemente a e o bien desvrita por la expresin o 1 U (r) = U0 2 r0 r
12

r0 r

con U0 = 2 eV (eV es una unidad de energ usada en la f a sica atmica llamada o 9 m). electron-volt) y r0 = 0,5 nm (1 nm=10 a) Demuestre que r = r0 es la separacin de equilibrio de la molcula. o e b) Graque U (r) en el rango 0,4 nm< r < 0,7 nm. c) Desarrolle el potencial U (r) en torno a r = r0 , es decir exprese U (r) de la forma 1 U (r) = c0 + c1 s + c2 s2 + . . . 2 donde s = r r0 y encuentre los coecientes c0 , c1 y c2 . d) Convnzase de que la fuerza para pequeos desplazamientos de los tomos respece n a to a su posicin de equilibrio (que ocurre para s = 0) viene dada por F (s) = ks. o Evale k. u e) Si las masas de los tomos son m1 = m2 = m, cul ser la frecuencia vibracional a a a de la molcula? e

m
28. Considere cuatro masas iguales unidas por resortes de constante de restitucin k tal o como se muestra en la gura. Las masas slo se pueden mover en el plano en que se o ubican. Usando argumentos de simetr a, describa algunos de los modos normales de vibracin y encuentre la frecuencia de o ellos. Cuantos modos normales tiene esa te sistema? Cuntos de ellos tienen frea cuencia cero?

k k

k k

k
Figura 13.25

13.7 Problemas 29.

383

Un reloj de los abuelos se basa en un pndulo de longitud 1 m. El reloj se atrasa 1 e segundo por d En cunto se debe corregir la longitud del pndulo? a. a e

30.

Un resorte de constante de resitucin o k = 2 dina/cm y largo en reposo 0 se encuentra adosado rmemente a la base de un recipiente (ver gura). El recipiente est lleno de agua. a Suponga ahora que en el instante t = 0 se le adosa al extremo superior una esfera slida homognea de radio R = 1 cm, o e hecha de un material ms liviano que el a agua, y que la esfera luego se suelta (o sea, en el instante t = 0 la longitud del resorte es 0 y la esfera se suelta en reposo). Se observa que la esfera realiza oscilaciones armnicas de amplitud A = 0,8 cm. o

t=0

l0

Figura 13.26

a) Encuentre la densidad de la esfera. b) Encuentre el per odo T del movimiento armnico que la esfera realiza una vez o que se suelta.

(Al desarrollar el problema ignore los efectos debidos al roce viscoso entre la esfera y el agua).

31.

El pndulo de la gura est formado por e a una barra de masa despreciable y longitud L. La masa del extremo inferior se mantiene unido a un resorte de constante k dispuesto horizontalmente y jo, por su otro extremo a una pared. Cuando el pndulo e se encuentra en posicin vertical la longio tud del resorte es la de su largo natural. Calcule la frecuencia del sistema. Verique su resultado analizando el l mite de algunos sistemas conocidos.

Figura 13.27

384

Oscilador Armnico o

32.

Figura 13.28

13.8.

Solucin a algunos de los problemas o

Solucin al problema 8 o La forma general de la solucin para un oscilador armnico simple es o o x(t) = A cos(t) + B sin(t). La condicin x(0) = 0 implica que A = 0, luego queda o x(t) = B sin(t). Derivando se obtiene v(t) = B cos(t) y a(t) = 2 B sin(t)

Aplicando las condiciones de borde se encuentra que v(1) = B cos() = 2 y a(2) = 2 B sin(2) = 2 2 B sin() cos() = 4. Formando el cuociente entre las dos ultimas ecuaciones obtenemos sin() = 1

La gura 13.29 muestra un grco del lado izquierdo y derecho de esta ecuacin. La intera o seccin de menor frecuencia ocurre para 3,43 s1 . o Solucin al problema 15 o El sistema tiene 4 grados de libertad y por lo tanto existen cuatro modos normales. Sean j , j = 1,2,3,4 los cuatro ngulos de las cuatro masas respecto a sus posiciones de equilia brio. Los cuatro modos normales se encuentran por simple inspeccin del problema. o

Considere un cilindro de radio R y densidad , con una perforacin cil o ndrica de radio R/2, tal como se muestra en la gura. El cilindro rueda sin resbalar sobre una supercie horizontal realizando pequeas n oscilaciones en torno a su posicin de equio librio. Encuentre el per odo de las oscilaciones.

13.8 Solucin a algunos de los problemas o

385

1 0 1 2 3 4 5 1 2

sin()
3 4 5

0 3.2 3.25 0.05 0.1 0.15

3.3

3.35

3.4

3.45

3.5 3.55

3.6

1/

0.2

0.25 0.3 0.35 0.4 0.45

sin()

1/

Figura 13.29

i) Uno de los modos normales tiene frecuencia nula (1 = 0) y corresponde a la rotacin o uniforme y simultnea de las cuatro masas a lo largo del anillo, o sea, 1 (t) = 2 (t) = a 3 (t) = 4 (t) = t. ii) Otro modo normal se obtiene si las part culas 1 y 3 se mantienen en reposo y las part culas 2 y 4 oscilan con la misma amplitud pero en sentido contrario, o sea 1 (t) = 3 (t) = 0, 2 (t) = 4 (t), t. Al desplazar la masa 2 en un ngulo uno de los resortes a se comprime y el otro se alarga en una magnitud R. La fuerza sobre la masa ser igual a a 2kR, luego la frecuencia de este modo normal ser 2 = 2k/m. a iii) Otro modo normal se obtiene si las part culas 2 y 4 se mantienen en reposo y las part culas 1 y 3 oscilan con la misma amplitud pero en sentido contrario. Por simetr a este modo tiene la misma frecuencia que el modo normal anterior (3 = 2k/m). iv) El cuarto modo normal se obtiene si las cuatro masas oscilan con la misma amplitud, 1 y 3 en la misma direccin y 2 y 4 en la direccin contraria, es decir, 1 (t) = 3 (t) = o o 2 (t) = 4 (t) =, t. Al desplazarse una masa en un ngulo , uno de los resortes a se acorta y el otro se alarga en una magnitud 2R. La fuerza sobre la masa ser, por a lo tanto, igual a 4kR. Luego la frecuencia de oscilacin es 2 = 4k/m. o La gura 13.30 muestra esquemticamente el movimiento de las cuatro masas para los a cuatro modos normales. Solucin al problema 21 o Al girar el alambre con las esferas en un ngulo = el torque es a z = . z El torque genera un cambio del momento angular del sistema. Se tiene

386
w

Oscilador Armnico o
Modo #1

#2
tu

#3
vw v pq

#4

Figura 13.30

d d z = I z = I , dt dt

donde I es el momento de inercia de las dos esferas para rotaciones alrededor del eje z (que coincide con el alambre). De las dos ecuaciones anteriores se deduce que I = o sea,
2 + 0 = 0 2 con 0 = /I. Para el per odo T se obtiene

T = 2

I .

Usando el teorema de Steiner, para el momento de inercia se encuentra la expresin o I=2 2 14 14 4 3 2 56 mR2 + mR2 = mR2 = R R = R5 . 5 5 5 3 15

Usando esto en la expresin anterior para el per o odo y despejando R se encuentra





RS

xy

fg

1
T U

pq

2
p

tu

DE

4 2

( ) ( ) 01 &' $% 67 6 r s de 89 45 r s 23 ! "#   @ A @ A `a lm fg xy ~ ` l hi ~ bc XY h jk | } z{ n o de VW | } n o

Tiempo

13.8 Solucin a algunos de los problemas o

387

15 T 2 = 0, 99986 cm5 . 56 4 3 O sea, con esferas de dimetro igual a 1 cm, este pndulo tendr un per a e a odo de 1 segundo. R5 = Solucin al problema 22 o a) Sea x0 la magnitud que el resorte se comprimir respecto a su largo natural una vez a que llegue al equilibrio. Se tiene que kx0 = mg o sea, x0 = 0, 5 10 mg = k 2 m = 2, 5 m.

La velocidad v0 de la masa cuando choca con el resorte viene dada por v0 = 2gh = 2 10 5 m = 10 s m . s

Por consiguiente, las condiciones iniciales son x(0) = x0 = 2, 5 m y x(0) = v0 = 10 k/m = 2 1 . s m . s

La frecuencia angular natural del sistema es 0 = Derivando la expresin o

z(t) = (A + Bt)e0 t se obtiene z(t) = (B A0 B0 t)e0 t . Evaluando estas expresiones en t = 0 se obtiene z(0) = A y z(0) = B A0 .

Usando las condiciones iniciales se encuentra para A y B los valores A = x0 = 2, 5 y B = A0 + z(0) = (2, 5 2 10) m = 5 s m . s m

388

Oscilador Armnico o

0.6 0.5 0.4 0.3 0.2 0.1 0

t
1 2 3 4

z(t)

0.1 0.2 0.3 0.4

Figura 13.31

b) La velocidad z(t) es nula cuando (B A0 B0 t) = 0. De esta relacin se deduce o que ello ocurre en el instante to = 1 A = 0 B 1 2, 5 2 (5) s=1 s.

c) La gura 13.31 muestra el grco de la posicin z(t) en funcin del tiempo. a o o d) Del cambio de energ potencial U = mg(h + x0 ), a 1 2 kx 2 o queda como energ potencial del resorte; el resto se disipa. Por lo tanto, la energ a a disipada es 1 Q = mgh + mgx0 kx2 2 0 1 = mgh + kx2 2 0 1 1 = 10 5 + 2 (2, 5)2 2 2 Solucin al problema 30 o Una vez que se adosa la esfera al resorte el nuevo punto de equilibrio del resorte sube en una magnitud D que se puede evaluar de la relacin o 4 kD = R3 (0 )g, 3

Joule = 31, 25

Joule

13.8 Solucin a algunos de los problemas o

389

donde 0 = 1 g/cm3 es la densidad del agua. Observe que la amplitud de la oscilacin o coincidir con D, o sea, D = A = 0,8 cm. Despejando se encuentra a = 0 Ahora k/g = 2 dina/g = 2 gramos, luego = 1 3 2 0, 8 4 g/cm3 = 0, 618 g/cm3 . 3kA . 4R3 g

El per odo del movimiento viene dado por T = 2 m , k

donde m = 4R3 /3 = 4 0,618/3 g = 25,9 g. Para el per odo se encuentra T = 1,75 s.

You might also like